You are on page 1of 154

Criminal Procedure Rowena Daroy Morales

ABBARIAO v BELTRAN 468 SCRA 421 PANGANIBAN; August 31, 2005


NATURE Administrative matter in the Supreme Court FACTS - This stemmed from an administrative case filed by Abbariao against Judge Beltran. Beltran was charged with gross ignorance of the law and knowingly rendering unjust judgment. - Abarriao was the former branch manager of Country Bankers Assurance Corporation. In October 1992, Joseph Abraham procured 2 insurance policies from him and issued a post-dated check to serve as pasyment. But the cheque was subsequently dishonored. - Informations were filed before before Judge Beltran for estafa and violation of BP 22. Petitioners Claim - Petitioner claims that Beltrans ruling that there was no valid insurance contract with Abraham was erroneous and that he had no jurisdiction over the case in the first place. Respondents Comments - Beltran claims that there was no valid insurance contract between Abbariao and Abraham because the insurance policy form of Abraham was disapproved. - He also claims he has jurisdiction over the case because the accused was arraigned in his court and the prosecutor failed to withdraw the case. ISSUE 1. WON the judge is guilty of rendering unjust judgment 2. WON Beltrans ruling showed gross ignorance of the law in terms of assuming jurisdiction over the case HELD 1. NO, the judge must be absolved from this charge. Ratio The acts of judges pertaining to their judicial functions are not subject to disciplinary power, unless such acts are commited with fraud, dishonesty, corruption or bad faith. In the absence of proof to the contrary, an erroneous decision or order is presumed to have been issued in good faith. 2. YES. Beltran had no authority to rule over the case.

a2010

page 1

Prof.
warrant of arrest. The spouses then filed a motion to quash and sought the nullification of subsequent orders. They alleged that the MTCC had no jurisdiction and authority to conduct a preliminary investigation of a complaint filed by an offended party directly with the court. The authority to conduct a preliminary investigation was vested solely on the Office of the City Prosecutor. - While waiting for the resolution of their motion to quash, the spouses did not post bail. On April 10, 2002 Corazon Ribaya was apprehended by arresting officers in the public market by virtue of a warrant of arrest issued by respondent judge. - The complainant filed this administrative case against Judge Parcia. The motions basically questioned respondents authority to conduct a preliminary investigation. - In her answer, respondent judge claimed that complainant was not a party in Criminal Case No. 8617. Respondent explained that she conducted the preliminary investigation of the criminal complaint against the spouses because the Officer-in-Charge (OIC) of the Office of the City Prosecutor was too busy to do so. - To support her claim, respondent attached the affidavit of OIC City Prosecutor Vasquez of the then newly-created Ligao City. Vasquez stated that the City Prosecutors Office was still undergoing reorganization when the subject criminal complaint was filed. It had neither enough manpower nor office space in the Hall of Justice. Positions had not yet been filled. His workload as Asst Provincial Prosecutor and OIC City Prosecutor was so heavy that time constraints did not permit him to conduct preliminary investigations. Thus, it was the respondent judge who conducted the preliminary investigation. - the Office of the Court Administrator (OCA) found that respondent erred when she conducted the preliminary investigation of the subject criminal complaint even after the Municipality of Ligao, Albay had been converted into a city. The OCA recommended that: (1) the complaint be re-docketed as a regular administrative matter; and (2) the respondent be reprimanded, directed to concentrate her time and effort on performing her judicial tasks and warned that a repetition of the same or similar offense would be dealt with more severely. - A motion for reconsideration was filed by respondent on October 1, 2003. This time, respondent claimed that what she conducted

Ratio A court can only take cognizance of a case that falls within its jurisdiction. Reasoning - April 15, 1994 is the date of effectivity of RA 7691. RA 7691 expanded the jurisdiction of the first-level courts by providing that first-level courts shall have jurisdiction over criminal cases in which the offense is punishable with imprisonment not exceeding 6 years, regardless of the amount of the fine. - January 30, 1995 was the date the information was filed. The case had to do with the violation of BP 22 which is penalized by an imprisonment of not less than 30 days but not more than one year. Thus when the information was filed, RA 7691 was already in effect. - During the tenure of the former presiding judge, the issue of jurisdiction over the case was already pending resolution. He also displayed indecisiveness by relying on the public prosecutors assurance that his court had acquired jurisdiction. - Aside from this instance, there were two other occasions when Beltran was charged and found guilty of gross ignorance of the law (in De Austria v. Beltran and Andres v. Beltran). DISPOSITION Judge Beltran was found guilty of gross ignorance of the law for which he is fined P20,000 as recommended by the OCA and is warned that a repetition of the same act would merit a graver penalty.

RIBAYA v JUDGE BINAMIRA-PARCIA AM No. MTJ-04-1547 CORNEJO; April 15, 2005


NATURE Administrative case against Judge Aurora BinamiraParcia of the Municipal Trial Court in Cities (MTCC), Ligao City, Albay relative to Criminal Case No. 8617 (People v. Sps. Ribaya) FACTS - Asst Provincial Prosecutor Pedro Vega, in his personal capacity, filed before the MTCC, Ligao City a criminal complaint for estafa against the Spes Ribaya on November 29, 2001. The spouses, after receiving P12,000 from Vega, allegedly misappropriated the amount to the latters prejudice. The preliminary investigation was then conducted by respondent judge. - Complainant, the daughter of the accused spouses, observed several irregularities in the conduct of the preliminary investigation and the issuance of the

Criminal Procedure Rowena Daroy Morales


on November 29, 2001 was a preliminary examination to determine probable cause for the issuance of a warrant of arrest against the spouses. Respondent also claimed that the criminal complaint was governed by Sec. 9, Rule 112 of the Revised Rules of Criminal Procedure, the rule governing cases that did not require preliminary investigation. Since the amount involved in the estafa case was P12,000, no preliminary investigation was required. ISSUES 1. WON respondent judge had the authority to conduct preliminary investigation 2. WON respondent judge is guilty of misconduct in office 3. WON preliminary investigation was not required for the estafa case 4. WON warrant of arrest was valid HELD 1. NO - Although judges of inferior courts are authorized to conduct preliminary investigation of all crimes within their jurisdiction, the task is essentially an executive function. As far back as Collector of Customs v. Villaluz, we already held that: [w]hile we sustain the power of the x x x courts to conduct preliminary examination, pursuant to Our Constitutional power of administrative supervision over all courts, as a matter of policy, we enjoin x x x judge[s] x x x to concentrate on hearing and deciding before their courts. x x x [Judges] should not encumber themselves with the preliminary examination and investigation of criminal complaints, which they should refer to the municipal judge or provincial or city fiscal, who in turn can utilize the assistance of the state prosecutor to conduct such preliminary examination and investigation. - City judges then were clearly authorized to conduct preliminary investigation and examination. But even then, we also held that the provisions of Rule 112 granting city judges the authority to conduct preliminary investigation did not apply to judges of cities the charters of which authorized the city fiscal only to conduct preliminary investigation of criminal complaints. - This ruling was, in fact, integrated into the Revised Rules of Criminal Procedure. Under Sec. 1, Rule 110, criminal actions in chartered cities are

a2010

page 2

Prof.
judge rendering it must at all times maintain the appearance of fairness and impartiality. - Considering all this, respondent judge committed simple misconduct in office. Misconduct in office has a well-defined meaning. It refers to misconduct that affects the judges performance of her duties and not just her character as a private individual. To constitute an administrative offense, misconduct should relate to or be connected with the performance of the official functions and duties of a public officer. 3. YES - Respondent correctly observed that it was not needed in the estafa case. The maximum penalty for the crime allegedly committed there (6 months and 1 day to 4 years and 2 months) did not meet the minimum penalty (at least 4 years, 2 months and 1 day) required to make a preliminary investigation part of the spouses right to due process. 4. YES - As long as the constitutional mandate was complied with, that is, the warrant of arrest was issued upon a finding of probable cause personally by the judge after an examination under oath or affirmation of the complainant and the witnesses he may produce, the warrant of arrest was valid. - Respondent judge examined the complainant Pedro Vega on the day the complaint was filed and she was satisfied that probable cause existed. The warrant of arrest she issued against the spouses Ribaya was, therefore, justified and no violation of their constitutional rights occurred. DISPOSITION Respondent Judge Aurora Binamira-Parcia is hereby found guilty of simple misconduct and a fine of P11,000 is imposed on her. She is hereby directed to devote her time and effort exclusively to discharging her judicial functions. She is furthermore warned that a repetition of the same or similar act will merit a more severe penalty.

instituted by filing the complaint only with the City Prosecutor. The rule implies that the task of conducting preliminary investigation in these cities is now lodged with the Office of the City Prosecutor. Consequently, inferior court judges of cities whose charters authorize only the fiscal to conduct preliminary investigation are no longer allowed to perform this function. - The Municipality of Ligao was converted into a city by RA 9008 which took effect on Feb 21, 2001. This law, also known as the charter of the City of Ligao, provides in Sec. 50 that: (a) There shall be established in the city a prosecution service to be headed by a city prosecutor and such number of assistant prosecutors as may be necessary, who shall be organizationally part of the DOJ (b) The City Prosecutor shall handle the criminal prosecution in the MTC in the city as well as in the RTC for criminal cases originating in the territory of the city, and shall render to or for the city such services as are required by law, ordinance or regulation of the DOJ - Clearly, respondent judge had no more authority to conduct a preliminary investigation of the subject criminal complaint. The officer authorized to conduct preliminary investigations in the then newly-created City of Ligao was its City Prosecutor. At that time, the duty devolved upon OIC City Prosecutor Vasquez, despite the administrative difficulties he was encountering. 2. YES - We noticed the contradiction between her answer and her motion for reconsideration as to what she actually conducted on November 29, 2001. In her answer, she justified her authority to conduct a preliminary investigation. In her motion for reconsideration of the OCAs resolution, however, she declared that she conducted a preliminary examination to justify the issuance of a warrant of arrest -There appear just too many intriguing uncertainties surrounding the filing of the estafa case. We therefore direct our attention to respondent judges failure to erase our doubts over how she administers justice in her jurisdiction. - Respondent judge must be reminded that she should do honor to her position not only by rendering just, correct and impartial decisions but doing so in a manner free from any suspicion as to their fairness and impartiality, and as to her integrity. A spotless dispensation of justice requires not only that the decision rendered be intrinsically fair but that the

TUMANG v BAUTISTA 136 SCRA 682 ABAD-SANTOS; May 31 1985


NATURE Petition to review and annul orders of RTC Laguna FACTS - Emilio Javier filed a sakdal against Enrique Tumang and his daughter Georgia Tumang. - The sakdal was written in Tagalog and was unaccompanied by an English translation.

Criminal Procedure Rowena Daroy Morales


- The Tumangs prayed that Javier be ordered to file a copy of the complaint as translated in English, and a copy of the criminal complaint and Decision of acquittal in the unjust vexation case mentioned in Javiers complaint. - TC ruled on Oct. 21 1982 that the complaint written in Pilipino, which is an official language, is proper and is admitted. TC also ruled that it is not absolutely necessary that copies of the complaint and criminal case be attached as annexes. Javier should have, however, at least stated the docket number as well as name of the court and branch number. - The Tumangs failed to answer the sakdal and were declared in default. They sought to reconsider, not only the order of default but also the order admitting the complaint in Pilipino. - TC set aside its order of default that refused to reconsider its order of October 21, 1982. - The Tumangs filed a motion to dismiss, alleging that the complaint did not state a cause of action and that the venue was improperly laid. TC denied the motion on both grounds. ISSUE WON the sakdal should have been in English and not Tagalog HELD YES - In the ponentes lecture, Writing Decisions, he said in part: What language should the judge use? The constitution says that until otherwise provided by law, English and Pilipino shall be the official languages! (Art. XV, Sec. 3, Par 3.) If we are to be guided by this provision then either English of Pilipino can be used. But in fact English is almost exclusively used and with good reason. For Pilipino is still a gestating language. The constitution says so. It directs that the Batasang Pambansa shall take steps towards the development and formal adoption of a common national language to be known as Pilipino. - However, petitioner cannot now raise this question before the Supreme Court, As they have tacitly submitted to the TCs ruling that the sakdal did not have to be translated in English; they analyzed the sakdal in arguing that it stated no cause of action. Such analysis demonstrated that they understood its contents. DISPOSITION Denied for lack of merit.

a2010

page 3

Prof.
orders should be set aside and that the Feb. 1 Decision should be reinstated. ISSUES 1. WON the trial court, upon motion by a private complainant, can set aside a previous judgment of conviction and remand the records of a case to the Office of the Provincial Prosecutor for re-evaluation of the evidence and the filing of the corresponding charge 2. WON the manifestation by the accused that he is not appealing from the trial courts Decision render the judgment final 3. WON the trial court err in granting private complainants motion for reconsideration/retrial 4. WON the assailed orders violate petitioners constitutional right against double jeopardy HELD 1. NO Ratio Only the accused may ask for a modification or setting aside of a judgment of conviction which he must do before the said judgment becomes final or before he perfects his appeal. Reasoning - Sec. 7 Rule 120 of the Revised Rules on Criminal Procedure, as amended, provides: Sec. 7. Modification of judgment - A judgment of conviction may, upon motion of the accused, be modified or set aside before it becomes final or before appeal is perfected. Except where the death penalty is imposed, a judgment becomes final after the lapse of the period for perfecting an appeal, or when the sentence has been partially or totally satisfied or served, or when the accused has waived in writing his right to appeal, or has applied for probation. - It is thus clear that only the accused may ask for a modification or setting aside of a judgment of conviction. And this he must do before the said judgment becomes final or before he perfects his appeal. Such judgment becomes final in any of the following ways: (a) when no appeal is seasonably filed by the accused, except in case of automatic review of the decision imposing the capital penalty; (b) when he has partially or totally served his sentence; (c) when he expressly waives his right to appeal the judgment, except when the death penalty is imposed; or (d) when he applies for probation. When a judgment becomes final, the trial court which rendered the judgment of conviction loses jurisdiction to alter, modify, or revoke it. 2. YES

383 SCRA 449 SANDOVAL-GUTIERREZ; June 26, 2002


NATURE Review on certiorari on a decision of RTC of Catarman, Northern Samar FACTS - Dec.12,1999: Potot was charged with homicide before the RTC for assaulting and stabbing a certain Rodolfo Dapulag with a knife, thereby causing his death. - Feb.1, 2000: Upon arraignment, Potot pleaded guilty and invoked the mitigating circumstances of plea of guilty and voluntary surrender. He was later convicted of homicide w/ the above stated mitigating circumstances. - Feb.3, 2000: Potot filed a manifestation with motion informing the TC that he is not appealing from the Decision and praying that a commitment order be issued so he could immediately serve his sentence. - Feb.11, 2000: Private complainant Rosalie Dapulag (wife of the victim), with the conformity of the public prosecutor, filed a motion for reconsideration/retrial praying that the decision be set aside and that the case be heard again because of certain irregularities committed before and during the trial. She alleged that there were 2 other men involved in the commission of the crime and that the eyewitness deliberately withheld the information upon solicitation by a certain Mayor Dapulag and upon the eyewitnesses own belief that such inclusion would complicate the case and make it more difficult. - Petitioner opposed this motion, asserting that the decision can no longer be modified or set aside because it became final when he formally waived his right to appeal. - May 3, 2000: The trial court granted Rosalie Dapulags motion, set aside its previous Decision as well as ordered that the records of the case be remanded to the Office of the Provincial Prosecutor for re-evaluation of the evidence and filing of the corresponding charge. - Petitioner filed a MFR, contending that the trial court has no jurisdiction to issue the Feb.1 order as the Decision had become final, and that the said order would place him in double jeopardy. - May 26, 2000: The trial court denied the MFR for the reason that the State is not bound by the error or negligence of its prosecuting officers, hence, jeopardy does not attach. - Petitioner now assails the May 3rd and May 26 orders with the Sol.Gen. agreeing that the challenged

POTOT v PEOPLE

Criminal Procedure Rowena Daroy Morales


Ratio The waiver by the accused of his right to appeal from a judgment of conviction has the effect of causing the judgment to become final and unalterable. Reasoning - It is an undisputed fact that after the promulgation of the judgment of conviction, petitioner filed a manifestation expressly waiving his right to appeal therefrom. His intention not to appeal is further indicated by his prayer in the same manifestation for the immediate issuance of a commitment order so he could serve his sentence. Such waiver has the effect of causing the judgment to become final and unalterable. Thus, it was beyond the authority of the trial court to issue the order of May 3, 2000 setting aside its Feb.3, 2000 Decision which had attained finality. 3. YES Ratio When the MFR of the judgment of conviction is not initiated by the accused or at the instance of the trial court with the consent of the accused, the same should be denied outright. Reasoning - Sec. 1 Rule 121 of the same Rules provides: Sec.1. New trial or reconsideration At any time before a judgment of conviction becomes final, the court may, on motion of the accused or at its own instance but with the consent of the accused, grant a new trial or reconsideration. - Since the MFR of the judgment of conviction was not initiated by the accused or at the instance of the trial court with his consent, the same should have been denied outright as being violative of the above provision. At any rate, the records do not show any irregularity in the preliminary investigation of the case before the Provincial Prosecutors Office. 4. YES Ratio The right against double jeopardy prohibits any subsequent prosecution of any person for a crime of which he has previously been acquitted or convicted. Reasoning - To invoke the defense of double jeopardy, the following requisites must be present: (1) a valid complaint or information; (2) the court has jurisdiction to try the case; (3) the accused has pleaded to the charge; and (4) he has been convicted or acquitted, or the case against him dismissed or otherwise terminated without his express consent. - These requisites have been established. Records show that petitioner was charged with homicide under a valid information before the trial court which has jurisdiction over it. He was arraigned and

a2010

page 4

Prof.
institutes the civil action prior to the criminal action. Considering that the offended party had paid the corresponding filing fee for the estafa cases prior to the filing of the BP 22 cases with the Metropolitan Trial Court (MeTC), the RTC allowed the private prosecutor to appear and intervene in the proceedings. ISSUE WON a private prosecutor can be allowed to intervene and participate in the proceedings of the above-entitled estafa cases for the purpose of prosecuting the attached civil liability arising from the issuance of the checks involved which is also subject mater of the pending B.P. 22 cases HELD YES. Settled is the rule that the single act of issuing a bouncing check may give rise to two distinct criminal offenses: estafa and violation of Batas Pambansa Bilang 22 (BP 22). The Rules of Court allow the offended party to intervene via a private prosecutor in each of these two penal proceedings. However, the recovery of the single civil liability arising from the single act of issuing a bouncing check in either criminal case bars the recovery of the same civil liability in the other criminal action. While the law allows two simultaneous civil remedies for the offended party, it authorizes recovery in only one. In short, while two crimes arise from a single set of facts, only one civil liability attaches to it. Reasoning - Petitioner theorizes that the civil action necessarily arising from the criminal case pending before the MTC for violation of BP 22 precludes the institution of the corresponding civil action in the criminal case for estafa now pending before the RTC. She hinges her theory on the following provisions of Rules 110 and 111 of Rules of Court. - Based on the foregoing rules, an offended party may intervene in the prosecution of a crime, except in the following instances: (1) when, from the nature of the crime and the law defining and punishing it, no civil liability arises in favor of a private offended party; and (2) when, from the nature of the offense, the offended parties are entitled to civil indemnity, but (a) they waive the right to institute a civil action, (b) expressly reserve the right to do so or (c) the suit has already been instituted. In any of these instances, the private complainants interest in the case disappears and criminal prosecution becomes the sole function of the public prosecutor. None of these exceptions apply to the instant case. Hence,

pleaded guilty to the charge. On the basis of his plea, petitioner was convicted and meted the corresponding penalty. As petitioner has been placed in jeopardy for the crime of homicide, he cannot be prosecuted anew for the same offense, or any offense which necessarily includes or is necessarily included in the first offense charged. DISPOSITION The petition is granted. The assailed orders dated May 3, 2000 and May 26, 2000 issued by the trial court are set aside. Its decision dated Feb. 1, 2000 is reinstated.

RODRIGUEZ v PONFERRADA 465 SCRA 338 PANGANIBAN; July 29, 2005


NATURE Petition for Certiorari seeking to reverse the July 27, 2002 Order of the RTC of Quezon City: WHEREFORE, the appearance of a private prosecutor shall be allowed upon payment of the legal fees for these estafa cases pending before this Court. FACTS - Honorable Assistant City Prosecutor Rossana S. Morales-Montojo of Quezon City Prosecutors Office issued her Resolution: there being PROBABLE CAUSE to charge respondent for ESTAFA under Article 315 paragraph 2(d) as amended by PD 818 and for Violation of Batas Pambansa Blg. 22, it is respectfully recommended that the attached Information be approved and filed in Court. - As a consequence thereof, separate informations were separately filed against herein [p]etitioner before proper [c]ourts, for Estafa and [v]iolation of Batas Pambansa Blg. 22 - petitioner through counsel filed in open court before the [p]ublic [r]espondent an Opposition to the Formal Entry of Appearance of the Private Prosecutor - The [p]ublic [r]espondent court during the said hearing noted the Formal Entry of Appearance of Atty. Felix R. Solomon as [p]rivate [p]rosecutor as well as the Opposition filed thereto by herein [p]etitioner. - Ruling of the Trial Court Noting petitioners opposition to the private prosecutors entry of appearance, the RTC held that the civil action for the recovery of civil liability arising from the offense charged is deemed instituted, unless the offended party (1) waives the civil action, (2) reserves the right to institute it separately, or (3)

Criminal Procedure Rowena Daroy Morales


the private prosecutor cannot be barred from intervening in the estafa suit. True, each of the overt acts in these instances may give rise to two criminal liabilities -- one for estafa and another for violation of BP 22. But every such act of issuing a bouncing check involves only one civil liability for the offended party, who has sustained only a single injury. - criminal liability will give rise to civil liability only if the same felonious act or omission results in damage or injury to another and is the direct and proximate cause thereof. Damage or injury to another is evidently the foundation of the civil action. - Thus, the possible single civil liability arising from the act issuing a bouncing check can be the subject of both civil actions deemed instituted with the estafa case and the BP 22 violation prosecution. On Election of Remedies - In its broad sense, election of remedies refers to the choice by a party to an action of one of two or more coexisting remedial rights, where several such rights arise out of the same facts, but the term has been generally limited to a choice by a party between inconsistent remedial rights, the assertion of one being necessarily repugnant to, or a repudiation of, the other. In its more restricted and technical sense, the election of remedies is the adoption of one of two or more coexisting ones, with the effect of precluding a resort to the others. - no binding election occurs before a decision on the merits is had or a detriment to the other party supervenes - it was not the intent of the special rule to preclude the prosecution of the civil action that corresponds to the estafa case, should the latter also be filed. The crimes of estafa and violation of BP 22 are different and distinct from each other. There is no identity of offenses involved, for which legal jeopardy in one case may be invoked in the other. The offenses charged in the informations are perfectly distinct from each other in point of law, however nearly they may be connected in point of fact - In promulgating the Rules, this Court did not intend to leave the offended parties without any remedy to protect their interests in estafa cases. Its power to promulgate the Rules of Court is limited in the sense that rules shall not diminish, increase or modify substantive rights. Private complainants intervention in the prosecution of estafa is justified not only for the prosecution of her interests, but also for the speedy and inexpensive administration of justice as mandated by the Constitution.

a2010

page 5

Prof.
1979, a restraining order was issued by the CA against the threatened act of arraignment of the accused. However, in a decision of October 25 1979, the CA dismissed the petition and lifted the restraining order of Jan 23,1979. The motion for reconsideration of the accused was denied in a resolution. ISSUE WON the trial court acting on a motion to dismiss a criminal case filed by the Provincial Fiscal upon instructions of the Secretary of Justice to whom the case was elevated for review, may refuse to grant the motion and insist on the arraignment and trial on the merits HELD YES Ratio Once an information is filed in court, the courts prior permission must be secured if fiscal wants to reinvestigate the case. While it is true that the fiscal has the quasi judicial discretion to determine whether or not a criminal case should be filed in court or not, once the case had already been brought to Court, whatever disposition the fiscal may feel should be proper in the case thereafter should be addressed for the consideration of the Court. DISPOSITION Petition dismissed

DISPOSITION Petition is DISMISSED and the assailed order AFFIRMED

CRESPO v MOGUL 151 SCRA 462 GANCAYCO; June 30, 1987


NATURE Petition to review the decision of the Circuit Criminal Court of Lucena City (petitioner prays that respondent judge be perpetually enjoined from enforcing his threat to proceed with the arraignment and trail of petitioner, ordering respondent Judge to dismiss the said case, and declaring the obligation of petitioner as purely civil.) FACTS - Assistant Fiscal Proceso de Gala filed an information for estafa against Mario Crespo in Circuit Criminal Court of Lucena City. When the case was set for arraignment, the accused filed a motion to defer arraignment on the ground that there was a pending petition for review filed with the Secretary of Justice of the resolution of the Office of the Provincial Fiscal for the filing of the information. The presiding judge (leodegario Mogul) denied the motion through his order. - The accused filed a petition for certiorari and prohibition with prayer for a preliminary writ of injunction. In an order (Aug 17 1977), the CA restrained Judge Mogul from proceeding with the arraignment of the accused until further orders from the Court - On May 15 1978, a decision was made by the CA granting the writ and perpetually restraining the judge from enforcing his threat to compel the arraignment of the accused in the case until the Dept of Justice shall have finally resolved the petition for review. - On March 22, 1978, The Undersecretary of Justice Hon Catalino Macaraig Jr, resolving the petition for review, reversed the resolution of the Office of the Provincial Fiscal and directed the fiscal to move for immediate dismissal of the information filed against the accused. The Provincial Fiscal filed a motion to dismiss for insufficiency of evidence on April 10, 1978. On November 24 1978, The Judge denied the motion and set the arraignment - The accused filed a petition for certiorari, prohibition, and mandamus with petition for the issuance of preliminary writ of prohibition and/or temporary restraining order in the CA. On January 23

STA. ROSA MINING v ZABALA 153 SCRA 367 BIDIN; August 31, 1987
NATURE Mandamus to compel respondent Fiscal to prosecute Criminal Case No. 821 of the then Court of First Instance of Camarines Norte until the same is terminated. FACTS - On March 21, 1974, petitioner filed a complaint for attempted theft of materials (scrap iron) forming part of the installations on its mining property at Jose Panganiban, Camarines Norte against private respondents Romeo Garrido and Gil Alapan with the Office of the Provincial Fiscal of Camarines Norte, then headed by Provincial Fiscal Joaquin Ilustre. - The case was assigned to third Assistant Fiscal Esteban P. Panotes for preliminary investigation who, after conducting said investigation, issued a resolution dated August 26, 1974 recommending that an information for Attempted Theft be filed

Criminal Procedure Rowena Daroy Morales


against private respondents on a finding of prima facie case which resolution was approved by Provincial Fiscal Joaquin Ilustre. Private respondents sought reconsideration of the resolution but the same was denied by Fiscal Ilustre in a resolution dated October 14, 1974. - On October 29, 1974, Fiscal Ilustre filed with the Court of First Instance of Camarines Norte an Information dated October 17, 1987 docketed as Criminal Case No. 821, charging private respondents with the crime of Attempted Theft. - In a letter dated October 22, 1974, the private respondents requested the Secretary of Justice for a review of the Resolutions of the Office of the Provincial Fiscal dated August 26, 1974 and October 14, 1974. - On November 6, 1974, the Chief State Prosecutor ordered the Provincial Fiscal to elevate entire records PFO Case 577 against Garrido et al.The letter-request for review was opposed by petitioner in a letter to the Secretary of Justice dated November 23, 1974 alleging, among other things, that an information for Attempted Theft had already been filed against private respondents for which reason the request for review has become a moot question as the Provincial Fiscal has lost jurisdiction to dismiss the charge for attempted theft. - On March 6, 1975, the Secretary of Justice, after reviewing the records, reversed the findings of prima facie case of the Provincial Fiscal and directed said prosecuting officer to immediately move for the dismissal of the criminal case. Petitioner sought reconsideration of the directive of the Secretary of Justice but the latter denied the same in a letter dated June 11, 1975. - A motion to dismiss dated September 16, 1975 was then filed by the Provincial Fiscal but the court denied the motion on the ground that there was a prima facie evidence against private respondents and set the case for trial on February 25, 1976. - Private respondents sought reconsideration of the court's ruling but in an Order dated February 13, 1976, the motion filed for said purpose was likewise denied. Trial of the case was reset to April 23, 1976. - Thereafter, Fiscal Ilustre was appointed a judge in the Court of First Instance of Albay and respondent Fiscal Zabala became officer-in-charge of the Provincial Fiscal's Office of Camarines Norte. - On April 19, 1976, respondent Fiscal filed a Second Motion to Dismiss the case. This second motion to dismiss was denied by the trial court in an order dated April 23, 1976. Whereupon, respondent fiscal manifested that he would not prosecute the case and

a2010

page 6

Prof.
compelling as its obligation to govern at all; and whose interest, therefore, in criminal prosecution is not that it shall win a case, but that justice shall be done. As such, he is in a peculiar and very definite sense the servant of the law, the two-fold aim of which is that guilt shall not escape or innocence suffer" (Suarez vs. Platon, 69 Phil. 556). - Accordingly, if the fiscal is not at all convinced that a prima facie case exists, he simply cannot move for the dismissal of the case and, when denied, refuse to prosecute the same. He is obliged by law to proceed and prosecute the criminal action. He cannot impose his opinion on the trial court. At least what he can do is to continue appearing for the prosecution and then turn over the presentation of evidence to another fiscal or a private prosecutor subject to his direction and control (U.S. vs. Despabiladeras, 32 Phil. 442; U.S. vs. Gallegos, 37 Phil. 289). Where there is no other prosecutor available, he should proceed to discharge his duty and present the evidence to the best of his ability and let the court decide the merits of the case on the basis of the evidence adduced by both parties. - The mere fact that the Secretary of Justice had, after reviewing the records of the case, directed the prosecuting fiscal to move for the dismissal of the case and the motion to dismiss filed pursuant to said directive is denied by the trial court, is no justification for the refusal of the fiscal to prosecute the case. It is the court where the case is filed and not the fiscal that has full control of it. - In order therefore to avoid such a situation whereby the opinion of the Secretary of Justice who reviewed the action of the fiscal may be disregarded by the trial court, the Secretary of Justice should, as far as practicable, refrain from entertaining a petition for review or appeal from the action of the fiscal, when the complaint or information has already been filed in Court. The matter should be left entirely for the determination of the Court." DISPOSITION petition is hereby Granted Public respondent or any other person who may be assigned or appointed to act in his place or stead, is hereby ordered to continue prosecuting Criminal Case No. 821 until the same is terminated.

disauthorized any private prosecutor to appear therein. Hence, this petition for mandamus. - In this action, petitioner prays for the issuance of the writ of mandamus "commanding respondent fiscal or any other person who may be assigned or appointed to act in his place or stead to prosecute Criminal Case No. 821 of the Court of First Instance of Camarines Norte" There is no question that the institution of a criminal action is addressed to the sound discretion of the investigating fiscal. He may or he may not file the information according to whether the evidence is in his opinion sufficient to establish the guilt of the accused beyond reasonable doubt. (Gonzales vs. Court of First Instance, 63 Phil. 846) and when he decides not to file the information, in the exercise of his discretion, he may not be compelled to do so (People vs. Pineda, 20 SCRA 748). However, after the case had already been filed in court, "fiscals are not clothed with power, without the consent of the court, to dismiss or nolle prosequi criminal actions actually instituted and pending further proceedings. The power to dismiss criminal actions is vested solely in the court" (U.S. vs. Barredo, 32 Phil. 444, 450; Gonzales vs. Court of First Instance, supra). ISSUE WON the fiscal can be compelled to prosecute the case after his motion to dismiss has been denied HELD YES - This court is of the view that the writ prayed for should issue. Notwithstanding his personal convictions or opinions, the fiscal must proceed with his duty of presenting evidence to the court to enable the court to arrive at its own independent judgment as to the culpability of the accused. The fiscal should not shirk from his responsibility much less leave the prosecution of the case at the hands of a private prosecutor. At all times, the criminal action shall be prosecuted under his direction and control (Sec. 4, Rule 110, Rules of Court). Otherwise, the entire proceedings will be null and void (People vs. Beriales, 70 SCRA 361). - "In the trial of criminal cases, it is the duty of the public prosecutor to appear for the government since an offense is an outrage to the sovereignty of the State." (Moran, Comments on the Rules of Court, Vol. IV, 1980 Ed., p. 10). This is so because "the prosecuting officer is the representative not of an ordinary party to a controversy but of a sovereignty where obligation to govern impartially is as

PEREZ v HAGONOY 327 SCRA 588 DE LEON; March 9, 2000


NATURE: Review on Certiorari

Criminal Procedure Rowena Daroy Morales


FACTS - Private respondent Hagonoy Rural Bank, Inc. owns the Hagonoy Money Shop which employed petitioner Cristina O. Perez as Officer-In-Charge, Cashier and Teller, Alberto S. Fabian as Bookkeeper, and Cristina Medina and Milagros Martin as Solicitors/Field Managers. - For the period starting August 3, 1992 up to December 5, 1993, the Laya, Manabat, Salgado and Company, an independent management, consultancy and accounting firm, conducted an audit of the financial affairs of the Hagonoy Money Shop and found anomalies in more or less twenty-eight (28) savings accounts consisting of withdrawals which were recorded in the subsidiary ledgers of the money shop but not in the passbooks which were in the possession of the depositors. The audit also revealed that to cover-up the anomalous withdrawals, fake deposits were recorded in the money shop's subsidiary ledgers whenever the remaining balance in a particular savings account went below the amount of legitimate withdrawals made by a depositor.This prompted the private respondent to file an affidavit-complaint for estafa against the aforementioned employees of the money shop and two outsiders, Susan Jordan and Brigida Mangahas. Acting Provincial Prosecutor, Jesus Y. Manarang (hereinafter "prosecutor"), issued a resolution finding prima facie evidence that the petitioner and her coemployees had committed the crime of estafa thru falsification of commercial documents, and recommending the filing of the corresponding information against them with the Regional Trial Court (RTC) of Malolos, Bulacan. The charges against Susan Jordan and Brigida Mangahas were, however, dismissed. - Perez filed a petition for review with the Secretary of Justice praying for the dismissal of the charges against her. On the other hand, private respondent moved for a reconsideration of the portion of the same resolution dismissing the complaint against Susan Jordan. - The prosecutor granted private respondent's motion for reconsideration.8 Hence, on April 27, 1994, an information for estafa thru falsification of commercial documents was filed against herein petitioner, Alberto Fabian, Milagros Martin, Cristina Medina and Susan Jordan, - On September 23, 1994, then Secretary of Justice, Franklin M. Drilon, issued Resolution No. 696, series of 1994 ordering the prosecutor to cause the dismissal of the information against herein petitioner

a2010

page 7

Prof.
retains the right to bring a special civil action for certiorari in his own name in criminal proceedings before the courts of law. Reasoning - In the case of Dela Rosa v. Court of Appeals,we held that: "In a special civil action for certiorari filed under Section 1, Rule 65 of the Rules of Court wherein it is alleged that the trial court committed grave abuse of discretion amounting to lack of jurisdiction or on other jurisdictional grounds, the rules state that the petition may be filed by the person aggrieved. In such case, the aggrieved parties are the State and the private offended party or complainant. The complainant has an interest in the civil aspect of the case so he may file such special civil action questioning the decision or action of the respondent court on jurisdictional grounds. In so doing, the complainant should not bring the action in the name of the People of the Philippines. The action may be prosecuted in (the) name of the said complainant." - Furthermore, our ruling in the case of Dee v. Court of Appeals allowing the private offended party to file a special civil action for certiorari to assail the order of the trial judge granting the motion to dismiss upon the directive of the Secretary of Justice is apropos. It follows, therefore, that if the private respondent in this case may file a special civil action for certiorari, then with more reason does it have legal personality to move for a reconsideration of the order of the trial court dismissing the criminal charges against the petitioner. In fact, as a general rule, a special civil action will not lie unless a motion for reconsideration is first filed before the respondent tribunal, to allow it an opportunity to correct its assigned errors.

on the ground of insufficient evidence. The private respondent filed a motion for reconsideration of the order of the Secretary of Justice, which motion, however, was denied with finality by the latter.Pursuant to the said resolution, the prosecutor filed a motion in the RTC praying for the dismissal of the case against herein petitioner and the admission of an amended information excluding petitioner as one of the accused which motion was granted by the RTC. Private respondent assailed the dismissal of the case against the petitioner in a motion for reconsideration filed in the RTC which motion was denied by the RTC after finding that the private respondent, as private complainant, had no legal personality to question the dismissal of the criminal charges against the petitioner. ISSUES 1. WON Judge Masadao, presiding judge of RTC Branch 9, Malolos, Bulacan, committed grave abuse of discretion in granting the prosecutor's motion to dismiss the criminal case against petitioner without an independent assessment of the sufficiency or insufficiency of the evidence against the latter 2. WON the private respondent, as private complainant, in a criminal case has the legal personality to question the dismissal by the trial judge of the criminal charges against herein petitioner upon the motion filed by the prosecutor HELD 1. YES, Judge Masadao acted with grave abuse of discretion in granting the prosecutor's motion to dismiss the criminal charges against the petitioner on the basis solely of the recommendation of the Secretary of Justice. Reasoning - As aptly observed by the Office of the Solicitor General, in failing to make an independent finding of the merits of the case and merely anchoring the dismissal on the revised position of the prosecution, the trial judge relinquished the discretion he was duty bound to exercise. In effect, it was the prosecution, through the Department of Justice which decided what to do and not the court which was reduced to a mere rubber stamp in violation of the ruling in Crespo v. Mogul.. 2. YES Ratio While it is only the Solicitor General that may bring or defend actions on behalf of the Republic of the Philippines, or represent the People or State in criminal proceedings pending in the Supreme Court and the Court of Appeals, the private offended party

PEOPLE v BUBAN GR No. 166895 VELASCO, JR; January 24, 2007


NATURE Petition for review decision of CA FACTS - Romeo Buban is accused of raping his then 12 year old daughter 5 times, on separate occasions. The medical examination of the girl reveals that she was indeed raped. - Although the girls sworn statement mentioned five occasions of rape, the Complaint mentioned only the 5th incident. The charges of rape committed on other occasions were not supported with the required

Criminal Procedure Rowena Daroy Morales


complaints in accordance with Section 5, Rule 110 of the 1985 Rules on Criminal Procedure. - RTC found the accused guilty (sentence: death penalty plus moral damages, civil indemnity, exemplary damages) but only with regard the 5th incident only. The other 4 charges were dismissed for lack of legal basis to convict. The dispositive portion of the RTCs decision was not specific as to which charge it found the accused guilty, but the body of the decision implies the dismissal of the other 4 charges, thus it can be deduced that the conviction pertains to the 5th incident only. CA affirmed. ISSUES 1. WON CA erred in finding the accused guilty despite the alleged insufficiency of evidence 2. WON the accused may be convicted for the other counts of rape, where the complaint mentions only one instance of rape HELD 1. NO Ratio there is no error in the appreciation of evidence by the court. Reasoning - the argument of the accused that the testimony of the girl is not reliable for inconsistencies is untenable. People v. Antonio: Discrepancies and inconsistencies in the testimony of a witness referring to minor details, and not in actuality touching upon the central fact of the crime, do not impair her credibility. If at all, they serve as proof that the witness is not coached or rehearsed. 2. As can be gleamed from the case, the complaint should contain all instances of the crime charged. The other 4 counts of rape were dismissed because the complaint did not specify the same, and only mentioned the last instance of rape, despite the inclusion of the other 4 in the sworn statement of the girl. The prosecution did not question anymore the dismissal of the other 4 counts, so the court did not discuss it further. DISPOSITION judgment affirmed with modification. Sentence changed to reclusion perpatua (pursuant to RA 9346, abolishing the death penalty) and higher damages.

a2010

page 8

Prof.
2. WON respondent Judge should be disqualified from further proceeding with the criminal cases HELD 1. NO Ratio Private prosecutors cannot intervene independently of and take a position inconsistent with that of the Solicitor General. Reasoning - Participation of the private prosecution in the instant case was delimited by this Court in its Resolution of October 1, 1975, thus: "to collaborate with the Solicitor General in the preparation of the Answer and pleadings that may be required by this Court." To collaborate means to cooperate with and to assist the Solicitor General. It was never intended that the private prosecutors could adopt a stand independent of or in contravention of the position taken by the Solicitor General. - Since a criminal offense is an outrage to the sovereignty of the State, it is but natural that the representatives of the State should direct and control the prosecution. > Suarez v Platon: the prosecuting officer "'is the representative not of, an ordinary party to a controversy, but of a sovereignty whose obligation to govern impartially is as compelling as its obligation to govern at all; and whose interest, therefore, in a criminal prosecution is not that it shall win a case, but that justice shall he done. As such, he is in a peculiar and very definite sense the servant of the law, the twofold aim of which is that guilt shall not escape or innocence suffer. He may prosecute with earnestness and vigor-indeed, he should do so. But, while he may strike hard blows, he is not at liberty to strike foul ones. It is as much his duty to refrain from improper methods calculated to produce a wrongful conviction as it is to use every legitimate means to bring about a just one." > People v Esquivel: that there is an absolute necessity for prosecuting attorneys to lay "before the court the pertinent facts at their disposal with methodical and meticulous attention, clarifying contradictions and filling up gaps and loopholes in their evidence, to the end that the court's mind may not be tortured by doubts, that the innocent may not suffer and the guilty not escape unpunished. Obvious to all, this is the prosecution's prime duty to the court, to the accused, and to the state." - It is for the purpose of realizing the aforementioned objectives that the prosecution of offenses is placed

NATURE Original action for certiorari and prohibition FACTS - Solicitor General Estilito P. Mendoza, Assistant Solicitor General Alicia Simpio-Diy and Solicitor Eduardo L. Kilayko for respondents. Estanisloo A. Fernandez and Dakila F. Castro & Associate as private prosecutors. - petitioners seek the annulment of respondent Judge's Orders in the Criminal Case People of the Philippines v Jorge Tan, Jr, Cesar Tan, Teofanis Bondoc, Osmundo Tolentino, Mariano Bartido and Librado Sode for frustrated murder and Double Murder of the son and uncle of Mayor Inigo Larazzabal. - Judge Pedro Gallardo made the two life sentences to death penalty allegedly after meeting with Mayor Larazzabal and receipt of other paraphernalia such as whisky and wine according to the court stenographer. - Jan 14, 1976 - SolGen, on behalf of the People of the Philippines, submitted his Comment to the petition. They are "persuaded that there are bases for stating that the rendition of respondent Judge's decision and his resolution on the motion for new trial were not free from suspicion of bias and prejudice therefore, they interpose no objection to the remand of the aforementioned criminal cases "for the rendition of a new decision by another trial judge." - Jan 30, 1976 - private prosecutors submitted their Comment in justification of the challenged Orders of the respondent Judge and objected to the remand of this case. - Feb 12, 1976, the petitioners moved to strike out the "Motion to Admit Attacked Comment" and the "Comment" of the private prosecutor on the ground that the latter has "absolutely no standing in the instant proceedings before this Honorable Court and, hence, without any personality to have any paper of his entertained by this Tribunal - private prosecutors now contend that they are entitled to appear before this Court, to take part in the proceedings, and to adapt a position in contravention to that of the Solicitor General. ISSUES 1. WON private prosecutors have the right to intervene independently of the Solicitor General and to adopt a stand inconsistent with that of the latter

TAN, JR v GALLARDO 73 SCRA 308 ANTONIO; October 5, 1976

Criminal Procedure Rowena Daroy Morales


under the direction, control, and responsibility of the prosecuting officer. - Role of the private prosecutors is to represent the offended party with respect to the civil action for the recovery of the civil liability arising from the offense. This civil action is deemed instituted with the criminal action, unless the offended party either expressly waives the civil action or reserves to institute it separately. Thus, "an offended party may intervene in the proceedings, personally or by attorney, specially in case of offenses which can not be prosecuted except at the instance of the offended party The only exception to this is when the offended party waives his right to civil action or expressly reserves his right to institute it after the termination of the case, in which case he lost his right to intervene upon the theory that he is deemed to have lost his interest in its prosecution. in any event, whether an offended party intervenes in the prosecution of a criminal action, his intervention must always be subject to the direction and control of the prosecuting official." > Herrero v Diaz: "intervention of the offended party or his attorney is authorized by section 15 of Rule 106 of the Rules of Court, subject to the provisions of section 4 of the same Rule that all criminal actions either commenced by complaint or by information shall be prosecuted under the direction and control of the Fiscal." - the position occupied by the offended party is subordinate to that of the promotor fiscal because, as the promotor fiscal alone is authorized to represent the public prosecution, or the People of the Philippine Islands, in the prosecution of offenders, and to control the proceeding, and as it is discretionary with him to institute and prosecute a criminal proceeding, being at liberty to commence it or not or to refrain from prosecuting it or not, depending upon whether or not there is, in his opinion, sufficient evidence to establish the guilt of the accused beyond a reasonable doubt, except when the case is pending in the Court of First Instance, the continuation of the offended party's intervention depends upon the continuation of the proceeding. Consequently, if the promotor fiscal desists from pressing the charge or asks the competent Court of First Instance in which the case is pending for the dismissal thereof, and said court grants the petition, the intervention of the person injured by the commission of the offense ceases by virtue of the principle that the accessory follows the principal. Consequently, as the offended party is not entitled to represent the People of the Philippine

a2010

page 9

Prof.
It is evident, therefore, that since the Solicitor General alone is authorized to represent the State or the People of the Philippines the interest of the private prosecutors is subordinate to that of the State and they cannot be allowed to take a stand inconsistent with that of the Solicitor General, for that would be tantamount to giving the latter the direction and control of the criminal proceedings, contrary to the provisions of law and the settled rules on the matter. 2. It is already moot because the judge is no longer in the judicial service DISPOSITION SC grants the petition and hereby remands the case to the trial court in order that another Judge may hear anew petitioners' motion for new trial and to resolve the issue accordingly on the basis of the evidence

Islands in the prosecution of a public offense, or to control the proceeding once it is commenced, and as his right to intervene therein is subject to the promotor fiscal's right of control, it cannot be stated that an order of dismissal decreed upon petition of the promoter fiscal himself deprives the offended party of his right to appeal from an order overrruling a complaint or information, which right belongs exclusively to the promotor fiscal by virtue of the provisions of section 44 of General Orders, No. 58. To permit a person injured by the commission of an offense to appeal from an order dismissing a criminal case issued by a Court of First Instance upon petition of the promoter fiscal, would be tantamount to giving said offended party of the direction and control of a criminal proceeding in violation of the provisions of the above-cited section 107 of General Orders, No. 58. - from the nature of the offense, or where the law defining and punishing the offense charged does not provide for an indemnity, the offended party may not intervene in the prosecution of the offense. - Solicitor General represents the People of the Philippines or the State in criminal proceedings pending either in the Court of Appeals or in this Court. Section 1 of Presidential Decree No. 478, "Defining the Powers and Functions of the Office of the Solicitor General", provides: SECTION 1. Function and Organization, (1) The Office of the Solicitor General shall represent the Government of the Philippines, its agencies and instrumentalities and its officials and agents in any litigation, proceeding, investigation or matter requiring the services of a lawyer. * * * The office of the Solicitor General shall constitute the law office of the Government, and as such, shall discharge duties requiring the services of a lawyer. It shall have the following specific powers and functions: (a) Represent the Government in the Supreme Court and the Court of Appeals in all criminal proceedings; represent the Government and its officers in the Supreme Court, the Court of Appeals, and all other courts or tribunals in all civil actions and special proceedings in which the Government or any officer thereof in his official capacity is the party. (k) Act and represent the Republic and/or the people before any court, tribunal, body or commission in any matter, action or proceeding which, in his opinion, affects the welfare of the people as the ends of justice may require.

PEOPLE v DELA CERNA 390 SCRA 538 CORONA ; October 9, 2002


NATURE Automatic review of decision of Cebu City RTC FACTS - Ernesto dela Cuesta was charged on May 16, 1997 with raping his minor daughter, Irene, seven times over a period of eight years beginning 1989 when the victim was seven years old. - The victim testified in open court about the incidents of rape. However, prior to the rendering of judgment, the victim, on July 3, 1998, filed an affidavit of desistance stating among others that she was no longer interested in pursuing the case and that she had already forgiven her father. - The SC noted that the rape incidents in this case occurred prior to the effectivity of RA 8353, The Anti-Rape Law of 1997, which took effect on October 22, 1997. Under this statute, the crime of rape was classified as a crime against person. It should be further noted that the law at the time the crimes were committed treated rape as a private crime covered by Article 344 of the RPC. As provided for in the said article, offenses of seduction, abduction, rape, or acts of lasciviousness shall not be prosecuted except upon a complaint filed by the offended party or her parents, grandparents, or guardian, nor in any case, the offender has been expressly pardoned by the above named persons. - The trial court found the defendant guilty and sentenced him to the supreme penalty of death.

Criminal Procedure Rowena Daroy Morales


- In his appeal, the offender claimed that he should no have been found guilty considering that the affidavit of desistance created a reasonable doubt as to his guilt. ISSUE WON the trial court erred in convicting the defendant HELD NO - The affidavit did not in fact contain any retraction on the claim of rape. Hence the guilty verdict was proper considering that the testimony of the victim was considered by the trial court as credible and believable. There was as such no reasonable doubt to speak of. - Even using the old statute which considered rape as a private crime and the forgiveness of the victim or the parents, grandparents, or guardian as extinguishing the crime, the Supreme court held that the pardon or forgiveness must be prior to the institution of the criminal action. After the case has been filed the control of the prosecution is removed from the offended partys hand and any change of heart by the victim will not affect the states right to vindicate the atrocities committed against itself. - The Court also ruled that the death penalty is not applicable in this case as the prosecution was not able to establish beyond reasonable doubt the alleged minority of the victim. It cited its previous rulings to this effect.

a2010

page 10

Prof.
parents, grandparents, or guardian, nor, in any case, if the offender has been expressly pardoned by the above-named persons, as the case may be. - Section 5 of Rule 110 of the 1985 Rules of Criminal Procedure states: The offenses of seduction, abduction, rape or acts of lasciviousness shall not be prosecuted except upon a complaint filed by the offended party or her parents, grandparents, or guardian, nor, in any case, if the offender has been expressly pardoned by the above-named persons, as the case may be. In case the offended party dies or becomes incapacitated before she could file the complaint and has no known parents, grandparents, or guardian, the State shall initiate the criminal action in her behalf. The offended party, even if she were a minor, has the right to initiate the prosecution for the above offenses, independently of her parents, grandparents or guardian, unless she is incompetent or incapable of doing so upon grounds other than her minority. Where the offended party who is a minor fails to file the complaint, her parents, grandparents or guardian may file the same. - A complaint of the offended party or her relatives is required in crimes against chastity out of consideration for the offended woman and her family, who might prefer to suffer the outrage in silence rather than go through with the scandal of a public trial. The law deems it the wiser policy to let the aggrieved woman and her family decide whether to expose to public view or to heated controversies in court the vices, fault, and disgraceful acts occurring in the family. - The complaint in the instant case has complied with the requirement under the Revised Penal Code and the Rules of Criminal Procedure, which vest upon JONALYN, as the offended party, the right to institute the criminal action. As signed by JONALYN, the complaint started the prosecutory proceeding. The assistance of JONALYNs aunt, or even of her mother, was a superfluity. JONALYNs signature alone suffices to validate the complaint. - If a minor under the Rules of Court can file a complaint for rape independently of her parents, JONALYN, then 20 years of age who was found to have the mentality of an 8-year-old girl, could likewise file the complaint independently of her relatives. Her complaint can be rightfully considered filed by a minor. 2. YES - The determination of the competence of witnesses to testify rests primarily with the trial judge who sees them in the witness stand and observes their behavior or their possession or lack of intelligence,

PEOPLE v DELA CRUZ 384 SCRA 375 DAVIDE; July 11, 2002
FACTS - Upon a complaint signed by JONALYN with the assistance of her aunt Carmelita Borja, two informations were filed by the Office of the Provincial Prosecutor before the RTC of Malolos charging Bienvenido Dela Cruz with rape. BIENVENIDO entered a plea of not guilty. - When JONALYN was presented as its first witness, the prosecution sought to obtain from the trial court an order for the conduct of a psychiatric examination to determine her mental and psychological capability to testify in court. Trial court allowed the prosecutor to conduct direct examination on JONALYN so that if in its perception she would appear to be suffering from mental deficiency, the prosecutor could be permitted to ask leading questions. Noticing that

JONALYN had difficulty in expressing herself, the trial court decided to suspend the proceedings to give the prosecution sufficient time to confer with her. - Trial court allowed the prosecution to put on the witness stand a Medical Officer of the National Center for Mental Health. Dr. Tuazon testified that she found that JONALYN was suffering from a moderate level of mental retardation and that although chronologically the latter was already 20 years of age, she had the mental age of an 8-yearold child under the Wechsler Adult Intelligence Scale. - The trial court issued an order allowing leading questions to be propounded to JONALYN. Thus, JONALYN took the witness stand. She declared in open court that BIENVENIDO raped her twice. She stated that BIENVENIDO placed himself on top of her and inserted his private part into her womanhood. - The defense filed a demurrer to evidence, which was granted. It admitted that it could have moved to quash the information but it did not because the complaint on which the information was based was on its face valid, it having been signed by JONALYN as the offended party. However, the undeniable truth is that JONALYN had no capacity to sign the same considering her mental deficiency or abnormality. The defense also insisted on assailing the competency of JONALYN as a witness. It claimed that JONALYNs testimony, considering her mental state, was coached and rehearsed. - The trial court denied the Demurrer to Evidence and set the dates for the presentation of the evidence for the defense. Trial court convicted BIENVENIDO of the crime of rape in Criminal Case No. 1275-M-96, but acquitted him in Criminal Case No. 1274-M-96 for insufficiency of evidence. ISSUES 1. WON the complaint for rape filed was valid 2. WON Jonalyn was competent to testify 3. WON Jonalyn was credible as a witness 4. WON leading questions should have been allowed to be asked to Jonalyn HELD 1. YES - The pertinent laws existing at the time the crimes were committed were Article 344 of the Revised Penal Code (prior to its amendment by R.A. No. 8353 which took effect on 22 October 1997) and Section 5 of Rule 110 of the 1985 Rules of Criminal Procedure. - The offenses of seduction, abduction, rape or acts of lasciviousness, shall not be prosecuted except upon a complaint filed by the offended party or her

Criminal Procedure Rowena Daroy Morales


as well as their understanding of the obligation of an oath. - The prosecution has proved JONALYNs competency by the testimony of Dr. Tuazon. The finding of the trial court, as supported by the testimony of Dr. Tuazon that JONALYN had the understanding of an 8year-old child, does not obviate the fact of her competency. Its only effect was to consider her testimony from the point of view of an 8-year-old minor. 3. YES - The foregoing narrative has established not only JONALYNs competency but also her credibility. Considering her feeble mind, she could not have fabricated or concocted her charge against BIENVENIDO. Also, no improper motive was shown by the defense as to why JONALYN would file a case or falsely testify against BIENVENIDO. - Complainant has made herself clear about the sexual molestation she suffered in the hands of the accused. Plain and simple her testimony may have been, unembellished, as it is, with details, yet, it is in its simplicity that its credence is enhanced. 4. YES - It is usual and proper for the court to permit leading questions in conducting the examination of a witness who is immature; aged and infirm; in bad physical condition; uneducated; ignorant of, or unaccustomed to, court proceedings; inexperienced; feeble-minded; confused and agitated; terrified; timid or embarrassed while on the stand; lacking in comprehension of questions asked; deaf and dumb; or unable to speak or understand the English or imperfectly familiar therewith. - The leading questions were neither conclusions of facts merely put into the mouth of JONALYN nor prepared statements which she merely confirmed as true. DISPOSITION RTC decision finding accusedappellant BIENVENIDO DELA CRUZ guilty of the crime of rape and sentencing him to suffer the penalty of reclusion perpetua is AFFIRMED, with the modification that accused-appellant is ordered to pay the victim JONALYN YUMANG civil indemnity in the reduced amount of P50,000 and moral damages in the amount of P50,000.

a2010
NATURE Appeal from the decision of the RTC

page 11

Prof.
superiority in strength, enough proof was adduced, however, to show that the attackers had cooperated in such a way as to secure advantage of their superiority in strength certainly out of proportion to the means of defense available to the person attacked. - Article III, Section 14, of the 1987 Constitution, in particular, mandates that no person shall be held answerable for a criminal offense without due process of law and that in all criminal prosecutions the accused shall first be informed of the nature and cause of the accusation against him. The right to be informed of any such indictment is likewise explicit in procedural rules. - object of informing an accused in writing of the charges against him: First. To furnish the accused with such a description of the charge against him as will enable him to make his defense; and second, to avail himself of his conviction or acquittal for protection against a further prosecution for the same cause; and third, to inform the court of the facts alleged, so that it may decide whether they are sufficient in law to support a conviction, if one should be had. (United States vs. Cruikshank, 92 U.S., 542). In order that this requirement may be satisfied, facts must be stated, not conclusions of law. Every crime is made up of certain acts and intent; these must be set forth in the complaint with reasonable particularity of time, place, names (plaintiff and defendant), and circumstances. In short, the complaint must contain a specific allegation of every fact and circumstance necessary to constitute the crime charged DISPOSITION appellant Ronnie Quitlong is found guilty of the crime of murder for the killing of Jonathan Calpito. Appellants Salvador Quitlong and Emilio Senoto, Jr., are found guilty as accomplices in the commission of the crime.

FACTS - Calpito was a student from Baguio city. One time, he wanted some fishballs so he and Gosil bought some fishballs worth P15. When Calpito counted his change, he found out that he only received P35 for his P100. Confronted by Calpito and Gosil, the fishball vendor would not admit that he had shortchanged Calpito. The 3 men kept arguing. Moments later, Soriano saw eight men rushing towards Gosil and Calpito. Calpito got stabbed and fell to the ground. - The RTC found Ronnie Quitlong, Salvador Quitlong and Emilio Senoto guilty of murder for the killing of Jonathan Calpito. Accused-appellants, shortly after the filing of the information, submitted a motion for reinvestigation alleging that it was a certain Jesus Mendoza who stabbed the victim. The trial court acted favorably on the motion. The City Prosecutor filed a motion to admit an amended information on the basis of affidavits. The information, as amended, included Jesus Mendoza among the named accused. But unlike accused-appellants who were immediately arrested after the commission of the crime, Jesus Mendoza remained at large. At their arraignment, the detained accused pleaded not guilty to the crime charged. - On 21 April 1995, the trial court, following his evaluation of the respective submissions of the prosecution and the defense, including their rebuttal and sur-rebuttal evidence, rendered its now assailed decision. ISSUES

1. 2.

WON the RTC abused its discretion and/or acted in excess of or without jurisdiction in finding that there was conspiracy between and among the accused-appellants WON the RTC gravely abused its discretion and/or acted in excess of or without jurisdiction in finding the accused-appellants guilty of the crime of Murder instead of Homicide

ROCO v CONTRERAS 461 SCRA 505 GARCIA; June 28, 2005


NATURE Petition for review on certiorari under Rule 45 of the Rules of Court the decision dismissing appeal and resolution denying motion for reconsideration of the Court of Appeals FACTS

PEOPLE v QUITLONG 292 SCRA 360 VITUG ; July 10, 1998

HELD 1. YES, Quitlong is guilty of murder while the other 2 are only accomplices. 2. NO, the crime was qualified The crime committed was qualified by abuse of superiority. While superiority in number would not per se mean

Criminal Procedure Rowena Daroy Morales


- Domingo Roco, engaged in buying and selling of dressed chicken, purchased his supply from private respondent Cals Poultry Supply Corporation (Cals) - As payment for his purchase, petitioner drew 5 checks payable to Cals against his account with PCIB. PCIB dishonored the checks for having been drawn from a closed account. Cals then filed a criminal complaint for violation of BP22 - Before trial could commence, Roco filed with the BIR a denunciation letter against Cals in that it failed to issue commercial invoices. BIR found no prima facie evidence of tax evasion. - Trial for Rocos violation of BP 22 commenced. After the prosecution rested, the MTCC declared the cases submitted for decision on account of petitioners failure to adduce evidence in his behalf. Later, MTCC rendered a judgment of conviction against petitioner. - Petitioner went to appeal to the RTC contending that he was deprived of due process. RTC agreed and vacated the MTCC decision. - Pending the remanded cases, petitioner filed with the MTCC a Request for Issuance of Subpoena Ad Testificandum and Subpoena Duces Tecum, requiring Vivian Deocampo or Danilo Yap, both of Cals Corporation or their duly authorized representatives, to appear and testify in court and to bring with them certain documents, records and books of accounts for the years 1993-19991. Prosecution did not object. - Acting Judge Geomer C. Delfin, issued an order granting petitioners request and accordingly directed the issuance of the desired subpoenas. -Cals counsel manifested that it was improper for the trial court to have directed the issuance of the requested subpoenas, to which the Roco countered by saying that Judge Delfins had become final and hence, immutable. Nonetheless, the trial court issued an order allowing the prosecution to file its comment or opposition to petitioners request for the issuance of subpoenas. They argued that Deocampo had earlier attested that the documents, records and
1
Sales Journal for the year 1993; Accounts Receivable Journal for the year 1993; Sales Ledger for the year 1993; Accounts Receivable Ledger for the year 1993 (in its absence, Accounts Receivable Ledger for the years 1994, 1995, 1996, 1997, 1998 or 1999); Audited Income Statement for the years 1993, 1994, 1995, 1996, 1997, 1998 and Income Statements as of February 1999; Audited Balance Sheet for the years 1993, 1994, 1995, 1996, 1997, 1998 and pBalance Sheet as of February 1999; and Income Tax Returns for the years 1993, 1994, 1995, 1996 and 1997.

a2010

page 12

Prof.
case. However, in the matter of relevancy of those books and documents to the pending criminal cases that petitioner miserably failed to discharge his burden. - Based on the records below and as correctly pointed out by the CA, petitioner had been issued by Cals with temporary receipts in the form of yellow pad slips of paper evidencing his payments, which pad slips had been validated by the corporation itself. It is clear that the production of the books and documents requested by petitioner are not indispensable to prove his defense of payment. DISPOSITION the instant petition is DENIED and the challenged decision and resolution of the Court of Appeals AFFIRMED.

books of accounts were already burned, they did not maintain the requested sales ledger and that other documents could not be produced because of the recent computerization of records was still in the process of completion. They also maintained that the documents requested are immaterial and irrelevant to the crimes for which the petitioner was being prosecuted. - In a resolution, the MTCC, thru its Judge Edward B. Contreras, denied petitioners request on the following grounds: (a) the requested documents, book ledgers and other records were immaterial in resolving the issues posed before the court; and (b) the issuance of the subpoenas will only unduly delay the hearing of the criminal cases. - Judge Contreras similarly denied the MFR. RTC denied due course to petition for failure to prove grave abuse of discretion. Similarly, it denied MFR. Petitioner went to CA via certiorari. The petition was still dismissed. MFR was still dismissed. Petitioners claim The denial of the request for the issuance of subpoena ad testificandum and subpoena duces tecum is violative of his constitutional rights ISSUE WON the lower courts erred in denying the subpoena requested by Roco HELD Ratio NO. Before a subpoena duces tecum may issue, the court must first be satisfied that the following requisites are present: (1) the books, documents or other things requested must appear prima facie relevant to the issue subject of the controversy (test of relevancy); and (2) such books must be reasonably described by the parties to be readily identified (test of definiteness). Reasoning - A subpoena is a process directed to a person requiring him to attend and to testify at the hearing or trial of an action or at any investigation conducted under the laws of the Philippines, or for the taking of his deposition. The first, subpoena ad testificandum, is used to compel a person to testify, while the second, subpoena duces tecum, is used to compel the production of books, records, things or documents therein specified. - The books and documents that petitioner requested to be subpoenaed are designated and described in his request with definiteness and readily identifiable. The test of definiteness, therefore, is satisfied in this

ASTORGA v PEOPLE 437 SCRA 152 YNARES-SANTIAGO.; Aug 20, 2004


FACTS - Three (3) private offended parties who are members of the Regional Special Operations Group (RSOG) of the DENR Tacloban City, together with two (2) members of Philippine National Police Regional Intelligence Group, were sent to the Island of Daram, Western Samar to conduct intelligence operations on possible illegal logging activities. At around 4:305:00 p.m., the team found two boats measuring 18 meters in length and 5 meters in breadth being constructed at Barangay Locob-Locob. There they met petitioner Benito Astorga, the Mayor of Daram, who turned out to be the owner of the boats. A heated altercation ensued between petitioner and the DENR team. Petitioner called for reinforcements and, moments later, a boat bearing ten armed men, some wearing fatigues, arrived at the scene. The DENR team was then brought to petitioners house in Daram, where they had dinner and drinks. The team left at 2:00 a.m. - On the basis of the foregoing facts, petitioner was charged with and convicted of Arbitrary Detention by the Sandiganbayan. - SC affirmed the conconviction of Daram. Defendant filed MFR denied with finality Filed an Urgent Motion for Leave to File 2nd MFR granted ISSUES Procedural WON filing of 2nd MFR is proper Substantive

Criminal Procedure Rowena Daroy Morales


WON the guilt of the accused was proven beyond reasonable doubt HELD Procedural YES Ratio While a second motion for reconsideration is, as a general rule, a prohibited pleading, it is within the sound discretion of the Court to admit the same, provided it is filed with prior leave whenever substantive justice may be better served thereby. Reasoning - The rules of procedure are merely tools designed to facilitate the attainment of justice. They were conceived and promulgated to effectively aid the court in the dispensation of justice. Courts are not slaves to or robots of technical rules, shorn of judicial discretion. In rendering justice, courts have always been, as they ought to be, conscientiously guided by the norm that on the balance, technicalities take a backseat against substantive rights, and not the other way around. Thus, if the application of the Rules would tend to frustrate rather than promote justice, it is always within our power to suspend the rules, or except a particular case from its operation. Substantive NO Ratio When the guilt of the accused has not been proven with moral certainty, the presumption of innocence of the accused must be sustained and his exoneration be granted as a matter of right. For the prosecutions evidence must stand or fall on its own merit and cannot be allowed to draw strength from the weakness of the evidence for the defense. Furthermore, where the evidence for the prosecution is concededly weak, even if the evidence for defense is also weak, the accused must be duly accorded the benefit of the doubt in view of the constitutional presumption of innocence that an accused enjoys. When the circumstances are capable of two or more inferences, as in this case, one of which is consistent with the presumption of innocence while the other is compatible with guilt, the presumption of innocence must prevail and the court must acquit. It is better to acquit a guilty man than to convict an innocent man. Reasoning - No sufficient evidence to show that petitioner instilled fear in the minds of the private offended parties. It appears that Darma merely extended his hospitality and entertained the DENR team in his house.

a2010

page 13

Prof.
- Petitioner was arraigned on August 18, 1998, and pleaded not guilty to both charges. Pre-trial ensued and the cases were jointly tried. The prosecution presented its witnesses, after which the Siguion Reyna, Montecillio and Ongsiako Law Offices (SRMO) as private prosecutor filed a Formal Offer of Evidence.7 Petitioner opposed the pleading, contending that the private complainant was represented by the ACCRA Law Offices and the Balgos and Perez Law Office during trial, and it was only after the prosecution had rested its case that SRMO entered its appearance as private prosecutor representing the PCIB. Since the ACCRA and Balgos and Perez Law Offices had not withdrawn their appearance, SRMO had no personality to appear as private prosecutor. Under the Informations, the private complainant is Caltex and not PCIB; hence, the Formal Offer of Evidence filed by SRMO should be stricken from the records. - Petitioner further averred that unless the Informations were amended to change the private complainant to PCIB, his right as accused would be prejudiced. He pointed out, however, that the Informations can no longer be amended because he had already been arraigned under the original Informations.8 He insisted that the amendments of the Informations to substitute PCIB as the offended party for Caltex would place him in double jeopardy. - PCIB, through SRMO, opposed the motion. It contended that the PCIB had re-credited the amount to Caltex to the extent of the indemnity; hence, the PCIB had been subrogated to the rights and interests of Caltex as private complainant. Consequently, the PCIB is entitled to receive any civil indemnity which the trial court would adjudge against the accused. Moreover, the re-credited amount was brought out on cross-examination by Ramon Romano who testified for the Prosecution. PCIB pointed out that petitioner had marked in evidence the letter of the ACCRA Law Office to PCIBank dated October 10, 1997 and the credit memo sent by PCIB to Caltex - On July 18, 2001, the RTC issued an Order granting the motion of the private prosecutor for the substitution of PCIB as private complainant for Caltex. It however denied petitioners motion to have the formal offer of evidence of SRMO expunged from the record. Petitioner filed a motion for reconsideration which the RTC denied on November 14, 2001. - Petitioner filed a Petition for Certiorari under Rule 65 of the Rules of Court with Urgent Application for Temporary Restraining Order with the Court of

DISPOSITION REVERSED. Petitioner Benito Astorga is ACQUITTED of the crime of Arbitrary Detention on the ground of reasonable doubt.

PEOPLE v TULIN

RICARZE v CA (PEOPLE, CALTEX) G.R. No. 160451 CALLEJO, SR; February 9, 2007
NATURE Petition for review on certiorari of the Decision of the Court of Appeals FACTS - Petitioner Eduardo G. Ricarze was employed as a collector-messenger by City Service Corporation, a domestic corporation engaged in messengerial services. He was assigned to the main office of Caltex Philippines, Inc. (Caltex) in Makati City. His primary task was to collect checks payable to Caltex and deliver them to the cashier. He also delivered invoices to Caltexs customers. - On November 6, 1997, Caltex filed a criminal complaint against petitioner before the Office of the City Prosecutor of Makati City for estafa through falsification of commercial documents. Romano alleged that, on October 16, 1997, while his department was conducting a daily electronic report from Philippine Commercial & Industrial Bank (PCIB) Dela Rosa, Makati Branch, one of its depositary banks, it was discovered that unknown to the department, a company check, Check No. 74001 dated October 13, 1997 in the amount of P5,790,570.25 payable to Dante R. Gutierrez, had been cleared through PCIB on October 15, 1997, notwithstanding two missing checks and two other check forgeries, one of which amounted to P1,790,757.25. All of these were never issued by Caltex. - Further investigation revealed that said savings account had actually been opened by petitioner; the forged checks were deposited and endorsed by him under Gutierrezs name. - In the meantime, the PCIB credited the amount of P581,229.00 to Caltex on March 29, 1998. However, the City Prosecutor of Makati City was not informed of this development. After the requisite preliminary investigation, the City Prosecutor filed two (2) Informations for estafa through falsification of commercial documents on June 29, 1998 against petitioner before the Regional Trial Court (RTC) of Makati City, Branch 63.

Criminal Procedure Rowena Daroy Morales


Appeals (CA,) praying for the annulment of the RTCs Orders of July 18, 2001 and November 14, 2001. - According to petitioner, damage or injury to the offended party is an essential element of estafa. The amendment of the Informations substituting the PCIBank for Caltex as the offended party would prejudice his rights since he is deprived of a defense available before the amendment, and which would be unavailable if the Informations are amended. Petitioner further insisted that the ruling in the Sayson case did not apply to this case. - The appellate court declared that when PCIB restored the amount of the checks to Caltex, it was subrogated to the latters right against petitioner. It further declared that in offenses against property, the designation of the name of the offended party is not absolutely indispensable for as long as the criminal act charged in the complaint or information can be properly identified. The appellate court cited the rulings of this Court in People v. Ho and People v. Reyes. ISSUE 1. 2. 3. WON petitioners rights are prejudiced with the substitution of the complainant WON there was a valid subrogation of rights by Caltex to PCIB WON charges against him should be dismissed because the allegations in both Informations failed to name PCIB as true offended party

a2010

page 14

Prof.
designation by which it is known or by which it may be identified, without need of averring that it is a juridical person or that it is organized in accordance with law. Dispositive WHEREFORE, the petition is DENIED. The assailed decision and resolution of the Court of Appeals are AFFIRMED. This case is REMANDED to the Regional Trial Court of Makati City, Branch 63, for further proceedings.

HELD 1. NO - The test as to whether a defendant is prejudiced by the amendment is whether a defense under the information as it originally stood would be available after the amendment is made, and whether any evidence defendant might have would be equally applicable to the information in the one form as in the other. An amendment to an information which does not change the nature of the crime alleged therein does not affect the essence of the offense or cause surprise or deprive the accused of an opportunity to meet the new averment had each been held to be one of form and not of substance. - In the case at bar, the substitution of Caltex by PCIB as private complaint is not a substantial amendment. The substitution did not alter the basis of the charge in both Informations, nor did it result in any prejudice to petitioner. The documentary evidence in the form of the forged checks remained the same, and all such evidence was available to petitioner well before

the trial. Thus, he cannot claim any surprise by virtue of the substitution. 2. YES - The Court agrees with respondent PCIBs comment that petitioner failed to make a distinction between legal and conventional subrogation. Subrogation is the transfer of all the rights of the creditor to a third person, who substitutes him in all his rights. It may either be legal or conventional. Legal subrogation is that which takes place without agreement but by operation of law because of certain acts. Instances of legal subrogation are those provided in Article 1302of the Civil Code. Conventional subrogation, on the other hand, is that which takes place by agreement of the parties. Thus, petitioners acquiescence is not necessary for subrogation to take place because the instant case is one of legal subrogation that occurs by operation of law, and without need of the debtors knowledge. 3. NO - The rules on criminal procedure require the complaint or information to state the name and surname of the person against whom or against whose property the offense was committed or any appellation or nickname by which such person has been or is known and if there is no better way of Identifying him, he must be described under a fictitious name (Rule 110, Section 11, Revised Rules of Court; now Rule 110, Section 12 of the 1985 Rules on Criminal Procedure.] In case of offenses against property, the designation of the name of the offended party is not absolutely indispensable for as long as the criminal act charged in the complaint or information can be properly identified. - Legal Basis: Section. 12. Name of the offended party. The complaint or information must state the name and surname of the person against whom or against whose property the offense was committed, or any appellation or nickname by which such person has been or is known. If there is no better way of identifying him, he must be described under a fictitious name. (a) In offenses against property, if the name of the offended party is unknown, the property must be described with such particularity as to properly identify the offense charged. (b) If the true name of the person against whom or against whose property the offense was committed is thereafter disclosed or ascertained, the court must cause such true name to be inserted in the complaint or information and the record (c) If the offended party is a juridical person, it is sufficient to state its name, or any name or

PEOPLE v GUEVARRA 179 SCRA 740 PADILLA: December 4, 1989


NATURE Automatic Review FACTS -On or about April 8, 1980, in Gapan, Nueva Ecija, several armed men namely Jaime Guevarra y Arcega, Poncing Abergas, Dan Tolentino, Baldo de Jesus, Roming Longhair, Boy Tae, Boy Pogi, Chotse Doe alias Bernabe Sulaybar y Hernandez, and Vergel Bustamante alias "Dan Saksak", entered the house of the sps Cruz and robbed them of P3000 and jewelry. Thereafter, Luisito Cruz was threatened by the men and forced to give the keys to his car by Vergel Bustamante. The members of the household were then made to enter a room and were tied. After the robbery, Priscilla Cruz was forcibly boarded in her own car by 5 of her kidnappers where she was held at knife and gunpoint. She was then told she was being held for ransom of P50k but they had to stop in San Rafael Bulucan to hire a truck because the car broke down. However, she was left at Valenzuela Bulacan as the men said the kidnapping did not materialize. The five men then boarded a taxi and the truck driver later took her home. On the same night, Luisito reported the incident which led to the detention of Vergel Bustamante who was positively identified by Priscilla. -Bustamante denied the allegations and interposed the defense of alibi, claiming to be in Caloocan at the time of the crime. His defense was rejected considering the proximity of Gapan and Caloocan and since witnesses had positively identified him. -After a separate trial for Poncing Abergas and Vergel Bustamante alias "Dan Saksak," inasmuch as Dan Tolentino, who had previously entered of plea of "not guilty" could not be served with subpoenas, and the other accused were reported to have died, judgment was rendered finding the accused Vergel Bustamante

Criminal Procedure Rowena Daroy Morales


alias "Dan Saksak" guilty of the crime of Kidnapping and Serious Illegal Detention and sentenced to suffer the death penalty, and to indemnify the offended party, Mrs. Priscilla Cruz, in the amount of P5,000.00. The accused Poncing Abergas, upon the other hand, was acquitted of the charge. Hence, this appeal. ISSUES 1. WON TC erred in ordering the amendment of the information to include Vergel Bustamante alias Dan Saksak despite lack of proof that the 2 are 1 and the same person. 2. WON there was no reinvestigation conducted to justify the filing of the amended information 3. WON the TC erred in convicting Bustamante upon the prosecution witnesses contradictory and improbable testimonies and the appellants extrajudicial confession 4. WON the accused can be convicted of kidnapping for ransom HELD 1. NO. - The ff circumstances led the RTC judge of Nueva Ecija to believe that Vergel Bustamante and Dan Saksak are one and the same person as the accused is mentioned in each as Vergel Bustamante alias Dan Saksak: A subpoena issued by the MTC of Gapan; a Return of Service of one subpoena; an order issued by the Municipal Court of Gapan finding a prima facie case against the accused; and the letter of transmittal of the records of the cases to the RTC of Nueva Ecija stating Bustamante aka Dan Saksak was detained in the Manila City Jail. -In, any case, the issue cannot be raised for the first time on appeal as it is one affecting jurisdiction over the person and should have been raised before the trial court in a motion to quash the information. As the accused failed to do so, he is deemed to have waived his objection to the information and is assumed to be satisfied with its legality. 2. NO - The reinvestigation is evidenced by the certification of the Fiscal stating that there was reasonable ground to believe a crime had been committed and that the accused were informed of the complaint and given an opportunity to submit controverting evidence. 3. NO - The said discrepancies in the testimonies were minor details which could not destroy the substance of said testimonies. As the highest degree of respect is accorded to the factual findings of the TC, the

a2010

page 15

Prof.
- Subsection 5 of section 6 of General Orders No. 58 declares that a complaint or information shall show, among others things, the names of the persons against whom, or against whose property, the offense was committed, if known. The complaint in this case therefore properly contained an averment as to the ownership of the property; and upon principle, in charging the crime of robbery committed upon the person, the allegation of the owner's name is essential. But of course if his name cannot be ascertained, it may be alleged that it is unknown. - From the fact that the name of the injured person may, in case of necessity, be alleged as unknown it should NOT be inferred that the naming of such person, when known, is of no importance. Where the name of the injured party is necessary as matter of essential description of the crime charged, the complaint must invest such person with individuality by either naming him or alleging that his name is unknown. It is elementary that in crimes against property, ownership must be alleged as matter essential to the proper description of the offense. To constitute robbery, the property obtained must be that of another, and indictments for such offenses must name the owner; and a variance in this respect between the indictment and the proof will be fatal. It is also necessary in order to identify the offense. A complaint charging the commission of the complex offense of robbery with homicide must necessarily charge each of the component offenses with the same precision that would be necessary if they were made the subject of separate complaints. It is well recognized in this jurisdiction that where a complex crime is charged and the evidence fails to support the charge as to one of the component offenses the defendant can be convicted of the other. The mere circumstance that the two crimes are so related as to constitute one transaction in no way affects the principles of pleading involved in the case. To permit a defendant to be convicted upon a charge of robbing one person when the proof shows that he robbed an entirely different person, when the first was not present, is violative of the rudimentary principles of pleading; and in addition, is subject to the criticism that the defendant is thereby placed in a position where he could not be protected from a future prosecution by a plea of former conviction or acquittal. If we should convict or acquit these defendants today of the robbery which is alleged to have been committed upon the property of Roman Estriba, it is perfectly clear that they could be prosecuted tomorrow for robbery committed upon

issue of the credibility of the witnesses cannot be raised. Also, the evidence presented by the prosecution was sufficient to support a finding of guilt even without the said extra-judicial confession. 4. NO -No element of ransom exists as no ransom note was presented in court. Neither was there a demand for money in exchange for Priscillas safe return. Besides, the Amended Information failed to allege that the kidnapping was for the purpose of extorting a ransom. The rule is that an accused cannot be convicted of a higher offense than that charged in the complaint or information. -Hence, Bustamante can only be convicted of kidnapping of a female under Article 267 with the aggravating circumstances of (a) the use of a motor vehicle and (b) the aid of armed men bringing the penalty up to the maximum. However, due to Article 3 Sec. 19 of the Constitution, the death penalty is reduced to reclusion perpetua. Dispositive WHEREFORE, the judgment appealed from is hereby AFFIRMED

US v LAHOYLAHOY and MADANLOG 38 Phil. 330 STREET; July 15, 1918


NATURE Review of a decision of the CFI of Province of Iloilo, sentencing the defendants Pedro Lahoylahoy and Marcos Madanlog to death upon a complaint charging the crime of robbery with multiple homicide. FACTS - The information in a prosecution for robbery with quadruple homicide charged that the accused criminally and by force appropriated certain articles of value, the property of one Roman Estriba, and on occasion thereof killed the said Roman Estriba and three others. However, the proof showed that the money which was the subject of the robbery was taken from one Juana Seran who was robbed and killed separately from the other three victims. ISSUE WON the conviction for robbery with quadruple homicide can be sustained HELD NO

Criminal Procedure Rowena Daroy Morales


the property of Juana; and the plea of former jeopardy would be of no avail. - In the light of what has been said it is evident that, by reason of the lack of conformity between the allegation and the proof respecting the ownership of the property, it is impossible to convict the two accused of the offense of robbery committed by them in this case; and therefore they cannot be convicted of the complex offense of robbery with homicide. HOWEVER, the accused were sentenced by the Supreme Court for four separate homicides.

a2010

page 16

Prof.
- It is a constitutional right of any person who stands charged in a criminal prosecution to be informed of the nature and cause of the accusation against him. Reasoning. The offense carries two elements: first, the carrying outside one's residence of any bladed, blunt, or pointed weapon, etc. not used as a necessary tool or implement for a livelihood; and second that the act of carrying the weapon was either in furtherance of, or to abet, or in connection with subversion, rebellion, insurrection, lawless violence, criminality, chaos, or public disorder. There are other statutes (SECTION 26 OF ACT NO. 1780, ORDINANCE NO. 3820 OF THE CITY OF MANILA) which may be charged against the accused for their acts to constitute a crime. It is the second element which removes the act of carrying a deadly weapon, if concealed, outside of the scope of the statute or the city ordinance mentioned above. In other words, a simple act of carrying any of the weapons described in the presidential decree is not a criminal offense in itself. What makes the act criminal or punishable under the decree is the motivation behind it. Without that motivation, the act fans within the purview of the city ordinance or some statute when the circumstances so warrant. -ON SUFFICIENCY OF THE INFORMATION: for a complaint or information to be sufficient it must, inter alia, state the designation of the offense by the statute, and the acts or omissions complained of as constituting the offense. This is essential to avoid surprise on the accused and to afford him the opportunity to prepare his defense accordingly. It is necessary that the particular law violated be specified as there exists a substantial difference between the statute and city ordinance on the one hand and P.D. 9 (3) on the other regarding the circumstances of the commission of the crime and the penalty imposed for the offense.(PD 9 punishes the offender with 5-10 yrs imprisonment; Sec26, Act 1780 with a fine of P500 or by imprisonment not exceeding 6 months or both; Ordinance 3820 with a fine of not more than P200 or imprisonment for not more than 1 month or both). But since it was specified in the Informations that the accused were charged with violation of Par3, PD 9, it was necessary for the Court to elucidate the elements of the said PD to differentiate it from other statutes (see above) the rest of the discussion was on the intent of the PD: to justify their decision that Par3 should be interpreted with the Whereas clause. - there exists a valid presumption that undesirable consequences were never intended by a legislative measure, and that a construction of which the

PEOPLE v PURISIMA 86 SCRA 542 MUNOZ-PALMA; November 20, 1978


NATURE Petitions for review (26 petitions consolidated) of the decisions of the Courts of First of Manila and Samar. FACTS -The private respondents were all charged with illegal possession of deadly weapons (one (1) carving knife with a blade 1/2 inches and a wooden handle of 5-1/4 inches, or an overall length of 11-3/4 inches in the Information filed with J.Purisima; ice pick with an overall length of about 8 1/2 inches in the Information filed with J. Maceren; socyatan in the Information filed with J. Polo) in violation of PD 9, Par. 3. Informations were filed with respondent judges in their respective courts (2 Branches of CFI, then CFI Samar) but upon motion to quash filed by the several accused, the said judges dismissed the Informations on the common ground that the said Informations did not allege facts which constitute the offense penalized by PD 90 failed to state 1 of the 2 essential elements of the crime punished (the carrying outside of the accused's residence of a bladed, pointed or blunt weapon is in furtherance or on the occasion of, connected with or related to subversion, insurrection, or rebellion, organized lawlessness or public disorder.) - In the 2 cases filed before the different branches of CFI Manila, the orders of dismissal were given before arraignment of the accused. In the criminal case before the CFI Samar the accused was arraigned but at the same time moved to quash the Information. In all the cases where the accused were

under arrest, the three Judges ordered their immediate release unless held on other charges. -ON PD 9:THIS CASE INVOLVES THE INTERPRETATION AND THE EXPLANATION OF THE INTENT OF THIS P.D. The pertinent paragraphs of the said PD is its Whereas clause ("WHEREAS, subversion, rebellion, insurrection, lawless violence, criminality, chaos and public disorder mentioned in the aforesaid Proclamation No. 1081 are committed and abetted by the use of firearms, explosives and other deadly weapons) and par3 (It is unlawful to carry outside of residence any bladed, pointed or blunt weapon such as 'fan knife,' 'spear,' 'dagger,' 'bolo,' 'balisong,' 'barong,' 'kris,' or club, except where such articles are being used as necessary tools or implements to earn a livelihood and while being used in connection therewith; and any person found guilty thereof shall suffer the penalty of imprisonment ranging from five to ten years as a Military Court/Tribunal/Commission may direct.) -Petitioners Contention: (1) Par 3, PD 9 shows that the prohibited acts need not be related to the subversive activities; that the act proscribed is essentially malum prohibitum penalized for reasons of public policy; (3) that since it is malum prohibitum, the intention of the accused who commits it is immaterial; (4) that PD was enacted to eradicate lawless violence which characterized pre-martial law days; and (5) that the real nature of the criminal charge is determined not from the caption or preamble of the information nor from the specification of the provision of law alleged to have been violated but by the actual recital of facts in the complaint or information. ISSUE WON the Informations filed by the People sufficient in form and substance to constitute the offense of Illegal Possession of Deadly Weapon penalized under PD 9 HELD NO. The two elements of the offense covered by P.D. 9(3) must be alleged in the information in order that the latter may constitute a sufficiently valid charged. Ratio. The sufficiency of an Information is determined solely by the facts alleged therein. Where the facts are incomplete and do not convey the elements of the crime, the quashing of the accusation is in order.

Criminal Procedure Rowena Daroy Morales


statute is fairly susceptable is favored, which will avoid all objectionable, mischievous, indefensible, wrongful, evil, and injurious consequences. It is to be presumed that when P.D. 9 was promulgated by the President of the Republic there was no intent to work a hardship or an oppressive result, a possible abuse of authority or act of oppression, arming one person with a weapon to impose hardship on another, and so on. Penal statutes are to be construed strictly against the state and liberally in favor of an accused. -ON OTHER REMEDIES OF THE PEOPLE: Under Rule 117, Sec 7 and Rule 110, Sec 13, Information may be amended or ordered by the court to be amended. Or, the People could have filed a complaint either under Sec 26 of Act 1780 or under Manila City Ordinance 3820 since most of the cases were dismissed prior to arraignment of the accused and on a motion to quash. Dispositive. WHEREFORE, We DENY these 26 Petitions for Review and We AFFIRM the Orders of respondent Judges dismissing or quashing the Information concerned, subject however to Our observations made in the preceding pages 23 to 25 of this Decision regarding the right of the State or Petitioner herein to file either an amended Information under Presidential Decree No. 9, paragraph 3, or a new one under other existing statute or city ordinance as the facts may warrant. Without costs. SO ORDERED.

a2010

page 17

Prof.
defendant a necessary knowledge of the charge so that he may not be confused in his defense. (a) BUT it is likewise the rule that if ever duplicity of offenses is committed, the same constitutes a ground for a motion to quash the complaint; failure of the accused to interpose the objection constitutes waiver. Neither can he claim that he was denied information that he was to be tried for two crimes. The acts complained of were stated in ordinary and concise language that any person of common intelligence would be able to understand and thereby know what acts he was to defend himself against. (b) As clearly found by the trial court: Both accused have, obviously, conspired and confederated to commit the crime, considering that they entered the bathroom where Rebecca was, together and at the same time. Accused Fernandez then tied her with a piece of cloth tightly around her neck, while accused Conrado held her hands placing them behind her body. Then after Fernandez had raped Rebecca, Conrado raped her. Both fled from the scene of the crime together and at the same time. 2. NO - Appreciating the aggravating circumstance of ignominy is correct because of the greater perversity displayed by the offenders. The act of "plastering" mud on the victim's vagina right after she was raped is adequately described as "ignominy" (rather than cruelty or ignominy) 3. NO - The original death sentence was correctly imposed: Art 335 RPC states that when the crime of rape is committed by 2 or more persons, the penalty shall be reclusion perpetua to death; Art 63 RPC states that when the penalty prescribed is composed of 2 indivisible penalties and the offense is attended by an aggravating circumstance, the greater penalty shall be applied. - However, since the original death penalties imposed by the trial court are no longer imposable under the present Constitution and are reduced to reclusion perpetua, the sentence on appellant Conrado has to be reduced to 2 penalties of reclusion perpetua. But the indemnity he has to pay to the victim must be increased to P20T in line with prevailing jurisprudence. Dispositive Appeal has no merit. Decision affirmed.

the house to report the tragic incident to Amelita, Teofilos daughter. - TEOFILO stated that upon being informed that his housemaid Rebecca was raped by the accused, they all proceeded to the office of the INP Police Station of Malasiqui to report the crime and had Rebecca physically examined in that same afternoon. - In defense, the 2 denied any involvement in the offense, both claiming they were nowhere at the scene of the crime when it was committed. - CFI decision: Each of the accused MELQUIADES FERNANDEZ and FEDERICO CONRADO is guilty beyond reasonable doubt of two crimes of rape, aggravated by cruelty or ignominy. Court sentences each of them to suffer 2 penalties of death. - Appeal before SC: The accused filed this appeal to reduce penalty from death to reclusion perpetua. However, in light of the 1987 Consti specifically Sec 19(1), Art III, under which a death penalty already imposed is reduced to reclusion perpetua, Fernandez withdrew his appeal. The lone appellant therefore is Conrado who insists on his appeal, notwithstanding the advice of his counsel de officio to discontinue. ISSUES 1. WON CFI erred in convicting them for 2 crimes of rape 2. WON CFI erred in holding that the rape was attended by the aggravating circumstance of cruelty or ignominy 3. WON CFI erred in sentencing each to suffer 2 penalties of death HELD 1. NO Ratio The imposition on each of the accused of the penalty corresponding to 2 crimes of rape is proper, because of the existence of conspiracy. In multiple rape, each defendant is responsible not only for the rape personally committed by him, but also for the rape committed by the others, because each of them cooperated in the commission of the rape perpetrated by the others, by acts without which it would not have been accomplished. Reasoning CFI is accused of violating the rule against duplicity of offenses in that, the accused were convicted for 2 crimes of rape even when under the criminal complaint against them, there is only 1 crime of rape alleged. The rule invoked is Sec 13, Rule 110 of the ROC which states that there should be only 1 offense charged in a criminal complaint or information, the purpose of which is to afford the

PEOPLE v FERNANDEZ 183 SCRA 511 PADILLA; March 22, 1990


NATURE Appeal from CFI Pangasinan decision FACTS - Criminal complaint filed before the CFI alleged that the accused, conspiring and mutually helping one another, had sexual intercourse with the 15-yr old Rebecca SORIANO, by means of force and intimidation. Assisted by counsel, the accused FERNANDEZ and CONRADO pleaded not guilty on arraignment and underwent trial. - REBECCA is Teofilo Malongs househelper. Ater she had just taken a bath and still naked, the two accused, both in short pants, surreptitiously entered the bathroom and sexually abused her. Fernandez then got a handful of mud near the bathroom and placed it on her vagina. She ran to the upper floor of

PEOPLE v LUMILAN 323 SCRA 170 DE LEON; June 25, 2000

Criminal Procedure Rowena Daroy Morales


NATURE Appeal from a decision of the Regional Trial Court of Ilagan, Isabela FACTS - Regional Trial Court (RTC) of Ilagan, Isabela, found accused-appellants Leon Lumilan and Antonio Garcia guilty beyond reasonable doubt of three (3) counts of murder, two (2) counts of frustrated murder, and three (3) counts of attempted murder, under an Information charging them and accused Fred Orbiso with the crime of Qualified Illegal Possession of Firearms Used in Murder, in violation of Presidential Decree (P.D.) No. 1866. - The evidence of the prosecution reveals that in the early evening of October 12, 1987, Meliton Asuncion, Modesto Roque, Eliong dela Cruz, Jerry Palomo, Simeon Pacano, Benito Alonzo, Nolasco Estrada, Mario Palomo and Romeo Pacho were drinking liquor inside the house of Policarpio Palomo when it was sprayed with bullets. The successive gunshots emanated from the fence about six (6) meters away from where they were drinking, killing Meliton Asuncion, Modesto Rogue, and Eliong dela Cruz and seriously wounding Jerry Palomo, Simeon Pacano, Nolasco Estrada, Mario Palomo and Romeo Pacho. - Upon being arraigned, both Lumilan and Garcia entered the plea of not guilty, and during trial, they interposed the defense of alibi. - After an assessment of the evidence, the trial court declared that no proof beyond reasonable doubt was adduced by the prosecution to justify the conviction of appellants for Qualified Illegal Possession of Firearms Used in Murder. However, the trial court convicted the appellants for Murder, Frustrated Murder and Attempted Murder. - Appellants filed a motion for reconsideration which was, however, denied - Hence, the instant appeal. ISSUE WON the appellants may be properly convicted of murder, frustrated murder and attempted murder under an Information that charges them with qualified illegal possession of firearms used in murder in violation of Section 1 of presidential Decree (P.D.) No. 1866, as amended. HELD YES - At the time the trial court promulgated its judgment of conviction in September 1990, it had already been six (6) months since the Court held in People v. Tacan that the unlawful possession of an unlicensed

a2010

page 18

Prof.
Qualified Illegal Possession of Firearms Used in Murder, is defective, and their conviction for Murder, Frustrated Murder and Attempted Murder, is irregular. - However, such defect in the Information and the irregular conviction of appellants, does not invalidate the criminal proceedings had in the trial court because the appellants waived their right to quash the Information, and they effectively defended themselves against the charges for murder, frustrated murder and attempted murder. - While the Information specifically states that appellants are being accused of the crime of Qualified Illegal Possession of Firearms Used in Murder in violation of P.D. No. 1866, its text is so worded that it describes at least three (3) crimes: illegal possession of firearms, murder, and attempted/frustrated murder. - The Information is undeniably duplicitous. Sec. 13, Rule 110 of the Revised Rules of Court provides that a complaint or information must charge but one offense, except only in cases where the law prescribes a single punishment for various offenses. Duplicity or multiplicity of charges is a ground for a motion to quash under Sec. 2 (e), Rule 117 of the Revised Rules of Court. The accused, however, may choose not to file a motion to quash and be convicted of as many distinct charges as are alleged in the information and proved during the trial. In the same vein, failure to interpose any objection to the defect in the information constitutes waiver. - In the instant case, appellant did not file any motion to quash the Information. More significantly, the bulk of the evidence that they presented during the trial was intended to disprove their complicity in the murder, frustrated murder and attempted murder of the victims. -As such, appellants cannot pretend that the Information did not fully apprise them of the charges against them as to cause them surprise in the event of conviction. The appellation of the crime charged as determined by the provincial fiscal may not exactly correspond to the actual crimes constituted by the criminal acts described in the Information to have been committed by the accused, but what controls in the description of the said criminal acts and not the technical name of the crime supplied by the provincial fiscal. Since appellants defended themselves not only against the offense of Qualified Illegal Possession of Firearms Used in Murder as specified in the Information, but also, and more

firearm or ammunition, whether or not homicide or murder resulted from its use, on one hand, and murder or homicide, on the other, are offenses different and separate from and independent of, each other. While the former is punished under a special law, the latter is penalized under the Revised Penal Code. Consequently, the prosecution for one will not bar prosecution for the other, and double jeopardy will not lie. - Sec. 4. Rule 120 of the Revised Rules of Court provides that an accused may not be convicted of an offense other than that with which he is charged in the Information, unless such other offense was both established by evidence and is included in the offense charged in the Information. Since murder or homicide neither includes or is necessarily included in qualified illegal possession of firearms used in murder or homicide, the trial court may not validly convict an accused for the former crime under an Information charging the latter offense. Conversely, an accused charged in the Information with homicide or murder may not be convicted of qualified illegal possession of firearms used in murder or homicide, for the latter is not included in the former. -Further, a significant change was introduced to Sec. 1 of P.D. No. 1866 by Republic Act (R.A.) No. 8294, such that now, where an accused uses an unlicensed firearm in committing homicide or murder, he may no longer be charged with what used to be the two separate offenses of homicide or murder under the Revised Penal Code and qualified illegal possession of firearms used in homicide or murder under P.D. No. 1866. -As amended by R.A. No. 8294, P.D. No. 1866 now mandates that the accused will be prosecuted only for the crime of homicide or murder with the fact of illegal possession of firearms being relegated to a mere special aggravating circumstance. - The Information charging appellants with Qualified Illegal Possession of Firearms Used in Murder, violates Sec. 1 of P.D. No. 1866, as amended by R.A. No. 8294, which obliterated the now obsolete concept of qualified illegal possession of firearms or illegal possession of firearms in its aggravated form, i.e., where the penalty for illegal possession is increased to reclusion perpetua or death by the attendance of homicide or murder. In fact, qualified illegal possession of firearms, which used to be a distinct offense, no longer exists in our statute books. - Whether considered in the light of our ruling in Tacan and its progeny of cases or in the context of the amendments introduced by R.A. No. 8294 to P.D. No. 1866, the Information charging appellants with

Criminal Procedure Rowena Daroy Morales


seriously against the crimes of Murder, Frustrated Murder and Attempted Murder as described in the body of the Information, it cannot be said that their conviction for the latter crimes is infirm and invalid ***Appellants in this case were nonetheless acquitted on the ground of reasonable doubt. The constitutional presumption of innocence in favor of the appellants was not over-turned by the evidence adduced by the prosecution. The Court entertained doubts as to the prosecutions witnesses testimony that they were able to identify the appellants as the authors of the crime considering that it was dark outside, and the only source of light were two kerosene lamps inside the house. They also took note of the fact that Pacano, one of the witnesses, only executed his sworn statement more than five months atfer the incident. Disposition The decision of the Regional Trial Court of Ilagan, Isabela is REVERSED and SET ASIDE. The accused-appellants, Leon Lumilan and Antonio Garcia, are hereby ACQUITTED on the ground that their alleged guilt was not proven beyond reasonable doubt.

a2010

page 19

Prof.
objective of Presidential Decree No. 133 is to place a strong deterrent on workers from sabotaging the productive efforts of the industry where they are employed, it is essential, to qualify the offense and to justify the imposition of the heavier penalty prescribed by said Decree, that the information should aver that the articles stolen were materials or products which the accused was "working on or using or producing," and that a statement in the preamble of the information that the accused is charged with the crime of simple theft "in relation to Presidential Decree No. 133," does not suffice for the purpose envisioned by the constitutional guarantee that the accused should be informed of the nature and cause of the accusation against him. The Supreme Court said that the appropriate penalty is that under Article 309 (3) of the RPC-prision correccional in its minimum and medium periods if value of property stolen is more than 200 pesos but does not exceed 6,000 pesos. But with the mitigating plea of guilty, penalty is in its minimum period.

MATILDE v JABSON 68 SCRA 456 ANTONIO; December 29, 1975.


NATURE Certiorari to nullify the judgment of respondent Court of First Instance of Rizal, Branch XXVI, in Criminal Cases Nos. 9552, 9553 and 9554, imposing upon the accused Crisanto Matilde, Jr. y Cruz, for the crime of simple theft, the penalty prescribed in Presidential Decree No. 133 (which imposes a heavier penalty) instead of that imposed by Article 309, paragraph 3, of the Revised Penal Code. FACTS - An Assistant Provincial Fiscal of Rizal filed three informations in Criminal Cases Nos. 9552, 9553 and 9554 against Crisanto Matilde, Jr. y Cruz, Patricio Guiruela y Luna, Ricardo Abener y San Pascual, Edgardo Cape y Atienza, Servando Calpo y Caballero, and Ireneo Belver y Bale. In three criminal cases, respondent court imposed upon petitioner, for the crime of simple theft, the penalty prescribed in Presidential Decree No. 133, instead of that imposed by Article 309, paragraph 3, of the Revised Penal Code. The information charged that petitioner and his co-accused, being then laborers, conspired and confederated with, and

mutually aided one another, with intent of gain and without knowledge and consent of their employer, in stealing the articles mentioned therein belonging to their employer. Although the preamble of said informations stated that petitioner was charged with the crime of simple theft "in relation to Presidential Decree No. 133," nowhere was it alleged in the body of said information that the articles stolen were materials or products which petitioner was "working on, or using or producing" as employee or laborer of the complainant, as provided for in Presidential Decree No. 133. Except for the dates of commission and the amounts involved, the aforesaid three (3) informations uniformly stated that said accused were charged with the crime of qualified theft, in relation to Presidential Decree No. 133, committed as follows: "That on or about the 14th day of November, 1973 in the Municipality of Pasig, Province of Rizal, Philippines, and within the jurisdiction of this Honorable Court, the above-named accused, being then laborers working at the Markes Agro-Chemical Enterprises, conspiring and confederating together with one Renato Matuto y Ann, who is still at large, all of them mutually helping and aiding one another, with intent of gain, grave abuse of confidence, and without the knowledge and consent of the said firm, its President and General Manager, Marciano K. Espiritu, did then and there wilfully, unlawfully and feloniously take, steal and carry away the following, to wit: . . ." - When the informations were amended from Qualified Theft to Simple theft and deleting from the body of Information the phrase Grave abuse of confidence, Matilde pleaded GUILTY but the Court imposed the penalty under PD 133 and not those by Article 309 (3) of the RPC. From this decision, Matilde sought from the Court a quo a reconsideration contending that in the absence of any allegation in the body of information alleging specifically all the elements of the offense defined and penalized under PD. 133, he cannot be conviceted and penalized under the aforesaid decree. ISSUE WON the information that the accused is charged with the crime of simple theft in relation to PD 133 suffices HELD NO - The Supreme Court granted the writ of certiorari and set aside the judgment, and directed that another one be rendered. It held that since the

BALITAAN v CFI (DE LOS REYES) 115 SCRA 729. GUERRERO; July 30, 1982
FACTS - Luz Balitaan owns a baby dresses mending shop. Rita de los Reyes is the manager of her business. - Luz, thru Special Counsel Aguila, filed with the MTC of Bauan, Batangas an Information charging Rita of the crime of estafa. The information contains that Rita misappropriated P127.58, through grave abuse of confidence, despite of repeated demands of Luz. (See original for exact wording of Information.) - During trial at the MTC, Luz testified that Rita delivered the baby dresses to Uniware, and for this she (Rita) obtained 3 checks totaling P1,632.97. A cash voucher evidencing the receipt of said amount was entered into evidence. The lawyer for the defense moved: (1) to strike the testimonies with regard to the voucher evidence on the ground that said testimonies are at variance with the allegations in the information, that there is no allegation in the

Criminal Procedure Rowena Daroy Morales


information whatsoever regarding these checks and this cash voucher; and (2) in the nature of an objection to any other question or questions regarding these checks that were allegedly received by the herein accused from the Uniware Incorporated because there is no allegation in the information. The court overruled such objections as the lawyer of the complainant told the judge that the evidence was presented to prove that the P127.58 was misappropriated from the P1,632.97. The testimony thus continued. [It turns out that Rita told Luz that P127.58 was due a Cesar Dalangin for some of the dresses he made. Luz then instructed Rita to encash the checks and pay Cesar. Rita gave Luz the encashed amount minus the P127.58. Three weeks later, when she noticed that many baby dresses were lost, she verified the receipts of the payments. Cesar said he did not make the baby dresses Rita said he did, and he didnt receive the amount (he didnt even know Rita). Luz then demanded from Rita the said amount; but Rita kept the money.] - The defense then filed a petition for certiorari in the CFI of Batangas against the MTC judge for denying the motions to strike out the testimonies relating to the evidence. CFI granted the petition and ordered the testimonies stricken out of the record. ISSUE WON the testimonies are at variance with the allegations in the information. HELD NO - It is fundamental that every element of which the offense is composed must be alleged in the complaint or information. What facts and circumstances are necessary to be stated must be determined by reference to the definitions and the essentials of the specific crimes. The main purpose of requiring the various elements of a crime to be set out in an information is to enable the accused to suitably prepare his defense. He is presumed to have no independent knowledge of the facts that constitute the offense. - Inasmuch as the crime of estafa through misappropriation or with grave abuse of confidence is charged, the information must contain these elements: (a) that personal property is received in trust, on commission, for administration or under any other circumstance involving the duty to make delivery of

a2010

page 20

Prof.
Court of Balingasag, Misamis Oriental on January 9, 1995. MCTC Judge Alfredo Cain found sufficient ground to prosecute the appellant for the crime of rape. This was the same finding of the Office of the Provincial Prosecutor of Misamis Oriental upon examination of the records of the preliminary investigation forwarded to it. Consequently, on March 21, 1995, the corresponding Information was filed with the Regional Trial Court reading as follows: "INFORMATION "The undersigned Assistant Provincial Prosecutor II, upon sworn complaint of the offended party, Neddy Calayca, accuses ARTEMIO CALAYCA of the crime of RAPE, committed as follows: That on or about the 29th day of January, 1994 at about 1:00 o'clock in the morning, more or less, at Barangay Solo, Municipality of Balingasag, Province of Misamis Oriental, Philippines and within the jurisdiction of this Honorable Court, the above-named accused did then and there willfully, unlawfully and feloniously and by means of force and intimidation, succeeded in having carnal knowledge (sexual intercourse) with her (sic) own daughter, Neddy Calayca, against her will and consent. "CONTRARY TO and in VIOLATION OF Article 335 of the Revised Penal Code, as amended by Republic Act No. 7659. "Cagayan de Oro City, Philippines, March 6, 1995. (SGD.) ROBERTO S. CASIO "Asst. Provincial Prosecutor II" - When arraigned under the above-quoted Information, the appellant entered a plea of "Not guilty" to the crime charged. Trial on the merits ensued thereafter. - The evidence for the prosecution was anchored mainly on the testimony of 16-year old Neddy Calayca who, on May 2, 1995, narrated that at about 1:00 o'clock in the morning of January 29, 1994, she was sound asleep inside their house at Barangay Solo, Balingasag, Misamis Oriental when she was awakened by the weight of her father, herein appellant Artemio Calayca, who was already on top of her, naked and armed with a bolo. He forcibly undressed her, inserted his penis into her vagina and made a push and pull motion. Feeling the pain in her vagina, she resisted his onslaught by kicking and hitting him, telling him with bitter tears, "I wish you would die. You are a father without good morals." But she was helpless to resist his lustful desire as he threatened her with a knife saying, "I will kill you if

or to return the same, even though the obligation is guaranteed by a bond; (b) that there is conversion or diversion of such property by the person who has so received it; (c) that such conversion, diversion or denial is to the injury of another and (d) that there be demand for the return of the property. - The position of the defense is that the testimonies tend to prove another kind of estafa --- using false pretenses or fraudulent acts (Art 315 par 2a RPC)--and not thru abuse of confidence (Art 315 par 1b RPC). The elements of these two are different. Under par 2a, demand is not necessary and deceit or false representation must be shown. But this doesnt mean that proof of deceit is not allowed for par 1b. Abuse of confidence and deceit may co-exist. Even if deceit may be present, the abuse of confidence will characterize the estafa as the deceit will be merely incidental or, is absorbed by abuse of confidence. - As long as there is a relation of trust and confidence between the complainant and the accused and even though such relationship has been induced by the accused thru false representations and pretense and which is continued by active deceit without truthfully disclosing the facts to the complainant, the estafa committed is by abuse of confidence although deceit co-exists in its commission. - The presence of deceit would not change the whole theory of the prosecution that estafa with abuse of confidence was committed. Dispositive CFI decision to strike out testimonies is reversed and set aside.

PEOPLE v CALAYCA 301 SCRA 192 MARTINEZ; January 20, 1999


NATURE Automatic review FACTS - A daughter was again allegedly raped by her own father, herein appellant Artemio Calayca, who is now facing a death sentence after having been found guilty of said crime in a Decision 1 dated June 13, 1995, rendered by the Regional Trial Court (Branch 24) of Cagayan de Oro City in Criminal Case No. 95129. Hence, this automatic review. - A rape charge was initiated by Neddy Calayca through a sworn complaint with supporting affidavits and documents 3 filed with the Municipal Circuit Trial

Criminal Procedure Rowena Daroy Morales


you will not agree." After the sexual assault, she picked up her clothes, dressed up and was left weeping. She was then 15 years old when this incident happened. 7 - Neddy Calayca first thought of immediately filing a case against appellant but was prevented by his threat to kill her. She, however, reported her awful experience with the appellant to her relatives in Mambayaan. She informed them that even before the January 29, 1994 incident, appellant had sexually abused her many times. Her relatives, who were also afraid of appellant, merely advised her to sue him. She immediately went home in Solo because she feared her father. When she reached home, her eldest sister Betty Lani Calayca also arrived from Manila. Informed of the rape incident, Lani and Neddy decided to leave the appellant. The two then traveled to Don Carlos, Bukidnon and worked as servants of the mayor, thinking their father could no longer find them there. However, appellant was able to locate them. While in the house of the mayor, appellant harassed them, so Betty Lani had him arrested by the police. While appellant was in jail, Neddy reported to the police authorities that he raped her. The police then took her sworn statement on the rape incident. Thereafter, Neddy filed her complaint for rape against the appellant. - Appellant Artemio Calaycadid not deny the imputation of her daughter Neddy Calayca that he raped her in the early morning of January 29, 1994. All that he testified to was that he was a widower in 1998 and has six children by his late wife, two of whom he identified as Neddy, the private complainant, and Betty Lani. He claimed that Neddy was only nine years old when his wife died. The private complainant stayed with him together with his five other children, while Betty Lani stayed with his (appellant's) brother at San Juan, Misamis Oriental. Betty Lani and Neddy left his house on August 19, 1993. They took his savings from the proceeds of the sale of his pig in the amount of P5,000.00. He then looked for his two daughters and found them at Bocboc, Don Carlos, Bukidnon. When he asked them why they took his money, his two daughters did not say a word, forcing him to slap them. - The defense did not present any other witness nor any documentary evidence. A judgment convicting the appellant of the crime charged and imposing upon him the penalty of death was rendered by the trial court. ISSUE

a2010
WON the correct penalty was imposed

page 21
Dispositive Judgment modified

Prof.

HELD NO - While the Court agrees that the penalty of death should be imposed on him, regrettably this is not in accord with the law and jurisprudence. Although the matter of the proper imposition of the penalty is not assigned as an error by the appellant, nevertheless, it is a well-established rule in criminal procedure that an appeal in a criminal proceeding throws the whole case open for review and it becomes the duty of the appellate court to correct an error as may be found in the appealed judgment, whether it is made the subject of assignment of errors or not. - The trial court imposed the death penalty on appellant because of the presence of the circumstance of minority of the victim (she was only 15 years old at the time she was raped on January 29, 1994) as well as the relationship of the offender (father) and the victim (daughter), pursuant to Section 11 of Republic Act No. 7659 30 which amended Article 335 of the Revised Penal Code. Section 11 provides, inter alia, that where the victim of the crime of rape is under 18 years of age and the offender is a parent of the victim, the death penalty shall be imposed. This is among the seven (7) circumstances enumerated in Section 11 which, as we have held in the recent case of People v. Garcia, 32 are considered special qualifying circumstances specifically applicable to the crime of rape. - There being no allegation of the minority of the victim in the Information under which the appellant was arraigned, he cannot be convicted of qualified rape as he was not properly informed that he is being accused of qualified rape. Appellant's conviction of qualified rape violates his constitutional right to be properly informed of the nature and cause of accusation against him. In a criminal prosecution, it is the fundamental rule that every element of the crime charged must be alleged in the Information. The main purpose of this constitutional requirement is to enable the accused to properly prepare his defense. He is presumed to have no independent knowledge of the facts that constitute the offense. - The failure to allege the fact of minority of the victim in the Information for rape is fatal and consequently bars the imposition of the death penalty. Having been informed only of the elements of simple rape, the appellant can be convicted only of such crime and be punished accordingly with reclusion perpetua.

US v JAVIER DICHAO 27 Phil 421 MORELAND; March 30, 1914


NATURE Appeal From CFI Davaos Decision FACTS - Said CFI sustained a Demurrer to an INFORMATION and dismissed the case of rape against Antonio Javier Dichao. - The Information stated that Dichao committed the crime of rape on or about and during the interval between October 1910, to August 1912 (vague di ba?) in Davao and that Dichao was, at that period, the legal guardian, being the stepfather, of Isabel de la Cruz who was under 12 years old when he raped her; that as a result of said carnal knowledge Isabel gave birth to a child on August 5, 1912. - The Demurrer alleged that the facts set forth in the Information did not constitute a public offense and that the criminal complaint did not conform substantially to prescribed form and that complaint was vague and ambiguous. ISSUE WON CFI committed an error in dismissing the case based upon the Demurrer HELD NO. CFIs decision must be affirmed. Ratio The allegations of an information should, if possible, be sufficiently explicit and certain as to TIME to inform the defendant of the date on which the criminal act is alleged to have been committed. Unless the accused is informed of the day, or about the day, he may be, to an extent deprived of the opportunity to defend himself. Reasoning - While Sec 7 of the Code of CrimPro provides that except when time is a material ingredient of an offense, the precise time of commission need not be stated in a complaint or information, but the act may be alleged to have been committed at any time before the filing thereof, - this DOES NOT MEAN that the prosecuting officer may be careless in fixing the date of the alleged crime, or that he may omit

Criminal Procedure Rowena Daroy Morales


the date altogether, or that he may make the allegation indefinite as to amount to the same thing. - Where the exact date cannot be fixed, or where the prosecuting officer is NOT thoroughly satisfied that he can prove a precise date, he should allege in the information that the crime was committed ON or ABOUT a DATE NAMED. - Under such an allegation he is not required to prove any precise date but may prove any date which is NOT SO REMOTE as to surprise and prejudice the defendant. - In case of SURPRISE, the Court may allow an amendment of the information as to time and an adjournment to the accused, if necessary to meet the amendment. - SC then cited cases: US v De Castro~ While it is not necessary, unless time is a material ingredient of the offense, that the precise time of the commission of the offense should be stated, still the act should be alleged to have been committed at some time before the filing of the complaint. US v. Enriquez- question of time as alleged in the information was discussed in an incidental way for the purpose of determining whether it of itself or in connection with the other allegations sufficiently identified the transaction which constituted estafa so as to notify the defendant of the transaction referred to; Time is not a matl ingredient in the crime of estafa. US v. Cardona- question of time was raised in the demurrer (on appeal) as to the variance bet the date of the crime in the info and that proved on the trial; Court here said that time being not an ingredient of the theft of a carabao, it did not have to be proved as laid. - The question whether the allegations of the info are sufficiently definite as to time and the question which arises on a variance between the allegations and the proof are different in nature and legal effect, and are decided on different principles. - In this case, the statement of the time when the crime was committed is too indefinite to give the accused an opportunity to prepare his defense, and that indefiniteness is not cured by setting out the date when a child was born as a result of such crime. - Sec 7 Rules of CrimPros purpose is to permit the allegation of a date of the commission of a crime as NEAR to the ACTUAL date as the information of the prosecuting officer will permit and when that has been done, any date proved which does not surprise and substantially prejudice the defense.

a2010

page 22

Prof.
- Moleros alibi was readily refuted. - Trial court found Molero guilty beyond reasonable doubt of rape. - A double jeopardy issue arose because there were two complaints filed: - filed March 22, 1977: rape was committed Feb 13, 1976 - filed March 30, 1978: rape was committed Feb 5, 1976 - Molero was arraigned under the first complaint, he pleaded not guilty. - During trial, the provincial fiscal filed motion for leave to amend the complaint. This was granted. Thus, the new complaint. - Molero filed motion to quash 2nd criminal complaint on ground of double jeopardy. This was denied. ISSUES 1. WON Molero was under double jeopardy 2. WON Molero committed the rape HELD 1. NO - Section 9, Rule 117 of 1985 Rules on Criminal Procedure: Conviction or acquittal of the defendant or the dismissal of the case shall be a bar to another prosecution for the offense charged - Here, the case was not terminated because the dispositive portion of the order expressly directed the Provincial Fiscal and/or prosecuting fiscal to file a new complaint and/or information. - The case was dismissed for no other reason except to correct the date of the crime. - This dismissal did not amount to an acquittal. - There was no need for trial court to have used such procedure. It should just have denied motion for reconsideration of the order granting the prosecutions motion for leave to amend the complaint. After arraignment and where appellant pleaded not guilty, is it still proper to amend date of commission of crime? Applying Sections 10 and 13 of Rule 110 of Revised Rules of Court, amendment sought by prosecution should have been granted. The precise time is not an essential element of rape. The amendment was only a matter of form and did not prejudice the rights of the appellant. 2. YES - Molero argues that if a crime was committed by him at all, it was qualified seduction. - SC didnt agree. Appellant was shown to have employed force and intimidation against daughter. Also, he had moral ascendancy and influence over the victim. The victim is illiterate and unschooled,

- It does not authorize the total omission of a date or such an indefinite allegation with reference thereto as amounts to the same thing. - SC: the variance bet the date of the commission of the crime as alleged in the info and that as proved on trial DOES NO warrant necessarily the acquittal of the accused. IF such variance occurs and it is shown that the defendant is surprised thereby, and that, by reason of that surprise, he is unable to defend himself properly, the court may in the exercise of sound discretion based on ALL circumstances, order the information amended so as to set forth the correct date and may grant an adjournment for such a length of time as will enable the defendant to prepare himself to meet the variance in date which was the cause of his surprise. Dispositive Decision affirmed.

PEOPLE v MOLERO 144 SCRA 397 GUTIERREZ JR.; September 24, 1986
NATURE Appeal from decision of CFI FACTS - Molero was charged with rape by daughter in complaint filed in CFI Negros Oriental. Molero told daughter to go with him to the river to catch shrimps and fish. She was barely 17. She was hugged fr behind by Molero and she fell to the ground. He unsheathed his bolo. He succeeded in having sexual intercourse and warned her not to tell anyone. - The mother learned of the incident and told daughter to keep quiet for the moment; they were secretive of their plan to report because Molero is a fierce man. - Mother and daughter went to Station Commander. They were advised to report to the PC Headquarters. At the PC Headquarters, complaint was investigated, but accused didnt want investigation to continue because accdg to him, this was their own problem. - Internal and external exam of victim showed she had previous sexual intercourse. - Molero denied the charge, saying he couldnt have done it because he was already committed in the provincial jail that time. He also denied the sworn statement he made, saying hes illiterate. He said he was not informed of his rights to remain silent and to counsel; that he was not assisted by counsel during investigation.

Criminal Procedure Rowena Daroy Morales


and Molero threatened her with a bolo and rendered her practically helpless.

a2010

page 23

Prof.
- Furthermore, Section 10, Rule 110 of the 1964 Rules of Court provided: Sec. 10. Time of the commission of the offense.-It is not necessary to state in the complaint or information the precise time at which the offense was committed except when time is a material ingredient of the offense, but the act may be alleged to have been committed at any time as near to the actual date at which the offense' was committed as the information or complaint will permit. 2. NO Ratio Art. 344(3) of the Revised Penal Code prohibits a prosecution for seduction, abduction, rape, or acts of lasciviousness, except upon a complaint made by the offended party or her parents, grandparents, or guardian, nor, in any case, if the offender has been expressly pardoned by the above-named persons, as the case may be. It does not prohibit the continuance of a prosecution if the offended party pardons the offender after the cause has been instituted, nor does it order the dismissal of said cause. The only act that riding to Article 344 extinguishes the penal action and the penalty that may have been imposed, is the marriage between the offender and the offended party. Reasoning - The rationale of the law on the prosecution of private crimes is simple: The law deems it the wiser policy to let the aggrieved woman and her family decide whether to expose to public view or to heated controversies in court the vices, faults and disgraceful acts occurring in the family. However, when, as in the case at bar, the pardon is given after the filing of the complaint in court, it comes too late to hide the shameful occurrence from public notice. Dispositive Decision of trial court affirmed

PEOPLE v LUALHATI 171 SCRA 277, 283 GRINO-AQUINO; March 16, 1989
NATURE Petition for review of the Decision of the Trial court FACTS - Complainant Josephine Dimaunahan was born on January 7, 1967 - In 1970, her mother separated from her father and started to live with appellant Vicente Lualhati without the benefit of marriage. She likewise lived with appellant who supported her, took care of her studies and treated her like his own daughter. - Sometime in June, 1978, while complainant's mother was at work, appellant and complainant were alone in the house. Appellant had sexual intercourse with complainant. It appeared that even prior to June, 1978, appellant had already several sexual relations with complainant - Upon arraignment on, the accused pleaded not guilty - The defense filed a motion to dismiss on the ground that the complaint charged more than one offense, namely: That on or about the month of June, 1978, and for sometime prior and subsequent thereto, ... the accused Vicente Lualhati wilfully, unlawfully and feloniously have carnal knowledge of the complainant Josephine M. Dimaunahan ... - Fiscal alleged that the accused was being tried on the Information which charged only one offense committed "in or about the month of June 1978." - Trial judge denied motion to dismiss. - The accused filed another Motion to Dismiss, alleging that he had been pardoned by the offended party, her mother and grandmother. Attached, to the Motion to Dismiss was the joint affidavit of desistance signed by the offended party, her mother and grandmother - -The offended party executed and filed an affidavit alleging that her father abandoned her at the age of two years and three months, without providing for her support and studies, and that the same were provided by her mother and grandmother who, on

the same date, executed a joint affidavit to the same effect - The Prosecuting Fiscal filed an Addendum to the Opposition to the Motion to Dismiss. He alleged that the express pardon given the accused was invalid for the offended party did not have "a will of her own," being merely eleven years old when the crime was committed; that the father of the offended party, executed an affidavit objecting to the pardon given to the accused; and that, as the father, he still possessed the "patria potestas" over the offended party in spite of his having abandoned her. - Trial court denied the motion to dismiss on account of the insistence of the victim's father to prosecute the accused, absent judicial pronouncement depriving him of parental authority over the offended party, a child below twelve years old. - Accused filed Motion to Quash, which was denied by the trial court - Trial court convicted the accused of rape, and imposed upon him the penalty of reclusion perpetua. ISSUES 1. WON there was a valid complaint against the appellant 2. WON the pardon given to him by the offended party, her mother, and grandmother extinguished his criminal liability, in spite of the objection of the victim's father. HELD 1. YES Ratio Discrepancies between the accusation and the complaint as to time of occurrence of the carnal copulations in rape do not affect any essential right of the accused, where the acts occurred within the period of time alleged in both writings and the difference noted in other respects was of a formal, rather than a substantial, character. Reasoning - Appellant contends that the complaint is void because it charges at least three crimes of rape, namely: (1) that which was committed "on or about the month of June, 1978;" (2) that which was committed "sometime prior to said period;" and (3) that which was committed "subsequent thereto." - Argument has no merit. Attached to Josephine's complaint was her sworn statement wherein, she categorically affirmed that Vicente abused her before the start of classes in June 1978. That affidavit, which may be considered part of the complaint required by law, cures any ambiguity in the complaint regarding the number of offenses committed by the accused.

PEOPLE v RAZONABLE 330 SCRA 562 PUNO; April 12, 2000


NATURE - Appeal from a decision by the RTC of Camarines Norte, dated May 3, 1996, finding appellant Benjamin Razonable guilty beyond reasonable doubt of raping his daughter, Maria Fe Razonable, and sentencing

Criminal Procedure Rowena Daroy Morales


him to suffer the penalties of 3 reclusion perpetua and to pay the amount of P200,000 as moral damages. FACTS - Razonable was charged in 3 separate Informations with the crime of rape, which are identically worded, as follows: "That sometime in the year 1987, at Purok I, Brgy. IV, Mantagbac, Municipality of Daet, Province of Camarines Norte, and within the jurisdiction of this Honorable Court, the above-named accused did then and there wilfully, unlawfully and feloniously have carnal knowledge of his own daughter MARIA FE H. RAZONABLE, against the latter's will and by means of force and intimidation, to her damage and prejudice. "The crime was committed with the aggravating circumstances of relationship, the accused is the father of the offended party and that said offense was committed in their own dwelling and the offended party not having given provocation for it." Razonable pleaded not guilty and his case was tried on the merits. - Although Maria Fe was raped on 3 consecutive days in the middle of June 1987, she was able to disclose the dastardly acts of her father to her elder sister only in February of 1993 because her conscience would not allow her any peace of mind. She also feared recurrence of the bestial acts. Her father often drank with friends inside their house and she was wary that appellant might give her to his friends. Thus, accompanied by her sister Ana Marie, complainant went to the police station and filed a complaint. Then they proceeded to the Camarines Norte Provincial Hospital where she was examined. Based on the medical certificate, she had, at the time of examination, incompletely healed hymenal lacerations at 5, 6, 7, and 9 o'clock positions. ISSUE 1. WON the RTC erred in not considering the information insufficient to support a judgment of conviction for its failure to state the precise date of the alleged commission of the offense, it being an essential element of the crime charged 2. WON the lower court erred in finding that the guilt of Razonable of the three counts of rape has been proven beyond reasonable doubt HELD 1. NO

a2010

page 24

Prof.
living alone for a long time. It is the most natural reaction for victims of criminal violence to strive to see the appearance of their assailant and observe the manner in which the crime was committed. Most often, the face and body movements of the assailant create a lasting impression which cannot be easily erased from their memory. The impression becomes more profound where the malefactor is the victim's own father. - The fact that Maria Fe continued to live with Razonable will not likewise crumple her credibility. At the time of the incident, she was a simple, nave and hapless child of twelve years. She was living by her lonesome self with her father, entirely dependent on him for all her needs. Her mother was in Isabela and her nearest sibling lived in another town. It could hardly be expected that such a child of tender age would know what to do and where to go under the circumstances. It is not proper to judge the actions of children who have undergone traumatic experiences by the norms of behavior expected under the circumstances from mature persons. - The delay in the filing of the cases does not necessarily impair the credibility of the victim. Experience teaches us that many victims of rape never complain or file criminal charges against the rapist, for they prefer to silently bear the ignominy and pain, rather than reveal their shame to the world or risk the offender's making good on his threats. Dispositive Considering that the acts were committed prior to the effectivity of RA 7659, the trial court correctly imposed the penalty of reclusion perpetua in each of the three cases. However, consistent with recent rulings, the amount of P50,000 for each count of rape should be awarded by way of moral damages, and hence the award given by the trial court should be reduced to P150,000. Likewise, current case law dictates that the victim shall be entitled to civil indemnity in the amount of P50,000 for each count of rape. - Decision of the RTC AFFIRMED with MODIFICATION.

Ratio The rationale of the rule (Section 11, Rule 110 of the ROC) is to inform the accused of the nature and cause of the accusation against him. To claim this substantive right protected by no less than the Bill of Rights, the accused is duty bound to follow our procedural rules which were laid down to assure an orderly administration of justice. Reasoning - Firstly, it behooved the accused to raise the issue of a defective information, on the ground that it does not conform substantially to the prescribed form, in a motion to quash said information or a motion for bill of particulars. An accused who fails to take this seasonable step will be deemed to have waived the defect in said information. The only defects in an information that are not deemed waived are where no offense is charged, lack of jurisdiction of the offense charged, extinction of the offense or penalty and double jeopardy. Corollarily, we have ruled that objections as to matters of form or substance in the information cannot be made for the first time on appeal. Razonable did not raise either in a motion to quash or a motion for bill of particulars the defect in the Information regarding the indefiniteness of the allegation on the date of the commission of the offense. - Secondly, during the trial, the defense never objected to the presentation of evidence by the prosecution to prove that the offense was committed in the middle of June 1987. It has not been shown that Razonable was taken by surprise with the testimony of Maria Fe that she was raped in the middle of June 1987, and hence could not properly defend himself. On the contrary, he was able to give an alibi as to his whereabouts at that particular time. In fine, he cannot pretend that he was unable to defend himself in view of the vagueness of the allegation in the information as to when the crimes were committed. 2. NO Reasoning Appellant claims that his guilt has not been proven beyond reasonable doubt on the following grounds: (1) the identity of the perpetrator has not been established with certitude since the room was dark and it has not been shown that it was properly illuminated; (2) it was unnatural for Maria Fe to remain in their house if it was true that she was threatened and intimidated; and (3) there was an unreasonable delay in the filing of the complaint which rendered the rape charges doubtful. - It is highly inconceivable that Maria Fe would not recognize her own father with whom she has been

ALMEDA v VILLALUZ PEOPLE v CASEY and FELIX 103 SCRA 21 GUERRERO; February 24, 1981
NATURE Automatic review of the judgment of the Circuit Criminal Court imposing upon Casey and Felix the capital c\punishment for the death of Alfredo Valdez.

Criminal Procedure Rowena Daroy Morales


FACTS - On May 22, 1968, Assistant Fiscal Herminio I. Benito filed an Information for Murder against accused-appellant Joseph Casey alias "Burl", alleging: That on or about the 31st day of March, 1968, in the municipality of San Juan, province of Rizal, a place within the jurisdiction of this Honorable Court, the above- named accused, being then armed with a knife, together with one Ricardo Felix alias "Carding Tuwad" who is then armed with a firearm and who was (sic) still at large, and the two of them conspiring and confederating together and mutually helping and aiding one another, with intent to kill, evident premeditation and treachery and taking advantage of superior strength, did, then and there wilfully, unlawfully and feloniously attack, assault and shoot and stab with the said firearm and knife one Alfredo Valdez, thereby inflicting upon the latter fatal wounds which directly caused his death. - In June, 1968, upon arraignment, Casey pleaded not guilty to the crime charged in the said complaint. - September, 1968, accused ' appellant Ricardo Felix alias "Carding Tuwad" was arrested. Accordingly, an Amended Information was filed by the same fiscal to include Ricardo Felix as an accused, stating: That on or about the 31st day of March, 1968, in the municipality of San Juan, province of Rizal, a place within the jurisdiction of this Honorable Court, the above named accused Joseph Casey alias "Burl" being then armed with a knife, together with the accused Ricardo Felix alias "Carding Tuwad" who was then armed with a firearm, and the two of them conspiring and confederating together and mutually helping and aiding one another, with intent to kill, evident premeditation and treachery and taking advantage of superior strength, did, then and there wilfully, unlawfully and feloniously attack, assault and shoot and stab with the said firearm and knife one Alfredo Valdez, thereby inflicting upon the latter fatal wounds which directly - The court a quo rendered the aforementioned judgment of conviction. It found that two aggravating circumstances attended the commission of the crime, namely: employing or taking advantage of superior strength and evident premeditation, one of which qualified the killing to murder. ISSUES

a2010

page 25

Prof.
Reasoning - Indeed, accused-appellant Joseph Casey gave an extrajudicial sworn statement that he met accusedappellant Ricardo Felix and another person named Rudy in Cubao, Quezon City on that fateful day. However, there is no showing that this meeting was purposely arranged to plan the killing of the victim. In fact, the following questions and answers in the said sworn statement show that there was no preconceived design to kill the victim. - There is evident premeditation when the killing had been carefully planned by the offender or when he had previously prepared the means which he had considered adequate to carry it out, when he had prepared beforehand the means suitable for carrying it into execution, when he has had sufficient time to consider and accept the final consequences, and when there had been a concerted plan. 16 It has also been held that to appreciate the circumstances of evident premeditation, it is necessary to establish the following; (1) the time when the offender determined to commit the crime; (2) the act manifestly indicating that the culprit has clung to his determination; and (3) a sufficient lapse of time between the determination and execution to snow him to reflect upon the consequences of his act and to allow his conscience to overcome the resolution of his will had he desired to hearken to its warning. - From the answers of accused-appellant Casey in said sworn statement, it can be gleaned that the killing was not a preconceived plan. It was not preceded by any reflection or deep thought. It was just a spontaneous decision reached when the victim started to run away upon being approached by accused-appellant Ricardo Felix. - There are indeed two accused-appellants in this case charged with the murder of not one victim but superiority in number does not necessarily mean superiority in strength. It is necessary to show that the aggressors "cooperated in such a way as to secure advantage from their superiority in strength." 3. YES Reasoning - Although there is no direct showing that the accused had conspired together, but their acts and the attendant circumstances disclose that common motive that would make accused Ricardo Felix as a co-principal with the actual slayer, Joseph Casey. Without doubt, he performed overt acts in furtherance of the conspiracy. - Ricardo Felix's overt acts consist in instigating the pursuit of the deceased, in firing a shot at him and in giving Joseph Casey encouragement by his armed

1. WON the Court a quo erred in illegally trying appellant Casey on the amended information without arraignment 2. WON the Court a quo erred in holding that appellants acted with evident premeditation and abuse o of superior strength, and in qualifying the crime committed as aggravated murder 3. WON whether or not there is conspiracy between the two accused in the commission of the crime 4. WON the Court erred in discounting Caseys defense that he acted in legitimate self-defense HELD 1. NO Reasoning The lack of arraignment under the amended information is objected to by accused-appellant Joseph Casey allegedly on the ground that there is a violation of his constitutional right to be informed of the charge against him. There can be a violation of such right, however, only when the amendment pertains to matters of substance. In the case at bar, the alterations introduced in the information refer to the inclusion of accused appellant Ricardo Felix to the same charge of murder. They do not change the nature of the crime against accused-appellant Casey. Conspiracy, evident premeditation, treachery and taking advantage of superior strength are similarly alleged in both informations. No extenuating circumstance is likewise alleged in both. Thus the amendment of the information as far as accusedappellant Casey is concerned is one of form and not of substance as it is not prejudicial to his rights. - The test as to whether a defendant is prejudiced by the amendment of an information has been said to be whether a defense under the information as it originally stood would be available after the amendment is made, and whether any evidence defendant might have would be equally applicable to the information in the one form as in the other. A look into Our jurisprudence on the matter shows that an amendment to an information introduced after the accused has pleaded not guilty thereto, which does not change the nature of the crime alleged therein, does not expose the accused to a charge which could call for a higher penalty, does not affect the essence of the offense or cause surprise or deprive the accused of an opportunity to meet the new averment had each been held to be one of form and not of substance not prejudicial to the accused and, therefore, not prohibited by Section 13, Rule 110 of the Revised Rules of Court. 2. YES

Criminal Procedure Rowena Daroy Morales


presence while the latter inflicted the fatal wounds on the deceased. From the extrajudicial confession of the accused-appellant Joseph Casey, it can also be inferred that Ricardo Felix was the moving factor of the evil act perpetrated by the former against the victim. While it was Joseph Casey who inflicted the mortal wounds that caused the death of the victim, he did so out of his perverted sense of friendship or companionship with Ricardo Felix. 4. YES Reasoning - claim is uncorroborated and contrary to the testimony of the eyewitness, Mercedes Palomo. - The fact that the victim sustained four stab wounds while the accused complained merely of abrasions on his back indicates the falsity of the claim. Dispositive the judgment of the trial court under automatic review is MODIFIED in that the accusedappellants Joseph Casey and Ricardo Felix are found guilty beyond reasonable doubt of the crime of homicide without any attending circumstances and should be sentenced to reclusion temporal in its medium period. But applying the Indeterminate Sentence Law, each of the accused is sentenced to an indeterminate penalty of ten years of prision mayor, as minimum, to seventeen years and four months of reclusion temporal, as maximum. The accused are likewise sentenced to indemnify the heirs of the deceased Alfredo Valdez in the amount of TWELVE THOUSAND PESOS jointly and severally, and to pay the costs.

a2010

page 26

Prof.
the offense was committed in 1964. The defense refused to cross-examine witness Alcantara, asked respondent Court to strike off the testimony of Alcantara because it referred to an offense not mentioned in the information, and asked for a ruling by respondent Court on the prosecution's verbal motion to amend the information. - Respondent Judge required the prosecution and the defense to submit memoranda. The contested order of July 22, 1970, denying the prosecution's verbal motion to amend information on the ground that said amendment would prejudice the substantial rights of the accused was issued. ISSUE WON the respondent Court abused its discretion when it refused an amendment to the information to change the date of the alleged commission of the offense from "August 1969" to "August 1964", on the ground it would constitute an impairment of the substantial rights of the accused as guaranteed by the Constitution. HELD NO Ratio While it has been held that except when time is a material ingredient of an offense, the precise time of commission need not be stated in the information, this Court stated that this does not mean that the prosecuting officer may be careless about fixing the date of the alleged crime, or that he may omit the date altogether, or that he may make the allegation so indefinite as to amount to the same thing. The prosecution is given the chance to allege an approximation of time of the commission of the offense and the precise date need not be stated but it does not mean that it can prove any date remote or far removed from the given approximate date so as to surprise and prejudice the accused. Reasoning - The period of almost five years between 1947 and 1952 covers such a long stretch of time that one cannot help but be led to believe that another theft different from that committed by the co-defendants in 1952 was also perpetrated by them in 1947. Under this impression the accused, who came to Court prepared to face a charge of theft of large cattle allegedly committed by them in 1952, were certainly caught by sudden surprise upon being confronted by evidence tending to prove a similar offense committed in 1947. The variance is certainly unfair to them, for it violates their constitutional rights to be informed before the trial of the specific charge

PEOPLE v REYES 108 SCRA 203 CONCEPCION, JR; October 23, 1981
NATURE Petition for certiorari with prayer for preliminary injunction on the order dated July 10, 1970, of the respondent Judge, (Hon. Alfredo C. Reyes of the Circuit Criminal Court, 4th Judicial District, Cabanatuan City) in Criminal Case No. CCC-IV-170NE, "People v Francisco Estrella," which denied petitioner's verbal motion for the amendment of the information in said case, by deleting the year "1969" as alleged therein, and in lieu thereof to put the year "1964 ". FACTS - Sometime in October, an information for qualified theft was filed against private respondent Francisco Estrella and three others, as Criminal Case No. 6799,

in the Municipal Court of San Jose, Nueva Ecija, pertinent portion as follows: That in the month of August, 1964, in the municipality of San Jose, province of Nueva Ecija, Philippines and within the jurisdiction of this Honorable Court, the above named accused Narciso Mananing being the driver of complainant Maria Ignacio- Francisco, Florentino Alcantara, repair shop owner where the truck hereinafter described was found and recovered, Francisco Estrella, a Philippine Constabulary soldier stationed at Bulacan, and Melecio Guevarra, all conspiring together, without the knowledge and consent of the owner thereof, take, steal and carry away one (1) Bedford truck with Chassis No. 153559, with Motor No. 2/54/5/6, with Plate No. T-35049, Series of 1964, to the damage and prejudice of the owner, Maria Ignacio-Francisco in the amount of P23,000.00, value of said vehicle. - On November 15, 1969, the Acting City Fiscal of San Jose City, (converted into city) Nueva Ecija, filed an information (Crim. Case No. CCC-IV-170) with the respondent Court, charging private respondent Francisco Estrella and three others, with qualified theft. This time the information contained Aug. 1969 instead of Aug. 1964 in the previous information and alleged grave abuse of confidence and that accused dismantled the vehicle. - On January 28, 1970, private respondent Francisco Estrella was arraigned, and he pleaded not guilty. During the arraignment, respondent-Judge required his clerk to read the information to Francisco Estrella. From January 28, 1970, up to May 21, 1970, the latter date being the scheduled trial of the case, the prosecution never moved to amend the information. - On May 21, 1970 when the prosecution was scheduled to present its evidence, it verbally moved that it be allowed to amend the information so as to change the date of the commission of the offense from "August 1969" to "August 1964." Private respondent Francisco Estrella, having come to the trial court ready to defend himself from an offense allegedly committed in "August 1969", vigorously objected to the verbal motion. - Respondent Judge withheld his ruling on the prosecution's motion to amend, and instead, required the prosecution to present its first witness, to determine whether the sought amendment in the information would constitute a change of substance affecting the rights of the accused or merely of form. - Florentino Alcantara, originally a co-accused but discharged as a prosecution witness, testified that

Criminal Procedure Rowena Daroy Morales


against them and deprives them of the opportunity to defend themselves. Moreover, they cannot be convicted of an offense of which they were not charged. (People v Opemia) Dispositive WHEREFORE, the questioned orders dated July 10, 1970 and September 14, 1970, by respondent Judge are hereby AFFIRMED, the preliminary injunction issued on September 24, 1970 dissolved, and this petition DISMISSED for lack of merit. Without costs. Voting Fernandez,* Abad Santos and De Castro, JJ., concur.

a2010

page 27
-

Prof.
As a result of a shooting incident, two informations for frustrated homicide were filed against Sixto Ruiz in the Court of First Instance of Rizal. In Criminal Case No. 4747, Ernesto Bello was named as the victim, while in Criminal Case No 4748, Rogelio Bello was the complainant. Upon arraignment, Sixto Ruiz pleaded not guilty to the two informations. A reinvestigation of these two cases was made in the Dept. of Justice, following which State Prosecutor filed a motion for leave of court to amend the informations on the ground that the evidence disclosed a prima facie case against Luis Padilla and Magsikap Ongchenco who acted in conspiracy with Ruiz. Ruiz filed his opposition to the motion. The trial Judge denied the motion to amend the information saying that allowance of the amendment alleging conspiracy would be amending the manner of committing the crime and thereby would constitute substantial amendment. As a consequence, State Prosecutor filed two new informations for frustrated homicide against Luis Padilla and Magsikap Ongchenco (Criminal Cases Nos. 9673 and 9674) alleging that the two conspired with Ruiz who was referred to as accused in Criminal Cases Nos. 4747 and 4748. Padilla and Ongchenco moved to quash the two new informations. The motion was denied by the lower court. Ruiz also filed in Criminal Cases Nos. 9673 and 9674 a motion to permit to quash and/or strike out the allegation of conspiracy in the two informations. The trial Judge ordered the striking out from the records the aforesaid motion and clarified that the allegation of conspiracy does not alter the theory of the case, nor does it introduce innovation nor does it present alternative imputation nor is it inconsistent with the with the original allegations. From these orders of the lower court, Ruiz, Padilla, and Ongchenco went to the CA on a petition for certiorari with preliminary injunction alleging that the trial Judge exceeded his jurisdiction or abused his judicial discretion in issuing the orders in Criminal Cases Nos. 9673 and 9674.

WON an information for the crime of homicide can be amended so as to charge the crime of murder after the accused had entered a plea of not guilty HELD NO. - The provision which is relevant to the problem is Rule 110, Sec. 13 of the Rules of Court - The petitioner invokes the first paragraph, whereas the respondent relies on the second. - To amend the information so as to change the crime charged from homicide to the more serious offense of murder after the petitioner had pleaded not guilty to the former is indubitably proscribed by the first paragraph of the above-quoted provision. For certainly a change from homicide to murder is not a matter of form; it is one of substance with very serious consequences. - Can the amendment be justified under the second paragraph? The answer is, No. For the provision speaks not of amendment but of dismissal of the information. in other words the provision contemplates the filing of a substitute, not an amended information - Can not the information for homicide against the petitioner be dismissed since no judgment has yet been rendered and another information for murder be filed? The answer, again, is, No. For the petitioner having pleaded not guilty to homicide, to dismiss the charge against him so as to file another charge for murder will place him thereby in double jeopardy. Aquino concur: - respondent Judge relied on Dimalibot vs. Salcedo - The Dimalibot case is different from the instant case. The plea in the Dimalibot case was made during the preliminary investigation to a complaint for homicide filed in the justice of the peace court. That is not the plea contemplated in Section 9, Rule 117 of the Rules of Court. The plea in the instant case was made to an information filed in the Court of First Instance.

SEPARATE OPINION BARREDO [concur]


I concur, but I believe this decision cannot bar another prosecution of private respondent under another information charging theft committed in 1964.

DIONALDO v DACUYCUY 108 SCRA 736 ABAD SANTOS; October 30, 1981
NATURE Petition to nullify orders of respondent judge FACTS -Petitioner Rolando Dionaldo stands charged with the crime of homicide. After he entered a plea of not guilty, the prosecution filed a motion for leave to amend the information, attaching thereto an amended information charging the accused with murder qualified by treachery and evident premeditation-a more serious offense. -No explanation was given in the motion for alleging evident premeditation but as to the allegation of treachery it was explained that, "the affidavit of the complaining witness indicates that the attack was sudden and it was only after they sustained the wounds consequent to the treacherous attack that they were forced to fight back to repel further aggression." It can thus be seen that all along this claimed circumstance was known to the prosecution but it was not alleged. -Counsel for the accused opposed the motion to amend the information but the respondent judge granted the motion ISSUE

PEOPLE v CA (RUIZ) 121 SCRA 733 RELOVA: April 28, 1983


NATURE Petition for certiorari with preliminary injunction to review the decision and resolution of the CA FACTS -

Criminal Procedure Rowena Daroy Morales


CA granted petition. The motion for reconsideration by herein petitioners to the foregoing decision of the CA was denied for lack of merit.

a2010

page 28

Prof.
merely states with additional precision something which is already contained in the original information, and which, therefore, adds nothing essential for conviction for the crime charged is an amendment to form that can be made at anytime. - The proposed amendments in the amended information, in the instant case, are clearly substantial and have the effect of changing the crime charged from "Robbery" punishable under Article 209 to "Robbery in an Uninhabited Place" punishable under Art. 302 of the Revised Penal Code, thereby exposing the private respondents-accused to a higher penalty as compared to the penalty imposable for the offense charged in the original information to which the accused had already entered a plea of "not guilty" during their arraignment. - Moreover, the change in the items, articles and pieces of jewelry allegedly stolen into entirely different articles from those originally complained of, affects the essence of the imputed crime, and would deprive the accused of the opportunity to meet all the allegations in the amended information, in the preparation of their defenses to the charge filed against them. It will be observed that private respondents were accused as accessories-after-thefact of the minor Ricardo Cabaloza who had already been convicted of robbery of the items listed in the original information. To charge them now as accessories-after-the-fact for a crime different from that committed by the principal, would be manifestly incongruous as to be allowed by the Court. - The allegation of conspiracy among all the private respondents-accused, which was not previously included in the original information, is likewise a substantial amendment saddling the respondents with the need of a new defense in order to meet a different situation in the trial court. To allow at this stage the proposed amendment alleging conspiracy among all the accused, will make all of the latter liable not only for their own individual transgressions or acts but also for the acts of their co-conspirators. Dispositive Petition is DISMISSED. Orders of the respondent court AFFIRMED. TRO lifted.

ISSUE WON the CA erred in granting the petition of (Ruiz, Padilla, and Ongchenco) HELD YES -

First and foremost, the trial Judge should have allowed the amendment in Criminal Cases Nos. 4747 and 4748 considering that the amendments sought were only formal. The amendments of Criminal Cases Nos. 4747 and 4748 would not have prejudiced Ruiz whose participation as principal in the crimes charged did not change. But the fact that the trial court erred in denying the motion of the prosecution to amend the informations in Criminal Cases Nos. 4747 and 4748 was no bar to the filing of the new informations. The allegation in Criminal Cases Nos. 9673 and 9674 filed against Padilla and Ongchenco that the two conspired and confederated with Ruiz merely describe the fact that the latter was already charged with the same offense. It is incorrect to say that the allegations of conspiracy include Ruiz as a defendant in the said case. Thus, he cannot file a motion to quash the same. Dispositive Decision and resolution of the CA are SET ASIDE. Decisions of lower court allowing retention of the allegation of conspiracy and the reference to Criminal Cases Nos. 4747 and 4748 in the informations filed in Criminal Cases Nos. 9673 and 9674 are SUSTAINED.

Antonio Cimarra, Ulpiano Villar, Bayani Catindig and Avelino de Leon. Said accused (now private respondents) were all members of the police force of Quezon City and were charged as accessories-afterthe-fact in the robbery committed by the minor Ricardo Cabaloza, who had already pleaded guilty and had been convicted in a crimial case before the Juvenile and Domestic Relations Court of Quezon City. Ricardo Cabaloza was convicted for the robbery of the same items, articles and pieces of jewelry belonging to Ding Velayo, Inc. valued at P75,591.40. - Upon arraignment, all of the accused (now private respondents) entered a plea of "not guilty" to the charge filed against them. - However, before the trial could proceed, the prosecuting fiscal filed a Motion to Admit Amended Information seeking to amend the original information by: (1) changing the offense charged from "Robbery" to "Robbery in an Uninhabited Place," (2) alleging conspiracy among all the accused, and (3) deleting all items, articles and pieces of jewelry alleged to have been stolen in the original Information and substituting them with a different set of items valued at P71,336.80. - Private respondents opposed the admission of the Amended Information. The respondent court resolved to deny the proposed amendments contained in the Amended Information. Petitioner moved for reconsideration of the aforesaid order but the respondent court denied said motion; hence, this petition. ISSUE WON the amended information should be admitted HELD - Amendment of an information under Sec. 14, Rule 110 of the 1985 Rules on Criminal Procedure (formerly, Section 13, Rule 110 of the old Rules on Criminal Procedure) may be made at any time before the accused enters a plea to the charge. Thereafter and during the trial, amendments to the information may also be allowed, as to matters of form, provided that no prejudice is caused to the rights of the accused. - The test as to when the rights of an accused are prejudiced by the amendment of a complaint or information is when a defense under the complaint or information, as it originally stood, would no longer be available after the amendment is made, and when any evidence the accused might have, would be inapplicable to the complaint or information as amended. On the other hand, an amendment which

PEOPLE v MONTENEGRO 159 SCRA 236 PADILLA; March 25, 1988


NATURE Petition for certiorari with preliminary injunction and/or restraining order FACTS - The City Fiscal of Quezon City, thru Assistant Fiscal Virginia G. Valdez, filed an Information for "Robbery" before the Court of First Instance of Rizal against

GARCIA v FLORIDO 52 SCRA 420 ANTONIO; August 31, 1973


NATURE Appeal by certiorari from the decision of the Court of First Instance of Misamis Occidental, dismissing petitioners' action for damages against respondents,

Criminal Procedure Rowena Daroy Morales


Mactan Transit Co., Inc. and Pedro Tumala, "without prejudice to refiling the said civil action after conviction of the defendants in the criminal case filed by the Chief of Police of Sindangan, Zamboanga del Norte", and from the order of said Court dated January 21, 1972, denying petitioners' motion for reconsideration. FACTS - On August 4, 1971, petitioners, German C. Garcia, Chief of the Misamis Occidental Hospital, together with his wife, Luminosa L. Garcia, and Ester Francisco, bookkeeper of said hospital, hired and boarded a PU car with plate No. 241-8 G Ozamis 71 owned and operated by respondent, Marcelino Inesin, and driven by respondent, Ricardo Vayson, for a roundtrip from Oroquieta City to Zamboanga City, for the purpose of attending a conference of chiefs of government hospitals, hospital administrative officers, and bookkeepers of Regional Health Office No. 7 at Zamboanga City. - At about 9:30 a.m., while the PU car was negotiating a slight curve on the national highway at kilometer 21 in Barrio Guisukan, Sindangan, Zamboanga del Norte, said car collided with an oncoming passenger bus (No. 25) with plate No. 77-4 W Z.N. 71 owned and operated by the Mactan Transit Co., Inc. and driven by defendant, Pedro Tumala. As a result of the aforesaid collision, petitioners sustained various physical injuries which necessitated their medical treatment and hospitalization. - Alleging that both drivers of the PU car and the passenger bus were at the time of the accident driving their respective vehicles at a fast clip, in a reckless, grossly negligent and imprudent manner in gross violation of traffic rules and without due regard to the safety of the passengers aboard the PU car, petitioners, German C. Garcia, Luminosa L. Garcia, and Ester Francisco, filed on September 1, 1971 with respondent Court of First Instance of Misamis Occidental an action for damages (Civil Case No. 2850) against the private respondents, owners and drivers, respectively, of the PU car and the passenger bus that figured in the collision, with prayer for preliminary attachment. - The principal argument advanced by Mactan Inc. et. al to in a motion to dismiss was that the petitioners had no cause of action for on August 11, 1971, or 20 days before the filing of the present action for damages, respondent Pedro Tumala was charged in Criminal Case No. 4960 of the Municipal Court of Sindangan, Zamboanga del Norte, in a complaint

a2010

page 29

Prof.
effect abandoned their right to press recovery for damages in the criminal case, and have opted instead to recover them in the present civil case. - As a result of this action of petitioners the civil liability of private respondents to the former has ceased to be involved in the criminal action. Undoubtedly an offended party loses his right to intervene in the prosecution of a criminal case, not only when he has waived the civil action or expressly reserved his right to institute, but also when he has actually instituted the civil action. For by either of such actions his interest in the criminal case has disappeared. - As we have stated at the outset, the same negligent act causing damages may produce a civil liability arising from crime or create an action for quasi-delict or culpa extracontractual. The former is a violation of the criminal law, while the latter is a distinct and independent negligence, having always had its own foundation and individuality. Some legal writers are of the view that in accordance with Article 31, the civil action based upon quasi-delict may proceed independently of the criminal proceeding for criminal negligence and regardless of the result of the latter. Hence, "the proviso in Section 2 of Rule 111 with reference to . . . Articles 32, 33 and 34 of the Civil Code is contrary to the letter and spirit of the said articles, for these articles were drafted . . . and are intended to constitute as exceptions to the general rule stated in what is now Section 1 of Rule 111. The proviso, which is procedural, may also be regarded as an unauthorized amendment of substantive law, Articles 32, 33 and 34 of the Civil Code, which do not provide for the reservation required in the proviso." - But in whatever way We view the institution of the civil action for recovery of damages under quasidelict by petitioners, whether as one that should be governed by the provisions of Section 2 of Rule 111 of the Rules which require reservation by the injured party considering that by the institution of the civil action even before the commencement of the trial of the criminal case, petitioners have thereby foreclosed their right to intervene therein, or one where reservation to file the civil action need not be made, for the reason that the law itself (Article 33 of the Civil Code) already makes the reservation and the failure of the offended party to do so does not bar him from bringing the action, under the peculiar circumstances of the case, We find no legal justification for respondent court's order of dismissal. 2. YES, because the action in fact satisfies the elements of quasi-delict.

filed by the Chief of Police and that, with the filing of the aforesaid criminal case, no civil action could be filed subsequent thereto unless the criminal case has been finally adjudicated, pursuant to Sec. 3 of Rule 111 of the Rules of Court, and, therefore, the filing of the instant civil action is premature, because the liability of the employer is merely subsidiary and does not arise until after final judgment has been rendered finding the driver, Pedro Tumala, guilty of negligence; that Art. 33 of the New Civil Code, is not applicable because Art 33 applied only to the crimes of physical injuries or homicide, not to the negligent act or imprudence of the driver. - The lower court sustained Mactan Inc. et. Al. and dismissed the complaint ISSUES

1.

WON the lower court erred in dismissing the complaint for damages on the ground that since no express reservation was made by the complainants, the civil aspect of the criminal case would have to be determined only after the termination of the criminal case WON the lower court erred in saying that the action is not based on quasi-delict since the allegations of the complaint in culpa aquiliana must not be tainted by any assertion of violation of law or traffic rules or regulations and because of the prayer in the complaint asking the Court to declare the defendants jointly and severally liable for moral, compensatory and exemplary damages

2.

HELD 1. YES Ratio An action based on quasi-delict may be maintained independently from a criminal action. By instituting a civil action based on a quasi-delict, a complainant may be deemed to abandon his/her right to press recovery for damages in the criminal case. Reasoning - In the case at bar, there is no question that petitioners never intervened in the criminal action instituted by the Chief of Police against respondent Pedro Tumala, much less has the said criminal action been terminated either by conviction or acquittal of said accused. - It is, therefore, evident that by the institution of the present civil action for damages, petitioners have in

Criminal Procedure Rowena Daroy Morales


Ratio An action shall be deemed to be based on a quasi-delict when all the essential averments under Articles 2176-2194 of the New Civil Code are present, namely: (a) act or omission of the private respondents; (b) presence of fault or negligence or the lack of due care in the operation of the passenger bus No. 25 by respondent Pedro Tumala resulting in the collision of the bus with the passenger car; (c) physical injuries and other damages sustained by petitioners as a result of the collision; (d) existence of direct causal connection between the damage or prejudice and the fault or negligence of private respondents; and (e) the absence of pre-existing contractual relations between the parties. Reasoning - The circumstance that the complaint alleged that respondents violated traffic rules in that the driver drove the vehicle "at a fast clip in a reckless, grossly negligent and imprudent manner in violation of traffic rules and without due regard to the safety of the passengers aboard the PU car" does not detract from the nature and character of the action, as one based on culpa aquiliana. The violation of traffic rules is merely descriptive of the failure of said driver to observe for the protection of the interests of others, that degree of care, precaution and vigilance which the circumstances justly demand, which failure resulted in the injury on petitioners. Certainly excessive speed in violation of traffic rules is a clear indication of negligence. Since the same negligent act resulted in the filing of the criminal action by the Chief of Police with the Municipal Court (Criminal Case No. 4960) and the civil action by petitioners, it is inevitable that the averments on the drivers' negligence in both complaints would substantially be the same. It should be emphasized that the same negligent act causing damages may produce a civil liability arising from a crime under Art. 100 of the Revised Penal Code or create an action for quasidelict or culpa extra-contractual under Arts. 21762194 of the New Civil Code. This distinction has been amply explained in Barredo vs. Garcia, et all (73 Phil. 607, 620-621). - It is true that under Sec. 2 in relation to Sec. 1 of Rule 111 of the Revised Rules of Court which became effective on January 1, 1964, in the cases provided for by Articles 31, 33, 39 and 2177 of the Civil Code, an independent civil action entirely separate and distinct from the civil action, may be instituted by the injured party during the pendency of the criminal case, provided said party has reserved his right to institute it separately, but it should be noted,

a2010

page 30

Prof.
before the RTC, Branch 12, seeking to set aside the MTC order of suspension. The petition was docketed as Civil Case No. CEB-26195. - Respondents filed a motion to dismiss the petition on the ground that the petition was filed by the private complainant, instead of the government prosecutor representing the People of the Philippines in criminal cases. RTC dismissed the petition for lack of conformity or signature of the government prosecutor. Petitioner moved MFR but was denied. From these orders, petitioner filed the instant petition for review. Petitioners Claim That a person aggrieved may file a special civil action for certiorari and that person includes the complainant or the offended party. A special action on an order issued by a lower court in a criminal case may be filed by the private offended party. Respondentss Comment In all criminal cases, all initiatory pleadings, as well as subsequent proceedings, must be initiated by the government counsel because the injured party is the People of the Philippines and the private complainant is a mere witness to the offense allegedly committed by the accused. People v. Dacudao and Metropolitan Bank and Trust Company v. Veridiano II apply, such that a private prosecutor in a criminal case has no authority to act for the People of the Philippines. It is the governments counsel, the Sol-Gen, who appears in criminal cases or incidents before SC. ISSUE WON a private offended party in a criminal proceeding may file a special civil action for certiorari under Rule 65, assailing an interlocutory order, without the conformity of the public prosecutor HELD YES Ratio If criminal case is dismissed by the trial court or if there is an acquittal, the appeal on the criminal aspect of the case must be instituted by the Solicitor General in behalf of the State. The capability of the private complainant to question such dismissal or acquittal is limited only to the civil aspect of the case. (Metrobank v. Veridiano II). But if the order which is assailed is not one dismissing the case or acquitting respondents / defendants, there is no limitation to the capacity of the private complainant to seek judicial review of the assailed order. Reasoning - [1] A special civil action for certiorari may be filed by an aggrieved party alleging grave abuse of

however, that neither Section 1 nor Section 2 of Rule 111 fixes a time limit when such reservation shall be made.

SEPARATE OPINION BARREDO [concur]


- I would like to limit my concurrence. - Article 2176 and 2177 definitely create a civil liability distinct and different from the civil action arising from the offense of negligence under the Revised Penal Code. Since Civil Case No. 2850 is predicated on the above civil code articles and not on the civil liability imposed by the Revised Penal Code, I cannot see why a reservation had to be made in the criminal case. As to the specific mention of Article 2177 in Section 2 of the Rule 111, it is my considered view that the latter provision is inoperative, it being substantive in character and is not within the power of the Supreme Court to promulgate, and even if it were not substantive but adjective, it cannot stand because of its inconsistency with Article 2177, an enactment of the legislature superseding the Rules of 1940. - Besides, the actual filing of Civil Case No. 2850 should be deemed as the reservation required, there being no showing that prejudice could be caused by doing so. - Accordingly, I concur in the judgment reversing the order of dismissal of the trial court in order that Civil Case No. 2850 may proceed, subject to the limitation mentioned in the last sentence of Article 2177 of the Civil Code, which means that of the two possible judgments, the injured party is entitled exclusively to the bigger one.

RODRIGUEZ v GADIANE 495 SCRA 368 TINGA; July 17, 2006


NATURE Petition for review on certiorari FACTS - Thomasita Rodriguez (petitioner) was the private complainant in a criminal case filed against Rolando Gadiane and Ricardo Rafols, Jr. (respondents), for violation of B.P. 22. The MTC hearing the complaint had suspended the criminal proceeding on the ground that a prejudicial question was posed in a separate civil case then pending. On 28 Feb. 2001, petitioner filed a petition for certiorari under Rule 65

Criminal Procedure Rowena Daroy Morales


discretion amounting to excess or lack of jurisdiction on the part of the trial court. In a long line of cases, this Court construed the term aggrieved parties to include the State and the private offended party or complainant. The complainant has an interest in the civil aspect of the case so he may file such special civil action questioning the decision or action of the respondent court on jurisdictional grounds. In so doing, complainant should not bring the action in the name of the People of the Philippines. The action may be prosecuted in name of said complainant. - [2] In this case, there is no doubt that petitioner maintains an interest in the litigation of the civil aspect of the case against respondents. Section 1(b), Rule 111 of 2000 Rules of Criminal Procedure states that the criminal action for violation of B.P. 22 shall be deemed to include the corresponding civil action. Hence, the possible conviction of respondents would concurrently provide a judgment for damages in favor of petitioner. The suspension of the criminal case which petitioner decries would necessarily cause delay in the resolution of the civil aspect of the said case which precisely is the interest and concern of petitioner. Such interest warrants protection from the courts. Dispositive: Petition is GRANTED. The assailed orders of RTC are SET ASIDE. Civil Case No. CEB26195 is REINSTATED

a2010

page 31

Prof.
-Private respondents contend that since petitioner did not comply with the agreement, he was not entitled to the 10% rebate in price, and as a consequence, the previous payments made by petitioner did not amount to full payment as required for all the lots and which would have entitled petitioner to the issuance and delivery of the certificates of title to all the lots. -On 26 July 1984, petitioner, filed a complaint for specific performance with damages, with the Regional Trial Court of Angeles City, Branch LX, docketed as Civil Case No. 4224. In his complaint, petitioner prayed, among others, that judgment be rendered ordering private respondents to deliver to him the transfer certificates of title covering the three (3) lots which he had allegedly fully paid for, and which private respondents had refused to do so. Also, it was prayed that judgment be rendered ordering the private respondents to jointly and severally pay the petitioner, actual damages equal to P320,000.00, representing unrealized gross profits; moral damages at the discretion of the court; and, attorney's fees equal to P15,000.00, plus the costs of the action. -Before the civil action was filed, petitioner also filed on 5 June 1984 with the City Fiscal of Angeles City a criminal complaint against herein respondent Manuel Lazatin, for violation of Presidential Decree No. 957, specifically Section 25 thereof, which provides: "PRESIDENTIAL DECREE NO. 957 REGULATING THE SALE OF SUBDIVISIONS LOTS AND CONDOMINIUMS, PROVIDING PENALTIES FOR VIOLATIONS THEREOF. SEC. 25. Issuance of Title. The owner or developer shall deliver the title of the lot or unit to the buyer upon full payment of the lot or unit. No fee, except those required for the registration of the deed of sale in the Registry of Deeds shall be collected for the issuance of such title. In the event a mortgage over the lot or unit is outstanding at the time of the issuance of the title to the buyer, the owner or developer shall redeem the mortgage or the corresponding portion thereof within six months such issuance in order that the title over any fully paid lot or unit may be secured and delivered to the buyer in accordance herewith. SEC. 39. Penalties. Any person who shall violate any of the provisions of this Decree and/or any rule or regulation that may be issued pursuant to this Decree, shall, upon conviction, be punished by a fine of not more than twenty thousand (P20,000.00) pesos and/or imprisonment of not more than ten years: Provided, that in the case of

NAGUIAT v IAC (TIMOG SILANGAN DEVELOPMENT CORP) 164 SCRA 505 PADILLA; August 18, 1988
NATURE Petition to review on certiorari the decision of the Intermediate Appellate Court FACTS -Timog Silangan Development Corporation (TSDC, for short) is a domestic corporation engaged in the business of developing and selling subdivision lots in

"Timog Park," located in Angeles City, with Manuel P. Lazatin (Lazatin, for short) as its President. - Antolin T. Naguiat purchased, on installment basis, four (4) lots from TSDC, identified as Lots Nos. 13, 14, 15 and 16, of Block 26 of Timog Park. Each lot consists of 300 square meters. The four (4) lots have a total area of 1,200 square meters, with a price of P60.00 per square meter. On the same date (7 February 1983) petitioner made a down payment of P7,200.00, representing 10% of the alleged total price of P72,000.00 for the four (4) lots. A corresponding receipt for the downpayment was issued by TSDC to the petitioner. While the Contract to Sell between TSDC and the petitioner stipulated a two-year period within which to pay the total contract price, the latter made substantial payments in the months of June to August 1983. On 10 August 1983, he paid the sum of P12,529.30 as his alleged full payment for Lot. No. 16, after which, TSDC caused to be issued in the name of the petitioner the title to said lot. On 7 November 1983, petitioner paid TSDC the amount of P36,067.97, which was allegedly his full payment for the remaining three (3) Lots, namely, Lots Nos. 13, 14 and 15. A corresponding receipt for said amount was also issued by TSDC to the petitioner. -Thereafter, from December 1983 up to June 1984, petitioner demanded from TSDC the issuance in his favor of the certificates of title for the three (3) lots, last paid for, but the private respondents (TSDC and Lazatin) refused on the ground that the petitioner had not fully paid for said three (3) lots. -Sometime in January, 1983, TSDC's Board of Directors approved the petitioner's contemplated purchase of the aforesaid lots. To confirm the agreement, respondent Lazatin wrote petitioner a letter reiterating standard conditions of the sale, which the petitioner allegedly accepted by affixing his conformity to said letter. The conditions for the sale of the lots were among others, "(i) 10% down payment with a commitment to commence construction therefrom (thereon) in one month's time; (ii) said construction to be finished within a period of six (6) months; and, (iii) the effective price was P70 per square meter with a rebate of P10.00 per square meter upon completion of the house in six (6) months." But, as alleged by the private respondents, petitioner commenced the construction of a house on one lot but failed to finish it within the stipulated period of six (6) months. And as to the other lots, petitioner allegedly failed altogether to construct houses on them.

Criminal Procedure Rowena Daroy Morales


corporations, partnership, cooperatives, or associations, the President, Manager or Administrator or the person who has charge of the administration of the business shall be criminally responsible for any violation of this Decree and/or the rules and regulations promulgated pursuant thereto." -On 13 September 1984, an information was filed against respondent Lazatin. -Petitioner filed on 23 February 1985 a motion to consolidate Civil Case No. 4224 and Criminal Case No. 6727. Despite the objection and opposition of the private respondents, in an Order dated 20 March 1985, the trial court granted the motion and ordered consolidation of the two (2) cases. -At the pre-trial hearing of both cases, petitioners's counsel appeared as counsel for the plaintiff in Civil Case and as private prosecutor in the Criminal Case. Private respondents objected, and filed their Motion and Opposition to Appearance of Plaintiff as Private Prosecutor with respect to the trial of the Criminal Case; the opposition was overruled by the trial court. -Private respondents filed a petition for certiorari and prohibition with the respondent appellate court, seeking the annulment of the orders of the trial court, dated 20 March 1985 and 29 May 1985. In due course, the respondent appellate court rendered a decision favorable to herein private respondents. -The decision of the respondent appellate court was received by petitioner's counsel on 16 October 1985. On 30 October 1985, petitioner's counsel filed with the respondent appellate court a Motion for Extension of Time to file a motion for reconsideration of aforesaid decision, praying for fifteen (15) days from 31 October 1985, within which to file said motion. -On 15 November 1985, petitioner's counsel filed a Second Motion for Extension of Time to file a motion for reconsideration, praying for another fifteen (15) days from 15 November 1985, within which to file said motion for reconsideration. It was denied stating among others that the fifteen (5) days period to file a motion for reconsideration is non-extendible. -On 2 December 1985, petitioner's counsel still filed his motion for reconsideration it was also denied. ISSUES 1. WON no motion for extension of time to file a motion for new trial or reconsideration may be filed with the Metropolitan or Municipal Trial Courts, the Regional Trial Courts, and the Intermediate Appellate Court as applied in the Habaluyas rule

a2010

page 32

Prof.
the same, which are, the Contract to Sell, the letter which contains the conditions for the purchase of the lots and, to which petitioner allegedly affixed his conformity, the official receipts for the alleged payments made by the petitioner, and other related documents. - Based on the foregoing, and considering that the criminal action filed is one for violation of a special law where, irrespective of the motives, mere commission of the act prohibited by said special law, constitutes the offense, then the intervention of the petitioner's counsel, as private prosecutor in the criminal action, will not prejudice the substantial rights of the accused. - The consolidation of the two cases in question, where petitioner's counsel may act as counsel for the plaintiff in the civil case and private prosecutor in the criminal case, will instead be conducive to the early termination of the two (2) cases, and will redound to the benefit and convenience of the parties; as well as to the speedy administration of justice. - As a ground for the consolidation of the criminal and civil cases, petitioner invokes Rule 111, Sec. 3(a), Rules of Court, which provides: "Sec. 3. Other civil actions arising from offenses. Whenever the offended party shall have instituted the civil action to enforce the civil liability arising from the offense, as contemplated in the first paragraph of Section 1 hereof, the following rules shall be observed: (a) 'After a criminal action has been commenced, the pending civil action arising from the same offense shall be suspended, in whatever stage it may be found until final judgment in the criminal proceeding has been rendered. However, if no final judgment has been rendered by the trial court in the civil action, the same may be consolidated with the criminal action upon application with the court trying the criminal action. If the application is granted, the evidence presented and admitted in the civil action shall be deemed automatically reproduced in the criminal action, without prejudice to the admission of additional evidence that any party may wish to present. Under the aforequoted provision, the civil action that may be consolidated with a criminal action, is one for the recovery of civil liability arising from the criminal offense, or ex delicto. In the case at bar, the civil action filed by the petitioner was for specific performance with damages. The main relief sought in the latter case, i.e., the delivery of the certificates of title to the lots which petitioner had allegedly fully paid for, was grounded on the

2. WON the civil and criminal case should be consolidated HELD 1. NO - In the case at bar, the petitioner filed his motions for extension of time to file a motion for reconsideration on 30 October 1985 and 15 November 1985, both within the periods sought to be extended. Hence the Habaluyas ruling did not yet apply to bar said motions for extension. As admitted by petitioner himself, he filed with the respondent appellate court two (2) motions for extension of time to file motion for reconsideration of the latter court's decision, with the justification that the two (2) motions were timely and properly presented, since they were filed before the expiration of the respective periods sought to be extended. - The case of Habaluyas Enterprises, Inc. v. Japzon, has ruled that: "Beginning one month after the promulgation of this Resolution, the rule shall be strictly enforced that no motion for extension of time to file a motion for new trial or reconsideration may be filed with the Metropolitan or Municipal Trial Courts, the Regional Trial Courts, and the Intermediate Appellate Court." - Based on the aforequoted ruling of the Habaluyas case, motions for extension of time to file a motion for new trial or reconsideration may no longer be filed before all courts, lower than the Supreme Court. The rule in Habaluyas applies even if the motion is filed before the expiration of the period sought to be extended, because the fifteen (15) day period for filing a motion for new trial or reconsideration with said courts, is non-extendible. But as resolved also in the Habaluyas case, the rule that no motion for extension of time to file a motion for new trial or reconsideration may be filed with the Metropolitan or Municipal Trial Courts, the Regional Trial Courts, and the Intermediate Appellate Court, shall be strictly enforced "beginning one month after the promulgation of this Resolution." The Court promulgated the Habaluyas resolution on 30 May 1986. Thus, the Habaluyas ruling became effective, and strictly enforced, only beginning 1 July 1986. 2. YES - In the cases at bar, the nature of the issues involved, at least, the factual issues in the civil and criminal actions are almost identical, i.e., whether or not petitioner had fully paid for the lots he purchased from the private respondents, so as to entitle him to the delivery of the certificates of title to said lots. The evidence in both cases, likewise, would virtually be

Criminal Procedure Rowena Daroy Morales


Contract to Sell between the petitioner and the private respondent. Hence the civil action filed by the petitioner was for the enforcement of an obligation arising from a contract, or ex contractu, and not one for the recovery of civil liability arising from an offense; hence, the law invoked by the petitioner is inapplicable. - But, as held in Canos v. Peralta, the consolidation of a criminal action with a civil action arising not ex delicto, may still be done, based upon the express authority of Section 1, Rule 31 of the Rules of Court, which provides: "Section 1. Consolidation. When actions involving a common question of law or fact are pending before the court, it may order a joint hearing or trial of any or all the matters in issue in the actions; it may order all the actions consolidated; and it may make such orders concerning proceedings therein as may tend to avoid unnecessary costs or delay." - In Canos v. Peralta, where the Court sustained the order of a trial court to consolidate a civil action (an action for the recovery of wage differential, overtime and termination pay, plus damages) with a criminal action (for violation of the Minimum Wage Law), it was held that: "A Court may order several actions pending before it to be tried together where they arise from the same act, event or transaction, involve the same or like issues, and depend largely or substantially on the same evidence, provided that the court has jurisdiction over the cases to be consolidated and that a joint trial will not give one party an undue advantage or prejudice the substantial rights of any of the parties . . . - The obvious purpose of the above rule is to avoid multiplicity of suits, to guard against oppression and abuse, to prevent delays, to clear congested dockets, to simplify the work of the trial court; in short the attainment of justice with the least expense and vexation to the parties litigants." Dispositive WHEREFORE, the petition is GRANTED. The decision of the respondent appellate court, dated 9 October 1985, is SET ASIDE. The Orders of the trial court, in Civil Case No. 4224 and Criminal Case No. 6727, dated 20 March 1985 and 29 May 1985 are REINSTATED. SO ORDERED.

a2010
REYES v SEMPIO-DIY 141 SCRA 208 PATAJO; January 29, 1986

page 33

Prof.
reservation therefore (Judgment in such a proceeding becomes binding as res judicata, according to Roa v Dela Cruz. Thus, plaintiff is barred). ISSUE WON intervention of private prosecution and failure to make a reservation bars plaintiff from filing a separate civil action for damages against the accused HELD NO, plaintiff is not barred. Ratio The mere appearance of a private prosecutor in the criminal case does not necessarily constitute such intervention on the part of the aggrieved party as could only import an intention on her part to press her claim for damages in said criminal case and a waiver of her right to file a separate civil action for damages. Reasoning - In the Roa case, there was a full-blown hearing where a private prosecutor participated actively and there could be no question that the aggrieved party's participation through the private prosecutor in said case clearly indicated her intention to have her claim for damages litigated in the criminal action against the accused. It was only after the trial of the case on the merits that a decision was rendered finding the accused guilty of slight slander and sentencing her to pay a fine of P50.00 but making no award of damages in favor of the aggrieved party. The reason for the Court's not making any award of damages is because of the failure of the aggrieved party to submit evidence to support her claim for damages. - In the present case, while it is true that Reyes was represented by a private prosecutor for the purpose of proving damages, the unexpected plea of guilt by the accused and her being sentenced immediately to a fine of P50.00 prevented petitioner from proving her claim for damages and making a reservation to file a separate civil action. Controlling case should be Meneses v Luat, and not Roa v Dela Cruz. In the Roa case, not only was the offended party represented by a private prosecutor in the criminal action, the action went through trial on the merits. In the Luat case, defendant Luat did not proceed to trial, as he pleaded guilty upon arraignment. The mere appearance of private counsel in representation of the offended party did not constitute such active intervention as could only import an intention to press a claim for damages in the same action.

NATURE Direct appeal on a question of law from a resolution of the Regional Trial Court (Malabon). FACTS - MTC (Navotas): Cristina Malicsi was charged with the crime of intriguing against honor. Zenaida Cruz Reyes (petitioner) was the aggrieved party. In said criminal case, Reyes was represented by a private prosecutor named Atty. Barayang. - Malicsi pleaded guilty to the information and was sentenced to pay P50. Because of her plea of guilty, the aggrieved party was unable to present evidence to prove damages against the accused. Reyes was not able to make a reservation of her right to file a separate civil action for damages. - Instead, she filed a new action against Cristina Malicsi and her husband with the Regional Trial Court for damages arising from defamatory words which were the subject of the information in the Criminal action. - At the pre-trial plaintiff admitted that she was represented by a private prosecutor in the criminal case against defendant Cristina Malicsi and in said case she did not reserve the right to file a separate action for damages. There was also admission that the private prosecutor was for proving damages against the accused. - The issue in the RTC was WON the plaintiff, represented by a private prosecutor and the failing to make a reservation to file a separate action, was barred from filing a separate civil action for damages against the accused Cristina Malicsi. RTC ruled in favor of the defendant. - RTC: There is no question that in defamation cases (such as the present) as in cases of fraud and physical injuries, a civil action for damages entirely separate and distinct from the criminal action may be brought by the injured party, and such action shall proceed independently of the criminal prosecution and shall require only a preponderance of evidence. An exception to the above rule; when the offended party actually intervenes in the criminal action by appearing therein through a private prosecutor for the purpose of recovering indemnity for damages, he is deemed to have waived his right to file a separate civil action for damages if he failed to make a

CORPUS v PAJE BORDAS v CANADALLA

Criminal Procedure Rowena Daroy Morales


- The failure of petitioner to make a reservation to file a separate civil action did not foreclose her right to file said separate complaint for damages. Under Article 33 of the Civil Code there is no requirement that as a condition to the filing of a separate civil action for damages a reservation to file said civil action be first made in the criminal case and such reservation is not necessary, the provision of Rule 111, Section 2 notwithstanding. Dispositive Petition is granted.

a2010

page 34

Prof.
failed to file MR within the reglementary period. Plaintiffs claimed that the motion to set aside the order of November 8, 1983 and the amplificatory motion for reconsideration was filed for all the plaintiffs, although signed by only some of the lawyers. -September 21, 1984: RTC issued order dealing with both motions (1) to reconsider its order of May 11, 1984 declaring that with respect to certain plaintiffs, the resolution of November 8, 1983 had already become final, and (2) to set aside its resolution of November 8, 1983 granting the defendants' motion to dismiss. In effect, the case against the defendants (except for Major Rodolfo Aguinaldo, and Master Sgt. Bienvenido Balabaere) was dismissed. -March 15, 1985: petitioners (plaintiffs below) filed the instant petition for certiorari seeking to annul and set aside RTC's resolution of November 8, 1983, its order of May 11, 1984, and its resolution dated September 21, 1984. -Respondents filed comment on the petition, November 9, 1985. -A reply was filed by petitioners on August 26, 1986. ISSUES 1. WON the suspension of the privilege of the writ of habeas corpus bars a civil action for damages for illegal searches conducted by military personnel and other violations of rights and liberties guaranteed under the Constitution 2. WON a superior officer under the notion of respondent superior be answerable for damages, jointly and severally with his subordinates, to the person whose constitutional rights and liberties have been violated 3. WON RTC was correct in dismissing the complaint with respect to plaintiffs Rogelio Aberca, Danilo de la Puente, Marco Palo, Alan Jazminez, Alex Marcelino, Elizabeth Protacio-Marcelino, Alfredo Mansos and Rolando Salutin, on the basis of the alleged failure of said plaintiffs to file a motion for reconsideration of the court's resolution of November 8, 1983, granting the respondent's motion to dismiss HELD 1. NO - The suspension of the privilege of the writ of habeas corpus does not destroy petitioners' right and cause of action for damages for illegal arrest and detention and other violations of their constitutional rights. The suspension does not render valid an otherwise illegal arrest or detention. What is suspended is merely the right of the individual to

ABERCA v VER 160 SCRA 590 YAP; April 15, 1988


NATURE Petition for certiorari FACTS -This case stems from alleged illegal searches and seizures and other violations of the rights and liberties of plaintiffs by various intelligence units of the AFP, known as Task Force Makabansa (TFM) ordered by General Fabian Ver "to conduct preemptive strikes against known communist-terrorist (CT) underground houses in view of increasing reports about CT plans to sow disturbances in Metro Manila," -Plaintiffs allege, among others, that complying with said order, elements of the TFM raided several places, employing in most cases defectively issued judicial search warrants; that during these raids, certain members of the raiding party confiscated a number of purely personal items belonging to plaintiffs; that plaintiffs were arrested without proper warrants issued by the courts; that for some period after their arrest, they were denied visits of relatives and lawyers; that plaintiffs were interrogated in violation of their rights to silence and counsel; that military men who interrogated them employed threats, tortures and other forms of violence on them in order to obtain incriminatory information or confessions and in order to punish them; that all violations of plaintiffs constitutional rights were part of a concerted and deliberate plan to forcibly extract information and incriminatory statements from plaintiffs and to terrorize, harass and punish them, said plans being previously known to and sanctioned by defendants. -Plaintiffs sought damages (actual/compensatory, moral, exemplary), and attorney's fees.

-Defendants filed motion to dismiss alleging that (1) plaintiffs may not cause a judicial inquiry into the circumstances of their detention in the guise of a damage suit because, as to them, the privilege of the writ of habeas corpus is suspended; (2) assuming that the courts can entertain the present action, defendants are immune from liability for acts done in the performance of their official duties; and (3) the complaint states no cause of action against the defendants. -Plaintiffs filed two separate oppositions to the motion to dismiss. -Defendants filed a Consolidated Reply. -RTC NCR Branch 95 Judge Willelmo C. Fortun issued a resolution granting the motion to dismiss. "After a careful study of defendants' arguments, the court finds the same to be meritorious and must, therefore, be granted. On the other hand, plaintiffs' arguments in their opposition are lacking in merit." -Plaintiffs filed motion to set aside the order dismissing the complaint and a supplemental motion for reconsideration. -Defendants filed a comment on the aforesaid motion of plaintiffs, furnishing a copy thereof to the attorneys of all the plaintiffs. -December 15, 1983: Judge Fortun issued an order voluntarily inhibiting himself from further proceeding in the case and leaving the resolution of the motion to set aside the order of dismissal to Judge Lising, "to preclude any suspicion that he (Judge Fortun) cannot resolve [the] aforesaid pending motion with the cold neutrality of an impartial judge and to put an end to plaintiffs assertion that the undersigned has no authority or jurisdiction to resolve said pending motion." -Plaintiffs resolved an amplificatory motion for reconsideration signed in the name of the Free Legal Assistance Group (FLAG) of Mabini Legal Aid Committee. -May 2,1984: defendants filed a comment on said amplificatory motion for reconsideration. -May 11, 1984: RTC Judge Esteban Lising, without acting on the motion to set aside order of November 8, 1983, issued an order declaring that since certain plaintiffs failed to file a motion to reconsider the Order of November 8, 1983, dismissing the complaint, nor interposed an appeal therefrom within the reglementary period, as prayed for by the defendants, said Order is now final against said plaintiffs. -on May 28,1984: plaintiffs filed a motion for reconsideration, alleging that it was not true that the plaintiffs mentioned in the order of May 11, 1984

Criminal Procedure Rowena Daroy Morales


seek release from detention through the writ of habeas corpus as a speedy means of obtaining his liberty. - At the heart of petitioners' complaint is Article 32 of the Civil Code. It is obvious that the purpose of the above codal provision is to provide a sanction to the deeply cherished rights and freedoms enshrined in the Constitution. Its message is clear; no man may seek to violate those sacred rights with impunity. Certain basic rights and liberties are immutable and cannot be sacrificed to the transient needs or imperious demands of the ruling power. The rule of law must prevail, or else liberty will perish. Our commitment to democratic principles and to the rule of law compels us to reject the view which reduces law to nothing but the expression of the will of the predominant power in the community. Democracy cannot be a reign of progress, of liberty, of justice, unless the law is respected by him who makes it and by him for whom it is made. Now this respect implies a maximum of faith, a minimum of Idealism. On going to the bottom of the matter, we discover that life demands of us a certain residuum of sentiment which is not derived from reason, but which reason nevertheless controls. 2 - We find respondents' invocation of the doctrine of state immunity from suit totally misplaced. The cases invoked by respondents actually involved acts done by officers in the performance of official duties written the ambit of their powers. It may be that the respondents, as members of the AFP, were merely responding to their duty, as they claim, "to prevent or suppress lawless violence, insurrection, rebellion and subversion" in accordance with Proclamation No. 2054 of President Marcos, despite the lifting of martial law on January 27, 1981, and in pursuance of such objective, to launch pre-emptive strikes against alleged communist terrorist underground houses. But this cannot be construed as a blanket license or a roving commission untramelled by any constitutional restraint, to disregard or transgress upon the rights and liberties of the individual citizen enshrined in and protected by the Constitution. The Constitution remains the supreme law of the land to which all officials, high or low, civilian or military, owe obedience and allegiance at all times. - Article 32 of the Civil Code which renders any public officer or employee or any private individual liable in damages for violating the Constitutional rights and liberties of another, as enumerated therein, does not exempt the respondents from responsibility. Only judges are excluded from liability under the said article, provided their acts or omissions do not

a2010

page 35

Prof.
meaning and a larger dimension. No longer may a superior official relax his vigilance or abdicate his duty to supervise his subordinates, secure in the thought that he does not have to answer for the transgressions committed by the latter against the constitutionally protected rights and liberties of the citizen. Article 32 of the Civil Code makes the persons who are directly, as well as indirectly, responsible for the transgression joint tortfeasors. -RTC was therefore mistaken in dropping defendants General Fabian Ver, Col. Fidel Singson, Col. Rolando Abadilla, Col. Gerardo Lantoria, Jr., Col. Galileo Kintanar, Col. Panfilo Lacson, Capt. Danilo Pizarro, lst Lt. Pedro Tango, Lt. Romeo Ricardo and Lt. Ricardo Bacalso from the acts of their subordinates. -The responsibility of the defendants, whether direct or indirect, is amply set forth in the complaint. It is well established in our law and jurisprudence that a motion to dismiss on the ground that the complaint states no cause of action must be based on what appears on the face of the complaint. To determine the sufficiency of the cause of action, only the facts alleged in the complaint, and no others, should be considered. For this purpose, the motion to dismiss must hypothetically admit the truth of the facts alleged in the complaint. -Applying this test, it is difficult to justify the trial court's ruling, dismissing for lack of cause of action the complaint against all the defendants, except Major Rodolfo Aguinaldo and Master Sgt. Bienvenido Balaba. The complaint contained allegations against all the defendants which, if admitted hypothetically, would be sufficient to establish a cause or causes of action against all of them under Article 32 of the Civil Code. 3. NO -A timely motion to set aside said order of November 8, 1983 was filed by plaintiffs, through counsel. True, the motion was not signed by all the counsels for the several plaintiffs but the body of the motion itself clearly indicated that the motion was filed on behalf of all the plaintiffs.This must have been also the understanding of defendants' counsel himself for when he filed his comment on the motion, he furnished copies thereof, not just to the lawyers who signed the motion, but to all the lawyers of plaintiffs. -In filing the motion to set aside the resolution of November 8, 1983, the signing attorneys did so on behalf of all the plaintiff. They needed no specific authority to do that. The authority of an attorney to appear for and in behalf of a party can be assumed, unless questioned or challenged by the adverse

constitute a violation of the Penal Code or other penal statute. - In carrying out this task and mission to protect the Republic from its enemies, constitutional and legal safeguards must be observed. -Moreover, petitioners' right and cause of action for damages are explicitly recognized in P.D. No. 1755 which amended Article 1146 of the Civil Code by adding the following to its text: However, when the action (for injury to the rights of the plaintiff or for a quasi-delict) arises from or out of any act, activity or conduct of any public officer involving the exercise of powers or authority arising from Martial Law including the arrest, detention and/or trial of the plaintiff, the same must be brought within one (1) year. -Even assuming that the suspension of the privilege of the writ of habeas corpus suspends petitioners' right of action for damages for illegal arrest and detention, it does not and cannot suspend their rights and causes of action for injuries suffered because of respondents' confiscation of their private belongings, the violation of their right to remain silent and to counsel and their right to protection against unreasonable searches and seizures and against torture and other cruel and inhuman treatment. -However, we find it unnecessary to address the constitutional issue pressed upon us. On March 25, 1986, President Corazon C. Aquino issued Proclamation No. 2, revoking Proclamation Nos. 2045 and 2045-A and lifting the suspension of the privilege of the writ of habeas corpus. The question therefore has become moot and academic. 2. YES -The doctrine of respondent superior has been generally limited in its application to principal and agent or to master and servant (i.e. employer and employee) relationship. No such relationship exists between superior officers of the military and their subordinates. -Be that as it may, however, the decisive factor in this case, in our view, is the language of Article 32. The law speaks of an officer or employee or person 'directly' or "indirectly" responsible for the violation of the constitutional rights and liberties of another. Thus, it is not the actor alone (i.e. the one directly responsible) who must answer for damages under Article 32; the person indirectly responsible has also to answer for the damages or injury caused to the aggrieved party. -By this provision, the principle of accountability of public officials under the Constitution acquires added

Criminal Procedure Rowena Daroy Morales


party or the party concerned, which was never done in this case. Dispositive Petition granted. SC annuled and set aside the resolution of the respondent court, dated November 8, 1983, its order dated May 11, 1984 and its resolution dated September 21, 1984. Case remanded to the respondent court for further proceedings. Costs against private respondents.

a2010

page 36

Prof.
- Respondents Comment The decision of the trial court carries with it the final adjudication of her civil liability. Since petitioner chose to actively intervene in the criminal action without reserving his right to file a separate civil action for damages, he assumed the risk that in the event he failed to recover damages he cannot appeal from the decision of the lower court. ISSUES 1. WON the decision of the MCTC constitutes the final adjudication on the merits of private respondent's civil liability 2. WON petitioner is entitled to an award of damages HELD 1. NO - The decision of the MCTC has not yet become final due to the timely appeal filed by petitioner with respect to the civil liability of the accused in said case. It was only the unappealed criminal aspect of the case which has become final. - People vs. Coloma: from a judgment convicting the accused, two (2) appeals may, accordingly, be taken. The accused may seek a review of said judgment, as regards both civil and criminal actions; while the complainant may appeal with respect only to the civil action, either because the lower court has refused to award damages or because the award made is unsatisfactory to him. The right of either to appeal or not to appeal in the event of conviction of the accused is not dependent upon the other. Petitioner may, as he did, appeal from the decision on the civil aspect which is deemed instituted with the criminal action and such appeal, timely taken, prevents the decision on the civil liability from attaining finality. 2. YES - Civil obligations arising from criminal offenses are governed by Article 100 of the RPC which provides that "Every person criminally liable for a felony is also civilly liable," in relation to Article 2177 of the Civil Code on quasi-delict, the provisions for independent civil actions in the Chapter on Human Relations and the provisions regulating damages, also found in the Civil Code. - Underlying the legal principle that a person who is criminally liable is also civilly liable is the view that from the standpoint of its effects, a crime has dual character: (1) as an offense against the state because of the disturbance of the social order; and

SEPARATE OPINION TEEHANKEE [concur]


- All persons, be they public officers or employees, or members of the military or police force or private individuals who directly or indirectly obstruct, defeat, violate or in any manner impede or impair the constitutional rights and civil liberties of another person, stand liable and may be sued in court for damages as provided in Art. 32 of the Civil Code. - The decision herein upholds and reinstates the civil action for damages filed in the court below by petitioners-plaintiffs for illegal searches conducted by military personnel and other violations of their constitutional rights and liberties. At the same time it rejects the automatic application of the principle of respondeat superior or command responsibility that would hold a superior officer jointly and severally accountable for damages, including moral and exemplary, with his subordinates who committed such transgressions. However, the judgment gives the caveat that a superior officer must not abdicate his duty to properly supervise his subordinates for he runs the risk of being held responsible for gross negligence and of being held under the cited provision of the Civil Code as indirectly and solidarily accountable with the tortfeasor. - Brandeis:"In a government of laws, existence of the government be imperilled following it fails to observe the law scrupulously. Our government is the potent omnipresent teacher. For good or ill, it teaches the whole people by example. Crime is contagious. If the government becomes the law breaker, it breeds contempt for the law, it invites every man to become a law unto himself, it invites anarchy. To declare that in the administration of criminal law the end justifies the means ... would bring terrible retribution." - It need only be pointed out that one of the first acts of the present government under President Corazon C. Aquino after her assumption of office in February, 1986 was to file our government's ratification and access to all human rights instruments adopted under the auspices of the United Nations, declaring thereby the government's commitment to observe

the precepts of the United Nations Charter and the Universal Declaration of Human Rights. More than this, pursuant to our Constitution which the people decisively ratified on February 2, 1987, the independent office of the Commission on Human Rights has been created and organized with ample powers to investigate human rights violations and take remedial measures against all such violations by the military as well as by the civilian groups.

OCCENA v ICAMINA 181 SCRA 328 FERNAN; January 22, 1990


NATURE Petition for certiorari to review the decision of RTC FACTS - Petitioner Eulogio Occena instituted before the Second Municipal Circuit Trial Court of Sibalom, San Remigio Belison, Province of Antique, Criminal Case No. 1717, a criminal complaint for Grave Oral Defamation against private respondent Cristina Vegafria for allegedly openly, publicly and maliciously uttering the following insulting words and statements: "Gago ikaw nga Barangay Captain, montisco, traidor, malugus, Hudas," which, freely translated, mean: "You are a foolish Barangay Captain, ignoramus, traitor, tyrant, Judas" and other words and statements of similar import which caused great and irreparable damage and injury to his person and honor. - Private respondent as accused therein entered a plea of not guilty. Trial ensued, at which petitioner, without reserving his right to file a separate civil action for damages actively intervened thru a private prosecutor. - After trial, private respondent was convicted of the offense of Slight Oral Defamation and was sentenced to pay a fine of Fifty Pesos (P50.00) with subsidiary imprisonment in case of insolvency and to pay the costs. - No damages were awarded to petitioner. - Disagreeing, petitioner sought relief from the RTC, which denied his petition. - Petitioners Claim The RTC decision is contrary to Article 100 of the RPC providing that every person criminally liable for a felony is also civilly liable, and Article 2219 of the New Civil Code providing that moral damages may be recovered in libel, slander or any other form of defamation.

Criminal Procedure Rowena Daroy Morales


(2) as an offense against the private person injured by the crime unless it involves the crime of treason, rebellion, espionage, contempt and others wherein no civil liability arises on the part of the offender either because there are no damages to be compensated or there is no private person injured by the crime. In the ultimate analysis, what gives rise to the civil liability is really the obligation of everyone to repair or to make whole the damage caused to another by reason of his act or omission, whether done intentional or negligently and whether or not punishable by law. - As a general rule, a person who is found to be criminally liable offends two (2) entities: the state or society in which he lives and the individual member of the society or private person who was injured or damaged by the punishable act or omission. The offense of which private respondent was found guilty is not one of those felonies where no civil liability results because either there is no offended party or no damage was caused to a private person. - There is here an offended party, whose main contention precisely is that he suffered damages in view of the defamatory words and statements uttered by private respondent, in the amount of Ten Thousand Pesos (P10,000.00) as moral damages and the further sum of Ten Thousand Pesos (P10,000) as exemplary damages. - Article 2219, par. (7) of the Civil Code allows the recovery of moral damages in case of libel, slander or any other form of defamation This provision of law establishes the right of an offended party in a case for oral defamation to recover from the guilty party damages for injury to his feelings and reputation. The offended party is likewise allowed to recover punitive or exemplary damages. Calling petitioner who was a barangay captain an ignoramus, traitor, tyrant and Judas is clearly an imputation of defects in petitioner's character sufficient to cause him embarrassment and social humiliation. Petitioner testified to the feelings of shame and anguish he suffered as a result of the incident complained of. - Petitioner is entitled to moral damages in the sum of P5,000.00 and a further sum of P5,000.00 as exemplary damages. Dispositive The petition was granted.

a2010

page 37

Prof.
whether on reasonable doubt or not, is not a bar to a subsequent civil action for recovery of civil liability, arising not from criminal negligence, but from a quasi-delict or culpa aquiliana. - It has been held that Article 2176 of the Civil Code, in referring to "fault or negligence" covers acts "not punishable by law" as well as acts that may be criminal in character, whether intentional and voluntary or negligent. A separate civil action lies against the offender in a criminal act, whether or not he is criminally prosecuted and found guilty or acquitted, provided that the offended party is not allowed to recover damages on both scores. - Article 29 of the Civil Code does not state that the right to file an independent civil action for damages (under said article) can be availed of only in offenses not arising from a tortious act. The only requisite for the exercise of the right to file a civil action for damages is that the accused must have been acquitted in the criminal action based on reasonable doubt. - The respondents argument that the applicable provision is Article 33 is devoid of merit because of the abovementioned argument. In addition, Article 33 assumes defamation, fraud, or physical injuries were intentionally committed. - In the case at bar, Rule 111 of the former Rules of Criminal Procedure (i.e., that a reservation be made in the criminal case of the right to institute an independent civil action) is not applicable because Article 29 does not require it. - The requirement in Section 2 of Rule 111 of the former Rules on Criminal Procedure, that there be a reservation in the criminal case of the right to institute an independent civil action, has been declared as not in accordance with law. It is regarded as an unauthorized amendment to the substantive law, in this case the Civil Code. In fact, the reservation of the right to file an independent civil action has been deleted from Section 2, Rule 111 of the 1985 Rules on Criminal Procedure, in consonance with the decisions of this Court declaring such requirement of a reservation as ineffective. - The active participation of the Bonite heirs does not act as a bar from pursuing a civil action for damages because the civil action based on criminal liability and a civil action under Article 29 are two separate and independent actions. Dispositive WHEREFORE, the orders of the respondent court are hereby REVERSED and SET ASIDE, and a new one is entered reinstating the complaint in the civil case directing said court to

NATURE Petition for certiorari to review the order of the Court of First Instance of Misamis Occidental, Br. III. Zosa, J. FACTS - September 24, 1968, 2 PM Bonite, a caminero of the Bureau of Public Highways was killed when he was hit by a truck driven by Abamonga. A complaint for reckless imprudence resulting in homicide was filed by the surviving heirs of Bonite but Abamonga was acquitted because of insufficient evidence. - In the course of the trial, the petitioners actively participated in the proceedings through their lawyer, private prosecutor Atty. Dulalas. - December 28, 1970 The Bonite heirs filed an action for recovery of damages against Abamonga based on the same act but the court dismissed the complaint for damages because the Court believes that the Bonite heirs did not reserve the right to do so. ISSUE WON an independent civil action for damages, under Article 29 of the Civil Code, is deemed barred by petitioners' failure in the criminal action to make a reservation to file a separate civil action and by their active participation in the prosecution of such criminal action. HELD YES Ratio When the accused in a criminal case is acquitted on the ground that his guilt has not been proved beyond reasonable doubt, a civil action for damages for the same act or omission may still be instituted against him, and only a preponderance of evidence is required to hold the accused liable. The civil liability is not extinguished by acquittal of the accused, where the acquittal is based on reasonable doubt (based on Article 29 of the Civil Code). Reasoning - In the criminal case against Abamonga, the accused was acquitted because there was insufficient evidence to prove his guilt beyond reasonable doubt. Clearly, the Bonite heirs have the right to file an independent civil action for damages despite the acquittal of the accused in the criminal case. - Aside from basing their action for damages in Article 29 of the Civil Code, the petitioners may also rely on Article 2176 which provides that acquittal of the accused from a charge of criminal negligence,

BONITE v ZOSA 162 SCRA 180 PADILLA; June 20, 1988

Criminal Procedure Rowena Daroy Morales


proceed with the trial of the case. Costs against private respondent.

a2010

page 38

Prof.
for the injury sustained by reason of the same, are preserved and remain in force, and it is therein expressly provided that the court, upon conviction of the accused, may enter judgment in favor of the injured person, against the defendant in the criminal case for the damage occasioned by the wrongful act." - Rakes vs. Atlantic, Gulf & Pacific Co.: "According to article 112 (Spanish Code of Criminal Procedure) the penal action once started, the civil remedy should be sought therewith, unless it had been waived by the party injured or been expressly reserve by him for civil proceedings for the future. If the civil action alone was prosecuted, arising out of a crime that could be enforced only on private complaint, the penal action thereunder should be extinguished." - Almeida vs. Abaroa (8 Phil. Rep., 178), was a civil action for damages brought the plaintiff against a person who had been previously acquitted on a criminal charge. It was held that his acquittal in the criminal action was a complete bar to a civil action for damages based upon the alleged criminal act of which the defendant had been accused. In the course of this decision it was said: - "Instituting a criminal action only, it will be understood, brings the civil action as well, unless the damaged or prejudiced person waives the same or expressly reserves the right to institute the civil action after the termination of the criminal case, if there be any reason therefor. (Art. 112 of the said Law of Criminal Procedure.) - "The right to bring the civil action, as reserved by the person damaged or prejudiced, after the termination of the criminal case, is only permitted, if there be any reason therefore, and so says the law, in the event that the judgment rendered in the criminal cause is a finding of guilt against the accused; but if the accused be acquitted, then the compliant in the civil action must be based on some fact and or cause distinct and separate from the criminal act itself." - The court then quotes from article 114 of the Spanish Code of Criminal Procedure provides: "When a criminal proceeding is instituted for the judicial investigation of a crime or misdemeanor, no civil action arising from the same act can be prosecuted; but the same shall be suspended, if there be one, in whatever stage or state it may be found, until final sentence in the criminal proceeding is pronounced.

SEPARATE OPINION MELENCIO-HERRERA [concur]


- 4 of them merely concurred in the result- If there has been active participation in the prosecution of a criminal case by the offended party, the civil action arising from the crime is deemed to have been also brought in the criminal case. Consequently, a judgment finding the accused guilty and granting him damages is binding upon the offended party and he may not file a separate civil action under Article 33. - However if the accused is acquitted on the ground of reasonable doubt, as in this case, the civil action for damages for the same act may be instituted under Article 29 of the Civil Code, notwithstanding the fact that the offended party had actively participated in the criminal action. - The rule in Corpus vs. Paje which states that reckless imprudence is not included in Article 33 of the Civil Code, was note deemed as authoritative doctrine because, of 11 justices, only 9 took part in the decision and

defendant are to be considered as a modification of the punishment, by increasing the penalty or otherwise, meted out to the defendant for the commission of the crime. (WON there is double jeopardy) 2. WON award was excessive HELD 1. NO. - Civil damages are no part of the punishment for the crime. - What was the effect of the action of this court in affirming that judgment as to the guilt and punishment of the accused and of reversing it as to the question of civil damage, with instructions to execute the punishment imposed and to try the civil branch of the case? Bearing in mind the broad line of demarcation between the civil liability of the accused and his criminal liability, the bare fact that his civil liability was determined and fixed had nothing whatever to do with the punishment imposed. The latter was not thereby affected. This time intervening between the judgment of guilt and the judgment of civil damages could in no way give to the latter the character of the former. - (That) the defendant might serve the term of imprisonment fixed by the court as the punishment for his crime, and after the sentence for civil damages and in case of his insolvency, he would have to return to prison to serve the subsidiary imprisonment by reason of his insolvency, being argued that this would constitute double jeopardy. It is well settled that execution against the person will issue in civil actions in case of personal injuries, and that this is not imprisonment for debt or punishment for crime. It is in lieu of the payment of the indemnity and is considered as a discharge thereof. If the payment of the indemnity is not punishment for the crime, the imprisonment in lieu thereof is not punishment for the crime. - As the civil liability is no part of the punishment for the crime, there would have been no question of double jeopardy... In the present case, the civil liability of the defendant was established, and the sole question determined upon the second trial was the amount of civil damages. The plea of double jeopardy can not be allowed. -On civil liability of persons accused of crime: - Springer vs. Odlin: "By General Orders, No. 58, section 107, the privileges secured by the Spanish law to persons claiming to be injured by the commission of an offense to take part in the prosecution of the offense and to recover damages

US v HEERY 25 Phil 600 TRENT; Oct 22, 1913


FACTS -Heery was charged with attempted murder, but was convicted of the lesser crime of maliciously inflicting serious injury upon Alex Sternberg, and was sentenced to one year and three months of prision correccional by the trial court. -On appeal, the decision was affirmed but case was remanded because it did not include the evidence of civil damages suffered by the offended party. -Then lower court, in its ruling, restated the conviction (of one yr to 3 months of prision correccional) and then included P50,500 for indemnity, with subsidiary imprisonment, in case of insolvency. -Heery raised the question of double jeopardy, and the award being excessive. ISSUES 1. WON remanding the case for determination of civil damages and their assessment against the

Criminal Procedure Rowena Daroy Morales


"To prosecute a penal action it shall not be necessary that a civil action arising from the same crime or misdemeanor be previously instituted." - Under the Spanish criminal law, an injured person had the right to intervene in the prosecution of the accused for the purpose of having his damages ascertained. The trial court was required to include the amount of these damages in the judgment of conviction. The plain provisions of section 107 of our criminal procedure, quoted supra, expressly preserves this right to the injured person. The refusal of the trial court to allow the injured person to introduce evidence as to his damages is, therefore, clearly prejudicial error. 2. YES. - There can be no objection to allowing the physicians' fees of P500 and P1,300 for the three months' salary, being the time the injured party was incapacitated from performing the work in which he was then engaged. The remainder, P48,700, appears to have been allowed on account of the permanent diminution of Sternberg's ability to earn money. The evidence of record does not establish such disability with that degree of certainly which will justify an award for that purpose. We have reached this conclusion after a most careful examination of all the testimony upon this point. Dispositive The award of damages is reduced to P1,800, the defendant to suffer subsidiary imprisonment, which in no event can exceed onethird of the principal penalty, in case of insolvency.

a2010

page 39

Prof.
impliedly instituted with the criminal action, unless the offended party expressly waives the civil action or reserves the right to institute it separately, and that, inasmuch as petitioner had failed to expressly reserve her right to institute the civil action separately, she may not now institute another action under articles 1902-1910 of the Civil Code based on the act or omission complained of in the criminal action. ISSUES 1. WON CFI was correct in considering and applying Sec 1, Rule 107, of the Rules of Court 2. WON respondent judge erred in suspending the hearing HELD 1. NO - The present civil case is based upon a cause of action not arising from the civil liability involved in the criminal case instituted against the accused. The civil case is based on alleged culpa contractual incurred by PAL because of its failure to carry safely Richard Parker to his place of destination, whereas the criminal case involves the civil liability of the accused. - Rule 107 contemplates a case where the offended party desires to press his right to demand indemnity from the accused in the criminal case which he may assert either in the same criminal case or in a separate action. - The failure of Parker to reserve her right to institute the civil action in the criminal case cannot in any way be deemed as waiver on her part to institute a separate civil action against PAL based on its contractual liability. 2. NO - The present civil case is directly interwoven with the criminal case in the sense that the main issue involved in both cases is the determination of the failure of Richard Parker to reach safely his destination or the determination of the cause of his death. This was the main reason that guided the lower court in postponing the hearing of the civil case until final judgment in the criminal case has been rendered. - Inasmuch as the power to grant or refuse continuances is inherent in all courts unless expressly limited by statute, and there is no showing that the lower court has abused its discretion is suspending the hearing, the petition for certiorari must fail. Dispositive Petition denied

introduced as evidence and was not, therefore, considered by that court. If it had been introduced as evidence, the government would have had the right to meet it and be heard upon it in that court. To permit the question to be raised here for the first time, and in the resolution thereof, to consider evidence that was never introduced in the trial court and which the government has never had an opportunity to meet in an orderly way, is not only to take the prosecution by surprise but is to establish a precedent which may be dangerous in practice and subversive of orderly procedure.

PARKER v PANLILIO and PHIL AIR LINES 91 PHIL 1 BAUTISTA ANGELO; March 5, 1952
NATURE Certiorari and mandamus FACTS - Asuncion Parker and her minor daughter Kathleen filed a complaint for damages against Philippine Air Lines, Inc., based on the alleged failure of PAL to carry safely Richard Parker from Daet, Camarines Norte to Manila. - PAL set up as special defense that the plane exploded in mid-air due to dynamite surreptitiously introduced into said air craft by criminal hands. A criminal case was already filed in CFI Camarines Norte against the supposed guilty parties. - When the case was set for the continuation of the hearing, PAL presented an oral motion for the suspension of the hearing, invoking (then) sec 1 Rule 107, of the Rules of Court, which provides that no civil action arising from the same offense can be prosecuted until final judgment in the criminal proceeding has been rendered. Parker vehemently opposed. - CFI suspended the hearing until the final determination of the criminal case which was then pending appeal in the SC. Petitioners Claim It was a mistake on the part of respondent judge to consider and apply Sec 1, Rule 107 of the Rules of Court, as her cause of action in the civil case is based on culpa contractual and not on the civil liability arising from the offense involved in the criminal case. Respondents Comments When a criminal action is instituted, the civil action for the recovery of the civil liability arising from the offense charged is

SEPARATE OPINION MORELAND [concurring and dissenting]


The record being in this condition, I am inclined to believe that this court should not take up and discuss, much less decide, the question of former jeopardy. To do so it must not only hold, in violation of the provisions of section 24 of the Code of Criminal Procedure, that it is unnecessary to plead the defense in the trial court but must also hold that it is unnecessary to introduce evidence in that court to substantiate the plea. The plea of former conviction or once in jeopardy should, according to established rules and the provisions of the Code of Criminal Procedure, be substantiated by the production of the record of the former trial and the introduction of the same in evidence. That was not done in this case. While the evidence taken in the former trial was introduced in the present case for the purpose of establishing the extent of defendant's civil liability, the remaining part of the record was not

Criminal Procedure Rowena Daroy Morales

a2010

page 40

Prof.
- Petitioner moved for the suspension of the proceedings in the civil case against him, citing the pendency of the criminal case against his driver. But the trial court denied petitioners motion on the ground that pursuant to the Civil Code, the action could proceed independently of the criminal action, in addition to the fact that the petitioner was not the accused in the criminal case. - CA dismissed his petition - There is no dispute that private respondent, as offended party in the criminal case, did not reserve the right to bring a separate civil action, based on the same accident, either against the driver, Herminio Andaya, or against the latters employer, herein petitioner Ruben Maniago. - petitioner argues that the civil action against him was impliedly instituted in the criminal action previously filed against his employee because private respondent did not reserve his right to bring this action separately. (The records show that while this case was pending in the Court of Appeals, the criminal action was dismissed on July 10, 1992 for failure of the prosecution to file a formal offer of its evidence, with the consequence that the prosecution failed to prosecute its case. Accordingly, it seems to be petitioners argument that since the civil action to recover damages was impliedly instituted with the criminal action, the dismissal of the criminal case brought with it the dismissal of the civil action.) - Private respondent admits that he did not reserve the right to institute the present civil action against Andayas employer. He contends, however, that the rights provided in Arts. 2176 and 2177 of the Civil Code are substantive rights and, as such, their enforcement cannot be conditioned on a reservation to bring the action to enforce them separately. ISSUE WON despite the absence of reservation, Boado may nonetheless bring an action for damages against petitioner under the Art.2176, 2180 and 2177 of the Civil Code and Rule 111 of the Rules of Court.

YAKULT PHILIPPINES v CA [CAMASO] 190 SCRA 357 GANCAYCO; Oct. 5, 1990


NATURE Petition for review of decision of the CA FACTS - 5 year old Roy Camaso (standing on a sidewalk) was sideswiped by a motorcycle owned by Yakult Philippines and driven by its employee, Larry Salvado on Dec.1982. - An information was then filed on Jan.1983 against Salvado charging him with the crime of reckless imprudence resulting in slight physical injuries. - On Oct. 1984, a complaint for damages was filed in the RTC of Manila by Roy Camaso (represented by his father, David) against Yakult and Salvado. The RTC decided in favor of the Camasos and held the defendants (herein petitioners) jointly and severally liable for damages, which then moved said defendants Yakult and Salvado to appeal the judgment. They also filed a peitition for certiorari in the CA challenging the RTCs jurisdiction in the civil case. Their argument was that the civil action for damages for injuries arising from alleged criminal negligence, there being no malice, cannot be filed independently of the criminal action and that under Rule 111 Sec.1 of the 1985 Rules of Crim. Pro., such a separate civil action may not be filed unless reservation thereof is expressly made. - The CA on Nov. 1989, dismissed the petition and the subsequent MFR. ISSUE WON a civil action instituted after a criminal action was filed can prosper even if there was no reservation to file a separate civil action HELD YES - Although the separate civil action filed in this case was without previous reservation in the criminal case, it was nevertheless instituted before the prosecution presented evidence in the criminal action, and the presiding judge handling the criminal

action was duly informed thereof, such that no damages was awarded in the disposition of the criminal action. Reasoning - Under the aforecited provisions of the rule, the civil action for the recovery of civil liability is impliedly instituted with the criminal action unless the offended party waives the civil action, reserves his right to institute it separately or institutes the civil action prior to the criminal action. - Such civil action includes recovery of indemnity under the Revised Penal Code, and damages under Articles 32, 33, 34 and 2176 of the Civil Code of the Philippines arising from the same act or omission of the accused. It is also provided that the reservation of the right to institute the separate civil action shall be made before the prosecution starts to present its evidence and under circumstances affording the offended party a reasonable opportunity to make such reservation. ** The SC considered the actual filing of the civil action far better than a compliance with the requirement of an express reservation that should be made by the offended party before the prosecution presents its evidence. It added that the purpose of this rule requiring reservation is to prevent the offended party from recovering damages twice for the same act or omission. Dispositive petition DENIED. CA decision AFFIRMED.

MANIAGO vCA (BOADO) 253 SCRA 674 MENDOZA; February 20, 1996
FACTS - Petitioner Ruben Maniago was the owner of shuttle buses which were used in transporting employees of the Texas Instruments, Inc. from Baguio City proper to its plant site at the Export Processing Authority in Loakan, Baguio City. - One of his buses figured in a vehicular accident with a passenger jeepney owned by private respondent Alfredo Boado along Loakan Road, Baguio City. As a result of the accident, a criminal case for reckless imprudence resulting in damage to property and multiple physical injuries was filed against petitioners driver, Herminio Andaya, with the Regional Trial Court of Baguio City - A month later, a civil case for damages was filed by private respondent Boado against petitioner himself

HELD NO Ratio The right to bring an action for damages under the Civil Code must be reserved as required by Rule 111, 1, otherwise it should be dismissed. 1 quite clearly requires that a reservation must be made to institute separately all civil actions for the recovery of civil liability, otherwise they will be deemed to

Criminal Procedure Rowena Daroy Morales


have been instituted with the criminal case. The right of the injured party to sue separately for the recovery of the civil liability whether arising from crimes or from quasi delict under Art. 2176 of the Civil Code must be reserved otherwise they will be deemed instituted with the criminal action. Reasoning A. There are statements in some cases implying that Rule 111, 1 and 3 are beyond the rulemaking power of the Supreme Court under the Constitution. A careful examination of the cases, however, will show that approval of the filing of separate civil action for damages even though no reservation of the right to institute such civil action had been reserved rests on considerations other than that no reservation is needed. - In Garcia v. Florido the right of an injured person to bring an action for damages even if he did not make a reservation of his action in the criminal prosecution for physical injuries through reckless imprudence was upheld on the ground that by bringing the civil action the injured parties had in effect abandoned their right to press for recovery of damages in the criminal case. - In Abellana v. Marave in which the right of persons injured in a vehicular accident to bring a separate action for damages was sustained despite the fact that the right to bring it separately was not reserved. But the basis of the decision in that case was the fact that the filing of the civil case was equivalent to a reservation because it was made after the decision of the City Court convicting the accused had been appealed. - In Jarantilla v. CA the ruling is that the acquittal of the accused in the criminal case for physical injuries through reckless imprudence on the ground of reasonable doubt is not a bar to the filing of an action for damages even though the filing of the latter action was not reserved. This is because of Art. 29 of the Civil Code which provides that when an accused is acquitted on the ground that his guilt has not been proved beyond reasonable doubt, a civil action for damages for the same act or omission may be instituted. This ruling obviously cannot apply to this case because the basis of the dismissal of the criminal case against the driver is the fact that the prosecution failed to prove its case as a result of its failure to make a formal offer of its evidence. - the rulings in these cases are consistent with the proposition herein made that, on the basis of Rule 111, 1-3, a civil action for the recovery of civil liability is, as a general rule, impliedly instituted with the criminal action, except only (1) when such action

a2010

page 41

Prof.
- Nor does it matter that the action is against the employer to enforce his vicarious liability under Art. 2180 of the Civil Code. Though not an accused in the criminal case, the employer is very much a party, as long as the right to bring or institute a separate action (whether arising from crime or from quasi delict) is not reserved. The ruling that a decision convicting the employee is binding and conclusive upon the employer not only with regard to its civil liability but also with regard to its amount because the liability of an employer cannot be separated but follows that of his employee is true not only with respect to the civil liability arising from crime but also with respect to the civil liability under the Civil Code. Dispositive The decision appealed from is REVERSED and the complaint against petitioner is DISMISSED.

arising from the same act or omission, which is the subject of the criminal action, is waived; (2) the right to bring it separately is reserved or (3) such action has been instituted prior to the criminal action. Even if an action has not been reserved or it was brought before the institution of the criminal case, the acquittal of the accused will not bar recovery of civil liability unless the acquittal is based on a finding that the act from which the civil liability might arise did not exist because of Art. 29 of the Civil Code. - Through all the shifts or changes in policy as to the civil action arising from the same act or omission for which a criminal action is brought, one thing is clear: The change has been effected by this Court. The new rules require reservation of the right to recover the civil liability, otherwise the action will be deemed to have been instituted with the criminal action. - Contrary to private respondents contention, the requirement that before a separate civil action may be brought it must be reserved does not impair, diminish or defeat substantive rights, but only regulates their exercise in the general interest of orderly procedure. - It is the conduct of the trial of the civil action - not its institution through the filing of a complaint - which is allowed to proceed independently of the outcome of the criminal case. B. There is a practical reason for requiring that the right to bring an independent civil action under the Civil Code separately must be reserved. It is to avoid the filing of more than one action for the same act or omission against the same party. Any award made against the employer, whether based on his subsidiary civil liability under Art. 103 of the Revised Penal Code or his primary liability under Art. 2180 of the Civil Code, is ultimately recoverable from the accused. - In the present case, the criminal action was filed against the employee, bus driver. Had the driver been convicted and found insolvent, his employer would have been held subsidiarily liable for damages. But if the right to bring a separate civil action (whether arising from the crime or from quasi-delict) is reserved, there would be no possibility that the employer would be held liable because in such a case there would be no pronouncement as to the civil liability of the accused. In such a case the institution of a separate and independent civil action under the Civil Code would not result in the employee being held liable for the same act or omission. The rule requiring reservation in the end serves to implement the prohibition against double recovery for the same act or omission.

SAN ILDEFONSO LINES, INC. v CA (PIONEER INSURANCE AND SURETY CORPORATION) 300 SCRA 484 MARTINEZ; April 24, 1998
NATURE Petition for review after a motion for reconsideration of respondent court judgment was denied FACTS - In the afternoon of June 24, 1991, a Toyota Lite Ace Van being driven by its owner Annie U. Jao and a passenger bus of herein petitioner San Ildefonso Lines, Inc. (hereafter, SILI) collided with each other at the intersection of Julia Vargas Avenue and Rodriguez Lanuza Avenue in Pasig, Metro Manila, totally wrecking the Toyota van and injuring Ms. Jao and her two (2) passengers in the process. - A criminal case was thereafter filed with the Regional Trial Court of Pasig on September 18, 1991 charging the driver of the bus, herein petitioner Eduardo Javier, with reckless imprudence resulting in damage to property with multiple physical injuries. - About four (4) months later, or on January 13, 1992, herein private respondent Pioneer Insurance and Surety Corporation (PISC), as insurer of the van and subrogee, filed a case for damages against petitioner SILI with the Regional Trial Court of Manila, seeking to recover the sums it paid the assured under a motor vehicle insurance policy as well as other damages, totaling P564,500.00 (P454,000.00 as actual/compensatory damages; P50,000.00 as exemplary damages; P50,000.00 as attorney's fees;

Criminal Procedure Rowena Daroy Morales


P10,000.00 as litigation expenses; and P500.00 as appearance fees.) - With the issues having been joined upon the filing of the petitioners' answer to the complaint for damages and after submission by the parties of their respective pre-trial briefs, petitioners filed on September 18, 1992 a Manifestation and Motion to Suspend Civil Proceedings grounded on the pendency of the criminal case against petitioner Javier in the Pasig RTC and the failure of respondent PISC to make a reservation to file a separate damage suit in said criminal action. This was denied by the Manila Regional Trial Court in its Order dated July 21, 1993 - After their motion for reconsideration of said July 21, 1993 Order was denied, petitioners elevated the matter to this Court via petition for certiorari which was, however, referred to public respondent Court of Appeals for disposition. On February 24, 1995, a decision adverse to petitioners once again was rendered by respondent court, upholding the assailed Manila Regional Trial Court Order. Hence, this petition for review after a motion for reconsideration of said respondent court judgment was denied. ISSUES

a2010

page 42

Prof.
is bound to observe the procedural requirements which Ms. Jao ought to follow had she herself instituted the civil case. Dispositive The assailed decision of the Court of Appeals dated February 24, 1995 and the Resolution dated April 3,1995 denying the motion for reconsideration thereof are reversed. The "manifestation and motion to suspend civil proceedings" filed by petitioners is granted.

1. 2.

WON an independent civil action based on quasi-delict under Article 2176 of the Civil Code can be filed if no reservation was made in the said criminal case WON a subrogee of an offended party can maintain an independent civil action during the pendency of a criminal action when no reservation of the right to file an independent civil action was made in the criminal action and despite the fact that the private complainant is actively participating through a private prosecutor in the aforementioned criminal case

HELD 1. NO - On the chief issue of "reservation", at the fore is Section 3, Rule 111 of the Rules of Court which reads: "Sec. 3. When civil action may proceed independently. -- In the cases provided for in Articles 32, 33, 34 and 2176 of the Civil Code of the Philippines, the independent civil action which has been reserved may be brought by the offended party, shall proceed independently of the criminal action, and shall require only a preponderance of

evidence." - Even though these so-called "independent civil actions" based on the aforementioned Civil Code articles are the exceptions to the primacy of the criminal action over the civil action as set forth in Section 2 of Rule 111, it is easily deducible from the present wording of Section 3 as brought about by the 1988 amendments to the Rules on Criminal Procedure -- particularly the phrase " which has been reserved" -- that the "independent" character of these civil actions does not do away with the reservation requirement. In other words, prior reservation is a condition sine qua non before any of these independent civil actions can be instituted and thereafter have a continuous determination apart from or simultaneous with the criminal action. - According to Justice Jose Y. Feria, remedial law expert and a member of the committee which drafted the 1988 amendments, whose learned explanation on the matter was aptly pointed out by petitioners, the 1988 amendment expands the scope of the civil action which is deemed impliedly instituted with the criminal action unless waived, reserved or previously instituted. Under the present Rule as amended, such a civil action includes not only recovery of indemnity under the Revised Penal Code and damages under Articles 32, 33, 34 of the Civil Code of the Philippines, but also damages under Article 2176 (quasi-delicts) of the said code. - It should be noted that while it was ruled in Abella vs. Marave (57 SCRA 106) that a reservation of the right to file an independent civil action is not necessary, such a reservation is necessary under the amended rule. Without such reservation, the civil action is deemed impliedly instituted with the criminal action, unless previously waived or instituted. - Far from altering substantive rights, the primary purpose of the reservation is, to borrow the words of the Court in "Caos v. Peralta":" to avoid multiplicity of suits, to guard against oppression and abuse, to prevent delays, to clear congested dockets, to simplify the work of the trial court; in short, the attainment of justice with the least expense and vexation to the parties-litigants." 2. NO - Private respondent PISC, as subrogee, is not exempt from the reservation requirement with respect to its damages suit based on quasi-delict arising from the same act or omission of petitioner Javier complained of in the criminal case. As private respondent PISC merely stepped into the shoes of Ms. Jao (as owner of the insured Toyota van), then it

MARCIA v CA (PAJE and VICTORY LINER) 120 SCRA 190 RELOVA; January 27, 1983
NATURE Appeal by certiorari from the decision of the Court of Appeals affirming the judgment of the Court of First Instance of Rizal, which dismissed the complaint filed by the petitioners against private respondents in the concept of an independent civil action for damages for physical injuries resulting from reckless imprudence.

FACTS - On December 23, 1956, in the municipality of Lubao, Pampanga, a passenger bus operated by private respondent Victory Liner, Inc. and driven by its employee, private respondent Felardo Paje, collided with a jeep driven by Clemente Marcia, resulting in the latter's death and in physical injuries to herein petitioners, Edgar Marcia and Renato Yap. Thereupon, an information for homicide and serious physical injuries thru reckless imprudence was filed against Felardo Paje in the CFI of Pampanga. - On January 23, 1957, an action for damages was filed in the CFI of Rizal by Edgar Marcia and Renato Yap, together with their respective parents, against the Victory Liner, Inc. and Felardo Paje, alleging that the mishap was due to the reckless imprudence and negligence of the latter in driving the passenger bus. - While said Civil Case was in progress in the Court of First Instance of Rizal, the criminal action proceeded in the Court of First Instance of Pampanga. The accused Felardo Paje was convicted of the offense charged. However, on appeal to the Court of Appeals, he was acquitted with the CA holding that "CRIMINAL NEGLIGENCE is WANTING in this case, and that appellant was NOT even guilty of CIVIL NEGLIGENCE.

Criminal Procedure Rowena Daroy Morales


Insofar as appellant was concerned, the CA held that this was a case of PURE ACCIDENT." - As a consequence, herein private respondents, defendants in Civil Case of the Court of First Instance of Rizal, moved for the dismissal of the complaint invoking the decision of the Court of Appeals acquitting Felardo Paje and citing Section 1 (d), Rule 107 of the Rules of Court (now Section 3 (c), Rule 111 of the New Rules of Court). On August 10, 1966, the Court of First Instance of Rizal rendered a decision dismissing plaintiffs' complaint against the defendants Victory Liner, Inc. and Felardo Paje. Petitioners appealed the case to the CA, which basically affirmed the RTC decision. Hence, this recourse. ISSUE WON the decision of the Court of Appeals acquitting the accused in reckless imprudence on the ground that the incident was accidental, extinguished by implication the civil action for damages HELD YES Ratio Extinction of the penal action does not carry with it extinction of the civil, unless the extinction proceeds from a declaration in a final judgment that the fact from which the civil might arise did not exist. Since, the CA found that this case was of pure accident, it is as good as saying as if he did not commit the crime charged. There being no crime committed, no civil liability arises. Reasoning - It is the stand of herein petitioners that Section 2, Rule 111 of the Rules of Court, not Section 3 (c) thereof, should apply in the case at bar. "Sec. 2. Independent civil action. - In the cases provided for in Articles 31, 32, 33, 34 and 2177 of the Civil Code of the Philippines, an independent civil action entirely separate and distinct from the criminal action, may be brought by the injured party during the pendency of the criminal case, provided the right is reserved as required in the preceding section. Such civil action shall proceed independently of the criminal prosecution, and shall require only a preponderance of evidence." - We do not agree. Section 2 of Rule 111 merely refers to the institution of an independent civil action without waiting for the filing or termination of the criminal action and requires only preponderance of evidence to prosper and not proof beyond reasonable doubt as required for conviction in criminal cases. However, an acquittal based on the

a2010

page 43

Prof.
returned which humiliated Zenaida and compelled her to go back to her parents. Respondents Comment Conrado Bunag Jr.and Zenaida Cirilo had earlier made plans to elope and get married (same as first set of facts) . And that the reason why Conrado broke off their plan to get married was their bitter disagreements over money and Zenaidas threats to his life. - The Cirilos filed a complaint for damages against Conrado Bunag Jr. and his father Conrado Bunag Sr. (Zenaidas uncle claims that Bunag Sr. assured them that the couple were to be married). The Trial Court ordered Bunag Jr. to pay damages (80Kmoral damages,20K-exemplary damages, 20ktemperate damages and 10k attorneys fees) Bunag Sr. was absolved from any and all liability.CA affirmed in toto - Bunag Jr contends that both the trial court awarded the damages on the basis of a finding that he is guilty of forcible abduction with rape,despite the prior dismissal of the complaint therefore filed by Zenaida with the Pasay City Fiscals Office. ISSUE WON the Fiscals dismissal of the complaint for forcible abduction with rape extinguished the civil liability of Conrado Bunag Jr HELD NO - The dismissal did not in any way affect the right of Zenaida Cirilo to institute a civil action arising from the offense. - Extinction of the penal action does not carry with it the extinction of civil liability unless the extinction proceeds from a declaration in a final judgment that the fact from which the civil case might arise did not exist. Reasoning -Generally, every person criminally liable is also civilly liable. Criminal Liability will give rise to civil liability ex delicto only if the same felonious act or omission results in damage or injury to another and is the direct and proximate cause thereof. -The two proceedings involved are not between the same parties (the criminal action is between the State and the defendant and the civil case is between the offended party and the defendant). Also, there are different rules as to the competency of witnesses and the quantum of evidence in criminal and civil proceedings.(criminal action proof beyond reasonable doubt; civil actionpreponderance of evidence)

finding that the facts upon which civil liability did not exist, bars the filing of an independent civil action if it is based on the crime. As early as 1952, We have held in the case of Tan vs. Standard Vacuum Oil Company, 91 Phil. 672, that "the acquittal of the accused from the criminal charge will not necessarily extinguish the civil liability unless the court declares in the judgment that the fact from which the civil liability might arise did not exist. Where the court states 'that the evidence throws no light on the cause of fire and that it was an unfortunate accident for which the accused cannot be held responsible,' this declaration fits well into the exception of the rule which exempts the accused, from civil liability." - Also, the charge against Felardo Paje was not for homicide and physical injuries but for reckless imprudence or criminal negligence resulting in homicide (death of Clemente Marcia) and physical injuries suffered by Edgar Marcia and Renato Yap. They are not one of the three (3) crimes mentioned in Article 33 of the Civil Code and, therefore, no civil action shall proceed independently of the criminal prosecution.

BUNAG JR. v CA (CIRILO) 211 SCRA 440 REGALADO; July 10, 1992
NATURE Petition for review from the decision of the CA FACTS - On Sept. 8, 1973 Conrado Bunag Jr. brought Zenaida Cirilo to a motel or hotel where they had sexual intercourse and later that evening he brought Zenaida to the house of his grandmothers house where they lived together as husband and wife for 21 days until Sept. 29, 1973. They filed their application for marriage license with the Local Civil Registral of Bacoor, Cavite. However, after a few days, Conrado filed an affidavit withdrawing his application for a marriage license. Plaintiffs Claim Conrado Bunag Jr. abducted her in the vicinity of San Juan de Dios Hospital in Pasay City and brought her to a motel where she was raped. Afterwhich he said that he would not let her go unless they get married, as he intended to marry her , so much so that she promised not to make any scandal and to marry him. They went to his gradmothers house and lived together as husband and wife for 21 days until Bunag Jr. left and never

Criminal Procedure Rowena Daroy Morales


- In this case the dismissal of the complaint for forcible abduction with rape was by mere resolution of the fiscal at the preliminary investigation stage. There is no declaration in a final judgment that the fact from which the civil case might arise did not exist.

a2010

page 44

Prof.
exclusively to civil liability founded on Article 100 of the Revised Penal Code; whereas the civil liability for the same act considered as a quasidelict only and not as a crime is not extinguished even by a declaration in the criminal case that the criminal act charged has not happened or has not been committed by the accused . . . - The aforecited case of Lontoc vs. MD Transit & Taxi Co., Inc., et al. involved virtually the same factual situation. The Court, in arriving at the conclusion hereinbefore quoted, expressly declared that the failure of the therein plaintiff to reserve his right to file a separate civil case is not fatal; that his intervention in the criminal case did not bar him from filing a separate civil action for damages, especially considering that the accused therein was acquitted because his guilt was not proved beyond reasonable doubt; that the two cases were anchored on two different causes of action, the criminal case being on a violation of Article 365 of the Revised Penal Code while the subsequent complaint for damages was based on a quasi-delict; and that in the judgment in the criminal case the aspect of civil liability was not passed upon and resolved. Consequently, said civil case may proceed as authorized by Article 29 of the Civil Code. - Under the present jurisprudential milieu, where the trial court acquits the accused on reasonable doubt, it could very well make a pronounce ment on the civil liability of the accused and the complainant could file a petition for mandamus to compel the trial court to include such civil liability in the judgment of acquittal. And that the failure of the court to make any pronouncement, favorable or unfavorable, as to the civil liability of the accused amounts to a reservation of the right to have the civil liability litigated and determined in a separate action. The rules nowhere provide that if the court fails to determine the civil liability it becomes no longer enforceable. Dispositive Decision of CA affirmed, petion denied.

JARANTILLA v CA (SING) 171 SCRA 429 REGALADO; March 21, 1989


NATURE Appeal on the decision of the Court of Appeals upholding the decision of the trial court awarding damages to the private respondent. FACTS - Private respondent Jose Kuan Sing was "side-swiped by a vehicle in the evening of July 7, 1971 in lznart Street, Iloilo City" The respondent Court of Appeals concurred in the findings of the court a quo that the said vehicle which figured in the mishap, a Volkswagen (Beetle type) car, was then driven by petitioner Edgar Jarantilla along said street toward the direction of the provincial capitol, and that private respondent sustained physical injuries as a consequence. - Petitioner was accordingly charged before the then City Court of Iloilo for serious physical injuries thru reckless imprudence in Criminal Case No. 47207 thereof. Private respondent, as the complaining witness therein, did not reserve his right to institute a separate civil action and he intervened in the prosecution of said criminal case through a private prosecutor. Petitioner was acquitted in said criminal case "on reasonable doubt". - On October 30, 1974, private respondent filed a complaint against the petitioner in the former Court of First Instance of Iloilo, Branch IV, docketed therein as Civil Case No. 9976, and which civil action involved the same subject matter and act complained of in Criminal Case No. 47027. In his answer filed therein, the petitioner alleged as special and affirmative detenses that the private respondent had no cause of action and, additionally, that the latter's cause of action, if any, is barred by the prior judgment in Criminal Case No. 47207 inasmuch as when said criminal case was instituted the civil liability was also deemed instituted since therein plaintiff failed to reserve the civil aspect and actively participated in the criminal case.

- Thereafter, acting on a motion to dismiss of therein defendant, the trial court issued on April 3, 1975 an order of denial. Petitioner thereafter filed in this Court a petition for certiorari, prohibition and mandamus, which was docketed as G.R. No. L-40992, assailing the aforesaid order of the trial court. Said petition was dismissed for lack of merit in the Court's resolution of July 23, 1975, and a motion for reconsideration thereof was denied for the same reason in a resolution of October 28, 1975. - After trial, the court below rendered judgment on May 23, 1977 in favor of the herein private respondent and ordering herein petitioner to pay damages. Thus, petitioner appealed said decision to the CA but said respondent court affirmed in toto the decision of the trial court with a few changes in the amount of the damages to be paid. ISSUE WON the private respondent, who was the complainant in the criminal action for physical injuries thru reckless imprudence and who participated in the prosecution thereof without reserving the civil action arising from the act or omission complained of, can file a separate action for civil liability arising from the same act or omission where the herein petitioner was acquitted in the criminal action on reasonable doubt and no civil liability was adjudicated or awarded in the judgment of acquittal HELD YES - The action is based on a quasi-delict, the failure of the respondent to reserve his right to file a separate civil case and his intervention in the criminal case did not bar him from filing such separate civil action for damages. Ratio The allegations of the complaint filed by the private respondent supports and is constitutive of a case for a quasi-delict committed by the petitioner. The Court has also heretofore ruled in Elcano vs. Hill that: ... a separate civil action lies against the offender in a criminal act whether or not he is criminally prosecuted and found guilty or acquitted, provided that the offended party is not allowed, if he is also actually charged criminally, to recover damages on both scores; and would be entitled in such eventuality only to the bigger award of the two, assuming the awards made in the two cases vary. In other words, the extinction of civil liability referred to in Par. (c) of Sec. 3 Rule 111, refers

JIMENEZ v AVERIA 22 SCRA 380 DIZON; March 29, 1968


FACTS - Ofelia V. Tang and Estefania de la Cruz Olanday were charged with estafa in the CFI of Cavite with the information alleging that they misappropriated P20,000 received from Manuel Jimenez for the purchase of a fishing boat named Basnig. They also

Criminal Procedure Rowena Daroy Morales


have the obligation to return the money if they do not purchase the boat, which they did not do. - Before arraignment, the accused filed a civil suit against Jimenez in the Quezon CFI contesting the validity of a certain receipt signed by them on October 26, 1962 wherein they acknowledged having received from him the sum of P20,000.00 with which to purchase for him a fishing boat and its accessories, and the further sum of P240.00 as agent's commission, with the obligation, on their part, to return the aforesaid amounts on January 30, 1963 in case they were unable to buy the fishing boat. They assert now that they never received any amount from Jimenez and that they signatures were taken through the means of fraud and deceit by Jimenez - After a few days, they filed a motion to suspend the proceedings of the criminal case pending the resolution of the prejudicial question in the civil case whether or not their signatures were taken through means of fraud and deceit by Jimenez. - Judge Averia granted the motion and hence this certiorari petition - Pre-ratio: Jimenez erred in the filing of a certiorari petition, and should have filed a mandamus to the SC instead to compel the lower court to proceed with the case. ISSUE WON the determination of the issue raised in the civil case mentioned heretofore is a prejudicial question, in the sense that it must be first resolved before the proceedings in the criminal case for estafa may proceed HELD NO - The issue of fraud and deceit raised in the civil case does not constitute a prejudicial question. The criminal court must now try the estafa case against the two accused. Reasoning - A prejudicial question has been define to be one which arises in a case, the resolution of which, (question ) is a logical antecedent of the issued involved in said case, and the cognizance of which pertains to another tribunal. Simply put, the questions must be determinative of the case before the court, and that jurisdiction to try and resolve said question must be lodged in another tribunal. - Applying these to the case, it will be readily seen that the alleged prejudicial question is not determinative of the guilt or innocence of the

a2010

page 45

Prof.
on the civil action for the revocation of the management contract. He contended that a prejudicial question was involved, thus he could no longer be tried pending the termination of the civil suit. The respondents, in turn, contended that the resolution of the civil case will not determine the liability of Rojas in the criminal case (not a prejudicial question); and even granting that there was a prejudicial question, the cases could proceed independently pursuant to Art.33 of CC, which provides: In cases of defamation, fraud and physical injuries, a civil action for damages, entirely separate and distinct from the criminal action may be brought by the injured party. Such civil action shall proceed independently of the criminal prosecution, and shall require only a preponderance of evidence. ISSUE WON the is a prejudicial question, thus requiring the resolution of the civil action for the determination of the criminal case HELD NO Ratio: A prejudicial question, which is must be determinative of the case before the court, and jurisdiction to try the same must be lodged in another court, is not present in this case. Reasoning: - It is indispensable then for this petition to succeed that the alleged prejudicial question must be determinative of the criminal case before respondent Judge. It is not so in this case. - Pisalbor. v. Tesoro: CFI erred in holding that the criminal case should be suspended. In the present proceedings, the civil case does not involve a question prejudicial to the criminal case, for to whomsoever the land may be awarded after all the evidence has been presented in the civil case, may not affect the alleged crime committed by the notary public, which is the subject of the criminal case. But, even supposing that both the civil and the criminal case involve the same question and one must precede the other, it should be the civil case which should be suspended rather than the criminal, to await the result of the latter. - Dela Cruz v City Fiscal: Regardless of the outcome of the pending civil case for annulment of the affidavit of adjudication, determination of the charge of falsification would be based on the truth or falsity of the narration of facts in the affidavit of adjudication, * * *. Therefore, the civil case

parties charged with estafa, because even on the assumption that the execution of the receipt whose annulment they sought in the civil case was vitiated by fraud, duress or intimidation, their guilt could still be established by other evidence showing, to the degree required by law, that they had actually received from the complaint the sum of P20,000.00 with which to buy for him a fishing boat, and that, instead of doing so, they misappropriated the money and refused or otherwise failed to return it to him upon demand. The contention of the private respondents herein would be tenable had they been charged with falsification of the same receipt involved in the civil action. - If the ruling were otherwise, there would hardly be a case for estafa that could be prosecuted speedily, it being the easiest thing for the accused to block the proceedings by the simple expedient of filing an independent civil action against the complainant, raising therein the issue that he had not received from the latter the amount alleged to have been misappropriated.

ROJAS v PEOPLE (ALIKPALA) 57 SCRA 243 FERNANDO; May 31, 1974


NATURE Petition for certiorari and prohibition FACTS - Rojas was charged w/ violation of Art.319 (Removal, sale, pledge of mortgaged property) of RPC for executing a new chattel mortgage on personal property (Caterpillar Tractor) in favor of another party w/o the consent of the previous mortgagee. After the criminal case was instituted, a civil case was filed against him by the offended party (CMS Estate) for the termination of a management contract, one of the causes of action of which consisted of petitioner having executed a chattel mortgage when a prior chattel mortgage was still valid and subsisting, thus giving lie to his express manifestation that the property was free from all liens and encumbrances. - Note: the trigger for the filing of information re: art.319 violation was the filing of 5 estafa cases against Rojas. - CFI Judge Alikpala ordered the arraignment, then the trial for the criminal case. Rojas filed an action for certiorari against the arraignment order, and prohibition against the order setting the trial, based

Criminal Procedure Rowena Daroy Morales


aforementioned does not involve a prejudicial question. - Benitez v. Concepcion, Jr (more analogous): the fact that the principal issues in both cases are the same and did arise from the same facts would not show any necessity that the civil case be determined first before taking up the criminal case. - Isip v. Gonzales: there is a prejudicial question only when the matter that has to be priorly decided by another authority is one the cognizance of which pertains to that authority and should not, under the circumstances, be passed upon by the court trying the criminal case. - Moreover, Art.33 explicitly provides that in cases of xxx fraud, xxx, a civil action for damages entirely separate and distinct from the criminal action, may be brought by the injured party. Such civil action SHALL proceed independently of the criminal prosecution xxx. - in this case, fraud is the basis for both the civil and criminal actions, thus they are to proceed independently. The invocation of the doctrine of prejudicial question is thus attended with futility. Personal note: ang pangit ng case. theres realy no discussion, puro citations, thats why this digest is also full of it. Dispositive Petition DENIED.

a2010

page 46

Prof.
involves the same facts upon which the criminal prosecution is based, but also that the resolution of the issues raised in said civil action would be necessarily determinative of the guilt or innocence of the accused. - If the first alleged sale in favor of Pichel is void or fictitious, then there would be no double sale and petitioner would be innocent of the offense charged. A conviction in the criminal case (if it were allowed to proceed ahead) would be a gross injustice and would have to be set aside if it were finally decided in the civil action that indeed the alleged prior deed of sale was a forgery and spurious. Dispositive Order of respondent judge in Criminal Case dated December 12, 1978 is hereby set aside. The temporary restraining order issued by this Court on May 16, 1979 is hereby made permanent and respondent judge is enjoined from proceeding with the arraignment and trial of the criminal case unless the civil case shall have been finally decided and terminated adversely against petitioner.

> therefore the alleged deed of sale in Pichel's favor sought to be declared valid was fictitious and inexistent - September 5, 1978 - while Civil Case was being TRIED before CFI Basilan, the Provincial Fiscal of Basilan filed an INFORMATION for Estafa (criminal case) in the same court against Ras arising from the same double sale subject matter of the civil complaint filed by Luis Pichel. - November 6, 1978 - petitioner filed a MOTION FOR SUSPENSION OF ACTION in said Criminal Case claiming that same facts and issues were involved in both the civil and criminal case and that the resolution of the issues in the civil case would necessarily be determinative of the guilt or innocence of the accused. - December 4, 1978 - Provincial Fiscal of Basilan filed his opposition on - December 12, 1978 - respondent judge saw no prejudicial question and accordingly denied the motion ISSUE WON civil case would be prejudicial to the criminal case given that they would discuss same facts and issues HELD YES - there appears to be a prejudicial question in the case at bar, considering that Ras' defense in Civil Case of the nullity and forgery of the alleged prior deed of sale in favor of Pichel (plaintiff in the civil case and complaining witness in the criminal case) is based on the very same facts which would be necessarily determinative of Ras' guilt or innocence as accused in the criminal case. Ratio A prejudicial question is defined as that which arises in a case the resolution of which is a logical antecedent of the issue involved therein, and the cognizance of which pertains to another tribunal. The prejudicial question must be determinative of the case before the court but the jurisdiction to try and resolve the question must be lodged in another court or tribunal. It is a question based on a fact distinct and separate from the crime but so intimately connected with it that it determines the guilt or innocence of the accused. Reasoning - For a civil case to be considered prejudicial to a criminal action as to cause the suspension of the criminal action pending the determination of the civil, it must appear not only that the civil case

LIBRODO v COSCOLLUELA, JR. (GUANTERO) 116 SCRA 303 MELENCIO-HERRERA; August 30, 1982
NATURE Petition for certiorari to review Negros CFI order FACTS - Felipe Rivera died leaving certain properties in San Carlos, Negros Occidental. His estate was settled in a special proceeding on November 24, 1976 and was terminated on the basis of a Project of Partition among Rufino Rivera Damandaman, Democrata Guantero, and Zosimo Guantero. - Rufinos share of the estate comprise of lots designated as Lots 559-B, 1906-B, 1910-B, and a901B which were all sugar lands. On January 18, 1977, Rufino leased the properties to Dr. Librodo, the petitioner, for a period of ten agricultural crop years. - On August 31, Democrata filed a petition to re-open the intestate proceeding on the ground that she was not present when the subdivision plan was submitted and that the judgment has not become final as the boundaries on the partition have not been platted. - In the meantime, according to the petitioner, private respondents, Guanteros, harvested the sugar canes he planted on the land he leased from Rufino. On August 10, 1978, a Criminal Case (the Criminal Case) was filed against the respondents for theft

RAS v RASUL 100 SCRA 125 TEEHANKEE; September 18, 1980


NATURE Petition to review and set aside the order of respondent Judge dated December 12, 1978 of criminal case in CFI Basilan denying petitioner's motion as accused therein to suspend proceedings due to the existence of a prejudicial question in Civil Case of the same court FACTS - April 27, 1978 - Luis Pichel filed a COMPLAINT against Alejandro Ras and Bienvenido Martin before CFI Basilan praying for the nullification of the deed of sale executed by Ras in favor of Martin and for the declaration of the prior deed of sale allegedly executed in his favor by the defendant Alejandro Ras as valid. - RAS ANSWER > they never sold the property to Pichel > the signatures appearing in the deed of sale in favor of plaintiff Pichel were forgeries

Criminal Procedure Rowena Daroy Morales


demanding damages amounting to Pesos 15,120.00. During the pendency of the Criminal Case, another case for damages (the Damages Case) against the private respondents alleging damages to the petitioner caused by the private respondents theft of the sugar canes and their occupation of the leased properties thus preventing him from cultivating or taking possession of the same. He alleged that this resulted in his being deprived of income for two years amounting to Pesos 78,280.00. - In their answer, respondents asserted that the lots are still under co-ownership among the heirs and that this is the subject of another special proceeding (the Intestate Case). That said, Democrata contended that Rufino could not execute the lease contracts without her conformity without her conformity as co-owner. The Guanteros filed a motion to suspend the proceedings in the Criminal Case on the ground of pendency of the Damages Case, the Intestate Case, and the ejectment case (the Ejectment Case) which was filed by Rufino against Democrata on January 13, 1977. - The respondents took the position that the various cases focused on the issues of possession and ownership of the lots involved as well as of the improvements thereon, hence, determinative of their guilt in the criminal action and hence constitutive of a prejudicial question. - Despite the objections made by the petitioner, the lower court issued the order finding that a prejudicial question existed and suspending the Criminal case proceeding. Hence this appeal. ISSUE WON the issues raised in the three cases mentioned involve a prejudicial question that warrants a suspension of the Criminal Case HELD NO The issues raised in the three cases do not involve the pivotal question of who planted the sugar can and, therefore, are not determinative juris et jure of guilt or innocence in the Criminal Case. Reasoning - A prejudicial question is one based on a fact distinct and separate from the crime but so intimately connected with it that it determines the guilt or innocence of the accused., and for it to suspend the criminal action, it must appear not only that said case involves facts intimately related to those upon which the criminal prosecution would be based but also

a2010

page 47

Prof.
a motion for reinvestigation in the Tanodbayan. The same was granted. - The Tanodbayan ordered to dismiss the case for lack of merit and to withdraw the Information filed in Criminal Case No. 11414 as soon as possible in the interest of justice. - Tanodbayan filed with the Sandiganbayan a motion to withdraw the information against petitioners. This was denied. - BAlgos et al filed a motion to suspend proceedings in the criminal case against them on the ground of the existence of a prejudicial question in Civil Case No. 5307. This was likewise denied by the Sandiganbayan. ISSUE WON the denial by the Sandiganbayan of the motion to withdraw the information and of another motion to suspend proceedings on the ground of a prejudicial question in a pending civil action constitute a grave abuse of discretion. HELD NO. - While the public prosecutor has the sole direction and control in the prosecution of offenses, once the complaint or information is filed in court, the court thereby acquires jurisdiction over the case and all subsequent actions that may be taken by the public prosecutor in relation to the disposition of the case must be subject to the approval of the said court. Before a re-investigation of the case may be conducted by the public prosecutor, the permission or consent of the court must be secured. And if after such reinvestigation the prosecution finds a cogent basis to withdraw the information or otherwise cause the dismissal of the case, such proposed course of action must be addressed to the sound discretion of the court. - The only instance when the appellate court should stay the hand of the trial court in such cases is when it is shown that the trial court acted without jurisdiction or in excess of its jurisdiction or otherwise committed a grave abuse of discretion amounting to such lack or excess of jurisdiction. - Petitioners are public officers charged with having violated Section 3(c) of RA 3019, for evident bad faith and manifest partiality in enforcing the writ of execution in Civil Case No. 4047 against a Mustang car registered in the name of Leticia Acosta-Ang (complainant) who is not the judgment debtor thereby causing undue injury to said complainant

that in the resolution of the issue or issues raised in the civil case, the guilt or innocence of the accused would necessarily be determined. - In the case at bar, the issues raised would not constitute a prejudicial question to the Criminal Case. The Intestate Case involves only the co-heirs and the facts involved are totally unrelated to the Criminal Case. Even if the Intestate Court should annul the division and uphold the co-ownership, that would not be determinative of the criminal responsibility of private respondents for theft of the sugar cane, which petitioner claims he planted in good faith by virtue of the valid lease agreement. The Ejectment Case also does not constitute a prejudicial question to the Criminal Case. It involves the issue of possession between co-owners. A decision therein in favor of Democrata would not affect the rights of Librodo, which spring from the lease contract. With regard the Damages case, it is actually the civil aspect of the Criminal Case as the two cases are of the same facts, and the entitlement to damages being predicated on the unlawful taking treated of in the Criminal Case, no necessity arises for that civil Case to be determined ahead of the Criminal Case. Dispositive In the absence of a prejudicial question, the order of the judge is set aside and he is instructed to proceed without delay with the trial of the criminal case.

BALGOS v SANDIGANBAYAN 176 SCRA 287 GANCAYCO; August 10, 1989


NATURE Petition to review the decision of Sandiganbayan FACTS - Balgos et al were charged with violation of Section 3(c) of RA 3019, otherwise known as the Anti-Graft and Corrupt Practice Act, as amended, in an information that was filed with the Sandiganbayan by the Special Prosecutor which was approved by the Deputy Tanodbayan, after a preliminary investigation. - Lim, the plaintiff and prevailing party in Civil Case No. 4047 filed a complaint for rescission of the sale of the car by Juanito Ang to private respondent Leticia Acosta-Ang for being allegedly in fraud of creditors. The said complaint was filed with the RTC of Nueva Vizcaya. On the same day, petitioners filed

Criminal Procedure Rowena Daroy Morales


and giving unwarranted benefits to the judgment creditor in said case. - Upon reinvestigation of the criminal case by the Tanodbayan, he found evidence tending to show that the sale of said car to the complainant by Juanito Ang, the judgment debtor, was a sham intended to defraud his creditors; that the deed of absolute sale which ostensibly was executed before a notary public appeared to be fictitious inasmuch as the entry of the document in the notarial register of said notary public on said date referred to a catering contract of other parties; that the certificate of registration of the car was issued to complainant only on June 13, 1984 which showed that the document of sale was actually executed only on or about the same date, that is, seven days after Juanito Ang received copy of the adverse decision in Civil Case No. 4047; and that upon the execution of the judgment, the car was found in the possession of Alvin, the son of Juanito Ang, who admitted that the car belonged to his father by showing the receipt of its repair in the name of Juanito Ang. This is the basis of the motion for withdrawal of the information of the Tanodbayan. - The respondents are aware that the complainant is not a party to the civil case filed by the creditor against spouses Juanito and Lydia Ang and that a writ of execution cannot be implemented validly against one who is not a party to the action. All these, coupled with the under haste in which the levy on the Mustang car was made without first ascertaining the true owner thereof demonstrate quite convincingly the evident bad faith and manifest partiality of the respondents, thereby giving unwarranted benefits to the judgment creditor to the damage and prejudice of the complainant. - Although at the reinvestigation, the Tanodbayan was persuaded that in fact the sale of the car to Leticia Ang was fraudulent, this did not necessarily clear petitioners of the aforesaid Anti-Graft charge against them. Still the burden is on the petitioners to establish that they acted in good faith in proceeding with the execution on the car even they were presented evidence tending to show it did not belong to Juanito Ang anymore. - The denial of the motion to suspend the criminal proceedings on the ground of the pendency of a prejudicial question in Civil Case No. 5307 is well taken. The doctrine of prejudicial question comes into play usually in a situation where a civil action and a criminal action are both pending and there exists in the former an issue which must be preemptively resolved before the criminal action may proceed, because whatsoever the issue raised in the

a2010

page 48

Prof.
was set on September 4, 1981 but petitioners failed to appear. It was reset to October 5, 1981 but this was postponed upon motion of petitioners. - On October 14, 1981, OROSEA filed a Complaint in the Court of First Instance of Quezon against the Edano spouses for the annulment/rescission of the Contract of Sale for which the petitioners issued the checks, subject of the criminal case. - The estafa case was again set for arraignment. This was postponed. With the entry of a new counsel, petitioners filed a motion to quash the estafa case, on ground of improper venue, but this motion was withdrawn by petitioners before it could be resolved. - The arraignment was again postponed thrice. Petitioners then filed a 'Motion to Suspend Arraignment and Further Proceedings, with a Supplemental Motion To Suspend Proceedings. This was opposed by the Provincial Fiscal of Quezon. Resolving the motion to suspend, respondent Judge issued his orders, now under question, denying the motion. CFI of Zambales also denied the same motion. A petition for certiorari is filed with CA and CA affirmed. ISSUE WON proceedings should be suspended until the civil case is disposed of, since CV No. 8769 involves a prejudicial question. HELD NO. - CV No. 8769 seeks the annulment of the deed of sale in favor of Orosea on the gound that there was fraud in misrepresenting that the land is free from all liens and encumbrances, and that it is not tenanted, when in truth and fact, the land is covered by the land reform program and that vast portions thereof are timber land, hence, allegedly indisposable public land. Therefore, according to petitioners, CV No. 8769 involves issues, the resolution of which will determine whether or not petitioners are criminally liable in CR No. 1423-I. They further argue that, if and when the court hearing CV No. 7869 annuls the subject deed of sale, then, their obligation to pay private respondents under the said deed would be extinguished, resulting in the dismissal of CR No. 1423-I. The contracts are thus voidable with the existence of fraud vitiating their consent. - However, it cannot be denied that at the time the acts complained of in the estafa case were committed, the deed of sale they seek to be annulled, was still binding to the parties.

civil action is resolved would be determinative juris et jure of the guilt or innocence of the accused in the criminal case. - The pending civil case for the annulment of the sale of the car to Leticia Ang is not determinative of the guilt or innocence of the petitioners for the acts allegedly committed by them in seizing the car. Even if in the civil action it is ultimately resolved that the sale was null and void, it does not necessarily follow that the seizure of the car was rightfully undertaken. The car was registered in the name of Leticia Ang six months before the seizure. Until the nullity of the sale is declared by the courts, the same is presumptively valid. Thus, petitioners must demonstrate that the seizure was not attended by manifest bad faith in order to clear themselves of the charge in the criminal action. Dispositive The petition is DENIED for lack of merit and the restraining order dated June 6, 1989 is hereby lifted. No costs.

UMALI v IAC (EDANO) 219 SCRA 339 PADILLA; June 21, 1990
NATURE Review on certiorari FACTS - Petitioners (Umali, Calleja, Ledesma) are officers of the Orosea Devt Corporation. Sometime on Sept. 4, 1979, Umali purchased from spoused Homorio and Solina Edano a lot in Mulanay, Province of Queazon for P1, 036,500 payable on 4 installments (P225,000, P271,500, P270,000, P 270,000) They issued for this purpose 4 checks drawn against the Chartered Bank, Manila Branch. The first check for P225,000.00 was honored upon its presentment. By arrangement the petitioners made with the Edano spouses, a deed of absolute sale in the name of Orosea Devt Corp. was executed even of the full purchase price has not yet been fully paid. Thereafter, OROSEA secured a loan of P1,000,000.00 from the Philippine Veterans Bank using this property as security. When the check for the second installment fell due, petitioners twice asked for deferment. The checks they have issued were dishonored. As a consequence of the dishonor of these checks, the Edano spouses filed a complaint for estafa against petitioners. - The information was filed by the Provincial Fiscal against petitioners on May 21, 1981, and it was docketed as Criminal Case No. 1423-I. Arraignment

Criminal Procedure Rowena Daroy Morales


- The two (2) essential elements for a prejudicial question to exist are: (a) the civil action involves an issue similar or intimately related to the issue raised in the criminal action; and (b) the resolution of such issue in the civil action determines whether or not the criminal action may proceed. - Given the nature of a prejudicial question, and considering the issues raised in CV No. 8769 and CR No. 1423-I, we agree with the ruling of the respondent Court of Appeals that the resolution of the issues in CV No. 8769 is not determinative of the guilt or innocence of the petitioners-accused in CR No. 1423-I, hence, no prejudicial question is involved between the said two (2) cases. Dispositive WHEREFORE, the petition is DENIED. The decision dated 23 September 1982 of the Court of Appeals in CA-GR SP No. 14504 is hereby AFFIRMED.

a2010

page 49

Prof.
guilty parties. And such condition has been imposed "out of consideration for the offended woman and her family who might prefer to suffer the outrage in silence rather than go through with the scandal of a public trial." - The gist of petitioner's pretense is that there are some elements of the latter which are not included in the former, and, not alleged, according to him, in the complaint filed herein, namely: 1) that the offended party is a virgin; and 2) that she is over 12 and under 18 years of age. The second element is clearly set forth in said complaint, which states that Ester Ulsano is "a minor . . . 17 years of age . . .", and, hence, over 12 and below 18 years of age. - As regards the first element, it is settled that the virginity mentioned in Art 343 RPC, as an essential ingredient of the crime of abduction with consent, should not be understood in its material sense and does not exclude the idea of abduction of a virtuous woman of good reputation because the essence of the offense "is not the wrong done to the woman, but the outrage to the family and the alarm produced in it by the disappearance of one of its members." - The complaint in the case at bar alleges not only that Ester Ulsano is a minor 17 years of age, but also that petitioner "willfully, unlawfully and feloniously" took her by force and violence . . . against her will and taking advantage of the absence of her mother" from their dwelling and carried "her to a secluded spot to gain carnal intercourse with the offended party against her will, using force, intimidation and violence, with lewd designs." This allegation implies that Ester is a minor living under patria protestas, thus leading to the presumption that she is a virgin apart from being virtuous and having a good reputation. The presumption of innocence includes that of morality and decency, and of chastity. Dispositive Wherefore, the decision appealed from is hereby affirmed, with costs against the petitioner Maximino Valdepenas. It is so ordered.

matter of the action wrt the offense of abduction with consent. MFR was denied Petitioners claims there was no complaint for abduction with consent filed and that the lower court acquired no jurisdiction over his person or over the crime of abduction with consent. ISSUE WON CA erred in not reversing he decision of the TC for lack of jurisdiction over the accused and the subject matter of the action for the offense abduction with consent HELD NO. - Jurisdiction over the person of an accused is acquired upon either his apprehension, with or without warrant, or his submission to the jurisdiction of the court. It is not claimed that petitioner had not been apprehended or had not submitted himself to the jurisdiction of the court. His actions show that he never questioned the judicial authority of the CFI, the justice of peace and the CA. He is deemed to have waived whatever objection he might have had to the jurisdiction over his person, and, hence, to have submitted himself to the Court's jurisdiction. His behavior - particularly the motions therein filed by him implied, not merely a submission to the jurisdiction thereof, but also, that he urged the courts to exercise the authority thereof over his person. - On the other hand, it is well settled that jurisdiction over the subject matter of an action is and may be conferred only by law. That jurisdiction over a given crime, not vested by law upon a particular court, may not be conferred thereto by the parties involved in the offense; and that, under an information for forcible abduction, the accused may be convicted of abduction with consent. Art 344 (3) RPC states that: ". . . the offenses of seduction, abduction, rape or acts of lasciviousness, shall not be prosecuted except upon a complaint filed by the offended party or her parents, grandparents, or guardian, nor in any case, if the offended has been expressly pardoned by the above- named persons, as the case may be". - Art 344 RPC does not determine the jurisdiction of our courts over the offense therein enumerated. It could not affect said jurisdiction, because the same is governed by the Judiciary Act of 1948, not by RPC, which deals primarily with the definition of crimes and the factors pertinent to the punishment of the culprits. The complaint required in said Article 344 is merely a condition precedent to the exercise by the proper authorities of the power to prosecute the

VALDEPENAS V PEOPLE 16 SCRA 871 CONCEPTION; April 30, 1966


NATURE Appeal by Maximino Valdepenas from a decision of the CA, affirming that of the CFI of Cagayan, convicting him of the crime of abduction with consent. FACTS - Jan 25, 56 Ester Ulsano filed with the justice of peace a criminal complaint charging Valdepenas with forcible abduction with rape of Ester Ulsano. After the preliminary investigation, the second stage of which was waived by Valdepenas, the justice of peace found that there was probable cause and forwarded the complaint to the CFI. - CFI found him guilty as charged and sentenced him accordingly. - On appeal, CA modified the decision, convicting him of abduction with consent. - Valdepenas filed MFR and new trial contesting the findings of CA, to the effect that complainant was below 18 y/o at the time of the occurrence. Motion was granted. The decision was set aside and the case was remanded to the CFI - CFI rendered decision reiterating findings of CA. Petitioner again appealed to CA which affirmed the CFI decision. - MFR was filed on the ground that lower court had no jurisdiction over the person and the subject

PEOPLE v PLATEROS 83 SCRA 401 AQUINO; May 30 1978


FACTS One night, Pedro Candel together with other pedicab drivers and Tomas Metucua, a second year college student drank beer in the kitchenette. Seated at another table were Warlito Plateros and Murillo Lahoy who were also drinking beer.

Criminal Procedure Rowena Daroy Morales


- Metucua and Plateros were rivals for the affection of Estrella Silamro, the cashier in the kitchenette. When Metucua was talking with Estrella, his alleged sweetheart, Plateros went near them and refused to leave them, thereby annoying Metucua. - At about midnight. Piquero, Candel and Aora, accompanied by Metucua, left the kitchenette and went to their pedicab. Candel was seated in the sidecar of the tricycle. Metucua sat on the driver's seat. Lahoy and Plateros came out of the kitchenette. Lahoy appeared to be angry, hostile and menacing. Without any warning, he stabbed Candel (maybe thinking that it was Metucua who was inside the pedicab because Candel is the driver abberatio personae) two times. Plateros also stabbed Candel. Moved by the instinct of selfpreservation, Candel jumped out of the sidecar. He fell on the ground face down. Lahoy allegedly stabbed Metucua. Then, Plateros and Lahoy fled from the scene of the assault, - Candel was brought to the hospital but he died on that same morning. Procedure - Two informations were filed in the Court of First Instance of Bohol accusing Plateros and Lahoy of (1) Murder of Candel and (2) Attempted Murder of Metucua. - The trial court tried the two cases jointly and rendered only one decision. Plateros and Lahoy were found guilty of murder, sentencing each of them of reclusion perpetua. In that same decision, the trial court convicted Lahoy of attempted murder (Plateros, his co-accused, was acquitted) of Metucua. - Lahoy appealed to the Court of Appeals and the CA acquitted him. - The murder case was elevated to SC for review. Together with it, the Solicitor General elevated the attempted murder case be he believed that the decision of CA is void because Lahoys appeal ought to have been certified to the Supreme Court by the CA because the attempted murder imputed to Lahoy was committed on the same occasion and arose out of the same occurrence as the murder imputed to him and Plateros in this case, as contemplated in section 17(1), formerly section 17(4) of the Judiciary Law, which reads: "SEC. 17, Jurisdiction of the Supreme Court. -x x x " x x xx x x x x x "The Supreme Court shall have exclusive jurisdiction to review, revise, reverse, modify or affirm on appeal, as the law or rules of court may provide, final judgments and decrees of inferior courts herein provided, in "(1) All criminal cases involving offenses for which the penalty

a2010

page 50

Prof.
between the decision of the Court of Appeals and this Court's decision in the instant murder case inasmuch as the victims in the two cases are different. The attempted murder case decided by the Court of Appeals involved the wounding of a certain Tomas Metucua whereas, in the instant murder case the victim was Pedro Candel. The acquittal of Lahoy in connection with the wounding of Metucua would not affect the determination of his guilt or innocence in connection with the death of Pedro Candel. - This holding does not in anyway emasculate the rule in section 17(1) that criminal cases appealed to the Court of Appeals, involving offenses which arose out of the same occurrence, or which were committed on the same occasion as the offense punished by death or reclusion perpetua should be certified to this Court by the Court of Appeals. It is this Court that would determine whether or not the cases appealed to the Court of Appeals should be decided together with the case appealed to this Court. 2. YES The feeble denials of Plateros and Lahoy (who admittedly were near the owns of the crime, when it was perpetrated) cannot prevail over the positive and unequivocal declarations of the eyewitnesses, Aora and Piquero, that the appellants were the authors of the stab wounds which caused Candal's death. Their guilt was proven beyond reasonable doubt. 3. YES. There was a conspiracy between Plateros and Lahoy as shown in their concerted efforts to injure Candel. Plateros and Lahoy, as boon companions, had been together since four o'clock in the afternoon. They had gone to different places and repaired twice to the kitchenette. They were together when they left the scene of the stabbing. 4. NO Lahoy and Plateros, who could have stabbed Candel or Metucua inside the kitchenette, did not do so. They waited for Metucua and the pedicab drivers to leave the kitchenette. Their intention was to make a surprise attack without any risk to themselves. The assault was deliberate, sudden and unexpected. That is the characteristics manifestation of treachery (alevosia). Hence, the killing was properly categorized as murder by the trial court (Art. 14(16), Revised Penal Code). Dispositive WHEREFORE, the trial court's judgment is affirmed with costs against the appellants. They are entitled to credit for their preventive

imposed is death or life imprisonment; and those involving other offenses which, although not so punished, arose out of the same occurrence or which may have been committed by the accused on the same occasion, as that giving rice to the more serious offense, regardless of whether the accused are charged as principals, accomplices or accessories, or whether have been tried jointly or separately; x x x." - In other words, the attempted murder case like the instant murder case, comes within the exclusive appellate jurisdiction of the SCt and should have been decided together with the instant murder case. ISSUES On Attempted Murder Case 1. WON the decision of the Court of Appeals acquitting Lahoy of attempted murder should be set aside for lack of appellate jurisdiction or as a lawless thing On Murder case 2. WON the guilt of Lahoy and Plateros was proven beyond reasonable doubt 3. WON there was conspiracy between Lahoy and Plateros 4. WON the crime should be categorized as simple homicide only and not murder HELD 1. NO. Ratio: The rule in section 17(1) is designed to avoid conflicts between the decisions of this Court and the Court of Appeals in cases involving offenses which arose from the same occurrence or which were committed on the same occasion usually by the same accused. However, that general rule has an exception. Where, by allowing the Court of Appeals to decide a can involving an offense, which is not punishable by death or reclusion perpetua but which arose out of the same occurrence or was committed on the same occasion, as the case involving an offense punishable by death or reclusion perpetua pending in this Court, there will be no conflict between the decisions of this Court and the Court of Appeals, the former case need not be elevated to this Court (People vs. Cario, 101 Phil. 1206). The rationale of that exception to the general rule is found in the maxim: Cessanie ratione legis, cessat et ipsa lex. (The reason for the law ceasing, the law itself also ceases.) Reasoning: - The doctrine of the Cario case may be applied in this case because here there can be no conflict

Criminal Procedure Rowena Daroy Morales


imprisonment under the conditions laid down in article 29 of the Revised Penal Code.

a2010

page 51

Prof.
prescribe in accordance with the following rules: . . . Violations penalized by municipal ordinances shall prescribe after two months. Section 2. Prescription shall begin to run from the day of the commission of the violation of the law, and if the same be not known at the time, from the discovery thereof and the institution of judicial proceedings for its investigation and punishment. The prescription shall be interrupted when proceedings are instituted against the guilty person, and shall begin to run again if the proceedings are dismissed for reasons not constituting jeopardy. - Petitioner concludes that as the information was filed way beyond the two-month statutory period from the date of the alleged commission of the offense, the charge against her should have been dismissed on the ground prescription. Prosecutions position The prosecution contends that the prescriptive period was suspended upon the filing of the complaint against her with the OPP. The SolGen invokes Section 1, Rule 110 of the 1985 Rules on Criminal Procedure (RCP), providing as follows: Section 1. How Instituted For offenses not subject to the rule on summary procedure in special cases, the institution of criminal action shall be as follows: b) For offenses falling under the jurisdiction of the MTC and MCTC, by filing the complaint directly with the said courts, or a complaint with the fiscal's office. However, in Metropolitan Manila and other chartered cities, the complaint may be filed only with the office of the fiscal. In all cases such institution interrupts the period of prescription of the offense charged. Respondent maintains that the filing of the complaint with the OPP comes under the phrase "such institution" and that the phrase "in all cases" applies to all cases, without distinction, including those falling under the RSP. ISSUE WON the offense has prescribed HELD YES - The filing of the complaint in the MTC, even if it be merely for purposes of preliminary examination or investigation, should, and does, interrupt the period of prescription of the criminal responsibility, even if the court where the complaint or information is filed can not try the case on its merits. Even if the court where the complaint or information is filed may only

PEOPLE v LAGON 185 SCRA 442 FELICIANO.: May 18, 1990


FACTS -On July 7 1976 a criminal action was filed with the City Court of Roxas charging Lagon with estafa for allegedly issuing a P4,232 check as payment for goods knowing she had insufficient funds. However on Dec. 2, as the trial commenced, the City Court dismissed the information on the ground that the penalty prescribed by law for estafa was beyond the courts authority to impose. Hence this petition for review. ISSUE WON the City Court had jurisdiction over the case HELD NO - It is settled doctrine that jurisdiction of a court in criminal law matters is determined by the law in effect at the time of the commencement of the criminal action and not the law in effect at the time of the commission of the offense charged. -Under Sec 87 of the Judiciary Act of 1948, municipal judges in the capitals of provinces and sub-provinces and judges of city courts shall have like jurisdiction as the CFI to try parties charged with an offense within their respective jurisdictions, in which penalties provided do not exceed prision correccional or fines no exceeding P6,000 or both. -At the time of the commission of the crime, the imposable penalty under Art 315 of the RPC was arresto mayor in its maximum period to prision correccional it is minimum period, falling well within the jurisdiction of the City Court. But when the information was filed, PD 818 had increased the imposable penalty to prision mayor in its medium period. -The real question raised by petitioner is whether the said doctrine disregards the rule against retroactivity of penal laws. It has been repeatedly held that in criminal prosecutions, jurisdiction is not determined by what may be meted out to the offender in after trial but by the extent of the penalty which the law imposes. Once jurisdiction is acquired by the Court in which the information is filed, it is retained regardless of whether the evidence proves a lesser

offense which carries a penalty that would otherwise fall within the jurisdiction of an inferior court. -In the instant case, should the information be refiled with the RTC, the court may not impose a more onerous penalty upon Lagon. Although the RTC retains subject-matter jurisdiction to try and decide the refiled case under PD 818, given the date of the commission of the crime (before effectivity of PD 818), the lower penalty provided in Art 315 (otherwise within the jurisdiction of the City Court) should be imposed. Dispositive WHEREFORE, the Court resolved to DENY the petition

ZALDIVIA V REYES, JR. 211 SCRA 277 CRUZ; July 3, 1992


NATURE Petition for review on certiorari FACTS The petitioner Lus Zaldivia is charged with quarrying for commercial purposes without a mayor's permit in violation of Ordinance No. 2, Series of 1988, of the Municipality of Rodriguez, in the Province of Rizal, allegedly committed on May 11, 1990. The referral-complaint of the police was received by the Office of the Provincial Prosecutor (OPP) of Rizal on May 30, 1990 and the information was filed with the MTC of Rodriguez, presided by Judge Andres Reyes, Jr., on October 2, 1990. - The petitioner moved to quash the information on the ground that the crime had prescribed, but the motion was denied. On appeal, the RTC of Rizal affirmed the denial of the motion. Petitioners claims In this petition, the petitioner argues that the charge against her is governed by the following provisions of the Rule on Summary Procedure (RSP): Section 1. Scope. This rule shall govern the procedure in the MetTC, the MTC, and the MCTC in the following cases: B. Criminal Cases: 3. Violations of municipal or city ordinances; . - Petitioner also invokes Act No. 3326, "An Act to Establish Periods of Prescription for Violations Penalized by Special Acts and Municipal Ordinances and to Provide When Prescription Shall Begin to Run," reading as follows: Section 1. Violations penalized by special acts shall, unless otherwise provided in such acts,

Criminal Procedure Rowena Daroy Morales


proceed to investigate the case, its actuations already represent the initial step of the proceedings against the offender. It is important to note that this decision was promulgated on May 30, 1983, two months before the promulgation of the RSP on August 1, 1983. On the other hand, Section 1 of Rule 110 is new, having been incorporated therein with the revision of the RCP on January 1, 1985, except for the last paragraph, which was added on October 1, 1988. - Sec. 1 of the RCP begins with the phrase, "for offenses not subject to the rule on summary procedure in special cases," which plainly signifies that the section does not apply to offenses which are subject to summary procedure. The phrase "in all cases" appearing in the last paragraph obviously refers to the cases covered by the Section, that is, those offenses not governed by the RSP. - The charge against the petitioner, which is for violation of a municipal ordinance of Rodriguez, is governed by the RSP and not the RCP. - Where paragraph (b) of the section does speak of "offenses falling under the jurisdiction of the MTC and MCTC," the obvious reference is to Section 32 (2) of B.P. No. 129, vesting in such courts: (2) Exclusive original jurisdiction over all offenses punishable with imprisonment of not exceeding four years and two months, or a fine of not more than four thousand pesos, or both such fine and imprisonment, regardless of other imposable accessory or other penalties, including the civil liability arising from such offenses or predicated thereon, irrespective of kind, nature, value, or amount thereof; Provided, however, That in offenses involving damage to property through criminal negligence they shall have exclusive original jurisdiction where the imposable fine does not exceed twenty thousand pesos. - These offenses are not covered by the RSP. Under Section 9 of the RSP, "the complaint or information shall be filed directly in court without need of a prior preliminary examination or preliminary investigation." Both parties agree that this provision does not prevent the prosecutor from conducting a preliminary investigation if he wants to. However, the case shall be deemed commenced only when it is filed in court, whether or not the prosecution decides to conduct a preliminary investigation. This means that the running of the prescriptive period shall be halted on the date the case is actual filed in court and not on any date before that. - This interpretation is in consonance with the aforequoted Act No. 3326 which says that the period of

a2010

page 52

Prof.
charging petitioners with crime of falsification of private document. Allegedly, Aurora and Angelina made it appear that they were the guardians of minors George and Alexander Meijia (sons of the spouses?) when they werent the guardians at the date of the execution of the document, a certain Carolina M. de Castro was the judicial guardian of the said minors). -Petitioners asked for a reinvestigation. Angeles City Fiscal reinvestigated to give them opportunity to present exculpatory evidence. After reinvestigation, parties charged moved for the dismissal of the case mainly on the ground that the City Court of Angeles had no jurisdiction over the offense because the private document that contained the alleged false statement of fact was signed by them outside the territorial limits of said city (One in Makati, the other one in QC). -However, the resolution of their motion to dismiss was delayed and the City Court already set their criminal case for arraignment. Petitioners secured several postponements of the arraignment. But since City Fiscal continually failed to act on their motion to dismiss, petitioners filed a motion to quash instead, on the ground that court had no jurisdiction. Respondents (with conformity of City Fiscal) filed an opposition to the motion to quash. Respondent judge denied motion to quash, set arraignment. So petitioners filed present action. ISSUE 1. WON City Court of Angeles City had jurisdiction to try and decide the criminal case for alleged falsification of a private document allegedly done by the parties named in the info even if the acts of falsification was allegedly done in Makati and QC, and thus outside the jurisdiction of said court Other procedural issues 2. WON the motion to quash was improper, and should not be allowed since by filing the said motion, the petitioners necessarily assumes the truth of the allegation of the information to the effect that the offense was committed within the territorial jurisdiction of Angeles City 3. WON the prayer for writs of certiorari and prohibition is proper HELD 1. NO. Ratio. The place where the criminal offense was committed not only determines the venue of the action but is an essential element of jurisdiction [US vs. Pagdayuman].

prescription shall be suspended "when proceedings are instituted against the guilty party." The proceedings referred to in Section 2 thereof are "judicial proceedings," contrary to the submission of the SolGen that they include administrative proceedings. At any rate, the Court feels that if there be a conflict between the RSP and the RCP, the former should prevail as the special law. And if there be a conflict between Act No. 3326 and the RCP, the latter must again yield because this Court, in the exercise of its rule-making power, is not allowed to "diminish, increase or modify substantive rights" under Article VIII, Section 5 (5) of the Constitution Prescription in criminal cases is a substantive right. - The prescriptive period for the crime imputed to the petitioner commenced from its alleged commission on May 11, 1990, and ended two months thereafter, on July 11, 1990, in accordance with Section 1 of Act No. 3326. It was not interrupted by the filing of the complaint with the OPP on May 30, 1990, as this was not a judicial proceeding. The judicial proceeding that could have interrupted the period was the filing of the information with the MTC of Rodriguez, but this was done only on October 2, 1990, after the crime had already prescribed. Dispositive Petition is GRANTED. Case is DISMISSED on the ground of prescription.

LOPEZ v CITY JUDGE 18 SCRA 616 DIZON, October 29, 1966


NATURE Petition for review on Certiorari and Prohibition FACTS -Petitioners (Roy Villasor, Angelina Meijia Lopez and Aurora Mejia Villasor) and other heirs of spouses Manuel Meijia and Gloria Lazatin entered into a contract with respondent Trinidad Lazatin for the development and subdivision of 3 parcels of land belonging to the intestate estate. Lazatin transferred his rights to Terra Devt Co (TDC). -Petitioners and co-heirs filed an action in CFI QC for rescission of said contract with Lazatin for alleged gross and willful violation of its terms. -Respondents (Lazatin and TDC) filed with Fiscals Office of City of Angeles a complaint against petitioners for violation of A172 in relation to A171, par4, RPC. Preliminary investigation conducted. Fiscal filed with Court in Angeles City information

Criminal Procedure Rowena Daroy Morales


Reasoning. Petitioners are charged with having falsified a private document, not using a falsified document, so it is essential to determine when and where the offense of falsification of a private document is deemed consummated or committed. The crime of falsification of a private document is consummated when such document is actually falsified with the intent to prejudice a 3rd person, whether such falsified document is or is not put to use illegally. The improper and illegal use of the document is not material or essential element of the crime of falsification of a private document [US vs. Infante, US vs. Barreto] 2. NO Ratio. The motion to quash now provided for in Rule 117 of the Rules of Court is manifestly broader in scope than the demurrer, as it is not limited to defects apparent upon the face of the complaint or information but extends to issues arising out of extraneous facts, as shown by the circumstance that, among the grounds for a motion to quash, Section 2 of said Rule provides for former jeopardy or acquittal, extinction of criminal action or liability, insanity of the accused etc., which necessarily involve questions of fact in the determination of which a preliminary trial is required. Reasoning. The argument of the respondents refer to the now obsolete demurrer to an information. 3. YES Ratio. The general rule is that a court of equity will not issue a writ of certiorari to annul an order of a lower court denying a motion to quash, nor issue a writ of prohibition to prevent said court from proceeding with the case after such denial, it being the rule that upon such denial the defendant should enter his plea of not guilty and go to trial and, if convicted, raise on appeal the same legal questions covered by his motion to quash. In this as well as in other jurisdictions, however, this is no longer the hard and fast rule. -The writs of certiorari and prohibition, as extraordinary legal remedies, are, in the ultimate analysis, intended to annul void proceedings; to prevent the unlawful and oppressive exercise of legal authority and to provide for a fair and orderly administration of justice. Reasoning. In several cases, the court already took cognizance of said writs, overlooking the flaw in the procedure followed in the interest of a more enlightened and substantial justice. The lack of jurisdiction of the City Court of Angeles is patent and it would be highly unfair to compel the petitioners to

a2010

page 53

Prof.
who was also charged with estafa, in his capacity as the President of Yabut Transit Lines. The exact same thing happened in his case (motion to quash -> improper venue reason -> quashed -> MFR denied). ISSUE 1. WON CFI Bulacan had jurisdiction over the case 2. WON new law punishes the postdating or issuance thereof in payment of a pre-existing obligation 3. WON facts charged in the informations constitute estafa HELD 1. YES Ratio Estafa by postdating or issuing a bad check under Art. 315 par 2(d) of the RPC may be a transitory or continuing offense. Its basic elements of deceit and damage may independently arise in separate places. In the event of such occurrence, the institution of the criminal action in either place is legally allowed. - The venue of the offense lies at the place where the check was executed and delivered to the payee. Reasoning Section 14(a), Rule 110 of the ROC: In all criminal prosecutions the action shall be instituted and tried in the Court of the municipality or province wherein the offense was committed or any one of the essential ingredients thereof took place. - The estafa charged in the 2 informations involved here appear to be transitory or continuing in nature. Deceit has taken place in Malolos (thru issuance and delivery of worthless checks), while the damage in Caloocan, where the checks were dishonored by the drawee banks there. - The place where the bills were written, signed or dated does not necessarily fix the place where they were executed. What is decisive is the delivery of the instrument which is the final act essential to its consummation as an obligation. - The receipt of the bad checks by a certain Yambao in Caloocan cannot be taken as delivery of the checks to Freeway Tires because he did not take delivery of the checks as holder. - Place of business of Freeway Tires is at Malolos, Bulacan from where the tire and gas purchases were made by the private respondents. Payment should then be considered effected there. 2. YES - Due to the absence of concrete evidence on the specific nature of the obligation assumed or supposedly discharged by the issuance of the bad checks, resolution of this controversial issue on the

undergo trial in said court and suffer all the embarrassment and mental anguish that go with it. Dispositive WHEREFORE, judgment is hereby rendered declaring that the offense charged in the information filed in Criminal Case No. C-2268 of the City Court of Angeles City is not within the jurisdiction of said court and that, therefore, said court is hereby restrained and prohibited from further proceedings therein. Costs against the private respondents.

PEOPLE v YABUT 76 SCRA 624 MARTIN; April 29, 1977


NATURE Petition for review on certiorari of Orders of CFI Bulacan FACTS - Cecilia YABUT was accused of ESTAFA by means of false pretenses before the CFI Bulacan. She, as treasurer of the Yabut Transit Lines, made out 3 checks in the total sum of P6, 568.94 drawn against the Merchants Banking Corp (located in Caloocan City), payable to Freeway Tires Supply. The checks were dishonored because of insufficient funds. Yabut failed to deposit the necessary funds to cover the checks. - Instead of entering a plea, YABUT filed a MOTION TO QUASH contending that: (1) the acts charged do not constitute the offense as there is no allegation that the postdated checks were issued and delivered to the complainant prior to or simultaneously with the delivery of the merchandise; (2) estafa is not indictable when checks are postdated or issued in payment of pre-existing obligations; (3) venue was improperly laid because checks were issued and received by complainant in Caloocan, Yabuts office. - The People opposed and maintained that new law on checks, RA 4885, amending Art. 315 par.2(d) RPC, penalizes the postdating and that Malolos court can exercise jurisdiction since the last ingredient of the case, damage, transpired in Bulacan (residence of the complainant) after the dishonor of the checks for lack of funds. - The judge quashed the information for the reason of improper venue. The other issue was not resolved by the judge. - Peoples MFR for this dismissal was denied. ** This is actually a decision for two petitions: the other case involved Cecilias husband, GEMINIANO

Criminal Procedure Rowena Daroy Morales


basis of the averments in the informations alone is not ripe. 3. YES Reasoning In considering a motion to quash based on the ground that the facts charged do not constitute an offense, the point of resolution is whether the facts alleged, if hypothetically admitted, would meet the essential elements of the offense as defined in the law. Facts alleged should be taken as they are. Dispositive Appealed orders ordering the quashal of the estafa informations against the two private respondents are reversed and set aside. Arraignment of the private respondents in the criminal cases should be set at the earliest date, and thereafter, the trial on the merits to proceed immediately.

a2010

page 54

Prof.
"The criminal and civil action for damages in cases of written defamations as provided for in this chapter, shall be filed simultaneously or separately with the court of first instance of the province or city where the libelous article is printed and first published or where any of the offended parties actually resides at the time of the commission of the offense: "Provided, however, That where one of the offended parties is a public officer whose office is in the City of Manila at the time of the commission of the offense, the action shall be filed in the Court of First Instance of the City of Manila or of the city or province where the libelous article is printed and first published, and in case such public officer does not hold office in the City of Manila, the action shall be filed in the Court of First Instance of the province or city where he held office at the time of the commission of the offense or where the libelous article is printed and first published "Preliminary investigation of criminal actions for written defamations as provided for in the chapter shall be conducted by the provincial or city fiscal of the province or city, or by the municipal court of the city or capital of the province where such actions may be instituted in accordance with the provisions of this article. ". . . ." (As amended by Republic Act Nos. 1289 and 4363) - Before article 360 was amended, the rule was that a criminal action for libel may be instituted in any jurisdiction where the libelous article was published or circulated, irrespective of where it was written or printed. Under that rule, the criminal action is transitory and the injured party has a choice of venue. -Experience had shown that under that old rule the offended party could harass the accused in a libel case by laying the venue of the criminal action in a remote or distant place. -Republic Act No. 4363 was enacted so as to prevent the offended party in written defamation cases from inconveniencing the accused by means of out-oftown libel suits, meaning complaints filed in remote municipal courts -The rules on venue in article 360 may be restated thus: 1. Whether the offended party is a public official or a private person, the criminal action may be filed in the Court of First Instance of the province or city

SEPARATE OPINION TEEHANKEE [concurring]


- The motion to quash on the ground of improper venue must yield to the express allegations of the informations, bearing in mind that what determines jurisdiction are the allegations in the information and that venue is sufficiently conferred wherein any one of the essential ingredients of the offense charged took place. It also imports on the part of the accused a hypothetical admission of the facts alleged in the information.

AGBAYANI v SAYO 89 SCRA 699 AQUINO; April 30, 1979


NATURE Instant petition for certiorari and prohibition FACTS -Conrado B. Mahinan, a lawyer, was the manager of the Cagayan Valley Branch of the Government Service Insurance System (GSIS) stationed at Cauayan, Isabela. Among his subordinates were Wilson Agbayani, Carmelo N. Bautista, Pablo R. Pascual, and Renato Romeo P. Dugay. -On March 8, 1976, Mahinan filed with the fiscal's office at Bayombong, Nueva Vizcaya a complaint for written defamation against Agbayani, Bautista, Pascual and Dugay. -On July 23, 1976, the provincial fiscal of Nueva Vizcaya filed in the Court of First Instance of that province an information for libel charging Agbayani, Bautista, Pascual and Dugay with having maliciously

made defamatory imputations against Mahinan on or about February 17, 1976 in Bambang, Nueva Vizcaya. -Quoted in the information were the affidavits of Pascual and Bautista signed at Cauayan, Isabela, Bautista's undated letter asking for Mahinan's dismissal, and Agbayani's "unusual incident report" subscribed and sworn to before a Manila notary and enclosing documentary evidence to support his charges of malversation and falsification against Mahinan and praying for the latter's separation from the service. -According to the information, all those documents allegedly depicted Mahinan "as an incorrigible managerial misfit, despoiler of public office, spendthrift of GSIS funds, inveterate gambler, chronic falsifier", and an "unreformed ex-convict". -The four accused filed a motion to quash contending that the Court of First Instance of Nueva Vizcaya has no jurisdiction over the offense charged because Mahinan was a public officer holding office at Cauayan, Isabela when the alleged libel was committed and, under Article 360 of the Revised Penal Code, the offense charged comes within the jurisdiction of the Court of First Instance of Isabela. They argued that the provincial fiscal of Nueva Vizcaya had no authority to conduct the preliminary investigation and to file the information. -It was denied by the trial court in its order of April 25, 1977 on the ground that Mahinan was not a public officer within the meaning of article 203 of the Revised Penal Code since the insurance business of the GSIS is not an inherently governmental function. -After petitioners' motion for the reconsideration of that order was denied, they filed in this Court the instant petition. ISSUE WON the CFI of Nueva Ecija was the proper venue of the criminal action for written defamation filed by Mahinan HELD NO -There is no issue as to whether Mahinan is a public officer. As GSIS branch manager, he is unquestionably a public officer. -Article 360, which lays down the rules on venue in cases of written defamation and which specifies the officer or court that should conduct the preliminary investigation, reads as follows: ART. 360.Persons responsible. . . .

Criminal Procedure Rowena Daroy Morales


where the libelous article is printed and first published. 2. If the offended party is a private individual, the criminal action may also be filed in the Court of First Instance of the province where he actually resided at the time of the commission of the offense. 3. If the offended party is a public officer whose office is in Manila at the time of the commission of the offense, the action may be filed in the Court of First Instance of Manila. 4. If the offended party is a public officer holding office outside of Manila, the action may be filed in the Court of First Instance of the province or city where he held office at the time of the commission of the offense. -As a corollary, the preliminary investigation of the criminal action for written defamation shall be conducted by the provincial or city fiscal of the province or city, or by the municipal court of the city or capital of the province where such action may be instituted. -Applying the foregoing rules, the proper venue of Mahinan's criminal action for written defamation against the petitioners is the Court of First Instance of Isabela, since as a GSIS branch manager, he was a public official stationed at Cauayan, Isabela and the alleged libel was committed when he was (as he still) in the public service. The preliminary investigation of the complaint should have been conducted by the provincial fiscal of Isabela, or by the municipal judge of Ilagan, the provincial capital, or by the Court of First Instance of the same province. -The criminal action could have been filed also in the Court of First Instance of the province or in the city court of the city where the libel was printed and first published. -The information in this case is defective or deficient because it does not show that the Court of First Instance of Nueva Vizcaya, where it was filed, has jurisdiction to entertain the criminal action for written defamation initiated by Mahinan against the petitioners and that the provincial fiscal of that province had the authority to conduct the preliminary investigation. -Venue in criminal cases is an essential element of jurisdiction Dispositive Petition granted. The trial court's order denying petitioners' motion to quash is set aside. It is directed to dismiss Criminal Case No. 509, the libel case against the petitioners, without prejudice to the filing of another criminal action for written defamation in the Court of First Instance of Isabela

a2010

page 55

Prof.
WON CFI of Manila has jurisdiction to continue with the trial of the offense as charged in view of the evidence presented by the prodecution HELD YES. - Rule 110 of the Revised Rules of Court, Sec. 14(a) provides: "Sec. 14. Place where action is to be instituted. (a) In all criminal prosecutions, the action shall be instituted and tried in the court of the municipality or province wherein the offense was committed or any one of the essential ingredients thereof took place. -Petitioner could have been charged and tried in Cagayan de Oro City for it is not disputed that he received the sweepstakes tickets from the PCSO, Cagayan de Oro branch. The essential ingredient of receiving the sweepstakes tickets took place in Cagayan de Oro City. He could also be charged in the City of Manila since the final accounting must be rendered in the Central Office, Manila. This is therefore, a case of concurrent jurisdiction by the proper court of the place wherein "anyone of the essential ingredients thereof took place." But the choice of venue lies with the prosecuting officer and not with the accused. Dispositive Decision of CA Affirmed. Remand to the trial court for further proceedings in the ordinary course of law

CATINGUB v CA (PCSO) 121 SCRA 106. GUERRERO; March 25, 1983.


NATURE This is an appeal by certiorari from the decision of the Court of Appeals in CA-G.R. No. 38698-R entitled "PEDRITO L. CATINGUB, Petitioner, versus HON. RICARDO C. PUNO, Judge of the CFI Manila, Branch 24, and the PHILIPPINE CHARITY SWEEPSTAKES OFFICE, Respondents." FACTS - Catingub was designated Temporary Sales Supervisor of the Philippine Charity Sweepstakes Office (PCSO) assigned at the Cagayan de Oro Branch. As such, he received sweepstakes tickets on consignment, with the express obligation to turn over the proceeds of the sales of these tickets to the Philippine Charity Sweepstakes Office. Later, he was informed by the Auditing Examiner of the PCSO, Cagayan de Oro Branch that he has been found short of P12,307.45. Petitioner was ordered to explain the shortage in writing and to produce the missing amount. He failed to do so. His services were terminated without prejudice to whatever court action the PCSO will take for the recovery of the amount involved. In a letter, petitioner proposed to the General Manager of the PCSO, Manila, to settle his shortages by making monthly payments in the amount of at least P200.00, which proposal was, however, denied by the General Manager (there was already an admission in this letter). -Catingub, was charged with the crime of malversation (take note: crimes of estafa and malversation are similar in nature: difference is that the funds in malversation are public in character) in the Court of First Instance of Manila. He filed motion to dismiss after arraignment on the sole ground that "the prosecution made a wrong choice of jurisdiction." He contended that "on the basis of the prosecution's evidence, the offense charged, together with all its essential ingredients occurred and the consummation thereof (was) completed, in Cagayan de Oro. -TC and CA dismissed motion hence this petition before the SC ISSUE

PEOPLE v GROSPE 157 SCRA 154. MELENCIO-HERRERA ; January 20, 1988


FACTS - Manuel Parulan is a wholesale dealer of San Miguel Corp (SMC). He issued two checks in favor of SMC (P86,071.20 and P11,918.80) that were dishonored for insufficiency of funds. - The checks were received at the SMC Bulacan branch, then forwarded to the SMC Regional Office in San Fernando, Pampanga. - SMC Finance Officer deposited the check in BPI San Fernando, Pampanga branch. - (Parulans bank is Planters Development Bank in Bulacan)

Criminal Procedure Rowena Daroy Morales


- SMC filed for violation of BP22 (1st check) and for estafa under par. 2d2 (2nd check) with the RTC in Pampanga. - After hearing the facts and evidence, Judge Grospe of the Pampanga RTC dismissed the case because he said that the two essential elements, deceit and damage, of the offenses charged occurred and took place in Bulacan. DECEIT took place when Parulan gave the checks to SMC in Bulacan, with the false assurance that it had sufficient funds. DAMAGE occurred at the moment the checks issued by the accused were dishonored by the Planters Development Bank, the drawee bank, at Santa Maria, Bulacan which received them from the BPI, San Fernando, Pampanga branch for clearing purpose. ISSUES 1. WON any of the essential elements of the offenses charged occurred or took place within the jurisdiction of RTC Pampanga. 2. WON this petition for Certiorari places accused in double jeopardy. HELD 1. YES - A person charged with a transitory crime may be validly tried in any municipality or province where the offense was in part committed. In transitory or continuing offenses in which some acts material and essential to the crime and requisite to its consummation occur in one province and some in another, the Court of either province has jurisdiction to try the case, it being understood that the first Court taking cognizance of the Case will exclude the others. ESTAFA under par 2d - Elements: (1) Deceit took place in Pampanga, where it was uttered and delivered. The rule is that the issuance as well as the delivery of the check must be to a person who takes it as a holder, which means the payee or indorsee of a bill or note, who is in possession of it, or the bearer, thereof, who in this case is the Financial Officer of SMC (2) Damage took place in Bulacan, where the check was dishonored by the drawee bank. - Therefore, jurisdiction may be entertained by either the Bulacan or the Pampanga court. BP 22 violation
2

a2010

page 56

Prof.
public or official document. The trial court adjudged petitioner Bala in Criminal Case No. 24443, guilty of the crime of falsification of a public document. The petitioner seasonably appealed, but the Court of Appeals, on April 9, 1980, affirmed in toto the lower court's decision. After the case had been remanded to the court of origin for execution of judgment, the petitioner applied for and was granted probation by the respondent judge in his order dated August 11, 1982. The petitioner was then placed under probation for a period of one (1) year, subject to the terms and conditions enumerated therein. - The probationer (petitioner) asked his supervising probation officer for permission to transfer his residence from BF Homes to Phil-Am Life Subdivision in Las Pias specifically 33 Jingco Street. The probation officer verbally granted the probationer's request as he found nothing objectionable to it. - By the terms of the petitioner's probation, it should have expired on August 10, 1983, one year after the order granting the same was issued. But, the order of final discharge could not be issued because the respondent probation officer had not yet submitted his final report on the conduct of his charge. Subsequently, the respondent People of the Philippines, through Assistant City Fiscal Jose D. Cajucom of Manila, filed a motion to revoke the probation of the petitioner before Branch XX of the Regional Trial Court (RTC) of Manila, presided over by the respondent judge. The motion alleged that the petitioner had violated the terms and conditions of his probation. - On January 4, 1984, the petitioner filed his opposition to the motion on the ground that he was no longer under probation, his probation period having terminated on August 10, 1983, as previously adverted to. As such, no valid reason existed to revoke the same, he contended. As if to confirm the Manila Assistant City Fiscal's motion to revoke the petitioner's probation, the respondent probation officer filed on January 6, 1984, a motion to terminate Manuel Bala's probation, at the same time attaching his progress report on supervision dated January 5, 1984. The same motion, however, became the subject of a "Manifestation," dated January 10, 1984, which stated that the probation officer was not pursuing the motion to terminate dated January 6, 1984; instead, he was submitting a supplemental report which recommended the revocation of probation "in the light of new facts, information, and evidences." - Thereafter, the petitioner filed a motion to dismiss and/or strike out the motion to revoke probation,

- In respect of the Bouncing Checks Case, the offense also appears to be continuing in nature. It is true that the offense is committed by the very fact of its performance; and that the Bouncing Checks Law penalizes not only the fact of dishonor of a check but also the act of making or drawing and issuance of a bouncing check. The case, therefore, could have been filed also in Bulacan. The determinative factor (in determining venue) is the place of the issuance of the check. However, it is likewise true that knowledge on the part of the maker or drawer of the check of the insufficiency of his funds, which is an essential ingredient of the offense is by itself a continuing eventuality, whether the accused be within one territory or another. Accordingly, jurisdiction to take cognizance of the offense also lies in the Regional Trial Court of Pampanga. - Jurisdiction or venue is determined by the allegations in the Information, which are controlling. The Information filed herein specifically alleges that the crime was committed in San Fernando, Pampanga, and, therefore, within the jurisdiction of the Court below. 2. NO The case was dismissed not on merits but on the erroneous conclusion of the judge that his court had no jurisdiction over the case. The dismissal being null and void, the proceedings before the RTC cant be said to have been lawfully terminated. Therefore, there is no second proceeding to place the accused in double jeopardy. Dispositive Petition granted. Judge is ordered to reassume jurisdiction over Criminal Cases Nos. 2800 and 2813 of his Court and to render judgment of either conviction or acquittal in accordance with the evidence already adduced during the joint trial of said two cases.

BALA v MARTINEZ 181 SCRA 459 MARTINEZ; January 20, 1999


NATURE Petition for certiorari and prohibition with preliminary injunction to review the order of the Court of First Instance of Manila FACTS - The petitioner had been indicted for removing and substituting the picture of Diazen which had been attached to her United States of America passport, with that of Notarte, in effect falsifying a genuine

Art. 315, par. 2(d) states: 'By postdating a check, or issuing a check in payment of an obligation when the offender had no funds in the bank, or his funds deposited therein were not sufficient to cover the amount of the check

Criminal Procedure Rowena Daroy Morales


questioning the jurisdiction of the court over his case inasmuch as his probation period had already expired. Moreover, his change of residence automatically transferred the venue of the case from the RTC of Manila to the Executive. Judge, of the RTC of Makati which latter court include under its jurisdiction the Municipality of Las Pias the probationer's place of residence, invoking Section 13, P.D. No. 968, which provides Sec. 13. Control and Supervision of Probationer. ... Whenever a probationer is permitted to reside in a place under the jurisdiction of another court, control over him shall be transferred to the Executive Judge of the, Court of First Instance of that place, and in such a case a copy of the probation order the investigation report and other pertinent records shall be furnished to said Executive Judge. Thereafter. the Executive Judge to whom jurisdiction over the probationer is transferred shall have the power with respect to him that was previously possessed by the court which granted the probation. - The respondent judge denied the motion to dismiss for lack of merit. Hence, this petition. ISSUE WON his transfer of residence automatically transferred jurisdiction over his probation from the Manila Regional Trial Court to the same court in his new address. HELD NO - In criminal cases, venue is an element of jurisdiction. Such being the case, the Manila RTC would not be deprived of its jurisdiction over the probation case. To uphold the petitioner's contention would mean a depreciation of the Manila court's power to grant probation in the first place. It is to be remembered that when the petitioner-accused applied for probation in the then CFI of Manila, he was a resident of Las Pias as he is up to now, although in a different subdivision. As pointed out earlier, he merely moved from BF Homes to Philam Life Subdivision 33 Jingco Street, also in Las Pias. On the other hand, pursuing the petitioner's argument on this score to the limits of it logic would mean that his probation was null and void in the place, because then the Manila CFI was without jurisdiction to grant him probation as he was a resident of Las Pias. It is therefore incorrect to assume that the petitioner's change of abode

a2010

page 57

Prof.
temporary release. Sola, Garcia and Cabral posted bail and have since been released. -The witnesses informed the prosecution of their fears that if the trial is held at the CFI Himamaylan which is but 10 kilometers from Kabankalan, their safety could be jeopardized. At least two of the accused are officials with power and influence in Kabankalan and they have been released on bail. In addition, most of the accused remained at large. Indeed, there have been reports made to police authorities of threats made on the families of the witnesses. - February 11, 1981: petition for cancellation of bail bonds and change of venue was filed. - February 12, 1981: the Court required the comment of the Solicitor General as well as of the private respondents. - March 4, 1981, the Comment was submitted by Solicitor General Mendoza. It opened with this preliminary statement: "The present petition was filed by the private prosecutors in Criminal Cases Nos. 1700-1706, People v. Pablo Sola, et al., pending trial before the CFI of Negros Occidental. Rightly, any petition before this Honorable Court on behalf of the People of the Philippines can, under the law, be instituted only by the Solicitor General. The assertion of the petitioner private prosecutors that they are instituting the action 'subject to the control and supervision of the Fiscal' will not, therefore, improve their legal standing." Nonetheless, it adopted the two-pronged trusts of the petition: 1. the setting aside, by certiorari, of the order of the Municipal Court of Kabankalan, presided over by Judge Gasataya, granting bail to the accused in the criminal cases mentioned above, and 2. the petition for a change of venue or place of trial of the same criminal cases to avoid a miscarriage of justice. - March 15, 1981: The Court Resolved to: (a) [Note] the comment of the Solicitor General on the urgent petition for change of venue and cancellation of bail bonds, adopting the plea of the petition, namely, (1) the setting aside, by certiorari, of the order of the Municipal Court of Kabankalan, presided over by Judge Rafael Gasataya, granting bail to the accused in the 7 Criminal Cases, and (2) the petition for a change of venue or place of trial of the same criminal cases to avoid a miscarriage of Justice; (b) [Transfer] the venue of the aforesaid criminal cases to Branch V of the Court of First Instance of Negros Occidental at Bacolod City, presided by Executive Judge Alfonso Baguio, considering that District Judge Ostervaldo Emilia of the Court of First Instance, Negros Occidental, Branch VI at Himamaylan has an

compels change of venue, and necessarily, control over the petitioner, to the Executive Judge of the RTC of his new residence. Thus, in the apportionment of the regional trial courts under Batas Pambansa Blg. 129, otherwise known as the Judiciary Reorganization Act of 1980, Las Pias is one among the municipalities included in the National Capital Judicial Region (Metro Manila) with a seat at Makati. 18 Needless to say, the Regional Trial Court in Makati, like the Manila Regional Trial Court, forms part of the Regional Trial Court of the National Capital Region. 19 Accordingly, the various branches of the regional trial courts of Makati or Manila under the National Capital Region, are coordinate and co-equal courts, the totality of which is only one Regional Trial Court. Jurisdiction is vested in the court, not in the judges. In other words, the case does not attach to the branch or judge. Therefore, in this case, RTC Branch XX of Manila, which granted the probation, has not lost control and supervision over the probation of the petitioner. Dispositive Petition dismissed

PEOPLE v SOLA 103 SCRA 393 FERNANDO; March 17, 1981


FACTS - September 15, 1980: acting on the evidence presented by the Philippine Constabulary commander at Hinigaran, Negros Occidental, the CFI issued a search warrant for the search and seizure of the deceased bodies of 7 persons believed in the hacienda of Pablo Sola at Sta. Isabel, Kabankalan, Negros Occidental. - September 16, 1980: elements of the 332nd PC/INP Company proceeded to the place of Sola. Diggings made in a canefield yielded two common graves containing the 7 bodies. - September 23 and October 1, 1980: the PC provincial commander filed 7 separate complaints for murder against the accused Pablo Sola, Francisco Garcia, Ricardo Garcia, Jose Bethoven Cabral, Florendo Baliscao and 14 other persons of unknown names. - After due preliminary examination of the complainant's witnesses and his other evidence, the municipal court found probable cause against the accused. It thus issued an order for their arrest. - Without giving the prosecution the opportunity to prove that the evidence of guilt is strong, the court granted them the right to post bail for their

Criminal Procedure Rowena Daroy Morales


approved leave of absence covering the period from January 12 to March 12, 1981 due to a mild attack of cerebral thrombosis and that the said Branch V is the nearest court station to Himamaylan: and (c) [Await] the comment of respondents on the petition to cancel bail, without prejudice to the public officials concerned taking the necessary measures to assure the safety of the witnesses of the prosecution." Thus, the issue of a change of venue has become moot and academic. ISSUE WON the bail bonds should be cancelled HELD YES Ratio Whether the motion for bail of a defendant who is in custody for a capital offense be resolved in a summary proceeding or in the course of a regular trial, the prosecution must be given an opportunity to present, within a reasonable time, all the evidence that it may desire to introduce before the court should resolve the motion for bail. If, as in the criminal case involved in the instant special civil action, the prosecution should be denied such an opportunity, there would be a violation of procedural due process, and the order of the court granting bail should be considered void on that ground. (People v San Diego) Reasoning - Bail was granted to the accused without hearing the prosecution -Justice Cardozo: "The law, as we have seen, is sedulous in maintaining for a defendant charged with crime whatever forms of procedure are of the essence of an opportunity to defend. Privileges so fundamental as to be inherent in every concept of a fair trial that could be acceptable to the thought of reasonable men will be kept inviolate and inviolable, however crushing may be the pressure of incriminating proof. But justice, though due to the accused, is due to the accuser also. The concept of fairness must not be strained till it is narrowed to a filament. We are to keep the balance true." - It does not suffice that the questions asked by the municipal judge before bail was granted could be characterized as searching. That fact did not cure an infirmity of a jurisdictional character. On change of venue - 1973 Constitution: The Supreme Court could order "a change of venue or place of trial to avoid a miscarriage of justice."

a2010

page 58

Prof.
- Municipal Judge Alfredo V. Granados of the Municipal Court of Pulilan received the complaint and conducted a preliminary investigation, first stage. - October 25, 1974: the Complaint was amended. - Rufino Bulanadi and Feliciano Gorospe were again named but Gerardo Fajardo was dropped and Oscar Alvaran was named instead. -The date when the crime was said to have been committed was changed from September 30, 1974 to September 25, 1974. - Again Judge Granados conducted a preliminary investigation and on November 18, 1974, he issued an order for the arrest of Bulanadi, Gorospe and Alvaran and fixed their bail at P15,000.00 each. -Bulanadi and Gorospe posted the requisite bail. Alvaran remained at large. -The second stage of the preliminary investigation was set on February 5, 1975, but on that day, neither Bulanadi or Gorospe appeared for which reason, Judge Granados declared that they had waived their right thereto and elevated the case to the CFI of Bulacan. - March 19, 1975: Provincial Fiscal Pascual C. Kliatchko filed with the CFI of Bulacan an Information for forcible Abduction with Rape against Gorospe and Bulanadi. But said information was later on amended. -Judge Nelly L. Romero Valdellon started the trial of the case on October 15, 1975. -The accused and their counsel de parte had long been notified that the case was to be tried on that day but they did not appear so the former were tried in absentia. -After hearing part of the testimony of Anastacia de Jesus, the complainant, Judge Valdellon was transferred to Metro Manila and she was replaced by Judge Fidel P. Purisima who finished the trial. -But Judge Purisima inhibited himself from deciding the case because J. Granados is his first cousin by affinity (to make sure that the decision to be rendered in this case shall be above suspicion) -So it was Judge Jesus R. de Vega who decided the case CFI: found Gorospe and Bulanadi guilty beyond reasonable doubt of Rape committed against Anastacia de Jesus as charged in the information; sentenced each of the accused to suffer 2 perpetual penalties of reclusion perpetua to be served in accordance with Art. 70 of the RPC, with all the accessory penalty of the law; to indemnify de Jesus in the amount of P40,000.00 for actual exemplary and moral damages, and to pay the costs. ISSUES

- People v. Gutierrez: To compel the prosecution to proceed to trial in a locality where its witnesses will not be at liberty to reveal what they know is to make a mockery of the judicial process, and to betray the very purpose for which courts have been established." It may be added that there may be cases where the fear, objectively viewed, may, to some individuals, be less than terrifying, but the question must always be the effect it has on the witnesses who will testify. The primordial aim and intent of the Constitution must ever be kept in mind. In case of doubt, it should be resolved in favor of a change of venue. As a matter of fact, there need not be a petition of this character filed before this Court. Such a plea could have been done administratively. In this particular case, however, there is justification for the procedure followed in view of the fact that along with the change of venue, the cancellation of the bail bonds was also sought. Dispositive The assailed order of Judge Gasataya granting bail to private respondents is nullified, set aside, and declared to be without force and effect. Exec. Judge Alfonso Baguio of the CFI of Negros Occidental, to whose sala the cases had been transferred is directed forthwith to hear the petitions for bail of private respondents, with the prosecution being duly heard on the question of whether or not the evidence of guilt against the respondents is strong. This decision is immediately executory. No costs.

PEOPLE v FELICIANO GOROSPE and RUFINO BULANADI 129 SCRA 233 ABAD SANTOS; May 15, 1984
FACTS - In a verified Complaint filed on October 8, 1974 with the Municipal Court of Pulilan, Bulacan, Anastacia de Jesus (14 yrs old) accused Gerardo Fajardo, Rufino Bulanadi and Feliciano Gorospe of the crime of Forcible Abduction with Rape. - The crime was said to have been committed on September 30, 1974, starting in Plaridel, Bulacan, thru Pulilan, and thence to Talavera, Nueva Ecija (in a hut where she was detained for 9 days and sexually abused during the night. She was made to lose her consciousness first by waiving a hankerchief on her face before they abducted her and eventually taking her to said place.)

Criminal Procedure Rowena Daroy Morales


1. WON there was error in filing the complaint since it was not filed in Plaridel, Bulacan or Talavera, Nueva Ecija but in Pulilan, Bulacan (and if yes, then WON an error was by the CFI of Bulacan in hearing the said case and not by the CFI of Nueva Ecija) 2. WON Judge Vega had authority to hear the case (***there are other issues but no longer related to the topic venue so I didnt include them anymore ~ eoc) HELD 1. NO - The Municipal Court of Pulilan had jurisdiction because the abductors and their captive passed Pulilan on their way from Plaridel to Talavera. And the CFI of Bulacan [as well as the CFI of Nueva Ecija] had jurisdiction because essential elements of the offense took place in Bulacan [and also in Nueva Ecija]. Reasoning Abduction is a persistent and continuing offense. (U.S. vs. Bernabe, 23 Phil. 154 [1912]). - Hence it may be "tried in the court of the municipality or province wherein the offense was committed or any one of the essential ingredients thereof took place." (Rules of Court, Rule 110, Sec. 14[a]). 2. YES. - Judge de Vega had the power to decide the case. Reasoning "Where a court of first instance is divided into several branches, each of the branches is not a court distinct and separate from the others. Jurisdiction is vested in the court, not in the judges, so that when a complaint or information is filed before one branch or judge, jurisdiction does not attach to said branch or judge alone, to the exclusion of the others. Trial may be had or proceedings may continue by and before another branch or judge." [Lumpay, et al. vs. Moscoso, 105 Phil. 968 (1959)]. Dispositive The judgment of the Court a quo is hereby affirmed in all respects.

a2010

page 59

Prof.
Petition for certiorari to annul orders of the City Court of San Carlos FACTS - October 19, 1980: Respondents entered the store and dining room of the Pacita Tandoc without her permission. There was an altercation between Tandoc and respondent, Arnold Payopay, regarding the stoning of the store and house. Payopay picked up stones and struck Tandoc but instead her helper, Bonifacio Menor, was hit and suffered physical injuries which according to the medico-legal certificate will heal in more than 30 days. Beda Acosta, who was behind Arnold Payopay, picked up the stone and struck Tandoc but her helper, Fred de la Vega, was hit instead and suffered injuries which injury will heal in less than 9 days. - 19 October 1980: a criminal complaint was lodged with the Office of the City Fiscal with the charges of Serious Physical Injuries, filed by Bonifacio Menor against Arnulfo (Arnold) Payopay; Slight Physical Injuries, filed by Fred de la Vega against Beda Acosta, and Trespass to Dwelling, filed by Pacita Tandoc against Arnulfo Payopay, Beda Acosta, Manuel Cancino, Nadong Fernandez and Arturo Syloria. - 2 December 1980: Arnulfo Payopay and his father Conrado Payopay, Sr., together with Manuel Cancino, also filed a complaint with the Office of the City Fiscal, against Pedro Tandoc, Pacita Tandoc, Rudy Diaz, Fred Menor, Rogelio Ercella, Juan Rosario and Fred de la Vega, with the charges of Trespass to Dwelling, Serious Oral Defamation, Grave Threats and Physical Injuries - 10 December 1980: the investigating fiscal found reasonable ground to believe that respondents Arnulfo Payopay, Beda Acosta, Manuel Cancino, Nadong Fernandez and Arturo Syloria committed the crimes charged. Informations were filed with the City Court. - With respect to the criminal complaint filed by Arnulfo Payopay and Manuel Cancino against petitioners for Serious Oral Defamation, Grave Threats and Physical Injuries, the Office of the City Fiscal recommended the dropping of said charges because they "were found to be in the nature of a counter charge, the same having been filed after more than 1 month from the date of the alleged incident." However, as to the charge of Trespass to Dwelling filed by Conrado Payopay, Sr. against Pedro Tandoc, a prima facie case was found by the investigating fiscal. Thus, an information was filed with the City Court.

and after investigation of fiscal, information was lodged. Case was docketed and Judge issued arrest warrant. Petitioners counsel filed motion asking fiscal to furnish clerk of court w/ testimony of witnesses who testified at preliminary investigation. Fiscal opposed. - Counsel for petitioner put in motion that should his first motion be acted upon adversely, that Court itself conduct the investigation under Sec 4 of Rule 108. Fiscal opposed. - Petitioners counsel asked that warrant of arrest be cancelled and the court conduct preliminary investigation. Judge denied motions and the MFRs. Hence the instant certiorari and mandamus petition. ISSUE WON in prelim investigation by fiscal, accused is entitled to be informed of substance of testimony and evidence against him HELD NO - Prelim investigation by fiscal is not within purview of Sections 13 and 11 of Rule 108. Sec 13 deals with transmission of records requirements and Sec 11 deals with prelim investigation by justices of peace and judges for purpose of issuance of warrant. - Sec 2, Act No 612: In cases triable only in CFI, defendant shall have speedy trial, but shall not be entitled as of right to a prelim investigation where prosecuting attorney, after investigation, shall have presented an information against him. - THE RIGHT TO A PRELIMINARY INVESTIGATION IS STATUTORY, NOT CONSTITUTIONAL. Its purpose is to secure the innocent against hasty prosecutions and protect him from public accusation, and also to protect the State from useless prosecutions. This investigation is called preliminary, to be followed by trial proper. Investigating judge or prosecuting officer acts upon probable cause and reasonable belief, not upon proof beyond reasonable doubt. - In this case, to ask for abstract of testimony at that stage for no other purpose than to scrutinize the same is, in effect, to ask for another prelim investigation.

HASHIM v CITY FISCAL OF MANILA 71 Phil 216 LAUREL; January 13, 1941
NATURE Certiorari and mandamus FACTS - Hashim was caught in possession of counterfeit treasury certificates, but was released upon filing of bond. Complaint was filed with Office of City Fiscal

TANDOC v RESULTAN 175 SCRA 37 PADILLA; July 5, 1989


NATURE

Criminal Procedure Rowena Daroy Morales


- 28 July 1981: Arnulfo Payopay, Conrado Payopay, Sr. and Manuel Cancino, directly lodged with the City Court of San Carlos City the following criminal complaints: (1) against Pedro Tandoc, Rogelio Ercella, Rudy Diaz, Juan Rosario and Fred Menor for Serious Physical Injuries, filed by Arnulfo (Arnold) Payopay; (2) against Rudy Diaz, Juan Rosario and Fred Menor for Trespass to Dwelling, filed by Conrado Payopay, Sr.; (3) against Pedro Tandoc, Rudy Diaz, Juan Rosario and Fred dela Vega for Less Serious Physical Injuries, filed by Manuel Cancino; (4) against Pedro Tandoc, Rudy Diaz, Rogelio Ercella, Juan Rosario & Fred Menor for Grave Threats to Kill, with Arnulfo Payopay as private complainant. - 13 August 1981: City Court, after conducting a preliminary examination of the 4 aforementioned cases, found reasonable ground to believe that the offenses charged may have been committed by the herein petitioners and that the latter were probably guilty thereof. - The issuance of warrants of arrest was ordered against them, although said warrants were later suspended upon motion of the petitioners. - A motion for reconsideration was denied. - Petitioners moved for a re-investigation of the cases by the Office of the City Fiscal. The court a quo denied said motion. Petitioners sought a reconsideration of said order, but it was likewise denied. ISSUE WON the city court has the power and authority to conduct a new a preliminary examination of charges, which were previously the subject of a preliminary investigation conducted by the Office of the City Fiscal and thereafter dismissed by the latter. HELD YES Ratio As long as the offense charged has not prescribed, the city court has the power and authority to conduct a preliminary examination and proceed with the trial of the case properly within its jurisdiction. Reasoning - The offenses charged against petitioners for Trespass to Dwelling, Grave Threats and Physical Injuries were all within the jurisdiction of the City Court. The complaints could be filed directly with the City Court which is empowered to conduct a preliminary examination for purposes of issuance of warrants of arrest, and thereafter to proceed with the trial of the cases on the merits. The preliminary

a2010

page 60

Prof.
not place the person against whom it is taken in jeopardy. - Under Section 10, Rule 112 of the 1964 Revised Rules of Criminal Procedure, in cases falling within the exclusive jurisdiction of an inferior court, as well as in cases within the concurrent jurisdiction of the city courts or municipal courts with Courts of First Instance, the accused was not entitled to be heard in a preliminary investigation proper. The reason behind this rule is as follows: " The loss of time entailed in the conduct of preliminary investigations, with the consequent extension of deprivation of the accused's liberty, in case he fails to post bail, which at times outlasts the period of the penalty provided by law for the offense, besides the mental anguish suffered in protracted litigations, are eliminated with the assurance of a speedy and expeditious trial for the accused, upon his arraignment (without having to undergo the second stage of the preliminary investigation), and of a prompt verdict on his guilt or innocence. On the other hand, the so-called first stage of preliminary investigation or the preliminary examination, conducted by the duly authorized officer, as borne out by the examination and sworn written statement of the complainants and their witnesses, generally suffices to establish the existence of reasonable ground to charge the accused with having committed the offense complained of." - The result of a preliminary investigation can neither constitute nor give rise to the defense of double jeopardy in any case, because such preliminary investigation is not and does not in itself constitute a trial or even any part thereof. In order that the defense of jeopardy may lie, there must be a former judgment, either of acquittal or of conviction, rendered by a court competent to render the same, not only by reason of the offense committed, which must be the same or at least comprised within it, but also by reason of the place where it was committed. Dispositive Petition dismissed.

investigation proper conducted by the City Fiscal could have been dispensed with. Neither did the earlier order of dismissal of the complaints by the investigating fiscal bar the filing of said complaints with the city court on the ground of double jeopardy. - The prescriptive period of a crime depends upon the penalty imposed by law. The penalties for the crimes charged are: arresto mayor for Trespass to Dwelling, Grave Threats and Less Serious Physical Injuries; and arresto mayor in its maximum period to prision correccional in its minimum period for Serious Physical Injuries. The prescriptive period of offenses punishable by arresto mayor is 5 years, while crimes punishable by correctional penalties prescribe in 10 years. The complaints were filed with the City Court only 9 months from said occurrence. - The re-investigation sought by petitioners applies only to instances where a case is cognizable by the Court of First Instance but filed with the City Court for purposes of preliminary investigation only and thereafter dismissed by the latter on the ground that no prima facie case exists. However, for cases cognizable by inferior courts and filed with the same not only for purposes of preliminary investigation but for trial on the merits, the Office of the City Fiscal has no authority to re-investigate. On Preliminary Investigation: - Purpose: to protect the accused from the inconvenience, expense and burden of defending himself in a formal trial unless the reasonable probability of his guilt shall have been first ascertained in a fairly summary proceeding by a competent officer; and to protect the state from having to conduct useless and expensive trials. - Stages: (1) the preliminary examination of the complainant and his witnesses prior to the arrest of the accused to determine whether or not there is ground to issue a warrant of arrest; (2) preliminary investigation proper, wherein the accused, after his arrest, is informed of the complaint filed against him and is given access to the testimonies and evidence presented, and he is also permitted to introduce evidence in his favor. The purpose of this stage of investigation is to determine whether or not the accused should be released or held before trial. - Nature: merely inquisitorial, and is often the only means of discovering the persons who may be reasonably charged with a crime, to enable the fiscal to prepare his complaint or information; not a trial of the case on the merits and has no purpose except that of determining whether a crime has been committed and whether there is probable cause to believe that the accused is guilty thereof, and it does

UNITED STATES v MARFORI 35 Phil 666 CARSON; December 9, 1916


FACTS - CASIANO MARFORI was convicted of the crime of injurias graves (aggravated slander), and sentenced to six months and one day of destierro (banishment) for a distance of 25 kilometers from the municipality where the crime was committed, to pay a fine of P65,

Criminal Procedure Rowena Daroy Morales


together with subsidiary destierro as prescribed by law on failure to pay this fine and to pay the costs. The complaint charges him of having spoken of the complainant in a manner which reflected adversely upon her virtue and good name in the presence of several witnesses. - The complaint was originally filed in the court of a justice of the peace who held a preliminary investigation and discharged the accused on the ground that he was not guilty of the crime with which he was charged. - A report of the proceedings was forwarded to the provincial fiscal by the justice of the peace. The complaining witness renewed the complaint in the CFI. An information was filed in that court and Marfori was brought to trial without further proceedings. - Upon arraignment, when Marfori was called upon to plead, his counsel declined to proceed on the ground that the court was without jurisdiction to bring Marfori to trial, no order remanding him for trial having been issued by a competent magistrate as a result of a preliminary trial (old term for PI, I think) held in accordance with law. - The trial judge overruled the objections of counsel, and ordered the parties to proceed with the trial on the ground that the report of the proceedings had at the preliminary trial held by the justice of the peace disclosed a reasonable probability that the crime charged had been committed and that the accused had committed it; that the justice of the peace had erred in discharging the accused; and that he should have remanded the accused for trial. - Marforis counsel then exempted to the ruling and insisted on the right to a preliminary trial. Marfori refused to enter a plea so that the court was compelled to direct the entry of a plea of not guilty in his behalf. ISSUE WON the trial court erred in bringing the accused to trial, over his objection, in the absence of an order remanding him for trial based upon a preliminary trial held in accordance with the provisions of law HELD YES. Ratio The right of an accused not to be brought to trial except when remanded as the result of a preliminary examination before a committing magistrate or, within the city of Manila, not to be brought to trial except in pursuance of like proceeding or the proceeding substituted by law, is a substantial one. Its denial, over the objection of the

a2010

page 61

Prof.
This is an appeal by the offended party, Petra Flores, from the order of the Court of First Instance of Laguna. FACTS - ORDER OF CFI:"The provincial fiscal having filed a motion in the above entitled case praying for the dismissal of the case, and the court having found meritorious the reasons alleged therein, add case is hereby dismissed, as prayed, with costs de oficio, and the bond filed for the temporary release of the accused is hereby ordered cancelled. It is so ordered." ISSUE WON the provincial fiscal has authority to conduct another preliminary investigation and thereafter ask the Court of First Instance to dismiss the criminal case remanded by the justice of the peace, after the latter had conducted the preliminary investigation and issued an order to the effect that there was probable cause to prosecute the offense charged which falls within the jurisdiction of the Court of First Instance." HELD After a criminal case has been remanded by the justice of the peace to the Court of First Instance which has jurisdiction to try it on the merits, and before the provincial fiscal has filed the necessary information, the latter not only has the power but also the duty to investigate the facts upon which the complaint filed in the justice of the peace court was based, to examine the evidence submitted to the justice of the peace and such other evidence as the parties may deem proper to submit on their own free will or on demand of the fiscal, for the purpose of determining whether there is at least prima facie evidence establishing the guilt of the accused and overcoming the presumption of innocence in his favor. If after he has done all this and considering all the circumstances of the case, the fiscal believes that the evidence is not sufficient to establish prima facie the guilt of the accused, he should submit to the court before which the case is pending the corresponding motion for dismissal. The provincial fiscal of Laguna complied with all these requirements before asking for the dismissal of the present case, thereby keeping within the powers conferred upon him by section 1687 of the Revised Administrative Code.

accused is prejudicial error, in that it subjects the accused to the loss of life, liberty or property without due process of law. Reasoning The accused was brought to trial, over his objection without having been committed or remanded for trial by an investigating magistrate. The justice of the peace who held the preliminary investigation dismissed the original complaint against the accused, being of opinion that there was no probable cause to believe him guilty of the offense; and although a so-called report of the proceedings was forwarded to the fiscal and doubtless submitted to the trial judge, original jurisdiction to commit the accused for trial as result of those proceedings was vested exclusively in the justice of the peace before whom they were had. - The order of the justice of the peace discharging the accused did not operate as a final acquittal, and was not a bar to re-arrest and prosecution for the offense originally charged. If the fiscal was not satisfied with the action of the justice of the peace, he could have secured the arrest of the accused upon a new complaint, and sought an order remanding the accused for trial in a second preliminary investigation had before either the justice of the peace who held the first investigation or before the judge of the CFI in the exercise of his functions as a committing magistrate. (Act 1627, Sec. 37) - But it would manifestly defeat the end sought to be attained by the provisions of law for the holding of preliminary investigations if either the fiscal, or the trial judge, or both acting together were permitted to make use of the record of the proceedings had before a justice of the peace at a preliminary trial, as a result of which the accused was discharged, for the purpose of bringing the accused to trial despite the order of discharge and over his objection based on the ground that he has not been remanded for trial as a result of a preliminary trial. Dispositive Judgment convicting and sentencing the accused REVERSED with the costs of both instances de officio and the record REMANDED to the court below for further proceedings.

PEOPLE v OVILLA 65 Phil 722 VILLA-REAL; June 27, 1938


NATURE

Criminal Procedure Rowena Daroy Morales


Dispositive Finding no merit in the sole error assigned by the appellant, the offended party in this case, the order appealed from is hereby affirmed,' with costs against the appellant.

a2010

page 62

Prof.
Atilano and two other unidentified persons, Richard Doe and John Doe. The prosecution also certified under oath that they conducted a preliminary investigation of the case Upon arraignment, Jesus Atilano, Prudencio Cichon and Lorenzo Delantar pleaded not guilty.

ISSUE WON the trial court had no jurisdiction to try the case for want of preliminary investigation HELD NO Reasoning - When Judge Templo set the case for preliminary investigation to afford the accused occasion to confront the witnesses against him, the accused instead filed a manifestation waiving his right to present evidence at the second stage of the preliminary investigation. When the case was forwarded to the CFI, the accused entered his plea without raising the question of lack of preliminary investigation. The aforesaid constitute waiver of the accuseds right to preliminary investigation. It is wellsettled that the right to preliminary investigation is not a fundamental right and that the same may be waived expressly or by silence. Such waiver carried with it the waiver of any procedural error or irregularity that may have attended the preliminary investigation. Dispositive The judgment under review is hereby affirmed

PEOPLE v VELOSO 112 SCRA 173 PER CURIAM; February 25, 1982
NATURE Automatic review of CFI decision imposing the capital penalty of death on accused Veloso for the crime of robbery with homicide and double serious physical injuries FACTS - Veloso and others entered the Odiamars house and robbed them around 7:30 in the evening. They stole money, tear gas gun, jewelry, old coins. - Hermenegildo Odiamar was shot and killed during the robbery, while the Odiamar spouses sustained serious physical injuries. - Veloso, among others, was charged for robbery with homicide and double serious physical injuries - July 5, 1970 (the case says july but I think its a typo probably june) : Judge Templo conducted preliminary examination - June 22, 1970 Judge Templo set the case for preliminary investigation to afford the accused the occasion to confront the witnesses against him and to present his own evidence - instead of availing himself of this opportunity, he filed a manifestation stating that he Waives his right to present evidence at the second stage of the preliminary investigation. - the case was forwarded to the CFI, and it appears that accused entered his non-guilty plea without raising the question of lack of preliminary investigation. Petitioners' Claim (1) the trial court had no jurisdiction to try the case for want of preliminary investigation (2) the extrajudicial confession he executed was obtained through force and intimidation and, therefore, inadmissible in evidence, and (3) in the absence of adequate proof that it was he who killed the deceased Hermenegildo Odiamar, he should be held guilty of the offense of robbery only, and not of the complex crime of robbery with homicide and double serious physical injuries.

3.

Criminal Case No. 3088.-- Estafa thru falsification of public/official documents was filed in the Court of First Instance of Zamboanga City against Prudencio Cichon and Paulino Duma, Also has certification of the State Prosecutors that they had conducted a preliminary investigation in the case The two accused pleaded not guilty.

PEOPLE v GOMEZ 117 SCRA 72 RELOVA; September 30, 1982


FACTS - In 1962, four (4) informations were filed by the prosecuting fiscals before the Court of First Instance of Zamboanga City. They were as follows:

1.

Criminal Case No. 3083.-- Edilberto Gomez, Prudencio N. Cichon, Cesar V. Castillo, Pedro Cuento and John Doe charged with Estafa thru falsification of public/official documents. The prosecuting officers certified under oath that they had conducted a preliminary investigation of the case in accordance with law; and that they believed that the offense charged had been committed and the accused were probably guilty thereof. The corresponding warrant of arrest for each of the accused was accordingly issued. The accused Pedro Cuento and Cesar Castillo pleaded not guilty. Criminal Case No. 3084.-- Estafa thru falsification of public/official documents against Lorenzo Delantar, Prudencio Cichon, Jesus F.

Criminal Case No. 3128.-- Prudencio Cichon, Jesus F Atilano and Pedro Cuento were charged with Estafa thru falsification of public/official documents. No certification that a preliminary investigation of the case had been made by the prosecutors. So District Judge himself made the preliminary investigation and once satisfied that a prima facie case against the three accused existed, issued warrants for their arrest on the same day. At the arraignment, all the accused pleaded not guilty. - On June 22, 1966, the accused in the four (4) cases, thru their counsel, filed a MOTION TO DECLARE INFORMATIONS AND WARRANTS OF ARREST null and void on the ground that the prosecution failed to observe the provisions of Section 13 and 14 of Rule 112 of the New Rules of Court regarding preliminary investigation and prayed the court to cancel the warrants of arrest issued. - On September 27, 1966, the lower court, for lack of merit, denied the aforesaid motion. - Upon a motion for reconsideration filed by the accused, thru counsel, the lower court, on November 2, 1966, reversed its former ruling and ordered the dismissal of all the four (4) cases against them, without prejudice to the refiling of the same. ISSUE WON the trial court erred in dismissing these cases on the ground that the preliminary investigations conducted therein were not in accordance with Sections 13 and 14 of Rule 112, in relation to Rule 144 of the Revised Rules of Court. HELD YES Ratio The preliminary investigations in these four (4) cases were terminated in 1962, or before the New

4.

2.

Criminal Procedure Rowena Daroy Morales


Rules of Court took effect on January 1, 1964. Rules 112 and 113 thereof cannot, therefore, apply to these cases at bar. Reasoning The government prosecutors certified under oath that they had conducted a preliminary investigation in said cases. And, in Criminal Case No. 3128, it was District Judge Gregorio Montejo who conducted the preliminary investigation and, finding the existence of a prima facie case, ordered the arrest of the defendant. It is clear, therefore, that the required investigations were complied with. - But then, assuming that the informations did not contain the requisite certificates regarding the Fiscal's having held a preliminary investigation, the omissions are not necessarily fatal. The absence of preliminary investigations does not affect the court's jurisdiction over the case. Nor do they impair the validity of the information or otherwise render it defective. If there were no preliminary investigations and the defendants, before entering their plea, invite the attention of the court to their absence, the court, "instead of dismissing the information, should conduct such investigation, order the fiscal to conduct it or remand the case to the inferior court so that the preliminary investigation may be conducted." (People vs. Casiano, 1 SCRA 478). The defendants in these cases did not question the validity of the informations on the ground of defective certifications or the right to preliminary investigations before they entered the plea of not guilty. They filed the motion to declare informations and warrants of arrest null and void only after more than one (1) year thereafter. Consequently, when they entered a plea of not guilty, they thereby waived all objections that are grounds for a motion to quash, except lack of jurisdiction or failure of the information to charge an offense. Thus, they waived the right to a preliminary investigation when they failed to invoke it prior to, or at least at, the time of the entry of their plea in the Court of First Instance. Dispositive ACCORDINGLY, the order dated November 2, 1966 of the Court of First Instance of Zamboanga is set aside and the said court is hereby ordered to proceed with the trial of the said criminal cases.

a2010

page 63

Prof.
-The primary requirement for the issuance of a warrant of arrest is the existence of probable cause (Sec. 3, Art. IV of the 1973 Constitution). P.D. No. 911 authorizes the fiscal or state prosecutor to determine the existence of probable cause. - There is thus no dispute that the judge may rely upon the fiscal's certification of the existence of probable cause and, on the basis thereof, issue a warrant of arrest. But this does not bind the judge to issue a warrant - Section 6, Rule 112 of the Rules of Court:: "Warrant of arrest, when issued. - If the judge be satisfied from the preliminary examination conducted by him or by the investigating officer that the offense complained of has been committed and that there is reasonable ground to believe that the accused has committed it, he must issue a warrant or order for his arrest." -the judge must satisfy himself of the existence of probable cause before issuing a warrant or order of arrest. If on the face of the information the judge finds no probable cause, he may disregard the fiscal's certification and require the submission of the affidavits of witnesses to aid him in arriving at a conclusion as to the existence of a probable cause. Without the affidavits of the prosecution witnesses and other evidence which, as a matter of longstanding practice had been attached to the informations filed in his sala, respondent found the informations inadequate bases for the determination of probable cause -Also, Rule on Summary Procedure in Special Cases, is applicable to some of the crimes in the said informations. This rule requires that the complaint or information must be accompanied by the affidavits of the complainant and of his witnesses in such number of copies as there are defendants plus two (2) copies for the court's files -judge also did not commit grave abuse of discretion in remanding some of the cases to the City Fiscal for further investigation. From the informations and affidavits presented to him, he found the charges patently without basis or merit. For respondent to issue the warrants of arrest and try the accused would only expose the latter to unnecessary harrassment, anxiety and expense. And as already pointed out, under the Rule on Summary Procedure in Special Cases, the respondent judge has the power to order the outright dismissal of the charge if, from the information and the affidavits attached thereto, he finds the same to be patently without basis or merit

-petitioners the City Fiscal of Butuan City and his assistants filed in the City Court of Butuan certain informations and certified them as follows: that a preliminary examination has been conducted by me in this case, having examined the complainant and his witnesses; that on the basis of the sworn statements and other evidence submitted before this Office, there is reasonable ground to believe that the crime charged has been committed and that herein accused is probably guilty thereof. -respondent judge set a hearing to determine the propriety of issuing warrants of arrest. After hearing, judge issued orders requiring petitioners to submit to the court the affidavits of the prosecution witnesses and other documentary evidence in support of the informations to aid him in the exercise of his power of judicial review of the findings of probable cause by petitioners -petitioners filed two separate motions for reconsideration stating that they were authorized to determine the existence of probable cause ni a preliminary investigation and that their findings constitute sufficient basis for the issuance of warrants of arrest. -respondent justifies his order as an exercise of his judicial power to review the fiscal's findings of probable cause. He further maintains that the failure of petitioners to file the required affidavits destroys the presumption of regularity in the performance of petitioners' official duties, particularly in the light of the long standing practice of the Office of the City Fiscal of Butuan of attaching to the informations filed with the court the affidavits of prosecution witnesses and other documentary evidence presented during the preliminary investigation -judge denied motion. And asked the submission of documents earlier asked for. *eventually, petitioners submitted the documents rendering the case mute (haha) and academic. But the Court decided to tackle the issue nonetheless. Some warrants were granted, some were remanded to ISSUE WON the respondent city judge may, for the purpose of issuing a warrant of arrest, compel the fiscal to submit to the court the supporting affidavits and other documentary evidence presented during the preliminary investigation HELD NO.

PLACER v VILLANUEVA 126 SCRA 463 ESCOLIN; December 29, 1983


FACTS

Criminal Procedure Rowena Daroy Morales


GO v CA (PELAYO) 206 SCRA 138 FELICIANO; February 11, 1992
NATURE Petition for review on certiorari from the decision of the Court of Appeals FACTS - July 2, 1991 Eldon Maguan entered a one-way street (Wilson St.) from the opposite direction (counterflow), heading towards P. Guevarra St. In so doing, he nearly collided with the car of accused Rolito Go. Go got out of his car and shot Maguan. - A security guard of a nearby bake shop witnessed the event and was able to note the plate number of the petitioner. The car was eventually traced to an Elisa Ang Go, wife of the accused. - The police were informed that the petitioner had a meal at the bake shop where his credit card was used to pay for the transaction. Police were able to identify the card owner as the accused Go and when his picture was shown to the security guard who positively identified him as the supposed assailant. Police then launched a manhunt for Go. - July 8, 1991 Go presented himself in the San Juan police station with his two lawyers in tow to verify reports that he was being hunted down by the police. 1. The police detained Go and a COMPLAINT for FRUSTRATED HOMICIDE was filed against him. 2. Asst. Prov. Prosecutor Villa Ignacio informed Go, in the presence of his lawyers, of his right to avail of preliminary investigation but in so doing, Go had to waive the provisions in Art. 125, RPC. Go refused. - July 9, 1991 Maguan died as a result of his gunshot wounds before an INFORMATION could be filed. - July 11, 1991: 3. The prosecutor filed an INFORMATION for murder, instead of an information for frustrated homicide. The prosecutor stated that no preliminary investigation was conducted because Go refused to waive provisions of Art. 125, RPC. 4. Gos counsel filed an OMNIBUS MOTION FOR IMMEDIATE RELEASE AND PROPER PRELIMINARY INVESTIGATION with the allegations that an illegal warrantless arrest had been effected and that no preliminary investigation had been conducted and prayed that Go be released on bail. - July 12, 1991:

a2010

page 64

Prof.
Respondents Comments: Go had been validly arrested because the crime had been committed 6 days before he was arrested. - Invoking Umali vs. Ramos where the Court upheld that a warrantless arrest was valid 14 days after the crime was committed. - The prosecutor proceeded under the erroneous supposition that Section 7 of Rule 112 was applicable and required petitioner to waive the provisions of Article 125 of the Revised Penal Code as a condition for carrying out a preliminary investigation. Go was entitled to a preliminary investigation and that right should have been accorded him without any conditions. ISSUES 1. WON the warrantless arrest was lawful 2. WON the accused Go had waived his right to preliminary investigation HELD 1. NO, the warrantless arrest was not lawful Ratio Rule 112, Sec. 7 states that a complaint for information can be filed sans preliminary investigation when a person has been lawfully arrested without a warrant except than an affidavit should be executed by the person who was responsible for the arrest. But the person arrested can ask for preliminary investigation by the proper officer before the complaint or information can be filed. In this case, the person arrested must waive the provisions of A125, RPC with the assistance of counsel (a lawyer or another person of his choice if a lawyer is not available). He may also apply for bail despite the waiver and the investigation must terminate within 15 days. Reasoning - Umil vs. Ramos only applies to continuing crimes so it does not apply in the case at bar. Murder is not a continuing crime because it happens in one place at a particular point in time and ends there as well. - The warrantless arrest does not follow the requisites in Rule 113, Sec. 5 because: >The arrest took place 6 days after Maguan was shot whereas the RoC provide that the crime should have been just committed, is about to be committed or is being committed. >None of the arresting officers had personal knowledge of the facts indicating that Go was the gunman as required in the RoC. The information that the police had was derived from eyewitness accounts.

5. Go filed an urgent EX-PARTE MOTION FOR SPECIAL RAFFLE in order to expedite action on the bail recommendation. The cash bond was approved and Go was released from jail. - July 16, 1991: 6. Prosecutor filed a MOTION FOR LEAVE TO CONDUCT PRELIMINARY INVESTIGATION and prayed that the court proceedings be suspended momentarily. 7. The trial court granted LEAVE to conduct preliminary investigation and cancelled the arraignment scheduled on August 15, 1991. - July 19, 1991: 8. Go contended through a PETITION FOR CERTIORARI, PROHIBITION AND MANDAMUS that the information was null and void because no preliminary investigation had been conducted. - July 23, 1991 Go surrendered to the police and the judge set the arraignment on August 23. - August 23, 1991: 9. Respondent judge issued a commitment order for Go. Upon arraignment, a plea of not guilty was entered because Go refused to enter a plea. 10. Go filed a PETITION FOR HABEAS CORPUS and the CA issued the writ. The petition for habeas corpus was consolidated with the petition for certiorari, prohibition and mandamus. - September 19, 1991 The trial started and the prosecution presented its first witness. This was followed by three more witnesses on October 3, 1991. - September 23, 1991 The CA dismissed the petition for habeas corpus and the petition for certiorari, prohibition and mandamus on the following grounds, among others: a) Validity of the warrantless arrest because the crime had been freshly committed. He was positively identified by the witness and his identity had been established when he came to the police station. b) Waiver of the right to preliminary investigation when he did not invoke it properly and waiver of any irregularity in his arrest when accused posted bail. c) Validity of the information against the accused precluded the grant of the petition for habeas corpus Petitioners Claim: Go contends that the crime had not been just committed because of the 6-day disparity. - None of the police officers who arrested him had any personal knowledge of the crime.

Criminal Procedure Rowena Daroy Morales


- When Go walked into the police station 6 days after Maguan was shot, he did not surrender (so as not to imply that he committed the crime) nor was he arrested but he placed himself in the disposal of the police authorities. 2. NO, Go had not waived his right to preliminary investigation. Ratio The rule is that the right to preliminary investigation is waived when the accused fails to invoke it before or at the time of entering a plea at arraignment. Reasoning - The right to have a preliminary investigation conducted before being bound over to trial for a criminal offense and hence formally at risk of incarceration or some other penalty, is not a mere formal or technical right; it is a substantive right. - The nature of the crime demanded that a preliminary investigation be conducted. Go did ask for a preliminary investigation from the start. On the day the information for murder was filed, he also filed an OMNIBUS MOTION for IMMEDIATE RELEASE and PRELIMINARY INVESTIGATION. The Court is not ready to ignore that act by Go and consider it as a waiver based simply on the contention of the SolGen that the motion should have been filed with the trial court and not the prosecutor. - According to Crespo vs. Mogul: The preliminary investigation conducted by the fiscal for the purpose of determining whether a prima facie case exists warranting the prosecution of the accused is terminated upon the filing of the information in the proper court. Should the fiscal find it proper to conduct a reinvestigation of the case, at such stage, the permission of the Court must be secured. After such reinvestigation the finding and recommendations of the fiscal should be submitted to the Court for appropriate action. - However, in the case at bar, Gos omnibus motion asked for a PRELIMINARY INVESTIGATION not REINVESTIGATION as discussed in Crespo vs. Mogul. The Prosecutor also filed a MOTION for LEAVE TO CONDUCT PRELIMINARY INVESTIGATION so the omnibus motion of Go was, in effect, filed in the trial court. Go did ask for a preliminary investigation on the very day that the information was filed without such preliminary investigation, and that the trial court was 5 days later apprised of the desire of the petitioner for such preliminary investigation. - There was no waiver of the right to preliminary investigation because Go had vigorously insisted on his right to preliminary investigation before his arraignment. At the time of his arraignment,

a2010

page 65

Prof.
- Meantime, petitioner is hereby ORDERED released forthwith upon posting of a cash bail bond of One Hundred Thousand Pesos (P100,000.00). This release shall be without prejudice to issue, should the any lawful order that the trial court Office of the Provincial Prosecutor move for cancellation of all at the conclusion of the preliminary investigation.

petitioner was already before the Court of Appeals on certiorari, prohibition and mandamus precisely asking for a preliminary investigation before being forced to stand trial. - Gos act of posting bail cannot be deemed to be a waiver of his right to preliminary investigation. Go asked for release on recognizance or on bail and for preliminary investigation in one omnibus motion. He had thus claimed his right to preliminary investigation before respondent Judge approved the cash bond posted by petitioner and ordered his release. Obiter - However, contrary to petitioner's contention, the failure to accord preliminary investigation, while constituting a denial of the appropriate and full measure of the statutory process of criminal justice, did not impair the validity of the information for murder nor affect the jurisdiction of the trial court. - In the case at bar, a trial for merits had already commenced and the prosecution had already presented 4 witnesses. > This, however, still entitles the accused to preliminary investigation. Trial on the merits should be suspended or held in abeyance and a preliminary investigation should accorded to petitioner, even if eventually, the prosecutor may or may not find probable cause. The point is that Go was not accorded his proper rights. > As for bail, Go is still entitled to be released on bail as a matter of right. Should the evidence against the accused be strong, the bail can then be cancelled. > To hold that the rights of Go were obliterated by the presentation of evidence in the proceedings in the trial court would be to legitimize the deprivation of due process. Dispositive ACCORDINGLY, the Court Resolved to GRANT the Petition for Review on Certiorari. The Order of the trial court dated 17 July 1991 is hereby SET ASIDE and NULLIFIED, and the Decision of the Court of Appeals dated 23 September 1991 hereby REVERSED. - The Office of the Provincial Prosecutor is hereby ORDERED to conduct forthwith a preliminary investigation of the charge of murder against petitioner Go, and to complete such preliminary investigation within a period of fifteen (15) days from commencement thereof. The trial on the merits of the criminal case in the Regional Trial Court shall be SUSPENDED to await the conclusion of the preliminary investigation.

SEPARATE OPINION CRUZ [concurring]


- There was no waiver of the right to preliminary investigation even if Go freely participated in his trial and his counsel even cross-examined the prosecution witnesses. - Go had from the start demanded a preliminary investigation and that his counsel had reluctantly participated in the trial only because the court threatened to replace him with a counsel de oficio if he did not. The petitioner was virtually compelled to go to trial. Such compulsion and the unjustified denial of a clear statutory right of the petitioner vitiated the proceedings as violative of procedural due process. - It appears that the trial court has been moved by a desire to cater to public opinion to the detriment of the impartial administration of justice. The petitioner as portrayed by the media is not exactly a popular person. Nevertheless, the trial court should not have been influenced by this irrelevant consideration, remembering instead that its only guide was the mandate of the law.

GUTIERREZ [concurring]
- The need for a trial court to follow the Rules and to be fair, impartial, and persistent in getting the true facts of a case is present in all cases but it is particularly important if the accused is indigent; more so, if he is one of those unfortunates who seem to spend more time behind bars than outside.

GRIO-AQUINO [dissenting]
- After 4 witnesses have already testified, among them an eyewitness who identified the accused as the gunman and a security guard who identified the plate number of the gunman's car, there is no need to conduct a preliminary investigation the sole purpose of which would be to ascertain if there is sufficient ground to believe that a crime was committed (which the petitioner does not dispute) and that he (the petitioner) is probably guilty thereof

Criminal Procedure Rowena Daroy Morales


(which the prosecutor, by filing the information against him, presumably believed to be so). - This case did not suffer from a lack of previous investigation. Diligent police work, with ample media coverage, led to the identification of the suspect who, 7 days after the shooting, appeared at the San Juan police station to verify news reports that he was the object of a police manhunt. There witnesses identified him to be the assailant. - It should be remembered that as important as is the right of the accused to a preliminary investigation, it is not a constitutional right. Its absence is not a ground to quash the information. It does not affect the court's jurisdiction, nor impair the validity of the information, nor constitute an infringement of the right of the accused to confront witnesses. - The petitioner's motion for a preliminary investigation is not more important than his application for release on bail, just as the conduct of such preliminary investigation is not more important than the hearing of the application for bail. The court's hearing of the application for bail should not be subordinated to the preliminary investigation of the charge. The hearing should not be suspended, but should be allowed to proceed because the parties will have an opportunity to show not only: (a) whether or not there is probable cause to believe that the petitioner killed Eldon Maguan, but more importantly (b) whether or not the evidence of his guilt is strong. The judge's determination that the evidence of his guilt is strong would naturally foreclose the need for a preliminary investigation to ascertain the probability of his guilt. - Go was indeed arrested by the police. Arrest is the taking of a person into custody in order that he may be bound to answer for the commission of an offense. An arrest is made by an actual restraint of the person to be arrested, or by his submission to the custody of the person making the arrest

a2010

page 66

Prof.
Ombudsman. The petitioners right to a preliminary investigation of the new charge is secured to him by Rule 112 of the 1985 Rules on Criminal Procedure. That right of the accused is substantial and its denial over his opposition is a prejudicial error in that it subjects the accused to loss of life, liberty, or property without due process of law. Since the right belongs to the accused, he alone may waive it. If he demands it, the State may not withhold it. However, as the absence of a preliminary investigation is not a ground to quash the complaint or information, the proceedings upon such information in the Sandiganbayan should be held in abeyance and the case should be remanded to the office of the Ombudsman for him or the Special Prosecutor to conduct a preliminary investigation. 2. NO - Since the petitioner is an incumbent public official charged in a valid information with an offense punishable under the Constitution and the laws, the laws command that he shall be suspended from office pendent lite must be obeyed. His approved leave of absence should not be a bar to his preventive suspension for, as indicated by the Solicitor General, and approved leave, whether it be for a fixed of indefinite period may be cancelled or shortened at will by the incumbent. However, since the preventive suspension has exceeded the reasonable maximum period of ninety days provided in Section 42 of the Civil Service Decree of the Philippines, it should now be lifted. Dispositive Petition for certiorari and prohibition is granted.

against petitioner, Quintin Doromal, a former commissioner of the Presidential Commission on Good Government for violation of the Anti-Graft and Corrupt Practices Act in connection with his shareholdings and position as president and director of the Doromal International Trading Corporation which submitted bids to supply equipment to the DECS and the National Manpower and Youth Council. - January 25, 1988, Special Prosecution Officer filed in the Sandiganbayan an information against petitioner. - The petitioner filed a petition for certiorari and prohibition in the SC questioning the jurisdiction of the Tanodbayan to file the information without the approval of the Ombudsman after the effectivity of the 1987 Constitution. - June 30, 1988, the SC annulled the information - Upon the annulment of the information against the petitioner, the Special Prosecutor sought clearance from the Ombudsman to refile it - The Ombudsman granted clearance but advised that some changes be made in the information previously filed - A new information was filed in the Sandiganbayan - Petitioner filed a motion to quash the information for being invalid because there had been no preliminary investigation and defective because the facts alleged do not constitute the offense charged. - The Sandiganbayan denied the motion to quash - The Special Prosecutor filed a motion to suspend accused pendente lite. Over the objection of the accused the Sandiganbayan ordered his suspension pendente lite from his position as PCGG Commissioner and from any other office he may be holding. ISSUES 1. WON the Sandiganbayan committed grave abuse of discretion in denying petitioners motion to quash the information 2. WON the Sandiganbayan committed grave abuse of discretion in suspending the petitioner from office despite the Presidents having previously approved his indefinite leave of absence until final decision in the case HELD 1. YES - A new preliminary investigation of the charge against the petitioner is in order not only because the first was a nullity but also because the accused demands it as his right. Moreover, the charge against him had been changed as directed by the

ALLADO v DIOKNO 232 SCRA 192 BELLOSILLO; May 5, 1994


NATURE Petition for certiorari and prohibition with prayer for a temporary restraining order FACTS - Petitioners Diosdado Jose Allado and Roberto L. Mendoza, alumni of the College of Law, University of the Philippines, are partners of the Law Firm of Salonga, Hernandez and Allado. In the practice of their profession, and on the basis of an alleged extrajudicial confession of a security guard (Umbal), they have been accused of the heinous crime of kidnapping with murder of a German national named Van Twest by the Presidential Anti-Crime Commission

DOROMAL v SANDIGANBAYAN (OMBUDSMAN and SPECIAL PROSECUTOR) 177 SCRA 1989 GRINO-AQUINO; September 7, 1989
NATURE Petition for Certiorari FACTS - October 1987, the Special Prosecution Officer conducted a preliminary investigation of the charge

Criminal Procedure Rowena Daroy Morales


(PACC) and ordered arrested without bail by respondent judge. - Petitioners filed this petition and principally contended that respondent judge acted with grave abuse of discretion and in excess of jurisdiction in "whimsically holding that there is probable cause against petitioners without determining the admissibility of the evidence against petitioners and without even stating the basis of his findings," and in "relying on the Resolution of the Panel and their certification that probable cause exists when the certification is flawed." Petitioners maintain that the records of the preliminary investigation which respondent judge solely relied upon failed to establish probable cause against them to justify the issuance of the warrant of arrest. Petitioners likewise assail the prosecutors' "clear sign of bias and partiality." - On the other hand, the Office of the Solicitor General argues that the determination of probable cause is a function of the judge who is merely required to personally appreciate certain facts to convince him that the accused probably committed the crime charged. ISSUE WON the respondent judge committed grave abuse of discretion in the preliminary inquiry which determines probable cause for the issuance of a warrant of arrest HELD - In the Order of respondent judge, it is expressly stated that "[t]his court after careful evaluation of the evidence on record, believes and rules that probable cause exists; and therefore, a warrant of arrest should be issued." However, we are unable to see how respondent judge arrived at such ruling. We have painstakingly examined the records and we cannot find any support for his conclusion. On the contrary, we discern a number of reasons why we consider the evidence submitted to be insufficient for a finding of probable cause against petitioners. - The PACC relies heavily on the sworn statement of Security Guard Umbal who supposedly confessed his participation in the alleged kidnapping and murder of Van Twest. For one, there is serious doubt on Van Twest's reported death since the corpus delicti has not been established, nor have his remains been recovered. Umbal claims that Van Twest was completely burned into ashes with the use of gasoline and rubber tires from around ten o'clock in the evening to six o'clock the next morning. This is

a2010

page 67

Prof.
preliminary investigation proper which ascertains whether the offender should be held for trial or released. Even if the two inquiries be conducted in the course of one and the same proceeding, there should be no confusion about their objectives. The determination of probable cause for the warrant is made by the judge. The preliminary investigation proper whether or not there is reasonable ground to believe that the accused is guilty of the offense charged and therefore, whether or not he should be subjected to the expense, rigors and embarrassment of trial is a function of the prosecutor. - ALLADO DOCTRINE: If upon the filing of the information in court, the trial judge, after reviewing the information and the document attached thereto, finds that no probable cause exists, he must either call for the complainant and the witnesses themselves or simply dismiss the case. There is no reason to hold the accused for trial and further expose him to an open and public accusation of the crime when no probable cause exists. Dispositive Petition granted

highly improbable, if not ridiculous. A human body cannot be pulverized into ashes by simply burning it with the use of gasoline and rubber tires in an open field. Even crematoria use entirely closed incinerators where the corpse is subjected to intense heat. Thereafter, the remains undergo a process where the bones are completely ground to dust. - Strangely, if not awkwardly, after Van Twest's reported abduction which culminated in his decimation by cremation, his counsel continued to represent him before judicial and quasi-judicial proceedings. Hence, even Asst. Solicitor General Estoesta believes that counsel of Van Twest doubted the latter's death. - Verily, respondent judge committed grave abuse of discretion in issuing the warrant for the arrest of petitioners it appearing that he did not personally examine the evidence nor did he call for the complainant and his witnesses in the face of their incredible accounts. Instead, he merely relied on the certification of the prosecutors that probable cause existed. For, otherwise, he would have found out that the evidence thus far presented was utterly insufficient to warrant the arrest of petitioners. - In Soliven v. Makasiar, we said that the judge (a) shall personally evaluate the report and the supporting documents submitted by the fiscal regarding the existence of probable cause and, on the basis thereof, issue a warrant of arrest; or, (b) if on the basis thereof he finds no probable cause, may disregard the fiscal's report and require the submission of supporting affidavits of witnesses to aid him in arriving at a conclusion on the existence of probable cause. - In People v. Inting, we emphasized the important features of the constitutional mandate: (a) The determination of probable cause is a function of the judge; it is not for the provincial fiscal or prosecutor to ascertain. Only the judge and the judge alone makes this determination; (b) The preliminary inquiry made by a prosecutor does not bind the judge. It merely assists him in making the determination of probable cause. The judge does not have to follow what the prosecutor presents to him. By itself, the prosecutor's certification of probable cause is ineffectual. It is the report, the affidavits, the transcript of stenographic notes (if any), and all other supporting documents behind the prosecutor's certification which are material in assisting the judge in his determination of probable cause; and, (c) Judges and prosecutors alike should distinguish the preliminary inquiry which determines probable cause for the issuance of a warrant of arrest from the

ROBERTS v CA 254 SCRA 307 DAVIDE, JR; March 5, 1996


FACTS - Several thousand holders of 349 Pepsi crowns in connection with the Number Fever Promotion filed with the Office of the City Prosecutor of Quezon City complaints against the petitioner officials of PEPSI. - The petitioners filed with the Office of the City Prosecutor a motion for the reconsideration of the Joint Resolution and with the DOJ a Petition for Review. The petitioners also Motions to Suspend Proceedings and to hold in Abeyance Issuance of Warrants of Arrest on the ground that they had filed the aforesaid Petition for Review. - Respondent Judge Asuncion issued the challenged order (1) denying the petitioners Motion to Suspend Proceedings and to Hold In Abeyance Issuance of Warrants of Arrest and the public prosecutors Motion to Defer Arraignment and (2) directing the issuance of the warrants of arrest after and setting the arraignment on 28 June 1993. - The petitioners filed with the Court of Appeals a special civil action for certiorari and prohibition with application for a temporary restraining order. They contended therein that respondent Judge Asuncion had acted without or in excess of jurisdiction or with grave abuse of discretion in issuing the

Criminal Procedure Rowena Daroy Morales


aforementioned order. - The Court of Appeals then issued a resolution denying the application for a writ of preliminary injunction. ISSUE WON public respondent Judge Asuncion committed grave abuse of discretion in ordering the issuance of warrants of arrest without examining the records of the preliminary investigation. HELD YES. - Section 2, Article III of the present Constitution provides that no search warrant or warrant of arrest shall issue except upon probable cause to be determined personally by the judge after examination under oath or affirmation of the complainant and the witnesses he may produce. - The determination of probable cause is a function of the Judge. It is not for the Provincial Fiscal or Prosecutor nor the Election Supervisor to ascertain. Only the Judge and the Judge alone makes this determination. - The preliminary inquiry made by a Prosecutor does not bind the Judge. It merely assists him to make the determination of probable cause. The Judge does not have to follow what the Prosecutor presents to him. By itself, the Prosecutors certification of probable cause is ineffectual. It is the report, the affidavits, the transcripts of stenographic notes (if any), and all other supporting documents behind the Prosecutors certification which are material in assisting the Judge to make his determination. - The teachings of the cases of Soliven3, Inting4, Lim5, Allado, and Webb reject the proposition that the investigating prosecutors certification in an information or his resolution which is made the basis
3
The Judge does not have to personally examine the complainant and his witnesses. The Prosecutor can perform the same functions as a commissioner for the taking of the evidence. However, there should be a report and necessary documents supporting the Fiscals bare certification. All of these should be before the Judge. 4 The supporting documents may consist of, viz., the affidavits, the transcripts of stenographic notes (if any), and all other supporting documents behind the Prosecutors certification which are material in assisting the Judge to make his determination of probable cause

a2010

page 68

Prof.

for the filing of the information, or both, would suffice in the judicial determination of probable cause for the issuance of a warrant of arrest. - In the present case, nothing accompanied the information upon its filing with the trial court. Clearly, when respondent Judge Asuncion issued the assailed order directing, among other things, the issuance of warrants of arrest, he had only the information, amended information, and Joint Resolution as bases thereof. He did not have the records or evidence supporting the prosecutors finding of probable cause. And strangely enough, he made no specific finding of probable cause; he merely directed the issuance of warrants of arrest. It may, however, be argued that the directive presupposes a finding of probable cause. But then compliance with a constitutional requirement for the protection of individual liberty cannot be left to presupposition, conjecture, or even convincing logic.

ISSUE WON the CA erred in finding that no probable cause exists to merit the filing of charges against private respondent Billy Cerbo

PEOPLE v CA (CERBO) (Republic v CA in page 5 of the outline) 301 SCRA 475 PANGANIBAN; January 21, 1999
NATURE Petition for Review FACTS - Private Respondent Jonathan Cerbo shot, at pointblank range, Rosalinda Dy in the presence and at the office of his father private respondent Billy Cerbo. - An information for murder was filed against Jonathan Cerbo. - The daughter of the victim executed an affidavitcomplaint charging private respondent Billy Cerbo of conspiracy in the killing. - Accordingly, the prosecution filed an amended information including Billy Cerbo in the murder case. A warrant for his arrest was later issued. - Billy Cerbo filed a motion to quash warrant of arrest arguing that the same was issued without probable cause. - The respondent judge issued an order dismissing the case against Billy Cerbo and recalling the warrant for his arrest. - The Court of Appeals debunked the petitioners assertion that the trial judge committed grave abuse of discretion and that the evidence presented thus far did not substantiate the charge.. Hence this petition.

HELD YES - The petition is meritorious. - The determination of probable cause during preliminary investigation is a function that belongs to the public prosecutor. It is an executive function. - The public prosecutor has the quasi-judicial authority to determine whether or not a criminal case must be filed in court. - The primary objective of a preliminary investigation is to free respondent from the inconvenience, expense, ignominy, and stress of defending himself/herself in the course of a formal trial, until the reasonable probability of his or her guilt in a more or less summary proceeding by a competent office designated by law for that purpose.6 - Secondarily, such summary proceeding also protects the state from the burden of the unnecessary expense of an effort in prosecuting alleged offenses and in holding trials arising from false, frivolous, or groundless charges.7 - The determination of probable cause to hold a person for trial must be distinguished from the determination of probable cause to issue a warrant of arrest, which is a judicial function. - A judge cannot be compelled to issue a warrant of arrest if he or she deems that there is no probable cause for doing so. - Corrollarily, the judge should not override the public prosecutors determination of probable cause to hold an accused for trial, on the ground that the evidence presented to substantiate the issuance of an arrest warrant was insufficient, as in the present case. - Therefore, if the information is valid on its face, and there is no showing of manifest error, grave abuse of discretion and prejudice on the part of the public prosecutor, the trial court should respect such determination. Dispositive Reversed.

The issuance of the warrants of arrest by a judge solely on the basis of the prosecutors certification in the information that there existed probable cause, without having before him any other basis for his personal determination of the existence of a probable cause, is null and void.

CASTILLO v VILLALUZ
6 7

Ledesma v. CA, 278 SCRA 657, Sept. 5, 1997. Id.

Criminal Procedure Rowena Daroy Morales


171 SCRA 39 NARVASA; March 8, 1989
NATURE Petition for certiorari and prohibition FACTS - In July 1971, a complaint and a Joint Affidavit were filed directly by Renato Montes and Jose de Silva against Manuel Laconico. The complaint charged the latter with estafa in the amount of P1K. Preliminary investigation (now in question) was conducted by respondent Judge of the Circuit Criminal Court, and thereafter issued a warrant of arrest. He ordered Provincial Fiscal to file the corresponding information against the respondent before the court of competent jurisdiction within 24 hours from receipt of said order. - Provincial Fiscal failed to file the information required within the time appointed, or at any time thereafter. Consequently, he was directed by His Honor to explain within 10 days "why he should not be punished for contempt of court for delaying the speedy administration of justice for disobeying a lawful order of the Court." Fiscal filed a MFR, but was denied. Hence, this petition for certiorari and prohibition was presented by petitioner Fiscal, seeking annulment of the aforesaid orders. ISSUES 1.WON respondent judge had no jurisdiction to conduct preliminary investigations, because the law creating Circuit Criminal Courts, R.A. 5179, did not confer on said courts the power to conduct preliminary investigations 2. WON judge erred in compelling fiscal under sanction of contempt, to file an information in court without conducting his own preliminary investigation HELD 1. YES Ratio: The conduct of a preliminary investigation is not a judicial function but part of the fiscals job, a function of the executive. Wherever there are enough fiscals or prosecutors to conduct preliminary investigations, courts are counseled to leave this job which is essentially executive to them, and the fact that a certain power is granted does not necessarily mean that it should be indiscriminately exercised. Reasoning: [a] Sec. 37 of BP. 129 reiterated the removal from Judges of Metropolitan Trial Courts in the National Capital Region of the authority to conduct preliminary investigations and Sec 2 of Rule

a2010

page 69

Prof.
FERNANDO; May 13, 1981
NATURE Writ of Certiorari FACTS -Counsel for Reynaldo Rodil who was charged with murder, asks to recall witnesses for the prosecution to enable such counsel to cross-examine them, on, to quote his words, "clarificatory and amplificatory matters" which was denied by Municipal Judge Segundo M. Garcia of Sta. Cruz, Marinduque. -What is prayed for is not only that such order denying counsel's request to recall government witnesses be set aside and nullified, but also that bail be granted petitioner, a petition to that effect having been denied with a subsequent motion for reconsideration still undecided. -Respondents were required to comment and the Court likewise issued a temporary restraining order. Such a comment was submitted on behalf of respondents by the Solicitor General seeking the dismissal of the petition on the ground that the right to cross-examine in a preliminary investigation is not a right granted an accused and that the exercise of discretion by respondent Judge considering the evidence of record sufficed to justify denial of the application for bail. -An examination of the record, as well as the pertinent doctrines, makes evident that the jurisdictional issue posed arises from the failure to accord petitioner a hearing on his application for bail. -A resolution of that question in the sense of respondent Judge affording petitioner his day in court is equally decisive of the other issue, whether or not counsel for petitioner could recall witnesses for the prosecution for the purpose of asking clarificatory questions. That he could very well do when they testify to prove evidence of guilt is strong. Under the present state of the law, it cannot be said that the right to cross-examine is guaranteed an accused at the stage of preliminary investigation. ISSUE WON counsel for petitioner could recall witnesses for the prosecution for the purpose of asking clarificatory questions (that he could very well do when they testify to prove evidence of guilt is strong) HELD

112 of 1985 Rules on Criminal Procedure no longer authorizes RTC Judges to conduct PIs. [b] The assignment of PI function to judges of inferior courts and to a very limited extent to courts of first instance was dictated by necessity and practical considerations, and the consequent policy, was that wherever there were enough fiscals or prosecutors to conduct preliminary investigations, courts were to leave that job which is essentially executive to them. It follows that the conclusions derived by a judge from his own investigation cannot be superior to and conclusively binding on the fiscal or public prosecutor, in whom that function is principally and more logically lodged. 2. YES The power to conduct PI is lodged in the fiscal. It is grave abuse of discretion on a judge to seek to foreclose the fiscal's prerogative to conduct his own preliminary investigation to determine for himself the existence or non-existence of probable cause, and to require him to show cause for not filing the information within 24 hours, on the sole basis of the Judge's conclusions. The fiscal has the duty to satisfy himself of the existence of probable cause, and could not shirk or be made to evade it by an unreasoning and indiscriminate reliance on the judge's investigation. Dispositive: Petition GRANTED. Challenged Orders annulled and set aside.

SEPARATE OPINION CRUZ [concurring]


- The fiscal prevails over the judge only in the determination of the existence of a prima facie case to justify the filing of a complaint or information. This task is executive. - But the determination of probable cause to justify the issuance of a search warrant or a warrant of arrest is the constitutional prerogative of the judge and may not be withdrawn from him or even only limited by statute or ROC. This task is judicial. The findings of fiscal in the PI do not control or foreclose the exercise of the power conferred personally on the judge under Sec. 2 the Bill of Rights. That power is his alone.

BALGOS v SANDIGANBAYAN [SUPRA, PAGE 34] RODIL v GARCIA 104 SCRA 362

Criminal Procedure Rowena Daroy Morales


YES. Counsel could recall the witnesses. - The Judge issued the denial for bail on the basis of the motion of petitioner that he be granted such right and the opposition filed by the First Assistant Provincial Fiscal without conducting any hearing on such motion. Clearly, he acted on the mistaken belief that the presentation of evidence by the prosecution for the purpose of the issuance of the warrant of arrest, the preliminary examination proper, suffices for the denial of the plea for bail. In the latest case on the subject, People v. Sola, decided on March 17, 1981, this Court relying on People v. San Diego, nullified an order of a municipal judge named respondent in that case as he granted bail to the accused without hearing the prosecution. The present case is much stronger; it is the accused himself, the explicit beneficiary of the constitutional right, who was not heard. - There was misapprehension on the part of respondent Judge of the import of the ruling in Ocampo v. Bernabe citing that The regular trial is, to a limited extent at least, anticipated. While the guilt or innocence of the accused is not to be determined, the quantity and character of the proofs on this point are, for the special purpose in hand, necessarily considered. Occasionally much time is thus consumed, and the court's attention is correspondingly diverted from other business. But these objections cannot avail against a positive constitutional command; if the Constitution requires the court to determine for itself whether or not the proof is evident or presumption great in a given case, all considerations of expediency or convenience, however potent they might be at the common law, must give way.'" This is so because the procedure to be followed in the hearing on an application for bail, while summary in character, is not to be a mere sham or pretense. It must not be an exercise in futility. The accused is not to be denied his day in court. - While it could be said that that the refusal of the Justice of the Peace to allow the defense to cross-examine the prosecution's witnesses presented prior to petitioner's arrest, cannot be utilized as argument for the contention that the prosecution should not have been allowed to cross-examine the defense witnesses and that an accused is not entitled to crossexamine the witnesses presented against him in the preliminary investigation before his arrest, this being a matter that depends on the

a2010

page 70

Prof.
was on its way to another place, Lalabuan, also in Masiu, had been ambushed - The next day, a lawyer (Atty. Batuampar) of one of the widows filed a letter-complaint with the fiscal, asking for a full blast preliminary investigation. The letter adverted to the possibility of innocent persons being implicated by the parties involved on both sides none of whom was, however, identified and promised that supporting affidavits would shortly be filed. Immediately the Provincial Fiscal addressed a "1st indorsement" to the respondent Judge, transmitting Atty. Batuampar's letter and requesting that "all cases that may be filed relative .. (to the incident) that happened in the afternoon of July 27, 1985," be forwarded to his office, which "has first taken cognizance of said cases. - On August 10, 1985, a criminal complaint for multiple murder was filed. On the same day, respondent Judge examined personally the 3 witnesses. Thereafter, the Judge approved the complaint and issued a warrant of arrest against the 14 petitioners (who were named by the witnesses) and 50 "John Does. - On Aug 14, 1985, an ex-parte motion was filed by Atty. Batuampar seeking recall of the warrant of arrest and subsequent holding of a "thorough investigation" on the ground that the Judge's initial investigation had been "hasty and manifestly haphazard" with "no searching questions" having been propounded. The respondent Judge denied the motion for "lack of basis;" hence the present petition. ISSUE WON the respondent Judge had the power to issue the warrant of arrest without completing the entire prescribed procedure for preliminary investigation HELD YES. - What the Rule provides is that no complaint or information for an offense cognizable by the Regional Trial Court may be filed without completing that procedure. Sec. 6 of Rule 112 clearly authorizes the MTC to issue a warrant even before opening the second phase. - This was equally true under the former rules, where the first phase of the investigation was expressly denominated "preliminary examination" to distinguish it from the second phase, or preliminary investigation proper - Sec 3 of rule 112 consists of 2 phases: - The first phase consists of an ex-parte inquiry into the sufficiency of the complaint and the affidavits

sound discretion of the Judge or investigating officer concerned (People v. Ramilo, \Dequito v. Arellano, Bustos v. Lucero) it could still be argued that the judge is not a ministerial officer reduced to recording what takes place and what witnesses say in the examination. Above all, his is the great responsibility of safeguarding the accused from groundless or vindictive prosecution. If the justice of the peace is to ascertain, as he must, whether a crime has been committed and, if so, whether there is probable cause that the accused committed it, his authority cannot be confined as in a straight jacket to the stiffness of medieval and outmoded technicalities of practice. It thus appears clearly that in the exercise of his discretion respondent Judge could have granted the request and thus avoided the necessity of a petition of this character having to be filed. The interest of a more speedy and a more efficient administration of justice would be best served if there is a greater awareness on the part of judges that in addition to safeguarding the express rights of an accused person, a matter mandated by the Constitution or the Rules of Court, they should likewise exercise their discretion in such a way that the purpose of a preliminary investigation, the avoidance of groundless or vindictive prosecutions, could be attained in as fair and objective manner as possible. Dispositive WHEREFORE, the writ of certiorari is granted. The order of respondent Judge denying bail is set aside. He, or whoever is now the Municipal Judge of Santa Cruz, Marinduque, must set forthwith the hearing on the application for bail of petitioner, to be conducted in accordance with the requirements of the Constitution, the Rules of Court, and this opinion. No costs.

PANGANDAMAN v CASAR 159 SCRA 599 NARVASA; April 14, 1988


FACTS - On July 27, 1985, a shooting incident occurred in Pantao, Masiu, Lanao del Sur, which left at least five persons dead and two others wounded. What in fact transpired is still unclear. According to one version, armed men had attacked a residence in Pantao, Masiu, with both attackers and defenders suffering casualties. Another version has it that a group that

Criminal Procedure Rowena Daroy Morales


and other documents offered in support thereof. And it ends with the determination by the Judge either: (1) that there is no ground to continue with the inquiry, in which case he dismisses the complaint and transmits the order of dismissal, together with the records of the case, to the provincial fiscal; or (2) that the complaint and the supporting documents show sufficient cause to continue with the inquiry and this ushers in the second phase. - This second phase is designed to give the respondent notice of the complaint, access to the complainant's evidence and an opportunity to submit counter-affidavits and supporting documents. At this stage also, the Judge may conduct a hearing and propound to the parties and their witnesses questions on matters that, in his view, need to be clarified. The second phase concludes with the Judge rendering his resolution, either for dismissal of the complaint or holding the respondent for trial, which shall be transmitted, together with the record, to the provincial fiscal for appropriate action. - The argument, therefore, must be rejected that the respondent Judge acted with grave abuse of discretion in issuing the warrant of arrest against petitioners without first completing the preliminary investigation in accordance with the prescribed procedure. The rule is and has always been that such issuance need only await a finding of probable cause, not the completion of the entire procedure of preliminary investigation - Also without appreciable merit is petitioners' other argument that there was scarcely time to determine probable cause against sixty-four persons (the fourteen petitioners and fifty "Does") within a matter of hours on a Saturday when municipal trial courts are open only from 8:00 a.m. to 1:00 p.m. Nothing in the record before this Court belies or discredits those affirmations which have, besides, the benefit of the legal presumption that official duty has been regularly performed. - Insofar, however, as said warrant is issued against fifty (50) "John Does" not one of whom the witnesses to the complaint could or would Identify, it is of the nature of a general warrant, one of a class of writs long proscribed as unconstitutional and once anathematized as "totally subversive of the liberty of the subject." Clearly violative of the constitutional injunction that warrants of arrest should particularly describe the person or persons to be seized, the warrant must, as regards its unidentified subjects, be voided. Dispositive Warrants against petitioners upheld; warrants against John Does denied

a2010

page 71

Prof.
Ratio 3 conditions must concur for the issuance of the warrant of arrest. The investigating judge must: (a) have examined in writing and under oath the complainant and his witnesses by searching questions and answers; (b) be satisfied that a probable cause exists; and (c) that there is a need to place the respondent under immediate custody in order not to frustrate the ends of justice. Reasoning The mandatory provision that the investigating judge "must issue a warrant of arrest" if he finds probable cause that the respondent committed the crime charged, found in all previous rules of criminal procedure, from General Orders No. 58 down to Rule 112 of the 1964 Revised Rules of Court, is absent in Section 1 of the 1985 Rules on Criminal Procedure. It is not obligatory, but merely discretionary, upon the investigating judge to issue a warrant for the arrest of the accused, for the determination of whether a probable cause exists and whether it is necessary to arrest the accused in order not to frustrate the ends of justice, is left to his sound judgment or discretion. In this particular case, since the robbery charge was the offshoot of a boundary dispute between two property owners, the investigating judge did not believe there was any danger of the accused absconding before the filing of the information against him by the fiscal, hence, he found no need to place him under immediate custody. Dispositive The appealed decision is SET ASIDE.

SAMULDE v SALVANI 165 SCRA 734 GRIO-AQUINO: September 26, 1988


NATURE Appeal from the decision of the RTC FACTS - Municipal Judge Samulde conducted a preliminary investigation upon a complaint for robbery. After making a preliminary investigation based on the affidavits of the complainant and her witnesses and counter-affidavits of the respondent and his witnesses, Judge Samulde transmitted the records of the case to Provincial Fiscal Salvani with his finding that "there is prima facie evidence of robbery as charge in the complaint". The fiscal returned the records on the ground that Judge Samulde failed to include the warrant of arrest against the accused as provided in Sec 5, Rule 112 of the 1985 Rules on Criminal Procedure. Judge Samulde sent back the records to Fiscal Salvani. He pointed out that under Sec 6, Rule 112, he may issue a warrant of arrest if he is satisfied "that a probable cause exists and that there is a necessity of placing the respondent under immediate custody in order not to frustrate the ends of justice, " implying that, although he found that a probable cause existed, he did not believe that the accused should be immediately placed under custody. Hence, he refused to issue a warrant of arrest. - A special civil action of mandamus was filed in the RTC by Provincial Fiscal Salvani against Judge Samulde to compel the latter to issue a warrant of arrest. The RTC dismissed the petition but nevertheless ordered Judge Samulde to issue a warrant of arrest, and to transmit the warrant to the Provincial Fiscal for appropriate action. He further advised the Municipal Judge "that henceforth he adheres to the same rule in similar cases where he conducts a preliminary investigation with a finding of a prima facie or probable cause." Unconvinced, Judge Samulde appealed to this Court. ISSUE WON a judge may be compelled to issue a warrant of arrest upon a finding of probable cause HELD NO

TANDOC v RESULTAN [SUPRA, PAGE 43] LIM vFELIX 194 SCRA 292 GUTIERREZ; February 19, 1991
NATURE Review for certiorari. FACTS - March 17, 1989: at the vicinity of the airport road of the Masbate Domestic Airport (Masbate, Masbate), Congressman Moises Espinosa, Sr. and his security escorts, namely Provincial Guards Antonio Cortes, Gaspar Amaro, and Artemio Fuentes were attacked and killed by a lone assassin. Dante Siblante, another security escort of Congressman Espinosa, Sr. survived the assassination plot, although, he himself suffered a gunshot wound.

Criminal Procedure Rowena Daroy Morales


- For the purpose of preliminary investigation, the designated investigator, Harry O. Tantiado, TSg (Legaspi) filed an amended complaint with the Municipal Trial Court of Masbate accusing, among others, Vicente Lim, Sr., Mayor Susana Lim of Masbate, Jolly T. Fernandez, Florencio T. Fernandez, Jr., Nonilon A. Bagalihog, Mayor Nestor C. Lim and Mayor Antonio Kho of the crime of multiple murder and frustrated murder in connection with the airport incident. - July 31, 1989: after the preliminary investigation court released an order stating after that a probable cause has been established for the issuance of a warrant of arrest of named accused in the amended complaint. - August 29, 1989: records of the case were transmitted to Provincial Prosecutor of Masbate. Respondent Acting Fiscal Antonio C. Alfane was designated to review the case. On September 22, 1989, Fiscal Alfane issued a Resolution which affirmed the finding of a prima facie case against the petitioners but differed in the designation of the crime (he said it should be MURDER for each case, with serious physical injuries). Fiscal Alfane filed with the Regional Trial Court of Masbate, four (4) separate informations of murder against the twelve (12) accused with a recommendation of no bail. - Respondents Lims filed a verified petition for a change of venue. Court granted the petition. The case was raffled to Judge Nemesio Felix. - Lims then prayed for the following: 1. An order be issued requiring the transmittal of the initial records of the preliminary inquiry or investigation conducted by the Municipal Judge Barsaga of Masbate for the best enlightenment of this Honorable Court in its personal determination of the existence of a probable cause or prima facie evidence as well as its determination of the existence of guilt, pursuant to the mandatory mandate of the constitution that no warrant shall issue unless the issuing magistrate shall have himself been personally convinced of such probable cause. 2. Movants be given ample opportunity to file their motion for preliminary investigation as a matter of right; - In another manifestation, the Lims reiterated that the court conduct a hearing to determine if there really exists a prima facie case against them in the light of documents which are recantations of some witnesses in the preliminary investigation. The respondent court issued an order denying for lack of merit.

a2010

page 72

Prof.
- What the Constitution underscores is the exclusive and personal responsibility of the issuing judge to satisfy himself of the existence of probable cause. In doing so, the judge is not required to personally examine the complainant and his witnesses. Following established doctrine and procedures, he shall: (1) personally evaluate the report and the supporting documents submitted by the fiscal regarding the existence of probable cause and, on the basis thereof, issue a warrant of arrest; or (2) if on the basis thereof he finds no probable cause, he may disregard the fiscal's report and require the submission of supporting affidavits of witnesses to aid him in arriving at a conclusion as to the existence of probable cause. - Sound policy dictates this procedure, otherwise judges would be unduly laden with the preliminary examinations and investigation of criminal complaints instead of concentrating on hearing and deciding cases filed before their courts. - The determination of probable cause for the warrant of arrest is made by the Judge. The preliminary investigation proper - whether or not there is reasonable ground to believe that the accused is guilty of the offense charged and, therefore, whether or not he should be subjected to the expense, rigors and embarrassment of trial - is the function of the Prosecutor. - The power to make a preliminary examination for the purpose of determining whether probable cause exists to justify the issuance of a warrant of arrest (or search warrant) has been and remains vested in every judge by the provisions in the Bill of Rights in the 1935, the 1973 and the present [1987] Constitutions securing the people against unreasonable searches and seizures, thereby placing it beyond the competence of mere Court Rule or Statute to revoke. - The problem lies with warrants of arrest especially in metropolitan or highly urban areas. If a Judge has to personally question each complainant and witness or go over the records of the Prosecutor's investigation page by page and word for word before he acts on each of a big pile of applications for arrest warrants on his desk, he or she may have no more time for his or her more important judicial functions. Dispositive Petition is granted.

ISSUE WON a judge may issue a warrant of arrest without bail by simply relying on the prosecution's certification and recommendation that a probable cause exists. HELD NO Ratio A Judge is not precluded from relying on the evidence earlier gathered by responsible officers. The extent of the reliance depends on the circumstances of each case and is subject to the Judge's sound discretion. However, the Judge abuses that discretion when having no evidence before him, he issues a warrant of arrest. Reasoning - Respondent Judge committed a grave error when he relied solely on the Prosecutor's certification and issued the questioned Order dated July 5, 1990 without having before him any other basis for his personal determination of the existence of a probable cause. If a Judge relies solely on the certification of the Prosecutor as in this case where all the records of the investigation are in Masbate, he or she has not personally determined probable cause. The determination is made by the Provincial Prosecutor. The constitutional requirement has not been satisfied. The Judge commits a grave abuse of discretion. - The records of the preliminary investigation conducted by the Municipal Court of Masbate and reviewed by the respondent Fiscal were still in Masbate when the respondent Fiscal issued the warrants of arrest against the petitioners. There was no basis for the respondent Judge to make his own personal determination regarding the existence of a probable cause for the issuance of a warrant of arrest as mandated by the Constitution. He could not possibly have known what transpired in Masbate as he had nothing but a certification - Art. III, Sec. 2. (Constitution) The right of the people to be secure in their persons, houses, papers and effects against unreasonable searches and seizures of whatever nature and for any purpose shall be inviolable, and no search warrant or warrant of arrest shall issue except upon probable cause to be determined personally by the judge after examination under oath or affirmation of the complainant and the witnesses he may produce, and particularly describing the place to be searched and the persons or things to be seized.

STONEHILL v DIOKNO 20 SCRA 383 CONCEPCION; June 19, 1967

Criminal Procedure Rowena Daroy Morales


NATURE Original action in the SC. Certiorari, prohibition, mandamus, injunction. -Petitioners: Harry S. Stonehill, Robert P. Brooks, John J. Brooks and Karl Beck; accused in certain deportation cases -Respondents-prosecutors: DOJ Sec Jose W. Diokno, NBI Acting Director Jose Lukban, Special Prosecutors Pedro D. Cenzon, Efren I. Plana and Manuel Villareal, Jr. and Manila City Asst. Fiscal Maneses G. Reyes -Repondents-judges: Judge Amado Roan of the Municipal (now City) Court of Manila, Judge Roman Cansino of the Municipal (now City) Court of Manila, Judge Hermogenes Caluag of CFI Rizal Quezon City Branch, Judge Eulogio Mencias of CFI Rizal, Pasig Branch, and Judge Damian Jimenez of the Municipal (now City) Court of Quezon City. FACTS -Upon application of the respondents-prosecutors, respondents-judges issued a total of 42 search warrants against petitioners and/or the corporations of which they were officers, directed to the any peace officer, to search the persons above-named and/or the premises of their offices, warehouses and/or residences, and to seize and take possession of books of accounts, financial records, vouchers, correspondence, receipts, ledgers, journals, portfolios, credit journals, typewriters, and other documents and/or papers showing all business transactions including disbursements receipts, balance sheets and profit and loss statements and Bobbins (cigarette wrappers) as "the subject of the offense; stolen or embezzled and proceeds or fruits of the offense," or "used or intended to be used as the means of committing the offense" of "violation of Central Bank Laws, Tariff and Customs Laws, Internal Revenue (Code) and the Revised Penal Code." Petitioners claim that the aforementioned search warrants are null and void, as contravening the Constitution and the Rules of Court; that the searches and seizures made in pursuance thereof are illegal; and that evidences obtained therein are consequently inadmissible. Respondents/prosecutors comments (1) that the contested search warrants are valid and have been issued in accordance with law; (2) that the defects of said warrants, if any, were cured by petitioners' consent; and (3) that, in any event, the effects seized are admissible in evidence against herein petitioners, regardless of the alleged illegality of the aforementioned searches and seizures.

a2010

page 73

Prof.
specific offense had been alleged in said applications. It was impossible for the judges who issued the warrants to have found the existence of probable cause, which presupposes the introduction of competent proof that the party against whom it is sought has performed particular acts, or committed specific omissions in violation of a given penal provision. -General search warrants are outlawed because they place the sanctity of the domicile and the privacy of communication and correspondence at the mercy of the whims, caprice or passion of peace officers. -To prevent the issuance of general search warrants, SC amended Sec. 3 of Rule 122 of the former Rules of Court by providing in the Revised Rules of Court that "no search warrant shall issue for more than one specific offense." -Search warrants authorizing the seizure of books of accounts and records pertaining to all business transactions of petitioners herein, regardless of whether the transactions were legal or illegal contravene the explicit command of the Bill of Rights that the things to be seized should be particularly described and defeat its major objective of eliminating general warrants. -SC resolved to adopt the doctrine in Mapp v Ohio (1961) and to finally abandon the 1948 ruling in Moncado vs. People's Court, 80 Phil. 1.
(*The latter case (citing Wigmore) held that illegally seized evidence is admissible, as long as it is relevant, but without prejudice to the criminal liability of the peace officers who made the seizure, for violation of domicile or under any other provision of the Penal Code. Justices Perfecto, Bengzon, Briones & Paras dissented from the majority opinion. ~marge~*)

Procedure: -March 22, 1962: SC issued the writ of preliminary injunction prayed for in the petition. -June 29, 1962: the writ was partially lifted or dissolved, insofar as the papers, documents and things seized from the offices of the corporations above mentioned are concerned; but, the injunction was maintained as regards the papers, documents and things found and seized in the residences of petitioners. NOTE: The ponencia splits the documents, papers, and things seized under the alleged authority of the warrants in question into two (2) major groups: (a) those found and seized in the offices of the aforementioned corporations, and (b) those found and seized in the residences of petitioners. ISSUES 1. As regards the first group, WON petitioners have a cause of action 2. As regards the second group, WON the search warrants in question, and the searches and seizures made under the authority thereof, are valid (and, WON said documents, papers and things may be used in evidence against petitioners) HELD 1. NONE. -The legality of a seizure can be contested only by the party whose rights have been impaired thereby, and that the objection to an unlawful search and seizure is purely personal and cannot be availed of by third parties. -Petitioners may not validly object to the use in evidence against them of the documents, papers and things seized from the offices and premises of the corporations, since the right to object to the admission of said papers in evidence belongs exclusively to the corporations, to whom the seized effects belong, and may not be invoked by the corporate officers in proceedings against them in their individual capacity. 2. NO. -1935 Constitution (Art. III, Sec. 1, par. 3) provides (a) that no warrant shall issue but upon probable cause, to be determined by the judge in the manner set forth in said provision; and (b) that the warrant shall particularly describe the things to be seized. -Search warrants, issued upon applications stating that the natural and juridical person therein named had committed offenses as abstract as "violation of Central Bank Laws, Tariff and Customs Laws, Internal Revenue (Code) and Revised Penal Code" do not satisfy the constitutional requirements because no

-The exclusionary rule is the only practical means of enforcing the constitutional injunction against unreasonable searches and seizures. -The non-exclusionary rule is contrary, not only to the letter, but also, to the spirit of the constitutional injunction against unreasonable searches and seizures. If there is competent evidence to establish probable cause of the commission of a given crime by the party against whom the warrant is intended, then there is no reason why the applicant should not comply with the requirements of the fundamental law. If he has no such evidence, then it is not possible for the Judge to find that there is probable cause, and, hence, no justification for the issuance of the warrant. The only possible explanation (not justification) for its issuance is the necessity of fishing evidence of the commission of a crime. But, then, this fishing expedition is indicative of the absence of evidence to establish a probable cause.

Criminal Procedure Rowena Daroy Morales


Foreign references cited to support this contention 1. Judge Learned Hand: Only in case the prosecution which itself controls the seizing officials, knows that it cannot profit by their wrong will that wrong be repressed. 2. Weeks v US (1914): The efforts of the courts and their officials to bring the guilty to punishment, praiseworthy as they are, are not to be aided by the sacrifice of those great principles established by years of endeavor and suffering which have resulted in their embodiment in the fundamental law of the land. 3. Mapp v Ohio (1961): all evidence obtained by searches and seizures in violation of the Constitution is, by that same authority, inadmissible in a State court. *Without that rule the freedom from state invasions of privacy would be so ephemeral and so neatly severed from its conceptual nexus with the freedom from all brutish means of coercing evidence as not to permit this Court's high regard as a freedom implicit in the concept of ordered liberty. *The exclusion of the evidence which an accused had been forced to give by reason of the unlawful seizure is the most important constitutional privilege. *The purpose of the exclusionary rule to "is to deter -- to compel respect for the constitutional guaranty in the only effectively available way -- by removing the incentive to disregard it." *We can no longer permit that right to remain an empty promise, to be revocable at the whim of any police officer who, in the name of law enforcement itself, chooses to suspend its enjoyment. Our decision, founded on reason and truth, gives to the individual no more than that which the Constitution guarantees him to the police officer no less than that to which honest law enforcement is entitled, and, to the courts, that judicial integrity so necessary in the true administration of justice. Obiter -In their MFR, petitioners further alleged possession of and control over the records, papers and effects found in the offices of the corporation, and the alleged "personal" nature thereof. -SC disposed of them by saying that this new theory was advanced, not in their petition or amended petition, but in the MR. At any rate, it is best to leave the matter open for determination in appropriate cases in the future. Dispositive Writs granted in part and denied in part; MR denied.

a2010

page 74

Prof.
(b) ownership and/or control or possession actual or constructive -- of premises searched gives "standing"; and (c) the "aggrieved person" doctrine where the search warrant and the sworn application for search warrant are "primarily" directed solely and exclusively against the "aggrieved person," gives "standing." -An examination of the search warrants in this case will readily show that, excepting three, all were directed against the petitioners personally. In some of them, the petitioners were named personally, followed by the designation, "the President and/or General Manager" of the particular corporation. The three warrants excepted named three corporate defendants. But the "office/house/warehouse/premises" mentioned in the said three warrants were also the same as those declared to be owned by or under the control of the petitioners in all the other search warrants. -Thus, the petitioners have full standing to move for the quashing of all the warrants regardless whether these were directed against residences in the narrow sense of the word, as long as the documents were personal papers of the petitioners or (to the extent that they were corporate papers) were held by them in a personal capacity or under their personal control. -SC, at all events, should order the return to the petitioners all personal and private papers and effects seized, no matter where these were seized, whether from their residences or corporate offices or any other place or places. The uncontradicted sworn statements of the petitioners in their, various pleadings submitted to this Court indisputably show that amongst the things seized from the corporate offices and other places were personal and private papers and effects belonging to the petitioners. -If there should be any categorization of the documents, papers and things which where the objects of the unlawful searches and seizures, I submit that the grouping should be: (a) personal or private papers of the petitioners, and (b) purely corporate papers belonging to corporations.

CASTRO [concurring and dissenting]


-Reasoning that the petitioners have not in their pleadings satisfactorily demonstrated that they have legal standing to move for the suppression of the documents, papers and effects seized in the places other than the three residences adverted to above, the opinion written by the Chief Justice refrains from expressly declaring as null and void the such warrants served at such other places and as illegal the searches and seizures made therein, and leaves "the matter open for determination in appropriate cases in the future." -It is with this position that Justice Castro is not in accord. -He says that All the search warrants, without exception, in this case are admittedly general, blanket and roving warrants and are therefore admittedly and indisputably outlawed by the Constitution; and the searches and seizures made were therefore unlawful. -He argues that assuming that the petitioners have no legal standing to ask for the suppression of the papers, things and effects seized from places other than their residences, this cannot in any manner affect, alter or otherwise modify the intrinsic nullity of the search warrants and the intrinsic illegality of the searches and seizures made thereunder. Whether or not the petitioners possess legal standing the said warrants are void and remain void, and the searches and seizures were illegal and remain illegal. -He insists that, upon the pleadings submitted to SC, the petitioners have the requisite legal standing to move for the suppression and return of the documents, papers and effects that were seized from places other than their family residences. -Since our constitutional provision on searches and seizures was derived almost verbatim from the Fourth Amendment to the United States Constitution, in the many years of judicial construction and interpretation of the said constitutional provision, our courts have invariably regarded as doctrinal the pronouncement made on the Fourth Amendment by federal courts, especially the Federal Supreme Court and the Federal Circuit Courts of Appeals. -The U.S. doctrines and pertinent cases on standing to move for the suppression or return of documents, papers and effects which are the fruits of an unlawful search and seizure, may be summarized as follows: (a) ownership of documents, papers and effects gives "standing;"

LUNA v PLAZA 26 SCRA 310 ZALDIVAR; November 29, 1968


FACTS - A criminal action was commenced by T-Sgt. Candido Patosa, PC investigator against Simon Luna, by filing with respondent Municipal Judge Lorenzo M.

SEPARATE OPINION

Criminal Procedure Rowena Daroy Morales


Plaza, of the Municipal Court of Tandag, charging the petitioner, with the crime of murder.
- Supporting the complaint were sworn statements of the witnesses for the prosecution, in the form of questions and answers taken by T-Sgt. Patosa, and subscribed and sworn to before the respondent Judge at the time of the filing of the complaint.

a2010

page 75

Prof.
court, under the rules and precedents, must be deemed final and binding upon this Court. 2. YES. - As provided in Republic Act No. 3828 Before a municipal judge may issue a warrant of arrest, the following conditions must first be fulfilled: (1) he must examine the witnesses personally; (2) the examination must be under oath; (3) the examination must be reduced to writing in the form of searching questions and answers. - The first condition was fulfilled. The trial court found as a fact that "the respondent judge personally examined the witnesses for the prosecution; that respondent judge adopted as his own personal examination the questions asked by T-Sgt. Patosa as appearing in the written statements, which he read over again to the witnesses together with the answers given therein, asking the witnesses whether said answers were theirs, and whether the same answers were true, to which the witnesses answered in the affirmative. Republic Act No. 3828 does not prohibit the municipal Judge from adopting the questions asked by a previous investigator. - The second condition was also fulfilled. The trial court found that the complaint was "supported by statements of the witnesses under oath." The record also shows there were documents to have been subscribed and sworn to before respondent Judge. - The third condition was likewise fulfilled. The examination of the witnesses was written down, in the form of searching questions and answers. The term searching questions and answers" means only, taking into consideration the purpose of the preliminary examination which is to determine "whether there is a reasonable ground to believe that an offense has been committed and the accused is probably guilty thereof so that a warrant of arrest may be issued and the accused held for trial," such questions as have tendency to show the commission of a crime and the perpetrator thereof. What would be searching questions would depend on what is sought to be inquired into, such as: the nature of the offense, the date, time, and place of its commission, the possible motives for its commission; the subject, his age, education, status, financial and social circumstances, his attitude toward the investigation, social attitudes, opportunities to commit the offense; the victim, his age, status, family responsibilities, financial and social circumstances, characteristics, etc. The points that are the subject of inquiry may differ from case to case. - The questions, therefore, must to a great degree depend upon the Judge making the investigation. At

- The respondent Judge examined the prosecution witnesses by reading to them "all over again the questions and answers" in their statements in writing, and the witnesses-affiants declared before said Judge that the questions were propounded by TSgt. Candido Patosa, and that the answers were made by them. - The affiants signed their respective affidavits in the presence of the respondent Judge, who also signed after the usual procedure of administering the oath. - Considering the answers of the affiants to the, questions contained in their sworn statements, together with the postmortem and autopsy report on the dead body of the victim Jaime Diaz Ng, the certificate of death, the sketch showing the position of the victim and the accused, the respondent Judge opine that there was reasonable ground to believe that the crime of murder had been committed and the amused was probably guilty thereof. - Respondent Judge issued the order and warrant of arrest, specifying therein that no bail should be accepted for the provisional release of the accused. - Upon motion of petitioner upon the ground that the evidence of guilt was not strong, respondent Judge issued an order, granting bail,; which order, however, respondent Judge later revoked, and petitioner was denied bail. - The case was subsequently remanded to the CFI of Surigao del Sur, after petitioner filed a waiver of his right to preliminary investigation. - Respondent Provincial Fiscal filed an information charging petitioner with the crime of murder. The petitioner was detained in the provincial jail. - Petitioner filed a petition for a writ of habeas corpus with the CFI of Surigao del Sur, claiming that he was being deprived of liberty without due process of law, on the ground that the imprisonment and detention was the result of a warrant of arrest issued by respondent Judge in violation of Republic Act No. 3828, and praying for the annulment of the order for his arrest and his discharge from confinement. - Respondents filed their answer, alleging that Republic Act Nor. 3828 had been substantially complied with; that a motion to quash, and not a petition for habeas corpus was the proper remedy, and that petitioner's application for bail constituted a

waiver of the right to question the validity of the arrest. - The CFI of Surigao del Sur ruled that respondent Municipal Judge had substantially complied with Republic Act No. 3828, and consequently denied the application for the writ of habeas corpus, and dismissed the case. - Hence the appeal. Petitioners Claim Republic Act No. 3828 imposes on a municipal judge, before he can issue a warrant of arrest, two specific duties, to wit: (1) personally examine the complainant and witnesses with "searching questions and answers," which means that the judge must cross-examine them in case their affidavits are presented; and (2) said examination must be reduced to writing and form part of the records of the case. The record of the instant case, does not show that said examination was performed by respondent Judge notwithstanding his testimony to the effect that he adopted the questions propounded to each of the prosecution witnesses by T-Sgt. Patosa. And assuming that the adoption of the questions made by T-Sgt. Patosa constituted substantial compliance with the requirement that the judge should examine the witnesses by asking searching questions, still the second requirement, that of reducing to writing the said procedure of adoption, has not been compiled with; and so, Republic Act No. 3828 was still violated, and the issuance of the warrant of arrest was in violation of said Act and the Constitution and constituted denial of due process. ISSUES 1. WON the trial court erred in giving absolute credence to the testimony of respondent Municipal Judge. 2. WON the requirements of Republic Act No. 3828 was satisfied. 3. WON the issuance of the warrant of arrest was a violation of the Constitution and of procedural due process. 4. WON the trial court erred in denying the writ of habeas corpus. HELD 1. NO - As a general rule, the lower court's findings, as to the credibility of witnesses will not be interfered with by appellate courts. Since petitioner appealed directly to this Court he must, raise only questions of law and he has thereby waived the right to raise any question of fact, and the findings of facts of the trial

Criminal Procedure Rowena Daroy Morales


any rate, the court a quo found that respondent Judge was "satisfied that the questions and answers contained in the sworn statements taken by T-Sgt. Patosa partake of the nature of his searching questions and answers as required by law," so the respondent Judge adopted them. 3. NO - The Constitution, in Section 1 (3), Article III, provides that no warrant shall issue but upon probable cause, to be determined by the judge after examination under oath or affirmation of the complainant and the witnesses he may produce. - The constitutional requirement of examination of witnesses under oath was, as shown above, fulfilled. The existence of probable cause depended to a large degree upon the finding or opinion of the judge conducting the examination. Respondent Judge found that there was a probable cause, as stated in his order of arrest. - Preliminary examination is not an essential part of due process of law. Preliminary examination may be conducted by the municipal judge, prior to the issuance of the warrant of arrest, either in the presence, or in the absence, of the accused. - The record shows that herein petitioner waived the preliminary investigation before respondent Municipal Judge, and instead, he riled a petition for bail. This conduct of petitioner indicates that he had waived his objection to whatever defect, if any, in the preliminary examination conducted by respondent Judge prior to the issuance of the warrant of arrest. 4. NO - Section 4 of Rule 102 of the Rules of Court provides in part, as follows: "Sec. 4 When writ not allowed or discharge authorized. If it appears that the person alleged to be restrained of his liberty is in the custody of an officer under process issued by a court or judge ... and that the court or judge had jurisdiction to issue the process ... or make the order, the writ shall not be allowed ... " - All the conditions, in the afore-quoted Section 4, set forth to deny the writ, are present in the instant case. - Petitioner is detained and is in the custody of the respondent Provincial Warden by virtue of the order of arrest and the order of respondent Judge, to confine petitioner in the provincial jail. It is not disputed by petitioner that respondent Judge had jurisdiction to issue the warrant of arrest and the order of commitment under the provisions of Section 47, Republic Act No. 409, as amended by Republic Act No. 1201, although petitioner did question the

a2010

page 76

Prof.
- September 29: While this case was pending, the Provincial Fiscal filed an information against Rosario, private respondent, for double physical injuries; double less serious physical injuries; and damage to property thru reckless imprudence, in CFI Urdaneta. - Rosario was prosecuted and convicted by the trial court in the criminal case. CA acquitted him from the crime charged on the ground that his guilt has not been proved beyond reasonable doubt. - April 3, 1972: Respondents filed a "Request for Admission" in the civil case, requesting petitioners to admit the truthfulness of the facts set forth as well as the correctness and genuineness of the documents attached. - May 5,1972: Petitioners filled a "Manifestation", admitting the allegations in the "Request for Admission" with some qualifications. Later, both parties submitted their respective memoranda. - December 28, 1972: On the basis of the testimonies and evidence submitted by the petitioners, as well as the records of the criminal case attached in the "Request for Admission" of the private respondents, CFI Manila rendered a decision, dismissing the complaint of the petitioners against private respondents as well as the counterclaim of private respondents against the petitioners. - February 13, 1978: CA affirmed Petitioners Version Bernabe Castillo was driving his jeep on the northbound lane of the McArthur Highway with his wife, father, and child at the rate of 25 kph. Just past San Nicolas bridge, he noticed, from a distance of 120 meters more or less, a speeding oncoming car along the same lane he was driving, overtaking a cargo truck ahead of it. - He switched on his headlights to signal the car to return to its own right lane as the way was not clear for it to overtake the truck. The signal was disregarded, as the car proceeded on its direction. - To evade the collision, he swerved his jeep to the right towards the shoulder and applied on the brakes, leaving his feet on it, even, immediately after the impact. The car rested on the shoulder of the right lane. The jeep's rear left wheel was on the road, leaving short tire marks behind it; while the car left long tire marks, specially its left rear wheel. Respodents Version Juanito Rosario who was driving the car, with his wife and daughter, were along MacArthur Highway going southwards. They saw ahead of them a big heavily loaded cargo truck. The truck was moving very slowly because of its heavy load so that Rosario decided to overtake it. But before doing so, he first saw to it that the road was clear and as additional precautionary measure,

validity of the warrant of arrest for allegedly having been issued in violation of Republic Act No. 3828 which was found to be untenable. - The remedy available to the petitioner herein, under the circumstances stated in this opinion, is not a petition for a writ of habeas corpus but a petition to quash the warrant of arrest or a petition for a reinvestigation of the case by the respondent Municipal Judge or by the Provincial Fiscal. - The Court stressed that what has been stated in the opinion was not intended to sanction the return to the former practice of municipal judges of simply relying upon affidavits or sworn statements that are made to accompany the complaints that are filed before them, in determining whether there is a probable cause for the issuance of a warrant of arrest. - That practice is precisely what is sought to be voided by the amendment of Section 87 (c) of Republic Act 296 (Judiciary Act of 1948) which requires that before a municipal judge issues a warrant of arrest he should first satisfy himself that there is a probable cause by examining the witnesses personally, and that the examination must be under oath and reduced to writing in the form of searching questions and answers. - It is obvious that the purpose of this amendment is to prevent the issuance of a warrant of arrest against a person based simply upon affidavits of witnesses who made, and swore to, their statements before a person or persons other than the judge before whom the criminal complaint is filed. Dispositive The decision of the trial court appealed from, was affirmed. Costs against petitionerappellant.

CASTILLO v CA (ROSARIO) 176 SCRA 591 FERNAN; August 21, 1989


NATURE Petition for review on certiorari FACTS - May 2, 1965: Parties figured in a vehicular accident which caused injuries to their persons and damage to their respective vehicles. They had conflicting versions of the accident. - June 30: Petitioners instituted a civil case for the recovery of damages for the injuries sustained and for the damage to the vehicle in CFI Manila.

Criminal Procedure Rowena Daroy Morales


he blew his horn several times at the time he was overtaking the truck. - As the car was about to overtake the slow moving cargo truck, the car's front left tire suddenly burst due to pressure causing the car to swerve to the left and naturally making steering and control difficult. - Because of the tendency of the car to veer towards the left due to the blown out tire, the driver steered the car towards the direction where he could find a safe place to park and fix the tire. He finally brought the car to a halt at the left shoulder of the road. - Just as he was about to get off to fix the flat tire, the car was suddenly bumped by the jeep which came from the opposite direction ISSUE WON petitioners were deprived of due process because their civil action was decided on the basis of private respondent Juanita Rosario's acquittal in the criminal case for reckless imprudence HELD NO Ratio Findings of fact of the Court of Appeals are conclusive on the parties and on the Supreme Court, unless (1) the conclusion is a finding grounded entirely on speculations, surmises and conjectures; (2) the inference made is manifestly mistaken; (3) there is grave abuse of discretion; (4) the judgment is based on misapprehension of facts; (5) the Court of Appeals went beyond the issues of the case and its findings are contrary to the admission of both appellant and appellee; (6) the findings of facts of the Court of Appeals are contrary to those of the trial court; (7) said findings of facts are conclusions without citation of specific evidence on which they are based; (8) the facts set forth in the petition as well as in the petitioner's main and reply briefs are not disputed by the respondent; and (9) when the finding of facts of the Court of Appeals is premised on the absence of evidence and is contradicted by evidence on record. Reasoning - The subject action for damages, being civil in nature, is separate and distinct from the criminal aspect, necessitating only a preponderance of evidence. - A quasi-delict or culpa aquiliana is a separate legal institution under the Civil Code, with a substantively all its own, and individuality that is entirely apart and independent from a delict or crime. A distinction exists between the civil liability arising from a crime and the responsibility for quasi-delicts or culpa extra-

a2010

page 77

Prof.
counsel had the opportunity to cross-examine the witnesses. Dispositive Petition denied

contractual. The same negligence causing damages may produce civil liability arising from a crime under the Penal Code, or create an action for quasidelictos or culpa extra-contractual under the Civil Code. Therefore, the acquittal or conviction in the criminal case is entirely irrelevant in the civil case. - But this rule is not without exception. Thus, Section 2 (c) of Rule 111 of the Rules of Court provides: Extinction of the penal action does not carry with it extinction of the civil, unless the extinction proceeds from a declaration from a final judgment that the fact from which the civil action might arise did not exist. - In a previous case, CA-G.R. No. 07684-CR, People v. Rosario, the CA after a painstaking analysis of. (a) the testimonial evidence; (b) the relative positions of the two vehicles as depicted in the sketches; (c) the distance of each of the two vehicles from the cemented edge of the road; (d) the point of impact; (e) the visible tire marks, and (f) the extent of the damage caused upon each of the two vehicles, ruled that it was the driver of the jeep and not the accused driver of the car who was negligent and accordingly acquitted the latter. - Negligence, being the source and foundation of actions of quasi-delict, is the basis for the recovery of damages. In the case at bar, the CA found that collision was not due to the negligence of Rosario but rather it was Castillo's own act of driving the jeep to the shoulder of the road where the car was, which was actually the proximate cause of the collision. With this finding, the CA exonerated Rosario from civil liability on the ground that the alleged negligence did not exist. - During the trial of the case before the CFI, respondents were not present because they were abroad. Their counsel introduced as part of their evidence, the records in the criminal case, in accordance with Section 41, Rule 130 of the Rules of Court. These records, mostly composed of transcripts of the hearing in the criminal case, were attached to their "Request for Admission" and were substantially admitted by petitioners. Petitioners raised, as one of their objections, the propriety and correctness of admitting and adopting these transcripts as part of the record in the civil case. According to them, this is a violation of Section 41, Rule 130, on the ground that petitioners were not given the opportunity to cross-examine. We disagree. A careful reading of the transcripts would reveal that counsel for petitioners actively participated during the proceedings of the criminal case. He raised various objections, in the course of the trial. Petitioners, therefore, thru

CALLANTA v VILLANUEVA 77 SCRA 377 FERNANDO; June 20, 1977


NATURE Original petitions in the Supreme Court, certiorari with preliminary injunction FACTS - Judge Villanueva of Dagupan refused to grant the motions to quash two complaints for oral defamation against Callanta. - Callantas counsel argued that there was an issue with regard to the validity of Villanuevas issuance of the warrants of arrest on the ground that it should have been the City Fiscal who conducted the preliminary investigation. - After the warrants were issued (with bail pegged at P600), Callanta posted the required bail bonds and was granted her provisional liberty. - The City Fiscal had manifested his intent to prosecute the case. - February 25, 1965 After the Court had conducted preliminary investigation and had acquired jurisdiction over the case, the Court referred the case to the Fiscal. - March 4, 1965 The arraignment was postponed because the Fiscal was still doing his investigation. - In the proceedings of April 20, 1965, the Fiscal entered his appearance for the government and manifested that he was ready for trial. ISSUE WON Callanta can contest the validity of his arrest HELD NO Ratio Posting of a bail bond constitutes waiver of any irregularity attending the arrest of a person and estops him from discussing the validity of his arrest. Reasoning - In the case of Luna vs. Plaza, the Court held that where petitioner has filed an application for bail and waived the preliminary investigation proper, he waived his objection to whatever defect, if any, in the preliminary examination conducted, prior to the issuance of a warrant of arrest.

Criminal Procedure Rowena Daroy Morales


- This doctrine has been upheld in a number of cases including People vs. Olandar, Zacarias vs. Cruz, Bermejo vs. Barrios, People vs. La Caste, Manzano vs Villa and People vs. Obngayan which stated that where the accused has filed bail and waived the preliminary investigation proper, he has waived whatever defect, if any, in the preliminary examination conducted prior to the issuance of the warrant of arrest. - The city fiscal had been quite active in the investigation and in the prosecution of the accused. It was he who manifested his readiness to appear in the trial. Obiter - With regard to the issue of whether or not the only person vested with authority to conduct a preliminary investigation is the city fiscal, the Charter of the City of Dagupan provides that the City Court of Dagupan City may also conduct preliminary investigation for for any offense, without regard to the limits of punishment and may release or commit any person charged with such offense to secure his appearance before the proper court. Dispositive WHEREFORE, these petitions for certiorari are dismissed. The restraining order issued by this Court is lifted and set aside. Costs against petitioner.

a2010

page 78

Prof.
that the books (etc) relating to the activities of . . . as usurer, are being kept and concealed in the house. . . all of which is contrary to the statute of law." ISSUE WON the search warrant and the seizure were illegal HELD YES -Reason 1: it appears that the affidavits, which served as the exclusive basis of the search warrants, are insufficient and fatally defective by reason of the manner in which the oaths were made and, therefore, it is hereby held that the search warrants in question and the subsequent seizure of the documents and papers are illegal and do not in any way warrant the deprivation to which the petitioners were subjected. -The oath required must refer to the truth of the facts within the personal knowledge of the petitioner or his witnesses, because the purpose thereof is to convince the committing magistrate, not the individual making the affidavit and seeking the issuance of the warrant, of the existence of probable cause. The true test of sufficiency of an affidavit to warrant issuance of a search warrant is whether it has been drawn in such a manner that perjury could be charged thereon and affiant be held liable for damages caused. -Sec 1, par 3, of Art III, Constitution: "The right of the people to be secure in their persons, houses, papers, and effects against unreasonable searches and seizures shall not be violated, and no warrants shall issue but upon probable cause, to be determined by the judge after examination under oath or affirmation of the complainant and the witnesses he may produce, and particularly describing the place to be searched, and the persons or things to be seized." Sec 97 of General Orders No. 58: "A search warrant shall not issue except for probable cause and upon application supported by oath particularly describing the place to be searched and the person or thing to be seized." - Both provisions require that there be not only probable cause before the issuance of a search warrant but that the search warrant must be based upon an application supported by oath of the applicant and the witnesses he may produce. In its broadest sense, an oath includes any form of attestation by which a party signifies that he is bound in conscience to perform an act faithfully and truthfully.

SEPARATE OPINION AQUINO [concurring]


- Sec. 77 of the Dagupan City charter expressly empowers its city court (formerly municipal court) to conduct preliminary investigation for any offense, without regard to the limits of punishment. - Every justice of the peace, municipal judge (meaning city judge), city or provincial fiscal, shall have authority to conduct preliminary examination or investigation in accordance with these rules of all offenses alleged to have been committed within his municipality, city or province, cognizable by the Court of First Instance (Sec. 87 of the Judiciary Law and Sec. 2, Rule 112).

RODRIGUEZ v VILLAMIEL 65 Phil 230 IMPERIAL; DEC 23, 1937


FACTS -Victor Villamiel, special agent for the Anti-Usury Board, made two affidavits for the purpose of obtaining search warrants against Rodriguez and

Evangelista. The text of both affidavits reads as follows: "Victor D. Villamiel having taken the oath prescribed by law, appears and states: that he has and there is just and probable cause to believe and he does believe that the books, lists, chits, receipts, documents, and other papers relating to the activities of Juan Evangelista, as usurer, are being kept and concealed in the house of said Juan Evangelista situated at Lucena, Tayabas, all of which is contrary to the statute of law." -The justice of the peace of the provincial capital issued the two search warrants against the petitioners (see original for the wording of the warrant) -Villamiel, with other agents and a constabulary soldier, executed the warrants, went to the residences of the petitioners, searched them and seized documents and papers belonging to petitioners. Villamiel issued a receipt to each of the petitioners, without specifying the documents and papers seized by him, which were taken to his office in Manila, keeping them there until he was ordered by the CFI to deposit them in the office of the clerk of court. -Petitioners filed a petition praying that the search warrants be declared null and void and illegal; that Villamiel be punished for contempt of court for having conducted the searches and for having seized the documents and papers without issuing detailed receipts and for not having turned them over to the court, and that said documents and papers be ordered returned to the petitioners. -the CFI found Villamiel guilty of contempt of court and fined him P10. The court declared the search warrants and the seizure of the documents and papers VALID, authorizing the agents of the AntiUsury Board to examine them and retain those that are necessary and material to whatever criminal action they may wish to bring against the petitioners. -Petitioners appealed. They contend that the search warrants issued by the court are illegal because they have been based on the affidavits of special agent Villamiel wherein he affirmed and stated that he had no personal knowledge of the facts that were to serve as basis for the issuance of the search warrants, but merely confined himself to asserting that he believed and there was probable cause to believe that the documents and papers were related to the activities of the petitioners as usurers. As has been seen, the special agent's affirmation in this respect consisted merely in the following: "that he has and there is just and probable cause to believe and he does believe

Criminal Procedure Rowena Daroy Morales


-Reason 2: At the hearing of the case, it was shown that the documents and papers had really been seized to enable the Anti-Usury Board to conduct an investigation and later use all or some of them as evidence against the petitioners in the criminal cases that may be brought against them. The seizure of books and documents by means of a search warrant, for the purpose of using them as evidence in a criminal case against the person in whose possession they were found, is unconstitutional because it makes the warrant unreasonable, and it is equivalent to a violation of the constitutional provision prohibiting the compulsion of an accused to testify against himself Therefore, it appearing that the documents and papers were seized for the purpose of fishing for evidence to be used against the petitioners in the criminal proceedings for violation of the Anti-Usury Law which might be instituted against them, this court holds that the search warrants issued are illegal and that the documents and papers should be returned to them. - Definition and rationale of search warrant: A search warrant is an order in writing, issued in the name of the People of the Philippine Islands, signed by a judge or a justice of the peace, and directed to a peace officer, commanding him to search for personal property and bring it before the court. Of all the rights of a citizen, few are of greater importance or more essential to his peace and happiness than the right of personal security, and that involves the exemption of his private affairs, books, and papers from the inspection and scrutiny of others. While the power to search and seize is necessary to the public welfare, still it must be exercised and the law enforced without transgressing the constitutional rights of citizens, for the enforcement of no statute is of sufficient importance to justify indifference to the basic principles of government.

a2010

page 79

Prof.
3, Project 6, Quezon City and 784 Units C & D, RMS Building, Quezon Avenue, Quezon City. Objection is interposed to the execution of Search Warrant No. 20-82[b] at the latter address on the ground that the two search warrants pinpointed only one place where petitioner Jose Burgos, Jr. was allegedly keeping and concealing the articles listed therein, i.e., No. 19, Road 3, Project 6, Quezon City. > although the warrants were directed against Jose Burgos, Jr. alone, articles belonging to his copetitioners Jose Burgos, Sr., Bayani Soriano and the J. Burgos Media Services, Inc. were seized. > real properties were seized under the disputed warrants. > that documents relied on by respondents could not have provided sufficient basis for the finding of a probable cause upon which a warrant may validly issue in accordance with Section 3, Article IV of the 1973 Constitution ISSUES WON the two search warrants are: 1. defective for stating only one and the same place to be searched 2. null and void for including properties not owned by the person named in the warrants 3. null and void for including real properties 4. null and void for being violative of the constitution, thus encroaching on petitioners' fundamental rights HELD 1. NO - The defect pointed out is a typographical error. Two search warrants were applied for and issued because the purpose and intent were to search two distinct premises. The addresses of the places sought to be searched were specifically set forth in the application, and since it was Col. Abadilla himself who headed the team which executed the search warrants, the ambiguity that might have arisen by reason of the typographical error is more apparent than real. - In the determination of whether a search warrant describes the premises to be searched with sufficient particularity, it has been held "that the executing officer's prior knowledge as to the place intended in the warrant is relevant. This would seem to be especially true where the executing officer is the affiant on whose affidavit the warrant had issued, and when he knows that the judge who issued the warrant intended the building described in the affidavit. And it has also been said that the executing

BURGOS SR v CHIEF OF STAFF 133 SCRA 800 ESCOLIN; December 26, 1984
NATURE Petition for certiorari, prohibition and mandamus with preliminary mandatory and prohibitory injunction FACTS - December 7, 1982 Judge Ernani Cruz-Pao CFI Rizal [Quezon City], issued two search warrants under

which the premises known as No. 19, Road 3, Project 6, Quezon City, business address of Metropolitan Mail newspaper, and 784 Units C & D, RMS Building, Quezon Avenue, Quezon City, business address of the "We Forum" newspaper were searched. - office and printing machines, equipment, paraphernalia, motor vehicles and other articles used in the printing, publication and distribution of the said newspapers, as well as numerous papers, documents, books and other written literature alleged to be in the possession and control of petitioner Jose Burgos, Jr. publisher-editor of the "We Forum" newspaper, were seized. - The questioned search warrants were issued by respondent judge upon application of Col. Rolando N. Abadilla, Intelligence Officer of the P.C. Metrocom. The application was accompanied by the Joint Affidavit of Alejandro M. Gutierrez and Pedro U. Tango, members of the Metrocom Intelligence and Security Group under Col. Abadilla which conducted a surveillance of the premises prior to the filing of the application for the search warrants on December 7, 1982. - Respondents aver that the case should be dismissed on the ground that petitioners had come to SC without having previously sought the quashal of the search warrants before the issuing judge. But this procedural flaw notwithstanding, SC took cognizance of this petition in view of the seriousness and urgency of the constitutional issues raised, not to mention the public interest generated by the search. - Respondents likewise urge dismissal of the petition on ground of laches, since said search warrants were issued on December 7, 1982, but the instant petition impugning the same was filed only on June 16, 1983. However, SC found that the extrajudicial efforts exerted by petitioners quite evidently negate the presumption that they had abandoned their right to the possession of the seized property, thereby refuting the charge of laches against them. Petitioners' Claims > Petitioners fault respondent judge for his alleged failure to conduct an examination under oath or affirmation of the applicant and his witnesses, as mandated by the constitution as well as Sec. 4, Rule 126 of the Rules of Court. However, SC found that as petitioners themselves conceded during the hearing on August 9, 1983, that an examination had indeed been conducted by respondent judge of Col. Abadilla and his witnesses, this issue is moot and academic. > Search Warrants No. 20-82[a] and No. 20-82[b] were used to search two distinct places: No. 19, Road

Criminal Procedure Rowena Daroy Morales


officer may look to the affidavit in the official court file to resolve an ambiguity in the warrant as to the place to be searched." 2. NO - Section 2, Rule 126 of the Rules of Court, enumerates the personal properties that may be seized under a search warrant. The rule does not require that the property to be seized should be owned by the person against whom the search warrant is directed. It may or may not be owned by him. Ownership, therefore, is of no consequence, and it is sufficient that the person against whom the warrant is directed has control or possession of the property sought to be seized. 3. NO - Under Article 415[5] of the Civil Code , "machinery, receptables, instruments or implements intended by the owner of the tenement for an industry or works which may be carried on in a building or on a piece of land and which tend directly to meet the needs of the said industry or works" are considered immovable property. Petitioners do not claim to be the owners of the land and/or building on which the machineries were placed. This being the case, the machineries in question, while in fact bolted to the ground remain movable property susceptible to seizure under a search warrant. 4. YES - Probable cause for a search is defined as such facts and circumstances which would lead a reasonably discreet and prudent man to believe that an offense has been committed and that the objects sought in connection with the offense are in the place sought to be searched. And when the search warrant applied for is directed against a newspaper publisher or editor in connection with the publication of subversive materials, as in the case at bar, the application and/or its supporting affidavits must contain a specification, stating with particularity the alleged subversive material he has published or is intending to publish. Mere generalization will not suffice. - In mandating that "no warrant shall issue except upon probable cause to be determined by the judge, . . . after examination under oath or affirmation of the complainant and the witnesses he may produce the Constitution requires no less than personal knowledge by the complainant or his witnesses of the facts upon which the issuance of a search warrant may be justified. In Alvarez v. CFI, SC ruled that "the oath required must refer to the truth of the facts within the personal knowledge of the petitioner or his witnesses, because the purpose

a2010

page 80

Prof.
Masamlok to join the NPA. He later attended an NPA seminar where Burgos, the first speaker, said very distinctly that he is an NPA together with his companions, to assure the unity of the civilian. That he encouraged the group to overthrow the government. To prove illegal possession, a person in charge of firearms and explosives of the PC HQ in Davao testified that accused was not among the list of firearm holders - On the other hand, accused-appellants claims that he was taken to the PC barracks and when he denied ownership of the gun, he was beaten, tortured, mauled and subjected to physical agony. He was forced to admit possession or ownership of the gun. 2 witnesses as well as Rubens wife Urbana, were presented by the defense in support of the accuseds denial of the charge against him. Urbana claimed that it was Masamlok who left the firearm there. - The RTC after considering the evidences presented by both prosecution and defense convicted accused Ruben Burgos guilty beyond reasonable doubt of the crime of illegal possession of firearms in furtherance of subversion. The RTC justified the warrantless arrest as falling under one of the circumstances when arrests may be validly made without a warrant, under Rule 113 Sec.6 of the Rules of Court. It stated that even if there was no warrant for the arrest of Burgos, the fact that the authorities received an urgent report of accused's involvement in subversive activities from a reliable source (report of Cesar Masamlok) the circumstances of his arrest, even without judicial warrant, is lawfully within the ambit of Sec. 6(a) of Rule 113 and applicable jurisprudence on the matter. If the arrest is valid, the consequent search and seizure of the firearm and the alleged subversive documents would become an incident to a lawful arrest as provided by Rule 126, Sec. 12. A person charged with an offense may be searched for dangerous weapons or anything which may be used as proof of the commission of the offense. ISSUES 1. WON the arrest was lawful and WON the search of his house and the subsequent confiscation of a firearm and documents conducted in a lawful manner. 2. WON there is enough evidence to prove his guilt beyond reasonable doubt. HELD 1. NO Art.III Sec.2 of the Constitution safeguards against wanton and unreasonable invasion of the privacy

thereof is to convince the committing magistrate, not the individual making the affidavit and seeking the issuance of the warrant, of the existence of probable cause." - the search warrants are in the nature of general warrants. - As a consequence of the search and seizure, the premises were padlocked and sealed, with the further result that the printing and publication of said newspapers were discontinued. Such closure is in the nature of previous restraint or censorship abhorrent to the freedom of the press guaranteed under the fundamental law, and constitutes a virtual denial of petitioners' freedom to express themselves in print. Dispositive Search Warrants Nos. 20-82[a] and 2082[b] issued by respondent judge on December 7, 1982 are null and void. All articles seized thereunder are ordered released to petitioners.

PEOPLE v BURGOS 144 SCRA 1 GUTIERREZ; Sept.4, 1986


NATURE Appeal from RTC decision convicting Ruben Burgos of the crime of Illegal Possession of Firearms in Furtherance of Subversion FACTS - Prosecution version: Upon obtaining information from one Cesar Masamlok, who personally and voluntarily surrendered to the Davao del Sur police HQ stating that accused Ruben Burgos forcibly recruited him to join the NPA with the use of a firearm against his life, a team was dispatched the following day to arrest Burgos. Through the help of Pedro Burgos, the brother of accused, the team was able to locate Ruben Burgos, who was plowing his field at the time. - When asked about the firearm, the accused denied possession of it, but after questioning the accuseds wife, the police were able to locate and retrieve the said firearm, a .38 caliber S & W, buried in the ground below their house. The police, after accused pointed them to the location, were also able to retrieve alleged subversive documents (a notebook and a pamphlet) hidden underground a few meters away from the house. - To prove accuseds subversive activities, Masamlok testified that accused came to his house and told him to join the NPA or his family will be killed along with him. The threat to his life and family forced

Criminal Procedure Rowena Daroy Morales


and liberty of a citizen as to his person, papers and effects. In this case, the arrest was made without warrant and since it does not fall within the exceptions of arrests that can be made without a warrant, it is unlawful and therefore, the fruit of the poisonous tree doctrine applies. Reasoning Under Sec.6 (a) of Rule 113, the officer arresting a person who has just committed, is committing, or is about to commit an offense must have personal knowledge of that fact. The offense must also be committed in his presence or within his view. There is no such personal knowledge in this case. Whatever knowledge was possessed by the arresting officers, it came in its entirety from the information furnished by Cesar Masamlok. The location of the firearm was given by Burgos wife. At the time of arrest, Burgos was not in actual possession of any firearm or subversive document. Neither was he committing any act which could be described as subversive. He was, in fact, plowing his field at the time. - The SolGen believes that the arrest may still be considered lawful under Sec.6(b) using the test of reasonableness. The SolGen submits that the info given by Masamlok was sufficient to induce a reasonable ground that a crime has been committed and that the accused is probably guilty thereof. In arrests without a warrant under Sec.6(b), however, it is not enough that there is reasonable ground to believe that the person to be arrested has committed a crime. A crime must in fact or actually have been committed first. That a crime has actually been committed is an essential precondition. It is not enough to suspect that a crime may have been committed. The fact of the commission of the offense must be undisputed. The test of reasonable ground applies only to the identity of the perpetrator. In this case, the accused was arrested on the sole basis of Masamlok's verbal report. Masamlok led the authorities to suspect that the accused had committed a crime. They were still fishing for evidence of a crime not yet ascertained. The subsequent recovery of the subject firearm on the basis of information from the lips of a frightened wife cannot make the arrest lawful. If an arrest without warrant is unlawful at the moment it is made, generally nothing that happened or is discovered afterwards can make it lawful. The fruit of a poisoned tree is necessarily also tainted. More important, We find no compelling reason for the haste with which the arresting officers sought to arrest the accused. We fail to see why they failed to first go through the process of obtaining a warrant of arrest, if indeed

a2010

page 81

Prof.
surrendered to the military, certainly his fate depended on how eagerly he cooperated with the authorities. Otherwise, he would also be charged with subversion. Masamlok may be considered as an interested witness. His testimony cannot be said to be free from the opportunity and temptation to be exaggerated and even fabricated for it was intended to secure his freedom. Moreover, despite the fact that there were other persons present during the alleged NPA seminar who could have corroborated Masamlok's testimony that the accused used the gun in furtherance of subversive activities or actually engaged in subversive acts, the prosecution never presented any other witness. Dispositive Judgment of conviction is REVERSED and SET ASIDE. Accused Burgos is ACQUITTED on grounds of reasonable doubt.

they had reasonable ground to believe that the accused had truly committed a crime. There is no showing that there was a real apprehension that the accused was on the verge of flight or escape. Likewise, there is no showing that the whereabouts of the accused were unknown. - The basis for the action taken by the arresting officer was the verbal report made by Masamlok who was not required to subscribe his allegations under oath. There was no compulsion for him to state truthfully his charges under pain of criminal prosecution. Consequently, the need to go through the process of securing a search warrant and a warrant of arrest becomes even more clear. The arrest of the accused while he was plowing his field is illegal. The arrest being unlawful, the search and seizure which transpired afterwards could not likewise be deemed legal as being mere incidents to a valid arrest. Neither can it be presumed that there was a waiver, or that consent was given by the accused to be searched simply because he failed to object. To constitute a waiver, it must appear first that the right exists; secondly, that the person involved had knowledge, actual or constructive, of the existence of such a right; and lastly, that said person had an actual intention to relinquish the right. The fact that the accused failed to object to the entry into his house does not amount to a permission to make a search therein. 2. NO. Since the extra-judicial confession, the firearm, and the alleged subversive documents are inadmissible in evidence, the only remaining proof to sustain the charge is the testimony of Masamlok, which is inadequate to convict Burgos beyond reasonable doubt. Reasoning Although it is true that the trial court found Masamloks testimony credible and convincing, the SC is not necessarily bound by the credibility which the trial court attaches to a particular witness. As stated in People v Cabrera (100 SCRA 424): When it comes to question of credibility the findings of the trial court are entitled to great respect upon appeal for the obvious reason that it was able to observe the demeanor, actuations and deportment of the witnesses during the trial. But We have also said that this rule is not absolute for otherwise there would be no reversals of convictions upon appeal. We must reject the findings of the trial court where the record discloses circumstances of weight and substance which were not properly appreciated by the trial court. In the instant case, Masamloks testimony was totally uncorroborated. Considering that Masamlok

ALIH v CASTRO 151 SCRA 279 CRUZ; June 23, 1987


NATURE Petition for prohibition and mandamus preliminary injunction and restraining order with

FACTS - On November 25, 1984, a contingent of more than two hundred Philippine marines and elements of the home defense forces raided the compound occupied by the petitioners at Gov. Alvarez street, Zamboanga City, in search of loose firearms, ammunition and other explosives. - The military operation was commonly known and dreaded as a "zona," which was like the feared practice of the kempeitai during the Japanese Occupation of rounding up the people in a locality, arresting the persons fingered by a hooded informer, and executing them outright (although the last part is not included in the modern refinement). - The initial reaction of the people inside the compound was to resist the invasion with a burst of gunfire. The soldiers returned fire and a bloody shoot-out ensued, resulting in a number of casualties. - 16 male occupants were arrested, later to be fingerprinted, paraffin-tested and photographed over their objection. The military also inventoried and confiscated nine M16 rifles, one M14 rifle, nine rifle grenades, and several rounds of ammunition found in the premises.

Criminal Procedure Rowena Daroy Morales


- On December 21, 1984, the petitioners came to this Court. Their purpose was to recover the articles seized from them, to prevent these from being used as evidence against them, and to challenge their finger-printing, photographing and paraffin-testing as violative of their right against self-incrimination. - The petitioners demand the return of the arms and ammunition on the ground that they were taken without a search warrant as required by the Bill of Rights. This is confirmed by the said report and in fact admitted by the respondents, "but with avoidance. ISSUE WON the search of petitioners premises was illegal. HELD YES. Ratio Even if were assumed for the sake of argument that they were guilty, they would not have been any less entitled to the protection of the Constitution, which covers both the innocent and the guilty. Reasoning Article IV, Section 3, of the 1973 Constitution: The right of the people to be secure in their persons, houses, papers, and effects against unreasonable searches and seizures of whatever nature and for any purpose shall not be violated, and no search warrant or warrant of arrest shall issue except upon probable cause to be determined by the judge, or such other responsible officer as may be authorized by law, after examination under oath or affirmation of the complainant and the witnesses he may produce, and particularly describing the place to be searched, and the persons or things to be seized. Article IV, Section 4(2): Any evidence obtained in violation of this or the preceding section shall be inadmissible for any purpose in any proceeding. -The respondents, while admitting the absence of the required such warrant, sought to justify their act on the ground that they were acting under superior orders. There was also the suggestion that the measure was necessary because of the aggravation of the peace and order problem generated by the assassination of Mayor Cesar Climaco. - Superior orders" cannot, of course, countermand the Constitution. The fact that the petitioners were suspected of the Climaco killing did not excuse the constitutional short-cuts the respondents took. - Zamboanga City at the time in question certainly did not excuse the non-observance of the constitutional guaranty against unreasonable

a2010

page 82

Prof.
hereby declared ILLEGAL and all the articles seized as a result thereof are inadmissible in evidence against the petitioners in any proceedings. However, the said articles shall remain in custodia legis pending the outcome of the criminal cases that have been or may later be filed against the petitioners.

searches and seizures. There was no state of hostilities in the area to justify, assuming it could, the repressions committed therein against the petitioners. - The record does not disclose that the petitioners were wanted criminals or fugitives from justice. At the time of the "zona," they were merely suspected of the mayor's slaying and had not in fact even been investigated for it. As mere suspects, they were presumed innocent and not guilty as summarily pronounced by the military. - lacking the shield of innocence, the guilty need the armor of the Constitution, to protect them, not from a deserved sentence, but from arbitrary punishment. Every person is entitled to due process. It is no exaggeration that the basest criminal, ranged against the rest of the people who would condemn him outright, is still, under the Bill of Rights, a majority of one. - The respondents cannot even plead the urgency of the raid because it was in fact not urgent. They knew where the petitioners were. They had every opportunity to get a search warrant before making the raid. If they were worried that the weapons inside the compound would be spirited away, they could have surrounded the premises in the meantime, as a preventive measure. - Conceding that the search was truly warrantless, might not the search and seizure be nonetheless considered valid because it was incidental to a legal arrest? Surely not. If all the law enforcement authorities have to do is force their way into any house and then pick up anything they see there on the ground that the occupants are resisting arrest, then we might as well delete the Bill of Rights as a fussy redundancy. - If the arrest was made under Rule 113, Section 5, of the Rules of Court in connection with a crime about to be committed, being committed, or just committed, what was that crime? There is no allegation in the record of such a justification. Parenthetically, it may be observed that under the Revised Rule 113, Section 5(b), the officer making the arrest must have personal knowledge of the ground therefor. - It follows that as the search of the petitioners' premises was violative of the Constitution, all the firearms and ammunition taken from the raided compound are inadmissible in evidence in any of the proceedings against the petitioners. These articles are "fruits of the poisonous tree. Dispositive WHEREFORE, the search of the petitioners' premises on November 25, 1984, is

POSADAS v CA (PEOPLE) 188 SCRA 288 GANCAYCO; August 2, 1990


NATURE Petition for review FACTS - Pat. Ursicio Ungab and Pat. Umbra Umpar, both members of the Integrated National Police (INP) of the Davao Metrodiscom assigned with the Intelligence Task Force, were conducting a surveillance along Magallanes Street, Davao City on October 16, 1986 at about 10:00 o'clock in the morning. They spotted petitioner carrying a "buri" bag and they noticed him to be acting suspiciously while they were within the premises of the Rizal Memorial Colleges They approached the petitioner and identified themselves as members of the INP. Petitioner attempted to flee but his attempt to get away was thwarted by the two notwithstanding his resistance. They then checked the "buri" bag of the petitioner where they found one (1) caliber .38 Smith & Wesson revolver with Serial No. 770196 two (2) rounds of live ammunition for a .38 caliber gun a smoke (tear gas) grenade, and two (2) live ammunitions for a .22 caliber gun. They brought the petitioner to the police station for further investigation. In the course of the same, the petitioner was asked to show the necessary license or authority to possess firearms and ammunitions found in his possession but he failed to do so. - He was then taken to the Davao Metrodiscom office and the prohibited articles recovered from him were indorsed to M/Sgt. Didoy the officer then on duty. He was prosecuted for illegal possession of firearms and ammunitions in the Regional Trial Court of Davao City wherein after a plea of not guilty and trial on the merits a decision was rendered on October 8, 1987 finding petitioner guilty of the offense. (It appearing that the accused was below eighteen (18) years old at the time of the commission of the offense (Art. 68, par. 2), he was sentenced to an indeterminate penalty ranging from TEN (10) YEARS and ONE (1)

Criminal Procedure Rowena Daroy Morales


DAY of prision mayor to TWELVE (12) Years, FIVE (5) months and Eleven (11) days of Reclusion Temporal, and to pay the costs. The firearm, ammunitions and smoke grenade are forfeited in favor of the government and the Branch Clerk of Court is hereby directed to turn over said items to the Chief, Davao Metrodiscom, Davao City.) - The petitioner interposed an appeal to the Court of Appeals wherein in due course a decision was rendered on February 23, 1989 affirming the appealed decision with costs against the petitioner. Hence, this petition for review, the main thrust of which is that there being no lawful arrest or search and seizure, the items which were confiscated from the possession of the petitioner are inadmissible in evidence against him. ISSUE WON the warrantless petitioner is valid search imposed on the

a2010

page 83

Prof.
circumstances involved, including the purpose of the search or seizure, the presence or absence of probable cause, the manner in which the search and seizure was made, the place or thing searched and the character of the articles procured. Clearly, the search in the case at bar can be sustained under the exceptions heretofore discussed, and hence, the constitutional guarantee against unreasonable searches and seizures has not been violated. Dispositive The petition is denied

HELD NO Ratio - The argument of the Solicitor General that when the two policemen approached the petitioner, he was actually committing or had just committed the offense of illegal possession of firearms and ammunitions in the presence of the police officers and consequently the search and seizure of the contraband was incidental to the lawful arrest in accordance with Section 12, Rule 126 of the 1985 Rules on Criminal Procedure is untenable. At the time the peace officers in this case identified themselves and apprehended the petitioner as he attempted to flee they did not know that he had committed, or was actually committing the offense of illegal possession of firearms and ammunitions. They just suspected that he was hiding something in the buri bag. They did now know what its contents were. The said circumstances did not justify an arrest without a warrant. However, there are many instances where a warrant and seizure can be effected without necessarily being preceded by an arrest, foremost of which is the "stop and search" without a search warrant at military or police checkpoints, the constitutionality or validity of which has been upheld by this Court in Valmonte vs. de Villa (to quote: Not all searches and seizures are prohibited. Those which are reasonable are not forbidden. A reasonable search is not to be determined by any fixed formula but is to be resolved according to the facts of each case. Where,

for example, the officer merely draws aside the curtain of a vacant vehicle which is parked on the public fair grounds, or simply looks into a vehicle or flashes a light therein, these do not constitute unreasonable search. True, the manning of checkpoints by the military is susceptible of abuse by the men in uniform in the same manner that all governmental power is susceptible of abuse. But, at the cost of occasional inconvenience, discomfort and even irritation to the citizen, the checkpoints during these abnormal times, when conducted within reasonable limits, are part of the price we pay for an orderly society and a peaceful community. Checkpoints may also be regarded as measures to thwart plots to destabilize the government in the interest of public security. In this connection, the Court may take judicial notice of the shift to urban centers and their suburbs of the insurgency movement, so clearly reflected in the increased killings in cities of police and military men by NPA "sparrow units," not to mention the abundance of unlicensed firearms and the alarming rise in lawlessness and violence in such urban centers, not all of which are reported in media, most likely brought about by deteriorating economic conditions ? which all sum up to what one can rightly consider, at the very least, as abnormal times.) - In this case, the warrantless search and seizure is more reasonable considering that unlike in the former, it was effected on the basis of a probable cause. The probable cause is that when the petitioner acted suspiciously and attempted to flee with the buri bag there was a probable cause that he was concealing something illegal in the bag and it was the right and duty of the police officers to inspect the same. It is too much indeed to require the police officers to search the bag in the possession of the petitioner only after they shall have obtained a search warrant for the purpose. Such an exercise may prove to be useless, futile and much too late. - In People vs. CFI of Rizal, the Court held as follows: . . . In the ordinary cases where warrant is indispensably necessary, the mechanics prescribed by the Constitution and reiterated in the Rules of Court must be followed and satisfied. But We need not argue that there are exceptions. Thus in the extraordinary events where warrant is not necessary to effect a valid search or seizure, or when the latter cannot be performed except without warrant, what constitutes a reasonable or unreasonable search or seizure becomes purely a judicial question, determinable from the uniqueness of the

ALLADO v DIOKNO [supra, page 48] MALALOAN v CA (FINEZA) 232 SCRA 249 REGALADO; May 6, 1994
NATURE Petition for review on certiorari of a decision of CA. FACTS - 1st Lt. Absalon V. Salboro of the CAPCOM Northern Sector (now Central Sector) filed with the Regional Trial Court of Kalookan City an application for search warrant. The search warrant was sought for in connection with an alleged violation of P.D. 1866 (Illegal Possession of Firearms and Ammunitions) perpetrated at No. 25 Newport St., corner Marlboro St., Fairview, QUEZON CITY. On March 23, 1990, respondent RTC Judge of KALOOKAN CITY issued Search Warrant No. 95-90. - On the same day, at around 2:30 p.m., members of the CAPCOM, armed with subject search warrant, proceeded to the situs of the offense alluded to, where a labor seminar of the Ecumenical Institute for Labor Education and Research (EILER) was then taking place. According to CAPCOM's 'Inventory of Property Seized,' firearms, explosive materials and subversive documents, among others, were seized and taken during the search. And all the sixty-one (61) persons found within the premises searched were brought to Camp Karingal, Quezon City but most of them were later released, with the exception of the herein petitioners, EILER Instructors, who were indicted for violation of P.D. 1866 in Criminal Case No. Q-90-11757 before Branch 88 of the Regional Trial Court of Quezon City, presided over by respondent Judge Tirso D.C. Velasco. - On July 10, 1990, petitioners presented a 'Motion for Consolidation, Quashal of Search Warrant and For the Suppression of All Illegally Acquired Evidence'

Criminal Procedure Rowena Daroy Morales


before the Quezon City court; and a 'Supplemental Motion to the Motion for Consolidation, Quashal of Search Warrant and Exclusion of evidence Illegally Obtained'. - On September 21, 1990, the respondent Quezon City Judge issued the challenged order, consolidating subject cases but denying the prayer for the quashal of the search warrant under attack, the validity of which warrant was upheld; opining that the same falls under the category of Writs and Processes, within the contemplation of paragraphs 3(b) of the Interim Rules and Guidelines, and can be serve not only within the territorial jurisdiction of the issuing court but anywhere in the judicial region of the issuing court (National Capital Judicial Region). - Respondent Court of Appeals rendered judgment, in effect affirming that of the trial court, by denying due course to the petition for certiorari and lifting the temporary restraining order it had issued on November 29, 1990 in connection therewith. This judgment of respondent court is now impugned in and sought to be reversed through the present recourse before us. ISSUE WON a court may take cognizance of an application for a search warrant in connection with an offense committed outside its territorial jurisdiction and to issue a warrant to conduct a search on a place likewise outside its territorial jurisdiction. HELD YES - No law or rule imposes such a limitation on search warrants, in the same manner that no such restriction is provided for warrants of arrest. The arguments of petitioners are not inferable by necessary implication from the statutory provisions which are presumed to be complete and expressive of the intendment of the framers. A contrary interpretation on whatever pretext should not be countenanced. - A bit of legal history on his contestation will be helpful. The jurisdictional rule heretofore was that writs and process of the so-called inferior courts could be enforced outside the province only with the approval of the former court of first instance. Under the Judiciary Reorganization Act, the enforcement of such writs and processes no longer needs the approval of the regional trial court. On the other hand, while, formerly, writs and processes of the then courts of first instance were enforceable throughout the Philippines, under the Interim or

a2010

page 84

Prof.
witnesses to and their examination in said places, with the attendant risk, danger and expense. Also, a further well-founded precaution, obviously born of experience and verifiable data, is articulated by the court a quo, as quoted by respondent court: "This court is of the further belief that the possible leakage of information which is of utmost importance in the issuance of a search warrant is secured (against) where the issuing magistrate within the region does not hold court sessions in the city or municipality, within the region, where the place to be searched is located." - The foregoing situations may also have obtained and were taken into account in the foreign judicial pronouncement that, in the absence of statutory restrictions, a justice of the peace in one district of the county may issue a search warrant to be served in another district of the county and made returnable before the justice of still another district or another court having jurisdiction to deal with the matters involved. In the present state of our law on the matter, we find no such statutory restrictions both with respect to the court which can issue the search warrant and the enforcement thereof anywhere in the Philippines. - NONETHELESS, TO PUT DOUBTS TO REST, THE SUPREME COURT LAID DOWN THE FOLLOWING POLICY GUIDELINES; 1. The Court wherein the criminal case is pending shall have primary jurisdiction to issue search warrants necessitated by and for purposes of said case. An application for a search warrant may be filed with another court only under extreme and compelling circumstances that the applicant must prove to the satisfaction of the latter court which may or may not give due course to the application depending on the validity of the justification offered for not filing the same in the court with primary jurisdiction thereover. 2. When the latter court issues the search warrant, a motion to quash the same may be filed in and shall be resolved by said court, without prejudice to any proper recourse to the appropriate higher court by the party aggrieved by the resolution of the issuing court. All grounds and objections then available, existent or known shall be raised in the original or subsequent proceedings for the quashal of the warrant, otherwise they shall be deemed waived. 3. Where no motion to quash the search warrant was filed in or resolved by the issuing court, the interested party may move in the court where the criminal case is pending for the suppression as evidence of the personal property seized under the

Transitional Rules and Guidelines, certain specified writs issued by a regional trial court are now enforceable only within its judicial region. - PRACTICAL CONSIDERATIONS The Court cannot be blind to the fact that it is extremely difficult, as it undeniably is, to detect or elicit information regarding the existence and location of illegally possessed or prohibited articles. The Court is accordingly convinced that it should not make the requisites for the apprehension of the culprits and the confiscation of such illicit items, once detected, more onerous if not impossible by imposing further niceties of procedure or substantive rules of jurisdiction through decisional dicta. For that matter, we are unaware of any instance wherein a search warrant was struck down on objections based on territorial jurisdiction. - We do not believe that the enforcement of a search warrant issued by a court outside the territorial jurisdiction wherein the place to be searched is located would create a constitutional question. Nor are we swayed by the professed apprehension that the law enforcement authorities may resort to what could be a permutation of forum shopping, by filing an application for the warrant with a "friendly" court. It need merely be recalled that a search warrant is only a process, not an action. Furthermore, the constitutional mandate is translated into specifically enumerated safeguards in Rule 126 of the 1985 Rules on Criminal Procedure for the issuance of a search warrant, and all these have to be observed regardless of whatever court in whichever region is importuned for or actually issues a search warrant. Said requirements, together with the ten-day lifetime of the warrant would discourage resort to a court in another judicial region, not only because of the distance but also the contingencies of travel and the danger involved, unless there are really compelling reasons for the authorities to do so. Besides, it does seem odd that such constitutional protests have not been made against warrants of arrest which are enforceable indefinitely and anywhere although they involve, not only property and privacy, but persons and liberty. - On the other hand, it is a matter of judicial knowledge that the authorities have to contend now and then with local and national criminal syndicates of considerable power and influence, political or financial in nature, and so pervasive as to render foolhardy any attempt to obtain a search warrant in the very locale under their sphere of control. Nor should we overlook the fact that to do so will necessitate the transportation of applicant's

Criminal Procedure Rowena Daroy Morales


warrant if the same is offered therein for said purpose. Since two separate courts with different participations are involved in this situation, a motion to quash a search warrant and a motion to suppress evidence are alternative and not cumulative remedies. In order to prevent forum shopping, a motion to quash shall consequently be governed by the omnibus motion rule, provided, however, that objections not available, existent or known during the proceedings for the quashal of the warrant may be raised in the hearing of the motion to suppress. The resolution of the court on the motion to suppress shall likewise be subject to any proper remedy in the appropriate higher court. 4. Where the court which issued the search warrant denies the motion to quash the same and is not otherwise prevented from further proceeding thereon, all personal property seized under the warrant shall forthwith be transmitted by it to the court wherein the criminal case is pending, with the necessary safeguards and documentation therefor. 5. These guidelines shall likewise be observed where the same criminal offense is charged in different informations or complaints and filed in two or more courts with concurrent original jurisdiction over the criminal action. When the issue of which court will try the case shall have been resolved, such court shall be considered as vested with primary jurisdiction to act on applications for search warrants incident to the criminal case. Dispositive WHEREFORE, on the foregoing premises, the instant petition is DENIED and the assailed judgment of respondent Court of Appeals in CA-G.R. SP No. 23533 is hereby AFFIRMED.

a2010

page 85

Prof.
favor of the unlimited power of a court to issue search warrants. - I have serious misgivings on the majority decision on the matter where another court may, because of extreme and compelling circumstances, issue a search warrant in connection with a criminal case pending in an appropriate court. To illustrate this exception, the Municipal Trial Court of Argao, Cebu, may validly issue a warrant for the search of a house in Davao City and the seizure of any property therein that may have been used in committing an offense in Manila already the subject of an information filed with the Metropolitan Trial Court of Manila. I submit that the exception violates the settled principle that even in cases of concurrent jurisdiction, the first court which acquires jurisdiction over the case acquires it to the exclusion of the other. (People vs. Fernando, 23 SCRA 867, 870 [1968]). This being so, it is with more reason that a court which does not have concurrent jurisdiction with the first which had taken cognizance of the case does not also have the authority to issue writs or processes, including search warrants, in connection with the pending case. Moreover, since the issuance of a search warrants is an incident to a main case or is an exercise of the ancillary jurisdiction of a court, the court where the main case is filed has exclusive jurisdiction over all incidents thereto and in the issuance of all writs and processes in connection therewith. Furthermore, instead of serving the ends of justice, the exception may provide room for unwarranted abuse of the judicial process, wreak judicial havoc and procedural complexities which effective law enforcement apparently cannot justify. I cannot conceive of any extreme and compelling circumstance which the court that first acquired jurisdiction over the case cannot adequately meet within its broad powers and authority. - In the light of the foregoing, and after re-examining my original view in this case, I respectfully submit that: 1. Any court within whose territorial jurisdiction a crime was committed may validly entertain an application for and issue a search warrant in connection with said crime. However, in the National Capital Judicial Region, Administrative Circulars No. 13 of 1 October 1985, and No. 19 of 4 August 1987 must be observed. 2. After the criminal complaint or information is filed with the appropriate court, search warrants in connection with the crime charged may only be issued by said court.

SEPARATE OPINION DAVIDE [concurring and dissenting]


- The absence of any express statutory provision prohibiting a court from issuing a search warrant in connection with a crime committed outside its territorial jurisdiction should not be construed as a grant of blanket authority to any court of justice in the country to issue a search warrant in connection with a crime committed outside its territorial jurisdiction. The majority view suggests or implies that a municipal trial court in Tawi-Tawi, Basilan, or Batanes can validly entertain an application for a search warrant and issue one in connection with a crime committed in Manila. Elsewise stated, all courts in the Philippines, including the municipal trial courts, can validly issue a search warrant in connection with a crime committed anywhere in the

Philippines. Simply put, all courts of justice in the Philippines have, for purposes of issuing a search warrant, jurisdiction over the entire archipelago. - I cannot subscribe to this view since, in the first place, a search warrant is but an incident to a main case and involves the exercise of an ancillary jurisdiction therefore, the authority to issue it must necessarily be co-extensive with the court's territorial jurisdiction. To hold otherwise would be to add an exception to the statutory provisions defining the territorial jurisdiction of the various courts of the country, which would amount to judicial legislation. The territorial jurisdiction of the courts is determined by law, and a reading of Batas Pambansa Blg. 129 discloses that the territorial jurisdiction of regional trial courts, metropolitan trial courts, municipal trial courts and municipal circuit trial courts are confined to specific territories. In the second place, the majority view may legitimize abuses that would result in the violation the civil rights of an accused or the infliction upon him of undue and unwarranted burdens and inconvenience as when, for instance, an accused who is a resident of Basco, Batanes, has to file a motion to quash a search warrant issued by the Metropolitan Trial Court of Manila in connection with an offense he allegedly committed in Itbayat, Batanes. - Nor can Stonehill vs. Diokno (20 SCRA 383) be an authoritative confirmation of the unlimited or unrestricted power of any court to issue search warrants in connection with crimes committed outside its territorial jurisdiction. While it may be true that the forty-two search warrants involved therein were issued by several Judges ---- specifically Judges (a) Amado Roan of the City Court of Manila, (b) Roman Cansino of the City Court of Manila, (c) Hermogenes Caluag of the Court of First Instance of Rizal (Quezon City Branch), (d) Eulogio Mencias of the Court of First Instance of Rizal (Pasig Branch), and (e) Damian Jimenez of the City Court of Quezon City (Footnote 2, page 387) ---- there is no definite showing that the forty-two search warrants were for the searches and seizures of properties outside the territorial jurisdiction of their respective courts. The warrants were issued against the petitioners and corporations of which they were officers and some of the corporations enumerated in Footnote 7 have addressed in Manila and Makati. (pp. 388-89). Rizal (which includes Makati) and Quezon City both belonged to the Seventh Judicial District. That nobody challenged on jurisdictional ground the issuance of these search warrants is no argument in

Criminal Procedure Rowena Daroy Morales


PEOPLE v CA (CERBO) 301 SCRA 475 PANGANIBAN; January 21, 1999
NATURE Petition for Review under Rule 45. FACTS - Rosalinda Dy was shot at pointblank range by Jonathan Cerbo in the presence and at the office of his father, Billy Cerbo - Elsa B. Gumban (eyewitness) identified Jonathan Cerbo as the assailant. - The 3rd Municipal Circuit Trial Court of NabunturanMawab, Davao, after a preliminary investigation, found "sufficient ground to engender a well-founded belief" that the crime of murder has been committed by private respondent Jonathan Cerbo and resolved to forward the entire records of the case to the provincial prosecutor at Tagum, Davao. - After an information for murder was filed against Jonathan Cerbo, petitioner Alynn Plezette Dy, daughter of the victim Rosalinda Dy, executed an affidavit-complaint charging private respondent Billy Cerbo of conspiracy in the killing , supported by a supplemental affidavit of Elsa B. Gumban, alleging that the shooting was done in the office and in the presence of Billy Cerbo who after the shooting did nothing (did not apply first aid nor bring the victim to the hospital) After a reinvestigation the prosecution filed an amended information including Billy Cerbo in the murder case. A warrant for his arrest was later issued. - Private respondent Billy Cerbo then filed a motion to quash warrant of arrest arguing that the same was issued without probable cause. Respondent Judge issued the first assailed order dismissing the case against Billy Cerbo for lack of probable cause and recalling the warrant for his arrest and ordered the withdrawal of the amended information and the filing of a new one charging Jonathan Cerbo only. - Private Prosecutor filed a motion for reconsideration which was denied by the respondent judge. - The Court of Appeals held that Judge Eugenio Valles did not commit grave abuse of discretion in recalling the warrant of arrest issued against Private Respondent Billy Cerbo and subsequently dismissing the Information for murder filed against the private respondent, because the evidence presented thus far did not substantiate such charge.

a2010

page 86

Prof.
or not that function has been correctly discharged by the public prosecutor is a matter that the trial court itself does not and may not be compelled to pass upon. It is not for instance permitted for an accused, upon the filing of the information against him by the public prosecutor, to preempt trial by filing a motion with the Trial Court praying for the quash or dismissal of the indictment on the ground that the evidence upon which the same is based is inadequate. Nor is it permitted, on the antipodal theory that the evidence is in truth inadequate, for the complaining party to present a petition before the Court praying that the public prosecutor be compelled to file the corresponding information against the accused. xxx xxx xxx - Indeed, the public prosecutor has broad discretion to determine whether probable cause exists and to charge those whom he or she believes to have committed the crime as defined by law. Otherwise stated, such official has the quasi-judicial authority to determine whether or not a criminal case list be filed in court. - Crespo v. Mogul: It is a cardinal principle that all criminal actions either commenced by complaint or by information shall be prosecuted under the direction and control of the fiscal. The institution of a criminal action depends upon the sound discretion of the fiscal. He may or may not file the complaint or information, follow or not follow that presented by the offended party, according to whether the evidence, in his opinion, is sufficient or not to establish the guilt of the accused beyond reasonable doubt. The reason for placing the criminal prosecution under the direction and control of the fiscal is to prevent malicious or unfounded prosecutions by private persons. Judicial Determination of Probable Cause - The determination of probable cause to hold a person for trial must be distinguished from the determination of probable cause to issue a warrant of arrest, which is a judicial function. The judicial determination of probable cause in the issuance of arrest warrants has been emphasized in numerous cases. - The rulings in Soliven, Inting and Lim, Sr. were iterated in Allado v. Diokno, where we explained again what probable cause means. Probable cause for the issuance of a warrant of arrest is the existence of such facts and circumstances that would lead a reasonably discreet and prudent person to believe that an offense has been committed by the

ISSUE WON the Trial Court had the authority to reverse the public prosecutor's finding of probable cause to prosecute accused and thus dismiss the case filed by the latter on the basis of a motion to quash warrant of arrest HELD NO Ratio The determination of probable cause during a preliminary investigation is a function that belongs to the public prosecutor. It is an executive function, the correctness of the exercise of which is matter that the trial court itself does not and may not be compelled to pass upon. - If the information is valid on its face and there is no showing of manifest error, grave abuse of discretion or prejudice on the part of the public prosecutor, courts should not dismiss it for 'want of evidence,' because evidentiary matters should be presented and heard during the trial. The functions and duties of both the trial court and the public prosecutor in "the proper scheme of things" in our criminal justice system should be clearly understood. Reasoning: Executive Determination of Probable Cause - The Separate (Concurring) Opinion of former Chief Justice Andres R. Narvasa in Roberts v. Court of Appeals : xxxthe Court is being asked to examine and assess such evidence as has thus far been submitted by the parties and, on the basis thereof, make a conclusion as to whether or not it suffices "to engender a well founded belief that a crime has been committed and that the respondent is probably guilty thereof and should be held for trial." - It is a function that this Court should not be called upon to perform. It is a function that properly pertains to the public prosecutor., one that, as far as crimes cognizable by a Regional Trial Court are concerned, and notwithstanding that it involves an adjudicative process of a sort, exclusively pertains, by law, to said executive officer, the public prosecutor. It is moreover a function that in the established scheme of things, is supposed to be performed at the very genesis of, indeed, prefatorily to, the formal commencement of a criminal action. The proceedings before a public prosecutor, it may well be stressed, are essentially preliminary, prefatory and cannot lead to a final, definite and authoritative adjudgment of the guilt or innocence of the persons charged with a felony or crime. Whether

Criminal Procedure Rowena Daroy Morales


person sought to be arrested. Hence, the judge, before issuing a warrant of arrest, "must satisfy himself that based on the evidence submitted, there is sufficient proof that a crime has been committed and that the person to be arrested is probably guilty thereof." At this stage of the criminal proceeding, the judge is not yet tasked to review in detail the evidence submitted during the preliminary investigation. It is sufficient that he personally evaluates such evidence in determining probable cause. - As held in Inting, the determination of probable cause by the prosecutor is for a purpose different from that which is to be made by the judge. Whether there is reasonable ground to believe that the accused is guilty of the offense charged and should be held for trial is what the prosecutor passes upon. The judge, on the other hand, determines whether a warrant of arrest should be issued against the accused, i.e., whether there is a necessity for placing him under immediate custody in order not to frustrate the ends of justice. - Verily, a judge cannot be compelled to issue a warrant of arrest if he or she deems that there is no probable cause for doing so. Corollary to this principle, the judge should not override the public prosecutor's determination of probable cause to hold an accused for trial on the ground that the evidence presented to substantiate the issuance of an arrest warrant was insufficient, as in the present case. Inapplicabilty of Allado and Salonga - Allado and Salonga constitute exceptions to the general rule and may be invoked only if similar circumstances are clearly shown to exist. However, the present case is not on all fours with Allado and Salonga. First, Elsa Gumban, the principal eyewitness to the killing of Rosalinda Dy, was not a participation or conspirator in the commission of the said crime. In Allado and Salonga, however, the main witnesses were the confessed perpetrators of the crimes, whose testimonies the court deemed 'tainted'. Second, in the case at bar, the private respondent was accorded due process, and no precipitate haste or bias during the investigation of the case can be imputed to the public prosecutor. On the other hand, the Court noted in Allado the "undue haste in the filing of the Information and in the inordinate interest of the government" in pursuing the case; and in Salonga, " . . . the failure of the prosecution to show that the petitioner was probably guilty of conspiring to commit the crime, the initial disregard of petitioner's constitutioner rights and the massive and

a2010

page 87

Prof.

damaging publicity against him." The rulings in the two aforementioned cases cannot apply to it.

PEOPLE v USANA and LOPEZ 323 SCRA 754 DAVIDE; January 28, 2000
NATURE Appeal from the decision of the Regional Trial Court convicting the two accused together with Julian D. Escano for the violation of R.A. 6425, as amended FACTS - On the 5th of April 1995 and during a COMELEC gun ban, some law enforcers of the Makati Police were manning a checkpoint at the corner of Senator Gil Puyat Ave. and the South Luzon Expressway. They were checking the cars going to Pasay City, stopping those they found suspicious, and imposing merely a running stop on the others. At about past midnight, they stopped a Kia Pride car with Plate No. TBH 493. One of the policemen saw a long firearm on the lap of the person seated at the passenger seat, who was later identified as Virgilio Usana. They asked the driver, identified as Escao, to open the door. PO3 Suba seized the long firearm, an M-1 US Carbine, from Usana. When Escao, upon order of the police, parked along Sen. Gil Puyat Ave., the other passengers were search for more weapons. Their search yielded a .45 caliber firearm which they seized from Escao. - The three passengers were thereafter brought to the police station Block 5 in the Kia Pride driven by PO3 Nonato. Upon reaching the precinct, Nonato turned over the key to the desk officer. Since SPO4 de los Santos was suspicious of the vehicle, he requested Escao to open the trunk. Escao readily agreed and opened the trunk himself using his key. They noticed a blue bag inside it, which they asked Escao to open. The bag contained a parcel wrapped in tape, which, upon examination by National Bureau of Investigation, was found positive for hashish. - An information for violation of RA 6425 thereafter was filed against them. The trial court found the three accused guilty of the said crime. - Accused-appellants assail the manner by which the checkpoint in question was conducted. They contend that the checkpoint manned by elements of the Makati Police should have been announced. They also complain of its having been conducted in an arbitrary and discriminatory manner. Also, they question the validity of the search.

ISSUES 1. WON the check point was illegal 2. WON the search was valid 3. WON the accused are guilty of violation of RA 6425 HELD 1. NO - Not all checkpoints are illegal. Those which are warranted by the exigencies of public order and are conducted in a way least intrusive to motorists are allowed. Ratio This Court has ruled that not all checkpoints are illegal. Those which are warranted by the exigencies of public order and are conducted in a way least intrusive to motorists are allowed. For, admittedly, routine checkpoints do intrude, to a certain extent, on motorists right to "free passage without interruption," but it cannot be denied that, as a rule, it involves only a brief detention of travelers during which the vehicles occupants are required to answer a brief question or two. For as long as the vehicle is neither searched nor its occupants subjected to a body search, and the inspection of the vehicle is limited to a visual search, said routine checks cannot be regarded as violative of an individuals right against unreasonable search. In fact, these routine checks, when conducted in a fixed area, are even less intrusive. - The checkpoint herein conducted was in pursuance of the gun ban enforced by the COMELEC. The COMELEC would be hard put to implement the ban if its deputized agents were limited to a visual search of pedestrians. It would also defeat the purpose for which such ban was instituted. Those who intend to bring a gun during said period would know that they only need a car to be able to easily perpetrate their malicious designs. - The facts adduced do not constitute a ground for a violation of the constitutional rights of the accused against illegal search and seizure. PO3 Suba admitted that they were merely stopping cars they deemed suspicious, such as those whose windows are heavily tinted just to see if the passengers thereof were carrying guns. At best they would merely direct their flashlights inside the cars they would stop, without opening the cars doors or subjecting its passengers to a body search. There is nothing discriminatory in this as this is what the situation demands. We see no need for checkpoints to be announced, as the accused have invoked. Not only would it be impractical, it would also forewarn those who intend

Criminal Procedure Rowena Daroy Morales


to violate the ban. Even so, badges of legitimacy of checkpoints may still be inferred from their fixed location and the regularized manner in which they are operated. 2. YES - Escano consented to the search and consented warrantless search is one of the exceptions from the warrant requirement. Ratio Jurisprudence recognizes six generally accepted exceptions to the warrant requirement: (1) search incidental to an arrest; (2) search of moving vehicles; (3) evidence in plain view; (4) customs searches; (5) consented warrantless search; and (6) stop-and-frisk situations. - Even though there was ample opportunity to obtain a search warrant, we cannot invalidate the search of the vehicle, for there are indications that the search done on the car of Escao was consented to by him. 3. NO - No fact was adduced to link Usana and Lopez to the hashish found in the trunk of the car and there was no showing that Usana and Lopez knew of the presence of hashish in the trunk of the car or that they saw the same before it was seized. Ratio Despite the validity of the search, we cannot affirm the conviction of Usana and Lopez for violation of R.A. No. 6425, as amended. The following facts militate against a finding of conviction: (1) the car belonged to Escao; (2) the trunk of the car was not opened soon after it was stopped and after the accused were searched for firearms; (3) the car was driven by a policeman from the place where it was stopped until the police station; (4) the cars trunk was opened, with the permission of Escao, without the presence of Usana and Lopez; and (5) after arrival at the police station and until the opening of the cars trunk, the car was in the possession and control of the police authorities. No fact was adduced to link Usana and Lopez to the hashish found in the trunk of the car. Their having been with Escao in the latters car before the "finding" of the hashish sometime after the lapse of an appreciable time and without their presence left much to be desired to implicate them to the offense of selling, distributing, or transporting the prohibited drug. In fact, there was no showing that Usana and Lopez knew of the presence of hashish in the trunk of the car or that they saw the same before it was seized. Dispositive Accused appellants are hereby acquitted.

a2010
PUNO; January 22, 1999

page 88

Prof.
committing, or is attempting to commit an offense; - Under Section 5 (a), as above-quoted, a person may be arrested without a warrant if he "has committed, is actually committing, or is attempting to commit an offense." - In the case, Doria was caught in the act of committing an offense. When an accused is apprehended in flagrante delicto as a result of a buybust operation, the police are not only authorized but duty-bound to arrest him even without a warrant. - However, the warrantless arrest, search and seizure of Gaddao is invalid - Accused-appellant Gaddao was not caught redhanded during the buy-bust operation to give ground for her arrest under Section 5 (a) of Rule 113. She was not committing any crime. Contrary to the finding of the trial court, there was no occasion at all for appellant Gaddao to flee from the policemen to justify her arrest in "hot pursuit." 114 In fact, she was going about her daily chores when the policemen pounced on her. - Neither could the arrest of appellant Gaddao be justified under the second instance of Rule 113. "Personal knowledge" of facts in arrests without warrant under Section 5 (b) of Rule 113 must be based upon "probable cause" which means an "actual belief or reasonable grounds of suspicion." In case, there was no reasonable suspicion especially as she was arrested solely on the basis of the alleged identification made by her coaccused - Doria did not point to appellant Gaddao as his associate in the drug business, but as the person with whom he left the marked bills. This identification does not necessarily lead to the conclusion that appellant Gaddao conspired with her co-accused in pushing drugs as Doria may have left the money in her house, with or without her knowledge, with or without any conspiracy. Save for accused-appellant Doria 's word, the Narcom agents had no reasonable grounds to believe that she was engaged in drug pushing. - As the arrest was illegal, the search and seizure is not incidental to the arrest 2. NO - The marijuana was not in plain view of the police officers and its seizure without the requisite search warrant was in violation of the law and the Constitution as the contents of the box where the marijuana was partially hidden was not readily apparent to PO Manlangit, one of the arresting officers.

FACTS - Philippine National Police (PNP) Narcotics Command (Narcom), received information from two (2) civilian informants (CI) that one "Jun" was engaged in illegal drug activities in Mandaluyong City. They decided to entrap him via a buy-bust operation. -The poseur-buyer, PO2 Manlangit set aside 1600 pesos as marked money for the entrapment operation, which was then handed to Jun upon transaction. Jun returned an hour later bringing marijuana where he and his associates subsequently arrested Jun but did not find the marked bills on him. Jun said he left the bills to his associate Neneth. Jun led the police to Neneths house. - The police went to Neneths house. Standing by the door, PO3 Manlangit noticed a carton box under the dining table. He saw that one of the box's flaps was open and inside the box was something wrapped in plastic. The plastic wrapper and its contents appeared similar to the marijuana earlier "sold" to him by "Jun." His suspicion aroused, PO3 Manlangit entered "Neneth's" house and took hold of the box. He peeked inside the box and found that it contained ten (10) bricks of what appeared to be dried marijuana leaves. They also found the marked bills. They arrested Jun and Neneth and brought them to headquarters. It was only then that the police learned that "Jun" is Florencio Doria y Bolado while "Neneth" is Violeta Gaddao y Catama. - The trial court found them guilty. ISSUES 1. WON the warrantless arrest of Doria and Gaddao, the search of the latters person and house, and the admissibility of the pieces of evidence obtained therefrom is valid 2. WON the marijuana was seized validly for being in plain view of the police officers HELD 1. YES - We also hold that the warrantless arrest of accusedappellant Doria is not unlawful. Warrantless arrests are allowed in three instances as provided by Section 5 of Rule 113 of the 1985 Rules on Criminal Procedure, to wit: Sec. 5. Arrest without warrant; when lawful. A peace officer or a private person may, without a warrant, arrest a person: (a) When, in his presence, the person to be arrested has committed, is actually

PEOPLE v DORIA 301 SCRA 668

Criminal Procedure Rowena Daroy Morales


- As a general rule, objects in plain view of arresting officers may be seized without a search warrant but must follow these requisites: (a) the law enforcement officer in search of the evidence has a prior justification for an intrusion or is in a position from which he can view a particular area; (b) the discovery of the evidence in plain view is inadvertent; (c) it is immediately apparent to the officer that the item he observes may be evidence of a crime, contraband or otherwise subject to seizure. - However, if it is not plain view of the police officers, it may not be seized without a warrant except if the package proclaims its contents, whether by its distinctive configuration, its transparency, or if its contents are obvious to an observer, then the contents are in plain view and may be seized. - The fact that the box containing about six (6) kilos of marijuana 137 was found in the house of accusedappellant Gaddao does not justify a finding that she herself is guilty of the crime charged. Dispositive the decision of the Regional Trial Court, Branch 156, Pasig City acting as a Special Court in Criminal Case No. 3307-D is reversed and modified as follows: 1. Accused-appellant Florencio Doria y Bolado is sentenced to suffer the penalty of reclusion perpetua and to pay a fine of five hundred thousand pesos (P500,000.00). 2. Accused-appellant Violeta Gaddao y Catama is acquitted.

a2010

page 89

Prof.
are presumed to have regularly performed their duty in the absence of proof to the contrary. -in many cases, drug pushers did sell their prohibited articles to prospective customers, be they strangers or not, in private as well as in public places, even in the daytime. Indeed, some drug pushers appear to have become exceedingly daring, openly defiant of the law. Hence, what matters is not the existing familiarity between the buyer and the seller, or the time and venue of the sale, but the fact of agreement as well as the act constituting sale and delivery of prohibited drugs 2. YES Ratio: The arrest was within the purview of Sec5 (a), Rule 113, Rules on Criminal Procedure, to wit: Arrest without warrant, when lawful. A peace officer or a private person may, without a warrant, arrest a person: (a) When, in his presence, the person to be arrested has committed, is actually committing, or is attempting to commit an offense; Reasoning: -Elamparo assails the legality of his arrest for failure of the apprehending officers to secure a search warrant. - for warrantless arrests, 2 elements must concur: (1) the person to be arrested must execute an overt act indicating the he has just committed, is actually committing, or is attempting to commit a crime; and (2) such overt act is done in the presence or within the view of the arresting officer. Thus, when he was seen repacking the marijuana, the police officers were not only authorized but also duty-bound to arrest him even without a warrant. Re: warrantless seizures: -However, not being absolute, the right against unreasonable searches and seizures is subject to exceptions. Thus, for example, Sec.12, Rule 126, Rules on CrimPro, provides that a person lawfully arrested may be searched for dangerous weapons or anything which may be used as proof of the commission of an offense, without a search warrant. -5 generally accepted exceptions to the right against warrantless searches and seizures have also been judicially formulated, viz: (1) search incidental to a lawful arrest, (2) search of moving vehicles, (3) seizure in plain view, (4) customs searches, and (5) waiver by the accused themselves of their right against unreasonable search and seizure. - this case falls squarely under the plain view doctrine People v Doria: Objects falling in plain view of an officer who has a right to be in the position to have

happened on the same day (12 Feb 1995), while the information for illegal possession of drugs was filed on 15 Feb 1995. - Arraignment: plea of not guilty. - Trial: prosecution presented the ff witnesses: police officer who was also poseur-buyer, another officer who took part in buy-bust, and NBI chemist who examined and confirmed the confiscated drugs to be marijuana. Defense presented as witnesses boarders of Elamparos house, saying that Elamparo was at their house when somebody knocked at their door. His father opened the same and was informed that somebody was looking for him. He went out and saw Spencer with handcuffs and being held by an arresting officer. When Elamparo persistently questioned Spencer as to why he was arrested, the arresting officers got mad at him prompting them to likewise bring him to the police station where he was detained. The officers demanded P15,000.00 for his release which he did not give. On the other hand, Spencer gave the sum and was released. - RTC: Guilty, under RA 6425. penalty of reclusion perpetua and fine of P9million. Elamparo appealed. ISSUE: 1. WON RTC was correct in the assessment of credibility of witnesses 2. WON the arrest of Elamparo was valid 3. WON the penalty imposed was correct HELD: 1. YES Ratio: Unless the trial court overlooked substantial facts which would affect the outcome of the case, we accord the utmost respect to their findings of facts. Reasoning: -Elamparo contends that it is highly unusual for arresting officers to act on an information of an unknown source without confirming the veracity of the report, and that it is incredible that a peddler of marijuana would be so brazen as to approach total strangers and offer to sell them marijuana. He insists that he was charged with illegal possession of marijuana because he failed to pay the police officers P15,000.00 for his release. - it is well-settled that the assessment of credibility of witnesses is within the province of the trial court which had an opportunity to observe the witnesses and their demeanor during their testimonies. As compared to the baseless claims of Elamparo, the version of the prosecution witnesses appears worthy of belief, coming as it does from law enforcers who

PEOPLE v ELAMPARO 329 SCRA 404 QUISUMBING; March 31, 2000


NATURE Appeal from judgment of RTC. FACTS - Acting on a report by an informant, police officers conducted a buy-bust operation (of marijuana) in Caloocan. They arrested the person who sold them the marijuana (Spencer), but the same was able to escape. Then: the buy-bust team pursued Spencer, who ran inside a bungalow-type house. Having trapped Spencer inside the house, the police officers frisked him and recovered the marked money. The officers also found Elamparo repacking 5 bricks of marijuana inside the houses sala Elamparo was then arrested and were taken to a precinct and delivered to an inquest fiscal for further investigation. The buy-bust operation and arrest

Criminal Procedure Rowena Daroy Morales


that view are subject to seizure even without a search warrant and may be introduced in evidence. The "plain view" doctrine applies when the following requisites concur: (a) law enforcement officer in search of the evidence has a prior justification for an intrusion or is in a position from which he can view a particular area; (b) discovery of the evidence in plain view is inadvertent; (c) immediately apparent to the officer that the item he observes may be evidence of a crime, contraband or otherwise subject to seizure. The law enforcement officer must lawfully make an initial intrusion or properly be in a position from which he can particularly view the area. In the course of such lawful intrusion, he came inadvertently across a piece of evidence incriminating the accused. The object must be open to eye and hand and its discovery inadvertent. - members of the buy-bust team were justified in running after Spencer (when he escaped) and entering the house without a search warrant for they were hot in the heels of a fleeing criminal. Once inside the house, the police officers cornered Spencer and recovered the buy-bust money from him. They also caught Elamparo in flagrante delicto repacking the marijuana bricks which were in full view 3. YES Ratio: Minority serves as a privileged mitigating circumstance to a crime, thus entitling the accused to a reduction of penalty one degree lower than that imposable (by virtue of art.13 (2) RPC) Reasoning: - contends that if found guilty, the privileged mitigating circumstance of minority should be appreciated in his favor. - In drug cases, the quantity of prohibited drugs involved is determinative of the imposable penalty. Section 20 of R.A. No. 6425, as amended by Section 17 of R.A. No. 7659, provides that when the quantity of indian hemp or marijuana is 750 grams or more, as in this case, the penalty shall be reclusion perpetua to death and fine ranging from five hundred thousand pesos (P500,000.00) to ten million pesos (P10,000,000.00). - Appellant having been born on January 9, 1978, was only 17 years, 1 month, and 3 days old, at the time of the commission of the crime on February 12, 1995. - being a minor over 15 and under 18 at the time of the commission, he is entitled to a reduced penalty due to the privileged mitigating circumstance - Thus, penalty should be reduced to reclusion temporal. No fine is imposable in this case, for it is

a2010

page 90

Prof.
- January 13, 1995 - TC convicted the accused of murder - February 10, 1995 - both accused appealed to SC where accused questioned TCs failure (a) to hear the petition for bail (b) to consider defense of relative in favor of Ramil Manes and (c) to take note that Sergon Manes was a mere victim of Tamorite's unlawful aggression According to the prosecution > June 23, 1991 5 in the afternoon, ALAN Catequista with NICANOR Tamorite and JOSE Cubita, went to see a basketball game at the barangay plaza. When the game was over, Alan approached and invited Nicanor to go home; at that time, he was still seated. Accused RAMIL Manes approached Nicanor and pointed a 38 caliber revolver at him, saying "It is a bad luck you did not kill me during the fiesta in Barangay Cabayugan. Now I will be the one to kill you." Nicanor ran to Alan and used him as a shield from Ramil. At that point, Alan heard a thud and as he looked back, he saw accused SERGON Manes with a gory knife and he also saw Nicanor running away, with blood on his back. Ramil Manes pursued Nicanor and shot him hitting him at the back, just above the waistline. Both accused continued to chase Nicanor who ran towards the premises of the house of ADING Ablado. Ramil Manes fired two more shots. It could not be determined whether those shots hit Nicanor as he and the accused were already inside the premises of the fence of Ading. Jose who was near Nicanor when the two accused chased him did not render assistance to him. After Alan heard the two shots, he and Jose ran home. Alan told his father and uncle that Sergon stabbed Nicanor and that Ramil shot him. Alan, his father, uncle, Jose and the mother of Nicanor then went to where the body of Nicanor was in the downhill portion of the premises of the house of Ading. Nicanor was lying on his back, with 2 wounds on the breast, 1 gunshot wound and 1 stab wound. According to the accused(Ramil) > June 23, 1991 in the afternoon, he was at home cooking. At around 5:00 to 5:30, he heard shouts coming from the direction of the barangay basketball court, which was about ten (10) meters away from his house. He went to the window to check what it was. He saw his younger brother Sergon lying on the concrete pavement and several persons were ganging up on him, three of whom he identified as Nicanor, Alan and Jose. They kept on boxing and kicking his brother prompting him to come to the latter's aid. On his way out, he saw a gun on top of

imposed as a conjunctive penalty only if the penalty is reclusion perpetua to death. Dispositive Petition AFFIRMED with modification.

PEOPLE v MANES 303 SCRA 231 PARDO; February 17, 1999


NATURE An appeal taken by accused Sergon Manes and Ramil Manes from the judgment of RTC Iloilo City, convicting them of murder and sentencing them to each "suffer the penalty of reclusion perpetua with the accessory penalties as provided in Article 41 of the Revised Penal Code" and "to indemnify the family of their victim in the amount of P50,000.00 plus P21,250.00 as expenses for the burial, wake and other related matter and to pay the costs. FACTS - July 12, 1991, Provincial Prosecutor of Iloilo Province filed with RTC Iloilo City, an INFORMATION charging the accused with MURDER: "That on or about the 23rd of June, 1991, in the Municipality of Badiangan, Province of Iloilo, Philippines, and within the jurisdiction of this Honorable court, the above-named accused, conspiring, confederating and mutually helping one another to better realize their purpose armed with a knife and a .38 caliber revolver respectively, with treachery and/or evident premeditation, did then and there wilfully, unlawfully, and feloniously assault, attack, stab and shot Nicanor Tamorite with the knife and .38 caliber revolver with which they were then provided, inflicting upon the said Nicanor Tamorite stab wounds and gun shot wounds on the different parts of his body which caused his death immediately thereafter." - prosecution recommended NO BAIL for the provisional liberty of the accused. - July 22, 1991 - TC issued a WARRANT OF ARREST against the accused - October 18, 1991 TC ordered the case ARCHIVED for failure to locate the two accused - June 24, 1992 - Sergon and Ramil Manes were ARRESTED in Romblon, Romblon - September 17, 1992 - Upon ARRAIGNMENT, both accused pleaded NOT GUILTY to the information - August 25, 1992 - accused filed a PETITION FOR BAIL which was opposed by the prosecution. TC did not hear the petition for bail. Neither did the accused invoke the right to bail at any stage of the trial.

Criminal Procedure Rowena Daroy Morales


the table and brought it with him to the basketball court. > While on his way to the basketball court, Ramil fired a warning shot to prevent Nicanor from stabbing his brother Sergon. Nicanor persisted in the pursuit of Sergon, with a knife in his hand. Sergon was about three meters ahead of Nicanor who was about ten meters ahead of the pursuing Ramil. Ramil fired another shot that hit Nicanor who,, fell to the ground. Meanwhile, Sergon managed to flee. Ramil also fled to the direction of the sugarcane field as soon as he fired the second shot because he saw the group of Alan approaching,, armed with guns .12 Ramil and his brother Sergon went into hiding and only surfaced a year later when they were arrested in Romblon. - prosecutions set of facts was favored by the court ISSUE WON petitioner has a right to bail HELD NO Ratio When an accused is charged with a capital offense, or an offense punishable by reclusion perpetua, or life imprisonment or death, and evidence of guilt is strong, bail must be denied, as it is neither a matter of right nor of discretion Reasoning - In offenses punishable by reclusion perpetua, life imprisonment or death, the accused has no right to bail when evidence of guilt is strong. The court must hear a petition for bail to determine whether the evidence of guilt is strong before deciding to grant or deny bail to the accused. While the accused can apply for bail and have the court hear his application summarily and promptly, such right may be waived expressly or impliedly. In this case, the trial court proceeded to try the case without resolving the petition for bail that appellants filed. However, the latter did not call the attention of the trial court to their unresolved application for bail. It was only in the appeal that they raised this issue. Thus, for failure to bring to the attention of the trial Court at the earliest opportune time, appellants are deemed to have waived their right to bail. - defense of relative: FAILED TO PROSPER because 1) unlawful aggression, the essential element to defense of relative is absent because if it were true that Sergon was being attacked, he would have suffered injuries. 2) if indeed he acted in defense of his younger brother Sergon who was then under attack, he would

a2010

page 91

Prof.
ISSUE - WON the Court of Appeals acted with grave abuse despite a showing by the prosecutor that there is strong evidence proving respondents guilt for the crime charged. HELD YES - The SC held that the CA and the lower court failed to mention and include some facts which are significant factors and circumstances which are strong, clear, and convincing. Consideration of the said factors and circumstances would have resulted in the denial of bail. Reasoning - Article III, Section 13 of the Bill of Rights provides: All persons, except those charged with offenses punishable by reclusion perpetua when evidence of guilt is strong, shall before conviction, be bailable by sufficient sureties, or be released on recognizance as may be provided by law. the right to bail shall not be impaired even when the privilege of the writ of habeas corpus is suspended. Excessive bail shall not be required. - Section 7 Rule 4 of the Rules of court provides: No person charged with a capital offense, or an offense punishable by reclusion perpetua or life imprisonment, when the evidence of guilt is strong, shall be admitted to bail regardless of the stage of criminal prosecution. - In the case at bar, bail is discretionary and not a matter of right considering that the punishment for the offense is reclusion perpetua. the grant of the bail is dependent on the evidence of the guilt which should which should be strong to justify denial. this determination is a matter of judicial discretion. - By judicial discretion, the law mandates the determination of whether proof is evident or the presumption of guilt is strong. Proof evident or evident proof is this connection, has been held to mean clear, strong evidence which leads a well guarded dispassionate judgment to the conclusion that an offense has been committed as charged, that the accused is the guilty agent, and that he will probably be punished capitally if the law is administered. Presumption great exists when the circumstances testified to are such that the inference of guilt naturally to be drawn therefrom is strong, clear, and convincing to an unbiased judgment and excludes all reasonable probability of any other conclusion. In other words, the test is not whether

not harbor any fear in presenting himself to the proper authorities. - even though prosecution failed to show evident premeditation, trial court correctly considered treachery as qualifying the killing of the victim to murder. Dispositive we AFFIRM the judgment of the trial court convicting accused-appellants Sergon Manes and Ramil Manes of murder and sentencing each of them to suffer the penalty of reclusion perpetua with the accessory penalties of the law and to indemnify the heirs of the deceased Nicanor Tamorite in the amount of P50,000.00, plus P21,250.00, as actual damages.

PEOPLE v CABRAL 303 SCRA 361 ROMERO; February 18, 1999


NATURE Special Civil Action FACTS - Roderick Odiamar was charged with the rape of 15 year old Cecille Buenafe. In a bid to secure temporary liberty, the accused filed a motion for bail which was opposed by the petitioner. - The lower court grated the motion on the ground that despite the crime alleged to have been committed is punishable by reclusion perpetua, the evidence thus far presented is not strong enough to warrant denial of the bail. The judge in concluding thus cited the fact that the girl went with the offender voluntarily and did not resist during the commission of the rape. In addition, the judge quoted the medico legal report as not conclusion that rape was in fact committed consideration that the lacerations on the victim may have been weeks or months old when the medical examination was performed six days after the offense occurred. - The CA affirmed the decision saying that there was no abuse of discretion in this case. There is grave abuse of discretion where the power is exercised in an arbitrary or despotic manner by reason of passion, prejudice, or personal hostility amounting to an evasion of positive duty or to a virtual refusal to perform the duty enjoined or to act at all in contemplation of the law. The People filed the appeal on the ground that while the judge had discretion on the grant of bail, he had abused this discretion.

Criminal Procedure Rowena Daroy Morales


the evidence establishes guilt beyond reasonable doubt but rather whether it shows evident guilt or a great presumption of guilt. - In the case of an application for bail, the duties of the judge are as follows: 1. Notify the prosecutor of the hearing of the application for bail or require him to submit his recommendation; 2. Conduct a hearing of the application for bail regardless of whether or not the prosecution refuses to present evidence to show that the guilt of the accused is strong for the purpose of enabling the court to exercise its discretion 3. Decide whether the evidence of guilt of the accused is strong based on the summary of evidence of the prosecution 4. If the guilt of the accused is not strong, discharge the accused upon the approval of the bail bond. Otherwise the petition should be denied. - Based on the duties, the courts order granting or denying bail must contain a summary of the evidence for the prosecution. A summary is defined as a comprehensive and usually brief abstract or digest of a test or statement. HENCE, THE SUMMARY SHOULD NECESSARILY BE A COMPLETE COMPILATION OR RESTATEMENT OF ALL THE PIECES OF EVIDENCE PRESENTED DURING THE HEARING PROPER. The Lower court cannot exercise judicial discretion as to what pieces of evidence should be included in the summary. Otherwise, the same will be considered defective in form and substance which cannot be sustained or be given a semblance of validity. Dispositive Grant of bail is declared void. The court should issue a warrant of arrest of Odiamar if his bail bond has been approved.

a2010

page 92

Prof.
d) Approval of the bail bonds shall be made only after the arraignment to enable this Court to immediately acquire jurisdiction over the accused; - Petitioner filed a motion to quash the informations against him. Pending resolution of his motion, he asked the trial court to suspend the arraignment scheduled. He then filed a motion in which he prayed that the amounts of bail bonds be reduced to P40,000.00 for each case and that the same be done prior to his arraignment. - Trial court denied petitioners motions to reduce bail bonds, to quash the informations, and to suspend arraignment. - Petitioner was arraigned during which he pleaded not guilty to the charges against him and then ordered him released upon posting bail bonds in the total amount of P800,000.00, subject to the conditions and the "hold-departure" order. The pretrial conference was set. - Petitioner filed a petition for certiorari in CA, assailing the trial courts orders. - While the case was pending in the Court of Appeals, two more informations were filed against petitioner, bringing the total number of cases against him to 12, which were all consolidated. - CA: a) The accused shall not be entitled to a waiver of appearance during the trial of these cases. He shall and must always be present at the hearings of these cases; b) In the event that he shall not be able to do so, his bail bonds shall be automatically cancelled and forfeited, warrants for his arrest shall be immediately issued and the cases shall proceed to trial in absentia;] - CA invalidated the first two conditions imposed in the May 16, 1997 order for the grant of bail to petitioner but ruled that the issue concerning the validity of the condition making arraignment a prerequisite for the approval of petitioners bail bonds to be moot and academic. It noted "that petitioner has posted the cash bonds; that when arraigned, represented by lawyers, he pleaded not guilty to each offense; and that he has already been released from detention." CA thought that the aforesaid conditions in the May 16, 1997 order were contrary to Art. III, 14(2) of the Constitution which provides that "[a]fter arraignment, trial may proceed notwithstanding the absence of the accused provided that he has been duly notified and his failure to appear is unjustifiable." - With respect to the denial of petitioners motion to quash the informations against him, CA held that petitioner could not question the same in a petition

SEPARATE OPINION VITUG [dissenting]


- The extraordinary remedies under Rule 65 of the rules of Court are not open when the question is whether the trial judge has erred in the exercise of sound discretion. These special reliefs are available only when the judge has committed grave abuse of discretion amounting to lack or excess of jurisdiction in his decision or order such as by arbitrarily ignoring the evidence or completely acting on bias and whim.

LAVIDES v CA (PISON and PEOPLE) 324 SCRA 321 MENDOZA; February 1, 2000

FACTS - Lavides was arrested for child abuse under R.A. 7610. His arrest was made without a warrant as a result of an entrapment conducted by the police. - Parents of complainant Lorelie San Miguel reported to the police that their daughter, then 16 years old, had been contacted by petitioner for an assignation that night at petitioners room at the Metropolitan Hotel. This was not the first time the police received reports of petitioners activities. An entrapment operation was therefore set in motion. The police saw him with Lorelie, who was wearing only a shirt and an underwear, whereupon they arrested him. Based on the sworn statement of complainant and the affidavits of the arresting officers, which were submitted at the inquest, an information for violation of Art. III, 5(b) of R.A. 7610 was filed. - Petitioner filed an "Omnibus Motion (1) For Judicial Determination of Probable Cause; (2) For the Immediate Release of the Accused Unlawfully Detained on an Unlawful Warrantless Arrest; and (3) In the Event of Adverse Resolution of the Above Incident, Herein Accused be Allowed to Bail as a Matter of Right under the Law on Which He is Charged." - Nine more informations for child abuse were filed against petitioner by the same complainant, Lorelie San Miguel, and by three other minor children, Mary Ann Tardesilla, Jennifer Catarman, and Annalyn Talingting. In all the cases, it was alleged that, on various dates mentioned in the informations, petitioner had sexual intercourse with complainants who had been "exploited in prostitution and . . . given money [by petitioner] as payment for the said [acts of] sexual intercourse." - No bail was recommended. Nonetheless, petitioner filed separate applications for bail in the nine cases. - Trial court granted the right to post bail in the amount of P80,000.00 for each case or a total of P800,000.00 for all the cases under the following conditions: a) The accused shall not be entitled to a waiver of appearance during the trial of these cases. He shall and must always be present at the hearings of these cases; b) In the event that he shall not be able to do so, his bail bonds shall be automatically cancelled and forfeited, warrants for his arrest shall be immediately issued and the cases shall proceed to trial in absentia; c) The hold-departure Order of this Court dated April 10, 1997 stands; and

Criminal Procedure Rowena Daroy Morales


for certiorari before it, but what he must do was to go to trial and to reiterate the grounds of his motion to quash on appeal should the decision be adverse to him. ISSUE WON CA erred in not determining the validity of the conditions imposed in the trial courts order of May 16, 1997 for the grant of bail.. HELD YES - CA should have determined the validity of the conditions imposed in the trial courts order of May 16, 1997 for the grant of bail because petitioners contention is that his arraignment was held in pursuance of these conditions for bail. - Bail should be granted before arraignment, otherwise the accused may be precluded from filing a motion to quash. For if the information is quashed and the case is dismissed, there would then be no need for the arraignment of the accused. In the second place, the trial court could ensure the presence of petitioner at the arraignment precisely by granting bail and ordering his presence at any stage of the proceedings, such as arraignment. Under Rule 114, 2(b) of the Rules on Criminal Procedure, one of the conditions of bail is that "the accused shall appear before the proper court whenever so required by the court or these Rules," while under Rule 116, 1(b) the presence of the accused at the arraignment is required. - On the other hand, to condition the grant of bail to an accused on his arraignment would be to place him in a position where he has to choose between (1) filing a motion to quash and thus delay his release on bail because until his motion to quash can be resolved, his arraignment cannot be held, and (2) foregoing the filing of a motion to quash so that he can be arraigned at once and thereafter be released on bail. These scenarios certainly undermine the accuseds constitutional right not to be put on trial except upon valid complaint or information sufficient to charge him with a crime and his right to bail. [Under Art. III, 5 of R.A. 7610, the offenses with which petitioner is charged are punishable by reclusion temporal in its medium period to reclusion perpetua.] - It is the condition in the May 16, 1997 order of the trial court that "approval of the bail bonds shall be made only after arraignment," which CA should instead have declared void. The condition imposed in the trial courts order of May 16, 1997 that the

a2010

page 93

Prof.
PEOPLE v MARK JIMENEZ G.R. No. 148571 PANGANIBAN; September 24, 2002
NATURE Petition for certiorari praying for the lifting of the bail Order, the cancellation of the bond, and the taking of Jimenez into legal custody FACTS - The US govt through diplomatic channels sent to the Phil. govt a note requesting the extradition of Mark B. Jimenez, also known as Mario Batacan Crespo. Upon receipt of the Notes and documents, the secretary of foreign affairs (SFA) transmitted them to the secretary of justice (SOJ) for appropriate action, pursuant to Section 5 of the Extradition Law. - Upon learning of the request for his extradition, Jimenez sought and was granted a TRO by the RTC of Manila, which prohibited the DOJ from filing with the RTC a petition for his extradition. The TRO was assailed byt the Sec. of Justice. Initially, the court dismissed the petition but after acting upon the motion for reconsideration, it reversed its earlier decision. It held that Jimenez was bereft of the right to notice and hearing during the evaluation stage of the extradition process. - Finding no more legal obstacle, the US govt, represented by the Philippine DOJ, filed with the RTC the appropriate Petition for Extradition. The Petition alleged that Jimenez was the subject of an arrest warrant issued by the US District Court for the Southern District of Florida on April 15, 1999. The warrant had been issued in connection with the following charges: (1) conspiracy to defraud the United States and to commit certain offenses; (2) tax evasion; (3) wire fraud (4) false statements, and (5) illegal campaign contributions. - In order to prevent the flight of Jimenez, the Petition prayed for the issuance of an order for his "immediate arrest" pursuant to Section 6 of PD No. 1069 - Before the RTC could act on the Petition, Respondent Jimenez filed before it an "Urgent Manifestation/Ex-Parte Motion,"which prayed that application for an arrest warrant be set for hearing. RTC grantes the motion of Jimenez. In that hearing, he manifested its reservations on the procedure adopted by the trial court allowing the accused in an extradition case to be heard prior to the issuance of a warrant of arrest. After the hearing, the court a

accused cannot waive his appearance at the trial but that he must be present at the hearings of the case is valid and is in accordance with Rule 114. For another condition of bail under Rule 114, 2(c) is that "The failure of the accused to appear at the trial without justification despite due notice to him or his bondsman shall be deemed an express waiver of his right to be present on the date specified in the notice. In such case, trial shall proceed in absentia." - Art. III, 14(2) of the Constitution authorizing trials in absentia allows the accused to be absent at the trial but not at certain stages of the proceedings, to wit: (a) at arraignment and plea, whether of innocence or of guilt,9 [Rule 116, 1(b)] (b) during trial whenever necessary for identification purposes, and (c) at the promulgation of sentence, unless it is for a light offense, in which case the accused may appear by counsel or representative.11 [Rule 120, 6.] At such stages of the proceedings, his presence is required and cannot be waived. - Although this condition is invalid, it does not follow that the arraignment of petitioner on May 23, 1997 was also invalid. Contrary to petitioners contention, the arraignment did not emanate from the invalid condition that "approval of the bail bonds shall be made only after the arraignment." Even without such a condition, the arraignment of petitioner could not be omitted. In sum, although the condition for the grant of bail to petitioner is invalid, his arraignment and the subsequent proceedings against him are valid. - Petitioner concedes that the rule is that the remedy of an accused whose motion to quash is denied is not to file a petition for certiorari but to proceed to trial without prejudice to his right to reiterate the grounds invoked in his motion to quash during trial on the merits or on appeal if an adverse judgment is rendered against him. However, he argues that this case should be treated as an exception. He contends that the Court of Appeals should not have evaded the issue of whether he should be charged under several informations corresponding to the number of acts of child abuse allegedly committed by him against each of the complainants. Dispositive The decision of the Court of Appeals is SET ASIDE and another one is RENDERED declaring the orders dated May 16, 1997 and May 23, 1997 of the Regional Trial Court, Branch 107, Quezon City to be valid, with the exception of condition (d) in the second paragraph of the order of May 16, 1997 (making arraignment a prerequisite to the grant of bail to petitioner), which is hereby declared void.

Criminal Procedure Rowena Daroy Morales


quo required the parties to submit their respective memoranda. In his Memorandum, Jimenez sought an alternative prayer: that in case a warrant should issue, he be allowed to post bail in the amount of P100,000. The alternative prayer of Jimenez was also set for hearing. Thereafter, the court below issued its questioned July 3, 2001 Order, directing the issuance of a warrant for his arrest and fixing bail for his temporary liberty at one million pesos in cash. After he had surrendered his passport and posted the required cash bond, Jimenez was granted provisional liberty. Hence, this Petition. ISSUE WON he is entitled to bail and to provisional liberty while the extradition proceedings are pending HELD NO Ratio. After being taken into custody, potential extraditees may apply for bail. Since the applicants have a history of absconding, they have the burden of showing that (a) there is no flight risk and no danger to the community; and (b) there exist special, humanitarian or compelling circumstances. The grounds used by the highest court in the requesting state for the grant of bail therein may be considered, under the principle of reciprocity as a special circumstance. In extradition cases, bail is not a matter of right; it is subject to judicial discretion in the context of the peculiar facts of each case. Article III, Section 13 of the Constitution, is worded as follows: Art. III, Sec. 13. All persons, except those charged with offenses punishable by reclusion perpetua when evidence of guilt is strong, shall, before conviction, be bailable by sufficient sureties, or be released on recognizance as may be provided by law. The right to bail shall not be impaired even when the privilege of the writ of habeas corpus is suspended. Excessive bail shall not be required." - the constitutional right to bail "flows from the presumption of innocence in favor of every accused who should not be subjected to the loss of freedom as thereafter he would be entitled to acquittal, unless his guilt be proved beyond reasonable doubt." It follows that the constitutional provision on bail will not apply to a case like extradition, where the presumption of innocence is not at issue. - That the offenses for which Jimenez is sought to be extradited are bailable in the United States is not an argument to grant him one in the present case. To stress, extradition proceedings are separate and

a2010

page 94

Prof.
Art.III Sec 5, par (a), subpar (5) of RA 7610 8 stating that, said Fitzgerald, actuated by lust, and by the use of drugs willfully, unlawfully and feloniously induced complainant AAA, a minor, 13 years of age, to engage in prostitution by then and there showering said AAA with gifts, clothes and food and thereafter having carnal knowledge of her in violation of the aforesaid law and to her damage and prejudice. - RTC rendered a decision finding respondent Guilty of Sec 5, par (a), subpar (5) of RA 7610 and sentenced to an indeterminate term of prision mayor min (8Y 1D) to prision temporal max (17Y 4M 1D). Upon completion of his sentence, he shall be deported immediately and forever barred from entry to the Philippines. He was acquitted on the case of rape. On the basis of the evidence adduced, the court considered the view that the the circumstances of the accused indicate a probability of flight and that there is undue risk that the accused may commit a similar offense, if released on bail pending appeal. - On appeal, CA affirmed the conviction, modifying the penalty to imprisonment of prision temporal (14Y 8M 1D) to reclusion perpetua (20Y 1D) - Fitzgerald filed for a Motion for a New Trial and a supplemental appeal to the motion on the ground that new material of evidence not previously available has surfaced. CA granted the motion for new trial. The original records of the case were remanded to the RTC, which was also directed to receive new evidence. The motion to transfer the respondent to the National Penitentiary was denied. - The people file a MFR, while Fitzgerald filed a Motion to fix bail with Manifestation. Both motions were denied by CA. The bail application was denied pursuant to Sec.7 Rule 114 ROC. The maximum penalty imposable in accordance with RA 7610 is reclusion perpetua and the evidence of guilt is strong. - The people filed a petition fro review on certiorari. It was dismissed which became final and executory. - Fitzgerald filed with CA a Motion for Early Transmittal of the Records and for the ReExamination of the Penalty Imposed and a Motion for Bail. CA issued the assailed resolution granting bail. It stated that although the evidence of guilt is strong, Fitzgerald is of old age and not in the best of health. Bail was granted premised not on the grounds stated in the motion for bail, but on substantial justice and considering new trial was granted in the case.
8

distinct from the trial for the offenses for which he is charged. He should apply for bail before the courts trying the criminal cases against him, not before the extradition court. - Also, we cannot allow our country to be a haven for fugitives, cowards and weaklings who, instead of facing the consequences of their actions, choose to run and hide. Hence, it would not be good policy to increase the risk of violating our treaty obligations if, through overprotection or excessively liberal treatment, persons sought to be extradited are able to evade arrest or escape from our custody. In the absence of any provision -- in the Constitution, the law or the treaty -- expressly guaranteeing the right to bail in extradition proceedings, adopting the practice of not granting them bail, as a general rule, would be a step towards deterring fugitives from coming to the Philippines to hide from or evade their prosecutors. > - The denial of bail as a matter of course in extradition cases falls into place with and gives life to Article 14 of the Treaty, since this practice would encourage the accused to voluntarily surrender to the requesting state to cut short their detention here. Likewise, their detention pending the resolution of extradition proceedings would fall into place with the emphasis of the Extradition Law on the summary nature of extradition cases and the need for their speedy disposition. Dispositive the Petition is GRANTED. The bail bond posted by private respondent is CANCELLED. The Regional Trial Court of Manila is directed to conduct the extradition proceedings before it, with all deliberate speed pursuant to the spirit and the letter of our Extradition Treaty with the United States as well as our Extradition Law.

PEOPLE v FITZGERALD 505 SCRA 573 AUSTRIA-MARTINEZ; October 27, 2006


NATURE Petition for Review on Certiorari assailing the resolution of CA which granted the Motion for bail of accused-appellant and herein respondent Victor Keith Fitzgerald. FACTS - An information was filed in the RTC charging Fitzgerald, an Australian citizen, with the violation of

Special Protection of Children against Child Abuse, Exploitation and Discrimination Act

Criminal Procedure Rowena Daroy Morales


- RTC ordered Fitzgeralds temporary release upon filing a cash bond of P100 000.00. Petitioners Claim > People filed this petition to annul the CA Resolution arguing that the CA erred in granting the Motion for Bail despite the crime charged was punishable by reclusion perpetua and the evidence of guilt is strong. Plaintiffs Claim > the grant for new trail negated the previous findings of the existence of strong evidence of guilt. The justification for provisional release is on humanitarian grounds, citing his deteriorating health and old age. ISSUES 1. WON CA had jurisdiction over the motion to post bail after issuing the resolution granting new trial 2. WON CA erred in allowing bail HELD 1. (the ruling on this matter is limited to this specific case) When the SC grants new trial, it vacates the judgment of the TC convicting the accused and remands the case to the TC for reception of newlydiscovered evidence and promulgation of a new judgment. However, when CA grants new trial, notwithstanding Sec1 Rule 125 ROC providing for the uniformity of the procedure between the SC and CA, CA may decide questions of fact and of law. When it grants a new trial pursuant to Sec14 Rule 124 ROC, it may either a) receive the new evidence under Sec 12 or b) refer the case to the court of origin for reception of such evidence under Sec 15. in either case, it does not relinquish to the TC jurisdiction over the case. It retains sufficient authority to resolve the incidents in the case and decide its merits. - Even when CA remanded the case to the TC, CA retained appellate jurisdiction. CA retained its authority to act on the respondents bail application. 2. The right to bail emanates from the right to be presumed innocent. It is accorded to a person in the custody of law who may by reason of the presumption of innocence he enjoys, be allowed provisional liberty upon filing of a security to guarantee his appearance before any court, as required under specific conditions. - Bail is a matter of right to an accused person in custody for an offense not punishable by death, reclusion perpetua or life imprisonment, but a matter of discretion on the part of the court, concerning one facing an accusation for an offense punishable by

a2010

page 95

Prof.
witnesses for the prosecution; all of the other defendants were present, were duly arraigned, pleaded not guilty, and were represented by a lawyer; that after the prosecution had closed its case against all of the said defendants except Rufino Lavarias, the court discovered that Rufino Lavarias was outside the court room. Upon discovering this fact the court ordered the said defendant Rufino Lavarias to appear in the court room, and then and there recalled one of the witnesses for the prosecution, Regino Maminta, and proceeded to examine him with reference to the part Rufino Lavarias took in the said robbery charged in the complaint, without arraigning the said Rufino Lavarias, reading to him the complaint, or informing him that he had the right to be represented by an attorney during the trial, in accordance with the provisions of sections 16, 17, and 18 of General Orders, No. 58. ISSUE WON the court did not comply with the provisions of sections 16, 17, and 18 of General Order No. 58 HELD YES. Rufino Lavarias was not arraigned. Dispositive The judgment of the court with reference to Rufino Lavarias is reversed, and the cause is remanded to the CFI Pangasinan for the purpose of a new trial. The court is affirmed as to the sentence of Fabiano Diadib and Inocencio Valerio on the ground of sufficiency of evidence.

death, reclusion perpetua or life imprisonment when evidence of guilt is strong. 9 - RTC and CA were unanimous in their findings of the existence of strong evidence of guilt. Under Sec 6(b) Rule 121, the grant of a new trial allows for reception of newly discovered evidence, but maintains evidence already presented or on record. In the present case, no new evidence had been introduced negating the earlier findings of the RTC and CA. Bail was not a matter of right but a mere privilege subject to the discretion of CA. - However, the CA admitted that the bail was based on health reasons disregarding the substantive and procedural requirements on bail. - CA made no specific findings that the respondent suffered from an ailment of such gravity that his continued confinement during trial will permanently impair his health or put his life in danger. - Moreover, there is finding on the record on the potential risk of respondent committing a similar offense. Dispositive petition is granted and the CA resolution annulled and set aside. The bail bond posted is cancelled. Let an order of arrest issue against the person of the accused.

US v PALISOC 4 Phil 207 JOHNSON; February 13, 1905


FACTS The defendants were charged with the crime of robbery for entering the house of one Regino Maminta, all being armed with talibones took and carried carried away the sum of 20 pesos, Mexican, and various pieces of jewelry, all of the value of 120 pesos, Mexican. The said defendants were tried and each of the defendants, Paulino Palisoc, Fabiano Diadib, Inocencio Valerio, and Domingo Torres, was sentenced to be imprisoned for the period of five years of presidio correccional; and Rufino Lavarias, because the court found that he was the leader of the said band of robbers, was sentenced to a period of six years of presidio correccional. From this sentence Fabiano Diadib, Inocencio Valerio, and Rufino Lavarias appealed to this court. The record shows that at the beginning of the trial Rufino Lavarias was not present and did not appear in court until after the fiscal had presented all of the
9

PEOPLE v AMBROSIO 56 Phil 801 PEOPLE v CARIAGA 64 Phil 1057 CONCEPCION; June 29, 1937
NATURE Appeal from the judgment of the CFI of Manila FACTS - Appellant Deogracias Cariaga was convicted of the crime of theft and sentenced to 1 month and 1 day of arresto mayor and to indemnify the offended party in the sum of P1.50. - Cariaga alleges that the court erred in having found him guilty of the crime based of the fact that the appealed judgment states: Upon arraignment, he entered a plea of guilty. The appellant without

Sec 4 and 5 Rule 114 ROC, and Sec 13 Art III 1987 Consti

Criminal Procedure Rowena Daroy Morales


denying the fact in itself, contends that the record does not show when, where or how he was arraigned. ISSUE WON the statement in the judgment that the accused has been arraigned and pleaded guilty is sufficient HELD YES - The statement is in compliance with the provisions of section 16&25 of Gen. Order No. 58, inasmuch as it may be presumed from said statement that the law has been obeyed by causing the accused to appear before the court, and it is shown thereby that he has really been arraigned, his plea entered personally being that of guilty. All this is not a mere conclusion as appellant contends, because the court is of the opinion that generally a conclusion is the averment or denial of a fact deduced from some evidence, or the averment or denial of a point of view of law derived from a law or a principle of law. In this case, however, the courts averment that the accused was arraigned and that he pleaded guilty, is not a deduction or consequence of an evidence, legal provision or principle of law, but a positive statement of facts. - It does not make a difference if the court, after hearing the accused plead guilty upon arraignment, stated such facts in the judgment rendered immediately, in open court and in the presence of the accused, or caused said arraignment and plea to be stated in the minutes. What is important is that the accused be arraigned and that he enters his plea. It is immaterial how or in what manner such facts are stated. For legal purposes, it makes no difference whether they appear in the minutes or in the judgment itself.

a2010

page 96

Prof.
comprehended the meaning, full significance and consequences of his plea. Dispositive. WHEREFORE, the judgment under automatic review is hereby SET ASIDE and the case REMANDED to the trial court for further proceedings. SO ORDERED.

a fisherman when the former stole their P80 and on the occasion killed the 2 victims. -the 2 were charged with Robbery with Double Homicide, with the aggravating circumstances of use of motorized banca as a means for flight or concealment, plus recidivism as regards Serna since he was previously convicted by final judgment in CFI of Manila, and was sentenced to an imprisonment from 10 to 17 years in 1958. -upon arraignment, both pleaded guilty, invoking the mitigating circumstance of plea of guilt. Immediately, CFI of Samar found the 2 guilty, considering the mitigating circumstance of plea of guilt and the aggravating circumstance cited above. Cipriano was sentenced to Reclusion Perpetua (MC offset AC) while Serna was sentenced to death (1MC to 2AC), therefore sent to SC for automatic review. -the information was read to the appellants in English and translated in Samar dialect, thereafter, the 2 pleaded guilty. After the plea, the Fiscal asked the plea of guilty to be considered mitigating then asked the court to consider the 2 aggravating circumstance against Serna. The court did not explain the import of the plea to the 2 accused, did not even bother to ask if they understood their plea, and just imposed on Serna the death penalty. ISSUE WON the trial court erred in automatically rendering the penalty of death to Serna HELD NO. Ratio. Considering that the appellant was charged with an offense punishable by death, the trial court should have required the prosecution to present its evidence to prove the extent of his culpability. The taking of such testimony is the prudent and proper course to follow for the purpose of establishing not only the guilt but also the precise culpability of the defendant. Where a plea of guilty is entered by the defendant, in cases where the capital penalty may be imposed, the court should make certain that defendant fully understands the nature of the charge preferred against him and the character of the punishment provided by law before it is imposed. The trial court should therefore call witnesses for the purposes of establishing the guilt and degree of culpability of the defendant, not only to satisfy the trial judge, but also to aid the Supreme Court in determining whether the accused really and truly understood and

SEPARATE OPINION AQUINO [dissenting]


-Serna should be sentenced to reclusion perpetua because: (1) recidivism not aggravating, the information not alleging the prior crime for which Serna was convicted; (2) use of motorized banca not aggravating, it was a means of flight and not for the commission of the crime; (3) treachery generic aggravating but offset by plea of guilt; (4) fact of more than 1 homicide not aggravating, not enumerated under Art14, RPC. -Serna understood his plea because there was no improvident plea -dont need to remand, 14 years already passed

PEOPLE v TIONGSON 130 SCRA 614 CONCEPCION; July 25, 1984


NATURE Mandatory review for imposition of death penalty FACTS - Tiongson escaped from the Municipal Jail of Bulalacao, Oriental Mindoro, together with de la Cruz and Santiago, where they were detained under the charge of Attempted Homicide. While in the act of escaping, Tiongson killed a member of the police force who was guarding them and a PC Constable who went in pursuit. - By reason thereof, Tiongson was then charged with Murder, in two separate informations, alleging that the commission of the offense was qualified by the circumstance of treachery, and aggravated by the circumstances of evident premeditation, in contempt of or with insult to the public authorities, nocturnity, committed in an uninhabited place and with abuse of superior strength. - Upon arraignment, the said accused, assisted by counsel de oficio, pleaded guilty to both informations. The trial court did not render judgment outright, but ordered the prosecution to present its evidence, after which, it sentenced the said accused

PEOPLE v SERNA 130 SCRA 550 CONCEPCION, July 25, 1984


NATURE Automatic review to the Supreme Court FACTS -In Samar during the night of November 28, 1970, Rafael Serna and Antonio Cipriano took away P80 from Romualdo Villones and Leonardo Carlos. The latter were paying for the fish that they bought from

Criminal Procedure Rowena Daroy Morales


to suffer the death penalty in each case, and to indemnify the heirs of the victims. ISSUES 1. WON a plea of guilt is always binding upon the accused for all the contents of the information 2. WON the killing was qualified by treachery 3. WON there were aggravating circumstances present HELD 1. NO. Ratio It may be true that a judicial confession of guilt admits all the material facts alleged in the information, including the aggravating circumstances listed therein, as stated by the trial judge, yet where there has been a hearing and such circumstances are disproven by the evidence, they should be disallowed in the judgment. Reasoning The norm that should be followed where a plea of guilty is entered by the defendant, especially in cases where the capital penalty may be imposed, is that the court should be sure that defendant fully understands the nature of the charges preferred against him and the character of the punishment provided by law before it is imposed. For this reason, the Court requires that in every case under a plea of guilty, where the penalty may be death, the trial court should call witnesses for the purpose of establishing the guilt and degree of culpability of the defendant and not only to satisfy the trial judge but to aid the Supreme Court in determining whether accuse understood and comprehended the meaning, full significance and consequences of his plea. In the instant case, the trial judge required the taking of testimony as to the circumstances under which the crime was committed before passing judgment so that the resulting verdict cannot in any way be branded as deficient. 2. NO. Reasoning The circumstances qualifying or aggravating the act of killing a human being must be proved in an evident and incontestable manner, mere presumptions or deductions from hypothetical facts not being sufficient to consider them justified. - According to the RPC, "there is treachery when the offender commits any of the crimes against the person, employing means, methods, or forms in the execution thereof which tend directly and specially to insure its execution, without risk to himself arising from the defense which the offended party might make."

a2010

page 97

Prof.
MENDOZA; February 2, 2000
NATURE Automatic review of the decision of the Regional Trial Court of Oriental Mindoro FACTS -On April 28, 1996, Oleby and Maricris, assisted by a neighbor, Lita Macalalad, told their mother that they had been raped by their father, herein accusedappellant. Thereupon, they went to the police authorities of Naujan and filed a complaint against accused-appellant -After preliminary examination, on June 6, 1996, four informations charging accused-appellant with rape on various dates were filed in the Regional TrialCourt, Calapan, Oriental Mindoro. -The record shows that at his arraignment on July 23, 1996, accused-appellant, assisted by Atty. Manolo A. Brotonel of the Public Attorney's Office, pleaded not guilty to the charges filed against him. -However, on August 5, 1997, after the prosecution had presented Dr. Cynthia S. Fesalbon, accusedappellant pleaded guilty to the crime charged in all the informations. -On August 12, 1997, the prosecution formally offered its documentary evidence and rested its case thereafter. -Accused-appellant did not present any evidence in his defense. -On August 27, 1997, the trial court rendered judgment finding accused-appellant guilty of four counts of rape against his daughters. -Nadera appealed ISSUES 1. WON the trial court erred when it accepted his plea of guilty to a capital offense without making a searching inquiry to determine whether he understood the consequences of his plea 2. WON the conviction must be set aside HELD 1. YES - Rule 116 of the Rules on Criminal Procedure provides: Sec. 3. Plea of guilty to capital offense; reception of evidence. When the accused pleads guilty to a capital offense, the Court shall conduct a searching inquiry into the voluntariness and full comprehension of the consequences of his plea and require the prosecution to prove his guilt and the precise

- It does not appear how and in what position the victim was when he was killed so that it cannot be said for certain that the accused had adopted a mode or means of attack tending directly to insure or facilitate the commission of the offense without risk to himself arising from the defense or retaliation which the victim might put up. - Pat. Garcia of the Bulalacao police force merely declared that he was in his house, about 15 meters away from the municipal building when the accused Rudy Tiongson and his companions escaped from prison, and he did not see the accused shoot Pat. Gelera. Pat. Gelera was already dead when the other witness saw him. - Treachery is also not present in the killing of PC Constable since the deceased was actually warned by another PC not to remain standing but seek cover because of the known presence of the accused in the vicinity, but that the said deceased disregarded the warning. - Since treachery, which would qualify the killing of Pat. Gelera and PC Constable Canela to Murder, was not present, the crimes may only be punished as Homicide. 3. NO. Reasoning (a) Evident premeditation must be ruled out in view of the absence of sufficient proof that a plan to kill the victims existed, the execution of which was preceded by deliberate thought and reflection. (b) That the crimes were committed in contempt of or with insult to the public authorities cannot be appreciated since they are not persons in authority, but merely agents of a person in authority. (c) In order that commission of a crime in an uninhabited place may be considered, it is necessary that the place of occurrence be where there are no houses at all, a considerable distance from the village or town, or where the houses are a great distance apart. (d) Abuse of superior strength must also be ruled out since there is no direct evidence that the accused employed it. Dispositive Petition is affirmed with the modification that the accused Rudy Tiongson should be sentenced to suffer imprisonment of eight (8) years and one (1) day of prision mayor, as minimum, to fourteen (14) years and eight (8) months of reclusion temporal, as maximum, for each homicide committed by him. The indemnity to be paid to the heirs of the victims is hereby increased to P30,000.00 in each case.

PEOPLE v NADERA 324 SCRA 490

Criminal Procedure Rowena Daroy Morales


degree of culpability. The accused may also present evidence on his behalf. -Under this Rule, three things are enjoined upon the trial court when a plea of guilty to a capital offense is entered: (1) the court must conduct a searching inquiry into the voluntariness of the plea and the accused's full comprehension of the consequences thereof; (2) the court must require the prosecution to present evidence to prove the guilt of the accused and the precise degree of his culpability; and, (3) the court must ask the accused if he desires to present evidence on his behalf and allow him to do so if he desires. -As explained in People v. Alicando, a searching inquiry must focus on: (1) the voluntariness of the plea, and (2) the full comprehension of the consequences of the plea. -In the case at bar, the record does not show what exactly transpired at the re-arraignment of accusedappellant, for what reason he changed his plea from "not guilty" to "guilty," and whether he fully understood the consequences of his guilty plea. -In its decision, the trial court described the manner in which the accused pleaded guilty, thus: Upon arraignment, accused, assisted by Atty. Manolo A. Brotonel of the Public Attorney's Office, pleaded not guilty to the crime charged. However, when these cases were called for pre-trial and trial, counsel for the accused manifested that the accused, realizing the futility of entering into trial and considering that he actually committed the acts complained of, intimated his intention to enter a plea of guilty to the above-mentioned charges. The accused was then asked by this Court if he was aware of the consequences of a plea of guilty to a capital offense: that for the rape he committed on May 17, 1992 against his daughter, Oleby Nadera, who was 9 years old at the time, he would be sentenced to reclusion perpetua and for the three other counts of rape committed on April 17 and 24, 1995 [both against Oleby Nadera] and on March 3, 1996 [against Maricris Nadera, 11 years old at the time], he would be sentenced to death by lethal injection. After having been informed of this, he insisted that he is willing to enter a plea of guilty to the crimes charged and is ready to face the consequences thereof. -The warnings given by the trial court in this case fall short of the requirement that it must make a searching inquiry to determine whether accused-appellant understood fully the import of his guilty plea. As has been said, a mere

a2010

page 98

Prof.
their mother, Daisy, and the physician who conducted the medical examination of the two girls, Dr. Cynthia Fesalbon. Certain circumstances present in this case, however, persuade us that a remand of this case is necessary. -First, a perusal of the decision of the court reveals that the trial judge failed to state the factual and legal reasons on which he based accused-appellant's conviction. Except for the narration of the prosecution's evidence and a bare recital of R.A. No. 7659, amending Art. 335 of the Revised Penal Code, there is nothing else to indicate the reason for the decision. There is no evaluation of the evidence and no reason given why the court found the testimonies of the witnesses credible. -Second, the cavalier attitude of accused-appellant's counsel, Atty. Manolo A. Brotonel of the Public Attorney's Office, cannot go unnoticed. It is discernible in (a) his refusal to cross examine Oleby Nadera; (b) the manner in which he conducted Maricris Nadera's cross examination; and, (c) his failure not only to present evidence for the accused but also to inform the accused of his right to do so, if he desires. Dispositive The appealed decision is set aside.

warning that the accused faces the supreme penalty of death is insufficient. For more often than not, an accused pleads guilty upon bad advice or because he hopes for a lenient treatment or a lighter penalty. The trial judge must erase such mistaken impressions. He must be completely convinced that the guilty plea made by the accused was not made under duress or promise of reward. The judge must ask the accused the manner the latter was arrested or detained, and whether he was assisted by counsel during the custodial and preliminary investigations. In addition, the defense counsel should also be asked whether he conferred with the accused and completely explained to him the meaning and the consequences of a plea of guilt. Furthermore, since the age, educational attainment and socio-economic status of the accused may reveal insights for a proper verdict in the case, the trial court must ask questions concerning them. -In this case, absent any showing that these questions were put to accused-appellant, a searching inquiry cannot be said to have been undertaken by the trial court. -In People v. Sevillano, this Court held that: In every case where the accused enters a plea of guilty to a capital offense, especially where he is an ignorant person with a little or no education, the proper and prudent course to follow is to take such evidence as are available and necessary in support of the material allegations of the information, including the aggravating circumstances therein enumerated, not only to satisfy the trial judge himself but also to aid the Supreme Court in determining whether the accused really and truly understood and comprehended the meaning, full significance and consequences of his plea. -Clearly, the plea of guilty of accused-appellant in this case was made improvidently. 2. Yes. -Convictions based on an improvident plea of guilt are set aside only if such plea is the sole basis of judgment. If the trial court relied on sufficient and credible evidence to convict the accused, the conviction must be sustained, because then it is predicated not merely on the guilty plea of the accused but on evidence proving his commission of the offense charged. -The prosecution evidence consisted of the testimonies of Oleby and Maricris Nadera, the results of their medical examinations, and the testimonies of

PEOPLE v NAVARRO 75 Phil 516 BENGZON; December 4, 1945


FACTS - Information for arbitrary detention: "That from January 27, 1945, and for several days thereafter, in the municipality of Calapan, Province of Mindoro, Commonwealth of the Philippines, and within the jurisdiction of this Honorable Court, said defendants Juan Navarro and Anacleto Atienza, Acting Provincial Governor and Provincial Warden, respectively, both being public officials to whom the custody and responsibility of prisoners were entrusted for proper action, without any lawful or justifiable cause and without legal grounds therefor, did then and there wilfully, unlawfully and feloniously detain Esteban P. Beloncio in the Provincial Jail of Mindoro which continued for more than fifteen days but less than six months." - A pre-trial was held, the Judge asking the parties or their attorneys some questions, which the latter answered, with the result that admissions were made to the effect that Esteban P. Beloncio and Juan G. Beloncio II had been detained for several days after January 27, 1945, in the provincial jail of Mindoro by

Criminal Procedure Rowena Daroy Morales


order of the Commanding General, Western Visayan Task Force, United States Army. Whereupon, defendants' counsel lost no time in filing a motion to quash, upon the ground that the facts charged did not constitute a criminal offense. - The judge dismissed the cases. ISSUE WON the motion to quash on the ground that the facts charged do not constitute and offense must be restricted to the facts charged in the informations HELD NO - The section of the rule permitting a motion to quash on the ground that "the facts charged do not constitute an offense" omits reference to the facts detailed "in the information." Other sections of the same rule would imply that the issue is restricted to those alleged in the information. - Prima facie, the "facts charged" are those described in the complaint, but they may be amplified or qualified by others appearing to be additional circumstances, upon admissions made by the people's representative, which admissions could anyway be submitted by him as amendments to the same information. - It would seem to be pure technicality to hold that in the consideration of the motion, the parties and the judge were precluded from considering facts which the fiscal admitted to be true, simply because they were not described in the complaint. Of course, it may be added that upon similar motions the court and the fiscal are not required to go beyond the averments of the information, nor is the latter to be inveigled into a premature and risky revelation of his evidence. - But the Court sees no reason to prohibit the fiscal from making, in all candor, admissions of undeniable facts, because the principle can never be sufficiently reiterated that such official's role is to see that justice is done; not that all accused are convicted, but that the guilty are justly punished. Less reason can there be to prohibit the court from considering those admissions, and deciding accordingly, in the interest of a speedy administration of justice. - The Beloncios were thus deprived of their liberty by order of the military authorities, a few days after the liberations of Mindoro. Judicial notice may be taken of the fact, that upon military occupation and before the establishment of the normal processes of civil government the liberties and rights of citizens are likely to suffer temporary restrictions, what with the

a2010

page 99

Prof.
certificate, thereby directly incriminating or imputing to said Marcial Apolonio y Santos the commission of the crime of bribery." -defendants filed a motion to quash saying that (1) the facts charged in the information do not constitute an offense (because the two crimes cannot be complexed); and (2) the court trying the case has no jurisdiction over the offense charged -CFI granted motion to dismiss agreeing with defendants -MFR was denied -appeal by fiscal before SC ISSUE WON the CFI erred in granting motion to quash HELD YES - It is very apparent that by the use of the phrase "thru unlawful arrest" in the information an idea is conveyed that the unlawful arrest was resorted to as a necessary means to plant evidence in the person of the offended party, thereby incriminating him. From a reading of the info the SC finds a close connection between the act of the accused in first unlawfully arresting the offended party and then investigating him; and it was during that investigation that they plated incriminatory evidence against him. SC agrees with the Solicitor General in his contention that the accused first had to resort to unlawful arrest in order to be able to plant the P1.00 bill among the money taken from the offended party. Also the court a quo has jurisdiction to try the accused of the offense charged in the information. The crime of unlawful arrest is punishable with arresto mayor or imprisonment of from one month and one day to six months, and a fine not exceeding P500.00; 5 and the crime of incriminatory machinations is punishable with arresto mayor, or imprisonment of from one month and one day to six months. Dispositive The order appealed from is reversed and set aside

exigencies of military strategy, or the confusion usually resulting from the situation. While the infringement of constitutional precepts and privileges is not to be tolerated, war necessities and consequences cannot be overlooked. At any rate, no reasons are shown why the irregularity, if any, committed by others, should be visited upon defendants-appellees. The acts imputed to them, do not, of themselves, constitute a punishable offense. Dispositive Appealed decision affirmed.

PEOPLE v ALAGAO 16 SCRA 879 ZALDIVAR; April 30, 1966


NATURE This is an appeal by the City Fiscal of Manila from an order of the Court of First Instance of Manila sustaining the motion to quash the information FACTS -City Fiscal of Manila filed an information against the defendants-appellees charging them of the complex crime of incriminatory machinations thru unlawful arrest, as follows: "That on or about the 28th day of February, 1961, in the City of Manila, Philippines, the said accused, being then members of the Manila Police Department, conspiring and confederating together and helping one another, did then and there willfully, unlawfully and feloniously incriminate or impute to one Marcial Apolonio y Santos the commission of the crime by bribery thru unlawful arrest, in the following manner, to wit: the said accused, on the aforesaid date, without reasonable ground therefor and for the purpose of delivering said Marcial Apolonio y Santos to the proper authorities, did then and there willfully, unlawfully and feloniously arrest said Marcial Apolonio y Santos; that after the said Marcial Apolonio y Santos had been arrested in the manner aforestated, and while the latter was supposedly being investigated by the said accused, the said accused did then and there place on commingle a marked P1.00 bill together with the money taken from said Marcial Apolonio y Santos, supposedly given to the latter by one Emerita Calupas de Aresa, so that he (Marcial Apolonio y Santos), then an employee of the Local Civil Registrar's Office of Manila, would appear to have agreed to perform an act not constituting a crime, in connection with the performance of his (Marcial Apolonio y Santos) duties, which was to expedite the issuance of a birth

LOPEZ v CITY JUDGE [supra, page 41] GAMBOA v CRUZ 162 SCRA 642 PADILLA; June 27, 1988
NATURE

Criminal Procedure Rowena Daroy Morales


Peition for certiorari and prohibition to review the order of the Court of First Instance of Manila, Br. 29 FACTS Petitioner alleges that he was arrested for vagrancy without a warrant of arrest. Thereafter, he was brought to Precinct 2, Manila where he was booked for vagrancy and then detained therein together with several others. The next day, five detainees, including petitioner, complainant Bernal pointed to petitioner and said, that one is a companion. After the identification, the other detainees were brought back to their cell but petitioner was ordered to stay on. While the complainant was being interrogated by the police investigator, petitioner was told to sit down in front of her. Subsquently, an information for robbery was filed against petitioner. He was arraigned and thereafter hearings were held. The prosecution formally offered its evidence and then rested its case. Petitioner, by counsel, instead of presenting his defense, manifested in open court that he was filing a Motion to Acquit or Demurrer to Evidence. Petitioner filed said motion on the ground that the conduct of the line-up without notice to, and in the absence of, his counsel violated his constitutional rights to counsel and to due process. The respondent court issued an order denying the Motion to Acquit. Hence, the instant petition. ISSUE WON the respondent judge acted in excess of jurisdiction and with grave abuse of discretion in issuing the assailed order HELD NO If a defendant does not move to quash the complaint or information before pleading, defendant is deemed to have waived all objections which are grounds for a motion to quash, except where the complaint or information does not charge an offense, or the court is without jurisdiction of the same. Here, petitioner filed a Motion to Acquit only after the the prosecution presented its evidence and rested its case. Since the exceptions above-stated, are not applicable, petitioner is deemed to have waived objections which are grounds for a motion to quash. Besides, the grounds relied upon by petitioner in his Motion to Acquit are not among the grounds provided in Sec. 2, Rule 117 of the Rules of Court for quashing a complaint or information. Consequently, the lower court did not err in denying petitioner's Motion to Acquit.

a2010
Dispositive Petition dismissed.

page 100

Prof.
-that illegal possession of firearms and ammunition is still penalized under PD No. 1866 which was not repealed by said EO NO. 107 and 222. ISSUES 1. WON J. Asuncion erred in holding that the possession of loose firearms and explosives is not illegal per se during the period covered by EO Nos. 107 and 222 2. WON it was not necessary for the prosecution to allege in the information that the firearms and ammunition, subject matter of this case, were brought out of the residence of the accused or were used by him in the commission or another offense, since these circumstances are not essential ingredients of the crime of illegal possession of firearms and ammunition 3. WON under the allegation in the information, prosecution may prove that the accused earned the firearms and ammunition outside of his residence HELD 1. EO NO. 107, as amended by EO No. 222, is similar to RA Nos. 4 and 482. SC did NOT give it a different meaning because there is no basis for such a difference. 2. NO. IT IS NECESSARY TO ALLEGE IT IN THE INFO. 3. NO. The information, in this particular charge against Abadilla, is fatally defective. It would be fatally defective against any other accused charged with the same offense. J. Asunction, in dismissing the information, committed no reversible error or grave abuse of discretion. Ratio (citing People vs. Austria) the presentation of evidence "cannot have the effect of validating a void information, or proving an offense which does not legally exist. ... The information was not merely defective but it does not charge any offense at all. Technically speaking, that information does not exist in contemplation of law." -Abadilla is regarded with unusual ease and facility as the "hit man" of the Marcos regime. But the Court cannot be swayed by appellations for it has a duty, as a temple of justice, to accord to every man who comes before it in appropriate proceedings the right to due process and the equal protection of the laws. Reasoning 1. It may be true that there is nothing in EO Nos. 107 and 222 that expressly legalizes the unlicensed possession of firearms and ammunition, but this Court, applying statutes similar to the executive orders in question, and which also provided for a period within which a holder or possessor of

PEOPLE v ASUNCION 161 SCRA 490 PADILLA; May 24, 1988


NATURE Certiorari FACTS: -Rolando Abadilla, a former colonel of the Armed Forces of the Philippines, was charged before the QC RTC with the offense of Violation of PD No. 1866 [ILLEGAL POSSESSION OF FIREARMS AND AMMUNITION) --The Information read that he willfully, unlawfully and feloniously had in his possession and under his custody and control rifles, pistons, ammunitions and magazines (see orig case for the list) without first securing the necessary license and/or permit from the lawful authority. -Upon motion of the accused, Asuncion dismissed the Information on the ground that it did not allege sufficient facts to constitute an offense, since the possession of loose firearms and explosives is not illegal per se, in view of Executive Order No. 107 which gives holders or possessors of unlicensed firearms and ammunition a period of six months from its effectivity, extended to 31 December 1987 by EO No. 222, within which to surrender the same to the proper authorities, without incurring any criminal liability therefor, except if the unlicensed firearm or ammunition is carried outside of one's residence, not for the purpose of surrendering the same, or used in the commission of any other offense, and there is no allegation in said information that the firearms and ammunition enumerated therein were carried outside the accused's residence or used in the commission of some other crime. - In support thereof, the respondent judge cited the decision in People vs. Lopez, 79 Phil 658. -The prosecution filed a MR of said Resolution, but the motion was denied -Hence, the present recourse by the prosecution. Petitioners Claims -nothing is contained in said EOs which legalizes the possession of firearms and ammunition without a permit; -that said EOs merely authorized holders or possessors of unlicensed firearms and ammunition to surrender the same within a specified filing period without incurring criminal liability;

Criminal Procedure Rowena Daroy Morales


unlicensed firearms and ammunition may surrender the same to the proper authorities without incurring criminal liability, had ruled that a criminal liability was temporarily LIFTED for mere possession' of unlicensed firearms and ammunition during the period covered, although such person is not exempt from criminal liability filing within the period provided, he carries the firearm and ammunition (unless it is for the purpose of surrendering the same) or he commits any other offense with the use of such unlicensed firearm and ammunition. -People vs. Lopez~ It will be seen that sec 2 (of RA NO 4) excluded from the operation of sec 1 up to August 31, 1946, possession of firearms and ammunition so long as they were not used for any purpose other than self-defense or carried for any purpose other than of surrendering them to the proper authorities. The Government does not dispute this interpretation. Although the law does not categorically state that criminal liability was temporarily lifted for mere possession of filing firegems and ammunition, that is the only construction compatible with the spirit and purposes of the enactment as revealed by its context. -People vs. Feliciano~ SC ruled that RA No. 482 legalized mere unlicensed possession of firearms and ammunition for the limited period specified in said law, and punished only (1) the use of unlicensed firearm or ammunition, or (2) the carrying of such firearm or ammunition on the person, except to surrender them. The Court said: -Feliciano ruling was reiterated in People vs. Tabunares: RA No. 482, in effect legalized mere unlicensed on within one year from said date, and punished only (1) the use of a or ammunition or (2) the carriage thereof on the person except for purpose of surrender. Appellant's conviction cannot stand, since it is rested solely on unlicensed possession on or about November 6, 1950. 2. People vs. Lopez~ the Court already ruled that, under RA No. 4, the use or the carrying of firearms and/or ammunition was an ingredient, if not the sole ingredient, of the offense; i.e. the very acts which were punished, subject to certain conditions, and hence, should be alleged and proved. -People vs. Austria~ the Court also ruled that in order that an information charging illegal possession of firearm and ammunition, under RA No. 482, may be deemed suffident, it must allege that the accused was using the unlicensed firearm or carrying it in his person at the time he was apprehended by the authorities with said firearm. Dispositive Petition is DENIED.

a2010

page 101

Prof.
and replaced as collector of customs. Office of State Prosecutors sustained filing of information against Layosa. - Layosa did not submit memorandum. Respondent fiscal alleged that petitioner had abandoned contention as to lack of jurisdiction. Fiscal stressed that case had been scheduled for trial at instance of petitioner and that latter manifested his willingness to proceed. ISSUE WON trial court acted with grave abuse of discretion in ordering suspension HELD NO - Lower court acquired jurisdiction upon filing of information. Petitioner was notified of presuspension hearing. His counsel participated. Requirements of due process were observed. Public interest demands a speedy determination of that question. - It is true that petitioner was not yet arrested or taken into custody when pre-suspension hearing was held. However, voluntary appearance through counsel was submission to lower court's jurisdiction. (Note that in civil cases, defendant's voluntary appearance is equivalent to service of summons.) - "Where a court has jurisdiction of the offense or subject matter, the objection that it has no jurisdiction of the person of the accused may be waived. One who desires to object to the jurisdiction of the court over his person must appear in court for that purpose only, and if he raises other questions, he waives the objection." Layosa waived the objection based on lack of jurisdiction over his person when, as already noted, he appeared at the presuspension hearing and his counsel cross-examined the prosecution witness.

LOPEZ v CITY JUDGE [supra, page 41] LAYOSA v RODRIGUEZ 86 SCRA 300 AQUINO; November 10, 1978
NATURE Certiorari from order of CA FACTS - This is about suspension of Layosa, collector of customs, who was charged by city fiscal in CFI w/ having violated AntiGraft and Corrupt Practices Law (Republic Act No. 3019). Information was based on complaint filed by assistant director of District AntiSmuggling Action Center. It was one of 5 cases filed against Layosa, aside from malversation case. - Gravamen is that he demanded and received from M/V Lady Angelita I 2 to 3 cases of beer & soft drinks as consideration for giving preferential berthing facilities. - Fiscal, pursuant to sec 13 of RA No. 3019, filed motion for Layosa's suspension. Respondent Judge granted motion. He found that a valid information had been filed against Layosa. - Layosa filed instant petition for certiorari. He prayed that suspension be set aside. He contended that the court did not acquire jurisdiction over his person because no warrant of arrest had as yet been issued when hearing on his suspension was held and the case was not raffled to respondent Judge, that the Chief State Prosecutor in a telegram to the fiscal directed that the record of the case be elevated for review, and that respondent Judge gravely abused his discretion. - Because Layosa defied suspension, lower court adjudged him in contempt of court and penalized him by imprisonment for 3 mos and fine of P500. Layosa appealed to CA. - Respondent Judge explained that, to avoid delay, he acted on motion for suspension because case was filed after raffling between 2 branches of court had been terminated. He was scheduled to hold sessions and the other Judge was to begin one-month vacation. Judge pointed out that his action was sanctioned by Administrative Order No. 6 of SC which empowers Executive Judge to act on interlocutory matters prior to raffling. Case was eventually raffled to sala of respondent Judge. Layosa posted bail bond. He was arraigned

PEOPLE v CITY COURT OF MANILA 121 SCRA 637 RELOVA; April 27, 1983
NATURE Petition to review the order of the City Court of Manila, Branch XI FACTS - October 17, 1971: The incident occurred. - October 18, 1971: An information for serious physical injuries thru reckless imprudence was filed

Criminal Procedure Rowena Daroy Morales


against Francisco Gapay y Mallares, driver of the truck. On the same day, the victim Diolito de la Cruz died. - October 20, 1972: Gapay was arraigned on the charge of serious physical injuries thru reckless imprudence. He pleaded guilty, was sentenced to 1 month and 1 day of arresto mayor, and commenced serving sentence. - October 24, 1972: An information for homicide thru reckless imprudence was filed against Gapay - November 17, 1972: the City Court of Manila, upon motion of private respondent, issued an order dismissing the homicide thru reckless imprudence case on the ground of double jeopardy. ISSUES WON a person who has been prosecuted for serious physical injuries thru reckless imprudence and convicted thereof may be prosecuted subsequently for homicide thru reckless imprudence if the offended party dies as a result of the same injuries he had suffered HELD NO Ratio One who has been charged with an offense cannot be charged again with the same or identical offense though the latter be lesser or greater than the former. However, where after the first prosecution a new fact supervenes for which the defendant is responsible, which changes the character of the offense and, together with the facts existing at the time, constitutes a new and distinct offense, the accused cannot be said to be in second jeopardy if indicted for the new offense. Reasoning - Respondent court held that above rule does not apply in this case. It based its decision on the ruling in People v Buan, which held that Article 365 of the Penal Code punishes the negligent state of mind and not the resulting injury. The trial court concluded that once prosecuted for and convicted of negligence, the accused cannot again be prosecuted for the same negligence although for a different resulting injury. - In his memorandum, the Solicitor General made mention of the fact that on October 21, 1972, the City Fiscal filed an Urgent Motion asking that the "hearing and arraignment of this case be held in abeyance for there is information that the victim, Diolito dela Cruz died, and the information would have to be amended." - Be that as it may, the fact remains that the victim Diolito dela Cruz died on October 18 "one (1) day

a2010

page 102

Prof.
a military viewpoint, it was technically impossible to get inside such a cordon." The military investigators reported within a span of three hours that the man who shot Aquino (whose identity was then supposed to be unknown and was revealed only days later as Rolando Galman, although he was the personal friend of accused Col. Arturo Custodio who picked him up from his house on August 17, 1983) was a communist-hired gunman, and that the military escorts gunned him down in turn. - Marcos was constrained to create a Fact Finding Board to investigate. Both majority and minority reports were one in rejecting the military version as propounded by the chief investigator, respondent Gen. Olivas, that Rolando Galman was the NPA-hired assassin, stating that "the evidence shows [to the contrary] that Rolando Galman had no subversive affiliations." They were in agreement that "only the soldiers in the staircase with Sen. Aquino could have shot him;" that Galman, the military's "fall guy" was "not the assassin of Sen. Aquino" and that "the SWAT troopers who gunned down Galman and the soldiers who escorted Sen. Aquino down the service stairs, deliberately and in conspiracy with one another, gave a perjured story to us regarding the alleged shooting by Galman of Sen. Aquino and the mowing down, in turn, of Galman himself;" in short, that Ninoy's assassination was the product of a military conspiracy, not a communist plot. The only difference between the two reports is that the majority report found all the 26 private respondents headed by then AFP Chief General Fabian Ver involved in the military conspiracy while the chairman's minority report would exclude 19 of them and limit as plotters "the 6 persons who were on the service stairs while Senator Aquino was descending" and "General Luther Custodio because the criminal plot could not have been planned and implemented without his intervention." - As the accused were tried in the Sandiganbayan, Marcos through all his recorded public acts and statements from the beginning disdained and rejected his own Board's findings and insisted on the military version of Galman being Ninoy's assassin. [Note: His private acts in trying to control the outcome of the case were to be known much later after he was already deposed.] - Saturnina Galman and Reynaldo Galman, mother and son, respectively, of the late Rolando Galman, and 29 other petitioners, composed of three former Justices of this Court, five incumbent and former university presidents, a former AFP Chief of Staff, outstanding members of the Philippine Bar and solid

after the accident and the arrest of the respondent Gapay" and that on October 20, 1972, the accused was arraigned, pleaded guilty and sentenced accordingly. Thus, jeopardy had attached and no new fact supervened after the arraignment and conviction of the accused. Dispositive Order of dismissal of lower court affirmed.

SEPARATE OPINION GUTIERREZ [concurring]


- Knowing the volume of the caseload in the City Court of Manila and the inevitably slow pace of work, it is most surprising that the accused could have been arraigned for the charge of serious physical injuries only 3 days after the incident, 2 days after the filing of the information and the death of the victim. The accused does not appear to have been a detention prisoner necessitating his immediate arraignment right after the filing of the information. The only sensible conclusion is that the accused was hastily made to plead guilty to serious physical injuries to foreclose a charge for homicide even before it could be filed. In such a case, there would be a trifling with the processes of justice and a collusive effort amounting to fraud or deceit to deprive the State of its authority to prosecute an accused for the correct offense. - However, records are inadequate to show that the arraignment, while hasty and surrounded by seemingly suspicious circumstances, was tainted by fraud, collusion, or other form of chicanery sufficient to sustain a finding that the State was denied due process - At any rate, I concur in the affirmance of the order of dismissal in line with the many protections that the Constitution and the laws give to the accused in criminal prosecutions.

GALMAN v SANDIGANBAYAN 144 SCRA 43 TEEHANKEE; September 12, 1986


FACTS - Ninoy Aquino was cold-bloodedly killed while under escort from his plane that had just landed at the Manila International Airport on August 21, 1983. His brain was smashed by a bullet fired point-blank into the back of his head by a murderous assassin, notwithstanding that the airport was ringed by airtight security of close to 2,000 soldiers and "from

Criminal Procedure Rowena Daroy Morales


citizens of the community, filed the present action alleging that respondents Tanodbayan and Sandiganbayan committed serious irregularities constituting mistrial and resulting in miscarriage of justice and gross violation of the constitutional rights of the petitioners and the sovereign people of the Philippines to due process of law. They asserted that the Tanodbayan did not represent the interest of the people when he failed to exert genuine and earnest efforts to present vital and important testimonial and documentary evidence for the prosecution and that the Sandiganbayan Justices were biased, prejudiced and partial in favor of the accused, and that their acts "clouded with the gravest doubts the sincerity of government to find out the truth about the Aquino assassination." Petitioners prayed for the immediate issuance of a temporary restraining order restraining the respondent Sandiganbayan from rendering a decision on the merits in the pending criminal cases which it had scheduled on November 20, 1985 and that judgment be rendered declaring a mistrial and nullifying the proceedings before the Sandiganbayan and ordering a re-trial before an impartial tribunal by an unbiased prosecutor. - The Supreme Court resolved by nine-to-two votes to issue the restraining order prayed for. But ten days later on November 28, 1985, the Court by the same nine-to-two-vote ratio in reverse, resolved to dismiss the petition and to lift the temporary restraining order issued ten days earlier enjoining the Sandiganbayan from rendering its decision. [Hmmm tsk] - Petitioners filed a motion for reconsideration, alleging that the dismissal did not indicate the legal ground for such action and urging that the case be set for a full hearing on the merits because if the charge of partiality and bias against the respondents and suppression of vital evidence by the prosecution are proven, the petitioners would be entitled to the relief demanded: The People are entitled to due process which requires an impartial tribunal and an unbiased prosecutor. If the State is deprived of a fair opportunity to prosecute and convict because certain material evidence is suppressed by the prosecution and the tribunal is not impartial, then the entire proceedings would be null and void. Petitioners prayed that the Sandiganbayan be restrained from promulgating their decision as scheduled anew on December 2, 1985. - On December 5, 1985, the Court required the respondents to comment on the motion for reconsideration but issued no restraining order. Thus, on December 2, 1985, as scheduled, respondent

a2010

page 103

Prof.
remedy is a direct action to annul the judgment where the burden of proof falls upon the plaintiff to establish by clear, competent and convincing evidence the cause of the nullity. - The Supreme Court appointed a three-member commission composed of retired SC Justice Conrado Vasquez, chairman, and retired IAC Justices Milagros German and Eduardo Caguioa as members, to hear and receive evidence, testimonial and documentary, of the charges of collusion and pressures and relevant matters, upon prior notice to all parties, and to submit their findings to this Court for proper disposition. The Commission submitted the following recommendation: Considering the existence of adequate credible evidence showing that the prosecution in the Aquino-Galman case and the Justices who tried and decided the same acted under the compulsion of some pressure which proved to be beyond their capacity to resist, and which not only prevented the prosecution to fully ventilate its position and to offer all the evidences which it could have otherwise presented, but also predetermined the final outcome of the case, the Commission is of the considered thinking and belief, subject to the better opinion and judgment of this Honorable Court, that the proceedings in the said case have been vitiated by lack of due process, and hereby respectfully recommends that the prayer in the petition for a declaration of a mistrial in Sandiganbayan Cases Nos. 10010 and 10011 entitled 'People vs. Luther Custodio, et al.,' be granted." ISSUES 1. WON the petition for a declaration of a mistrial in Sandiganbayan Cases Nos. 10010 and 10011 entitled 'People vs. Luther Custodio, et al.,' be granted 2. WON a retrial would constitute double jeopardy HELD 1. YES Reasoning - The Supreme Court cannot permit such a sham trial and verdict and travesty of justice to stand unrectified. The courts of the land under its aegis are courts of law and justice and equity. They would have no reason to exist if they were allowed to be used as mere tools of injustice, deception and duplicity to subvert and suppress the truth, instead of repositories of judicial power whose judges are sworn and committed to render impartial justice to all alike who seek the enforcement or protection of a right or the prevention or redress of a wrong, without fear or favor and removed from the pressures of

Sandiganbayan issued its decision acquitting all the accused of the crime charged, declaring them innocent and totally absolving them of any civil liability. [Note: the word used by the Sandiganbayan was innocent instead of not guilty!] Respondents submitted that with the Sandiganbayan's verdict of acquittal, the instant case had become moot and academic. On February 4, 1986, the same Court majority denied petitioners' motion for reconsideration for lack of merit. [Note that EDSA I happened before the month ended.] - On March 20, 1986, petitioners filed their motion to admit their second motion for reconsideration. The thrust of the second motion for reconsideration was the startling and therefore unknown revelations of Deputy Tanodbayan Manuel Herrera as reported in the March 6, 1986 issue of the Manila Times entitled "Aquino Trial a Sham," that the then President had ordered the respondents Sandiganbayan and Tanodbayan Bernardo Fernandez and the prosecution panel headed by Herrera to whitewash the criminal cases against the 26 respondents accused and produce a verdict of acquittal. - Tanodbayan Fernandez claimed he never succumbed to any alleged attempts to influence his actuations in the premises, having instead successfully resisted perceived attempts to exert pressure to drop the case after preliminary investigation and actually ordered the filing and prosecution of the two murder cases against privateparty respondents. Respondents Justices of the Sandiganbayan First Division in their collective comment of April 9, 1986 stated that the trial of the criminal cases by them was valid and regular and decided on the basis of evidence presented and the law applicable, but manifested that "if it is true that the former Tanodbayan and the Deputy Tanodbayan, Chief of the Prosecution Panel, were pressured into suppressing vital evidence which would probably alter the result of the trial, Answering Respondents would not interpose any objection to the reopening of those cases, if only to allow justice to take its course." - Respondents-accused opposed the second motion for reconsideration and prayed for its denial. The accused-respondents raised the issue of double jeopardy, and invoked that the issues had become moot and academic because of the rendition of the Sandiganbayan's judgment of acquittal of all respondents-accused on December 2, 1985, with counsels for respondents Ver and Tigas, as well as Olivas, further arguing that assuming that the judgment of acquittal is void for any reason, the

Criminal Procedure Rowena Daroy Morales


politics and prejudice. More so, in the case at bar where the people and the world are entitled to know the truth and the integrity of our judicial system is at stake. In life, as an accused before the military tribunal Ninoy had pleaded in vain that as a civilian he was entitled to due process of law and trial in the regular civil courts before an impartial court with an unbiased prosecutor. In death, Ninoy is the victim of the "treacherous and vicious assassination" and the relatives and sovereign people as the aggrieved parties plead once more for due process of law and a retrial before an impartial court with an unbiased prosecutor. The Court is constrained to declare the sham trial a mock trial - the non-trial of the century and that the predetermined judgment of acquittal was unlawful and void ab initio. 2. NO Ratio Double jeopardy cannot be invoked against this Court's setting aside of the trial courts' judgment of dismissal or acquittal where the prosecution which represents the sovereign people in criminal cases is denied due process. Where the prosecution is deprived of a fair opportunity to prosecute and prove its case, its right to due process is thereby violated. The cardinal precept is that where there is a violation of basic constitutional rights, courts are ousted of their jurisdiction. Thus, the violation of the State's right to due process raises a serious jurisdictional issue which cannot be glossed over or disregarded at will. Where the denial of the fundamental right of due process is apparent, a decision rendered in disregard of that right is void for lack of jurisdiction. Reasoning - Legal jeopardy attaches only (a) upon a valid indictment, (b) before a competent court, (c) after arraignment, (d) a valid plea having been entered; and (e) the case was dismissed or otherwise terminated without the express consent of the accused. The lower court was not competent as it was ousted of its jurisdiction when it violated the right of the prosecution to due process. In effect, the first jeopardy was never terminated, and the remand of the criminal case for further hearing and/or trial before the lower courts amounts merely to a continuation of the first jeopardy, and does not expose the accused to a second jeopardy. - More so does the rule against the invoking of double jeopardy hold in the cases at bar where as we have held, the sham trial was but a mock trial where the authoritarian president ordered respondents Sandiganbayan and Tanodbayan to rig the trial and closely monitored the entire proceedings to assure the predetermined final outcome of acquittal and

a2010

page 104

Prof.
thereof, judgment is hereby rendered nullifying the proceedings in respondent Sandiganbayan and its judgment of acquittal in Criminal Cases Nos. 10010 and 10011 entitled "People of the Philippines vs. Gen. Luther Custodio, et al." and ordering a re-trial of the said cases which should be conducted with deliberate dispatch and with careful regard for the requirements of due process, so that the truth may be finally known and justice done to all.

total absolution as innocent of all the respondentsaccused. Notwithstanding the laudable efforts of Justice Herrera which saw him near the end "deactivating" himself from the case, as it was his belief that its eventual resolution was already a foregone conclusion, they could not cope with the misuse and abuse of the overwhelming powers of the authoritarian President to weaken the case of the prosecution, to suppress its evidence, harass, intimidate and threaten its witnesses, secure their recantation or prevent them from testifying. Fully aware of the prosecution's difficulties in locating witnesses and overcoming their natural fear and reluctance to appear and testify, respondent Sandiganbayan maintained a "dizzying tempo" of the proceedings and announced its intention to terminate the proceedings in about 6 months time or less than a year, pursuant to the scripted scenario. The prosecution complained of "the Presiding Justice's seemingly hostile attitude towards (it)" and their being the subject of warnings, reprimand and contempt proceedings as compared to the nil situation for the defense. Herrera likewise complained of being "cajoled into producing witnesses and pressed on making assurances that if given a certain period, they will be able to produce their witnesses," Herrera pleaded for "a reasonable period of preparation of its evidence" and cited other pending cases before respondent court that were pending trial for a much longer time where the "dizzying tempo" and "fast pace" were not maintained by the court. Manifestly, the prosecution and the sovereign people were denied due process of law with a partial court and biased Tanodbayan under the constant and pervasive monitoring and pressure exerted by the authoritarian President to assure the carrying out of his instructions. A dictated, coerced and scripted verdict of acquittal such as that in the case at bar is a void judgment. In legal contemplation, it is no judgment at all. It neither binds nor bars anyone. Such a judgment is "a lawless thing which can be treated as an outlaw". It is a terrible and unspeakable affront to the society and the people. To paraphrase Brandeis: If the authoritarian head of the government becomes the lawbreaker, he breeds contempt for the law, he invites every man to become a law unto himself, he invites anarchy. Dispositive Petitioners' second motion for reconsideration is granted. The resolutions of November 28, 1985 dismissing the petition and of February 4, 1986 denying petitioners' motion for reconsideration are hereby set aside and in lieu

PEOPLE V. RELOVA 148 SCRA 292 FELICIANO; March 6, 1987


NATURE Petition for certiorari and mandamus, the People seek to set aside the orders of the respondent Judge of the CFI of Batangas in a criminal case, quashing an information for theft filed against private respondent Manuel Opulencia on the ground of double jeopardy and denying the petitioner's motion for reconsideration. FACTS - CIRCUMSTANCES: On 1 February 1975, members of the Batangas City Police together with personnel of the Batangas Electric Light System, equipped with a search warrant issued by a city judge of Batangas City, searched and examined the premises of the Opulencia Carpena Ice Plant and Cold Storage owned and operated by the private respondent Manuel Opulencia. The police discovered that electric wiring, devices and contraptions had been installed, without the necessary authority from the city government, and "architecturally concealed inside the walls of the building" owned by the private respondent. These electric devices and contraptions were, in the allegation of the petitioner "designed purposely to lower or decrease the readings of electric current consumption in the electric meter of the said electric [ice and cold storage] plant." During the subsequent investigation, Manuel Opulencia admitted in a written statement that he had caused the installation of the electrical devices "in order to lower or decrease the readings of his electric meter. - CASE: On 24 November 1975, an information against Manuel Opulencia for violation of Ordinance No. 1, Series of 1974, Batangas City. Manuel Opulencia pleaded not guilty. On 2 February 1976, he filed a motion to dismiss the information upon the grounds that the crime there charged had already prescribed and that the civil indemnity there sought to be recovered was beyond the jurisdiction of the Batangas City Court to award. Batangas City

Criminal Procedure Rowena Daroy Morales


Court granted the motion to dismiss on the ground of prescription, it appearing that the offense charged was a light felony which prescribes two months from the time of discovery thereof, and it appearing further that the information was filed by the fiscal more than nine months after discovery of the offense charged in February 1975. - Fourteen (14) days later, another information against Manuel Opulencia, this time for theft of electric power under Article 308 in relation to Article 309, paragraph (1), of the Revised Penal Code. Before he could be arraigned thereon, Manuel Opulencia filed a Motion to Quash, alleging that he had been previously acquitted of the offense charged in the second information and that the filing thereof was violative of his constitutional right against double jeopardy. -Respondent Judge granted the accused's Motion to Quash and ordered the case dismissed. An MOR was denied. - On 1 December 1976, the present Petition for certiorari and mandamus was filed in this Court by the Acting City Fiscal of Batangas City on behalf of the People. ISSUE WON the defense of double jeopardy applies in this case. (Yes) HELD RATIO: Where one offense is charged under a municipal ordinance while the other is penalized by a statute, the critical inquiry is to the identity of the acts which the accused is said to have committed and which are alleged to have given rise to the two offenses: the constitutional protection against double jeopardy is available so long as the acts which constitute or have given rise to the first offense under a municipal ordinance are the same acts which constitute or have given rise to the offense charged under a statute. REASONING: Constitutional provision on double jeopardy reads: No person shall be twice put in jeopardy of punishment for the same offense. If an act is punished by a law and an ordinance, conviction or acquittal under either shall constitute a bar to another prosecution for the same act. (Article IV (22), 1973 Constitution) This case must be examined, not under the terms of the first sentence of Article IV (22) of the 1973 Constitution, but rather under the second sentence of the same section. The first sentence of Article IV (22) sets forth the general rule: the constitutional protection against double jeopardy is not available

a2010

page 105

Prof.
Petition for certiorari and mandamus is DENIED. Let the civil action for related civil liability be remanded to the CFI of Batangas City for further proceedings.

where the second prosecution is for an offense that is different from the offense charged in the first or prior prosecution, although both the first and second offenses may be based upon the same act or set of acts. The second sentence of Article IV (22) embodies an exception to the general proposition: the constitutional protection, against double jeopardy is available although the prior offense charged under an ordinance be different from the offense charged subsequently under a national statute such as the Revised Penal Code, provided that both offenses spring from the same act or set of acts. Put a little differently, where the offenses charged are penalized either by different sections of the same statute or by different statutes, the important inquiry relates to the identity of offenses charged: the constitutional protection against double jeopardy is available only where an Identity is shown to exist between the earlier and the subsequent offenses charged. In contrast, where one offense is charged under a municipal ordinance while the other is penalized by a statute, the critical inquiry is to the identity of the acts which the accused is said to have committed and which are alleged to have given rise to the two offenses: the constitutional protection against double jeopardy is available so long as the acts which constitute or have given rise to the first offense under a municipal ordinance are the same acts which constitute or have given rise to the offense charged under a statute. The question of Identity or lack of Identity of offenses is addressed by examining the essential elements of each of the two offenses charged, as such elements are set out in the respective legislative definitions of the offenses involved. The question of Identity of the acts which are claimed to have generated liability both under a municipal ordinance and a national statute must be addressed, in the first instance, by examining the location of such acts in time and space. When the acts of the accused as set out in the two informations are so related to each other in time and space as to be reasonably regarded as having taken place on the same occasion and where those acts have been moved by one and the same, or a continuing, intent or voluntary design or negligence, such acts may be appropriately characterized as an integral whole capable of giving rise to penal liability simultaneously under different legal enactments (a municipal ordinance and a national statute). DISPOSITION

PEOPLE v GROSPE [supra, page 43] CANIZA v PEOPLE (AGLORO) 159 SCRA 16 FELICIANO; March 18, 1988
NATURE Petition for Prohibition and certiorari directed at 1) the CFI Order of Nov. 27, 1979 issued by Branch 23 of CFI of Manila in Criminal Case 46768 and 2) said courts Order of March 20, 1980 in the same case denying Canizas Motion for Reconsideration FACTS - March 20, 1974: Assistant City Fiscal of Manila filed an Information for falsification of public documents allegedly committed on Nov. 5, 1968 by Caniza. - May 24, 1974: Caniza filed Motion to Quash saying that allegations in the information did not constitute an offense, and that the information contained averments which, if true, would constitute a legal excuse or justification - trial court granted Motion to Quash, dismissed case against Caniza - Fiscals Motion for Reconsideration of this Order was denied - June 13, 1979: a second Information (docketed as Criminal Case 46768) was filed charging Caniza with substantially the same offense as that charged under the previous information - Caniza moved to quash this second information on the grounds that 1) the offense charged had already prescribed, 2)quashal of the first Information had been on the merits, 3)the allegations of the second Information did not constitute and offense - Respondent judge issued an order denying the motion to quash - He also denied Canizas motion for reconsideration ISSUES 1. WON the offense charged had already prescribed 2. WON the filing of the second Information has placed the accused in jeopardy of punishment for the same offense a second time HELD 1. NO

Criminal Procedure Rowena Daroy Morales


Reasoning - 5 years, 4 months, and 16 days had elapsed between November 5, 1968 (the date of commission of the alleged offense) and March 20 1974 (date of filing the first information); 4 years, 2 months and 12 days had elapsed between April 3, 1975 (date of denial by the trial court of the Fiscals motion for reconsideration) and June 13, 1979 (date of filing of the second information). A total of 9 years, 6 months and 28 days had been consumed by the time the second Information was filed in court. - Under Article 90, in relation with Article 172 of the Revised Penal Code, the crime of falsification of public document committed by a private individual the offense with which petitioner Caiza is presently charged - prescribes in ten (10) years. In this respect, Article 91 of the Revised Penal Code states further: - Computation of prescription of offenses. The period of prescription shall commence to run from the day on which the crime is discovered by the offended party, the authorities, or their agents, and shall be interrupted by the filing of the complaint or information, and shall commence to run again when such proceedings terminate without the accused being convicted or acquitted, or are justifiably stopped for any reason not imputable to him. 2. NO Reasoning - Section 9 of Rule 117 of the Rules of Court 6 lists the following requisites in order that the defense of double jeopardy may be successfully invoked by an accused person: a. a valid Complaint or Information or other formal charge sufficient in form and substance to sustain a conviction; b. a Court of competent jurisdiction; c. that the accused had pleaded to the charge against him; d. that the accused had been convicted, or acquitted, or the case against him dismissed or otherwise terminated without his express consent; and e. that the second offense charged is the same as the first, or an attempt to commit the same or a petition thereof, or that the second offense necessarily includes or is necessarily included in the first offense charged. - Criminal Case No. 16879 was ordered dismissed by the trial court with the express consent of the accused i.e., upon Motion to Quash filed by petitioner Caiza. Generally, a dismissal under such circumstance win not bar another prosecution for the same offense; the defendant, in having the case against him dismissed, thereby waives his

a2010

page 106

Prof.
- During the trial for the estafa case, Atty. Rodriguez Dadivas, counsel for the accused Devera and Machado, orally moved for the inhibition of the presiding Judge Suplico on the ground that he had some doubts as to the impartiality of the judge against whom he and some nineteen (19) other practicing lawyers had filed serious administrative charges with the President of the Philippines, the Chief Justice of the Supreme Court, and the Minister of Justice. Following Atty. Dadivas, Atty. Roberto Barrios, former private prosecutor, also moved for the inhibition of the judge for the same reason alleged by Atty. Dadivas. The presiding judge, however, ruled Attys. Dadivas and Barrios out of order and asked the City Fiscal to present the evidence for the prosecution. Thereupon, the City Fiscal manifested that he was authorizing the private prosecutor to actively handle the prosecution of the case. Atty. Roberto Barrios, however, insisted that the presiding judge should first rule on their previous motion for inhibition. Instead of resolving the motion for inhibition, the presiding judge asked the comments of Atty. Lorenzo E. Coloso, counsel for the accused Bernabe Que and Amelia Que, and Atty. Alberto Villarruz, counsel for the accused Paz L. Martelino, who both invoked the constitutional right of their clients to a speedy trial. The presiding judge asked again the prosecution to present its evidence but the private prosecutor insisted that a ruling be made by the presiding judge with regard to the pending motion for inhibition. As a result, the presiding judge issued the order dated November 10, 1986 dismissing this case. - On November 21, 1986, the prosecution filed a motion for reconsideration from the order of dismissal. This was opposed by the defense. - In the meantime, the case was re-raffled to Judge Rodrigo Cosico. Judge Cosico in an order dated May 22, 1987, granted the prosecution's motion for reconsideration and caused the case to be reopened. The subsequent motion for reconsideration flied by the defense was denied in an order dated November 27, 1987. - G.R. No. 83114 - accused Martelino filed before the CA a petition for certiorari praying that the order of Judge Cosico reinstating the case be declared null and void on the ground of double jeopardy. CA found merit in the petition and set aside Judge Cosico's order as "it amounts to double jeopardy on the part of the petitioner." The decision of the appellate court is based on precedents which discuss the failure of the prosecution to appear for trial, produce its witnesses, or present its evidence.

constitutional right against double jeopardy for the reason that he effectively prevents the trial court from proceeding to trial on the merits and rendering a judgment of conviction against him - Application of the aforestated doctrine of waiver, however, is subject to two (2) sine qua non conditions: first, dismissal must have been sought or induced by the defendant, either personally or through counsel; and second, such dismissal must not have been on the merits and must not necessarily amount to an acquittal. In this respect, the record shows that petitioner Caiza moved to quash the first Information (Criminal Case No. 16879) on grounds that the allegations made therein did not constitute an offense and/or that the first Information contained allegations which, if true, constituted a legal excuse or justification. These grounds, upon which the trial court anchored its 27 November 1974 Order of dismissal, are clearly directed at the sufficiency of said information to sustain the conviction of petitioner Caniza and, hence, indicate the absence of the first requisite in double-jeopardy. Furthermore, and more importantly, dismissal of a criminal action on this basis is not properly considered as amounting to an acquittal on the merits; from a legal standpoint, the defendant is deemed as not having been charged with the commission of any offense whatsoever under the deficient information. Consequently, petitioner Caizas plea of second jeopardy cannot be sustained: he effectively waived his right to assert that plea when he moved to quash the first Information filed against him. Dispositive Petition for Prohibition and certiorari is DISMISSED. The 8 December 1980 Resolution of this Court giving due course to the Petition is withdrawn and the disputed Orders dated 27 November 1979 and 20 March 1980 issued by respondent judge in Criminal Case No. 46768 are hereby AFFIRMED. This case is remanded to the court a quo for trial on the merits.

QUE v COSICO 177 SCRA 410 GUTIERREZ, JR.; September 8, 1989


FACTS - Petitioners in G.R. No. 81861 and private respondent in G.R. No. 83114 all stand as the accused in Criminal Case No. C-2152 for estafa thru falsification of commercial documents which case was dismissed but subsequently reinstated.

Criminal Procedure Rowena Daroy Morales


- G.R. No. 81861 - the accused Bernabe Que and Amelia Que filed a petition for certiorari directly with this court seeking to declare Judge Cosico's orders dated May 22, 1987 and November 27, 1987 as null and void and to prohibit respondent from further proceeding with the criminal case. ISSUE WON the reinstatement of the criminal case placed the accused in double jeopardy. HELD Ratio the requisites that must concur for legal jeopardy to attach are, to wit: 1) a valid complaint or information; 2) a court of competent jurisdiction; 3) the accused has pleaded to the charge and 4) the accused has been convicted or acquitted or the case dismissed or terminated without the express consent of the accused. Reasoning The fourth requisite is lacking in the instant case. The case was dismissed upon motion and with the express consent of the accused. The accused Bernabe Que, Amelia Que and Paz Martelino invoked their constitutional right to a speedy trial when the prosecution refused to present evidence until the court had ruled on the motion for inhibition. It was on their oral motion that the lower court ordered the case to be dismissed. There were no oppressive delays on the part of the prosecution. The prosecution's insistence that Judge Suplico rule on the motion to inhibit before further proceedings in the case was not dilatory. There is no reason apparent from the records why Suplico should vacillate or show anger on a matter that affects the subsequent course of the trial. He could have easily granted or denied the motion, giving sound reasons for his ruling. He could have required that the motion be submitted formally. The subsequent behavior of the former Judge, especially his precipitate dismissal of the case shows that his reaction was not mere impetuousness or pique. It bears the earmarks of bias and prejudice. As noted by Judge Cosico in his order dated November 27, 1987: ... A closed scrutiny of the foregoing chronology of facts that transpired at the Regional Trial Court does not show that the prosecution deliberately delayed the prosecution of this case nor does it appear that the prosecution was unprepared to present its evidence. The two (2) postponements requested by the prosecution appear to be reasonable. Moreover, it appears that on November 10, 1986, Mr. Angel Yu, principal witness for the prosecution, was then present and ready to testify. On the other hand, it

a2010

page 107

Prof.
with the complex crime of estafa through falsification of two checks of the Philippine National Bank and were found guilty. Sentence for each of the three defendants to suffer in each case a penalty of not less than 6 years and 1 day and not more than 9 years, 4 months and 1 day of prision mayor, to pay a fine of P2,500 and the costs. In addition, they were sentenced to indemnify the Philip-pine Ryukyus Command, the payee of the checks, in the sum of P5,417.11 in each of the two cases - On appeal before the CA, Cruz and Vergara were given reduced penalties. Smason was only found guilty of committing the crime through gross imprudence and was accordingly sentenced to 4 months of arresto mayor in each of the two cases. ISSUES 1. WON the acts done by him, as found by the Court of Appeals constitute gross imprudence 2. WON he was correctly found guilty of the offense estafa through (falsification by) negligence HELD 1. YES - Appellant was, or must have been aware that the claim was for a sizeable amount, totalling over twelve thousand pesos, and ordinary prudence required that he should satisfy himself by all proper and adequate means of the identity of the persons claiming said amounts, since they we personally unknown to him. The mere assurance of a former class, mate would certainly not be a satisfactory identification to justify disbursement of such a large amount - appellant as a Lieutenant of the Army is sufficiently intelligent and educated to foresee the possibility that the certificates could be forged or stolen - appellant cooperated in the commission of the complex offense of estafa through falsification by reckless imprudence by acts without which it could not have been accomplished, and this being a fact, there would be no reason to exculpate him from liability 2. YES - counsel contends that: Samson cannot be convicted of the crime of estafa through falsification by imprudence for the reason that the information filed against him charges only a willful act of falsification and contains no reference to any act of imprudence on his part; the alleged imprudent act does not include or is not necessarily included in the offense charged in the information because a deliberate

appears that Atty. Lorenzo Coloso also asked for at least two (2) postponements. In invoking the right of the accused to speedy trial, Atty. Coloso is not therefore coming to this court with clean hands. Considering the two (2) postponements requested by Atty. Coloso, the accused in effect waived their right to speedy trial." Dispositive WHEREFORE, the petition docketed as G.R. 81861 is hereby DISMISSED for lack of merit. The petition docketed as G. R. No. 83114 is GRANTED and the questioned orders of Judge Cosico dated May 22, 1987 and November 27, 1978 are AFFIRMED. The decision of the Court of Appeals dated April 22, 1988 is SET ASIDE.

SAMSON v CA 103 PHIL 277 BAUTISTA ANGELO; MARCH 31, 1958


NATURE Petition for review by certiorari of a decision of the Court of Appeals FACTS - On October 2, 1948 Amado L. Cruz asked the help of his former classmate Rufino T. Samson in getting the checks of the two claimants who were with him at Camp Murphy. After being assured twice of the identity of the supposed claimants and after examining their residence certificates attached to the claim papers, Samson accompanied by Cruz and the supposed claimants went to talk to Lt. Manuel Valencia and requested him to act as guarantor to secure the claimants check. - acting on the assurance of Samson, Valencia helped them secure checks in the name of Rosalinda Perez and Espiridion Lascano. Hese were encashed by Mallari again believing Samson. - Paras didnt know how to write so she instead placed her thumbmark at the back of the check while Samson signed as witness. Lascano placed his signature while Samson signed as last endorser. - they then proceeded to Aristocrat to eat lunch. Here Samson received P300 supposedly to pay the officers who helped them and P10 for taxi money. - two days after, Samson was informed of the possibility that the people who claimed the checks werent the real people who they claimed to be. This he was able to verify. Upon knowing of this, Samson reported the incident to an officer. - Samson, Cruz and Vergara and two others whose names are unknown in two separate informations

Criminal Procedure Rowena Daroy Morales


intent to do an unlawful act is inconsistent with the idea of negligence. - The rule regarding variance between allegation and proof in a criminal case, is: "When there is variance between the offense charged in the complaint or information, and that proved or established by the evidence, and the offense as charged, is included in or necessarily includes the offense proved, the defendant shall be convicted of the offense proved included in that which is charged, or of the offense charged included in that which is proved" (Section 4, Rule 116 now rule 120. Rules of Court). -"An offense charged necessarily includes that which is proved, when some of the essential elements or ingredients of the former, as this is alleged in the complaint or information, constitute the latter. And the offense charged is necessarily included in the offense proves, when the essential ingredients of the former constitute or form a part of those constituting the latter" (Section 5, Rule 116, now rule 120) - conviction for a criminal negligent act can be had under an information exclusively charging the commission of a willful offense, upon the theory that the greater includes the lesser offense - appellant did not act with criminal intent but merely failed to take proper and adequate means to assure himself of the identity of the real claimants as an ordinary prudent man would do. In other words, the information alleges acts which charge willful, falsification but which turned out to be not willful but negligent. This is a case covered by the rule when there is a variance between the allegation and proof - Moreover, Section 5, Rule 116 now 120, of the Rules of Court does not require that all the essential elements of the offense charged in the information be proved, it being sufficient that some of said essential elements or ingredients thereof be established to constitute the crime proved. This conclusion is strengthened by the provisions of Section 9, Ruled 113, (I think sec 7 rule 117 na) of the Rules of Court under which appellant could no longer be prosecuted for estafa through falsification of commercial documents by reckless negligence were we to acquit him in the cases at bar on the obviously technical theory of the dissenters

a2010

page 108

Prof.
People of the Philippines v. Panfilo Lacson, et al. pending before Branch 81 of the RTC of Quezon City. - On May 18, 1995, then PNP Director-General Recaredo Sarmiento II announced, in a press conference, the killing of eleven (11) members of the Kuratong Baleleng Gang (KBG) in a shootout with police elements near the fly-over along Commonwealth Avenue, Quezon City at about 4:00 A.M. that day. - On May 22, 1995, morning papers carried the news that SPO2 Eduardo delos Reyes had claimed that the killing of the eleven (11) gang members was a rubout or summary execution and not a shootout. - In an affidavit he executed the following day, delos Reyes stated that he was part of a composite police team called the Anti-Bank Robbery and Intelligence Task Force Group (ABRITFG) composed of elements of the National Capital Region Commandand headed by Chief Superintendent Jewel Canson; Traffic Management Command, headed by Senior Superintendent Francisco Subia, Jr.; Presidential AntiCrime Commission (PACC), headed by Chief Superintendent Panfilo M. Lacson; Central Police District Command, headed by Chief Superintendent Ricardo de Leon; and Criminal Investigation Command (CIC), headed by Chief Superintendent Romeo Acop - Delos Reyes claimed that the police team arrested the eleven (11) gang members in early morning of May 18, 1995 at the gangs safe house in Superville Subdivision, Paraaque; that after their arrest, the gang members were made to board two vans, their hands tied behind their backs, and brought initially to Camp Crame where a decision to summarily execute them was made, and later to Commonwealth Avenue where they were shot to death by elements of the Anti-Bank Roberry Intelligence Task Force Group - On May 26, 1995, SPO2 Corazon dela Cruz, another CIC investigator, executed an affidavit corroborating the material allegations of delos Reyes - On May 31, 1995, Armando Capili, a reporter of Remate, executed an affidavit stating that he was present when the KBG members were arrested in Superville Subdivision - On June 1, 1995, Chief Superintendent Job A. Mayo, PNP Director for Investigation, filed murder charges with the Office of the Ombudsman against ninetyseven (97) officers and personnel of ABRITFG. The next-of-kin of the slain KBG members also filed murder charges against the same officers and personnel. - Ombudsman Aniano Desierto then created a panel of investigators to conduct a preliminary

that case that while willful crimes are punished according to their result in crimes of negligence, what the law punishes is the carelessness itself, the failure to take the precautions that society has a right to expect will be taken under the circumstances of each case - that intentional falsification and falsification by negligence not only differ in seriousness, but in essence; they are, by their nature, two different offenses altogether. Wherefore, an offender who is accused of intentional falsification cannot be held to answer for falsification by negligence, because the essential element of the latter offense, the ingredient that characterizes it and separates it from all other offenses, to wit, the criminal negligence or carelessness, is not involved in the elements of the crime charged. Not only is it not included: it is excluded by incompatibility, because malice or intent cannot co-exist with negligence - On the procedural side, the objections to appellant's conviction of estafa by falsification through negligence are much more serious. Section 5, Rule 116 (now 120), upon which the majority relies as justifying the conviction, expresses the following rule: An offense charged necessarily includes that which is proved, when some of the essential elements or ingredients of the former, as this alleged in the complaint or information, constitute the latter. (Italic mine) It is not enough, therefore, that the elements of the crime for which an accused is convicted should be proved, but then must also be charged or alleged

PEOPLE v PANFILO LACSON G.R. No. 149453 RESOLUTION: May 28, 2002
NATURE Petition for review on certiorari FACTS The assailed Decision of the appellate court granted respondent Lacsons Second Amended Petition for Prohibition with application for the issuance of a Temporary Restraining Order, (1) assailing the Order issued by Judge Herminia Pasamba of the Regional Trial Court (RTC) of Manila, Branch 40, that allowed the continuation of the reinvestigation of the Kuratong Baleleng cases; and (2) praying for the dismissal of Criminal Cases entitled

SEPARATE OPINION REYES JBL [dissent]


- Quizon vs. Justice of the Peace of Bacolor (97 Phil., 342), July 28, 1955, that criminal negligence is not a mere variant of the intentional misdeed; that it is a distinct and separate in itself. We also pointed out in

Criminal Procedure Rowena Daroy Morales


investigation of the murder charges. On October 20, 1995, the panel issued a resolution recommending the dismissal of the charges for lack of probable cause. - Ombudsman Desierto referred the resolution for review. On November 20, 1995, the review panel reversed the resolution and found probable cause for the prosecution of multiple murder charges against twenty-six (26) officers and personnel of ABRITFG. - On November 2, 1995, the Ombudsman filed before the Sandiganbayan eleven (11) Informations for MURDER, docketed as Criminal Cases Nos. 23047 to 23057, against respondent Panfilo M. Lacson and twenty-five (25) other accused. All twenty-six (26) of them were charged as principals. - Upon motion of the respondent, the criminal cases were remanded to the Ombudsman for reinvestigation. On March 1, 1996, Amended Informations were filed against the same twenty-six (26) suspects but the participation of respondent Lacson was downgraded from principal to accessory. Arraignment then followed and respondent entered a plea of not guilty. - With the downgrading of charges against him, respondent Lacson questioned the jurisdiction of the Sandiganbayan to hear the criminal cases as none of the principal accused in the Amended Informations was a government official with a Salary Grade (SG) 27 or higher, citing Section 2 of R. A. No. 7975 then prevailing. Accordingly, the Sandiganbayan ordered the cases transferred to the Regional Trial Court - The Office of the Special Prosecutor filed a motion for reconsideration of the transfer. Pending resolution of the motion, R. A. No. 8249 took effect on February 23, 1997, amending R. A. No. 7975. In particular, the amendatory law deleted the word principal in Section 2 of R. A. No. 7975, thereby expanding the jurisdiction of the Sandiganbayan to include all cases where at least one of the accused, whether principal, accomplice or accessory, is a government official of Salary Grade (SG) 27 or higher. The amendment is made applicable to all cases pending in any court in which trial has not yet begun as of the date of its approval. - In Lacson v. Executive Secretary, respondent Lacson challenged the constitutionality of the amendment and contended that the Sandiganbayan had no jurisdiction over the criminal cases. This Court, while dismissing the constitutional challenge, nonetheless ordered the transfer of the criminal cases to the Regional Trial Court on the ground that the Amended Informations for murder failed to indicate that the offenses charged therein were

a2010

page 109

Prof.
to attend the investigation of said Criminal Cases - On May 28, 2001, respondent Lacson, et al., invoking, among others, their constitutional right against double jeopardy, filed a petition for prohibition with application for temporary restraining order and/or writ of preliminary injunction with the Regional Trial Court of Manila, primarily to enjoin the State prosecutors from conducting the preliminary investigation. - The plea for temporary restraining order was denied - On June 6, 2001, eleven (11) Informations for murder involving the killing of the same members of the Kuratong Baleleng gang were filed before the Regional Trial Court of Quezon City - The new Informations charged as principals thirtyfour (34) people, including respondent Lacson and his twenty-five (25) other co-accused in the original informations. The criminal cases were assigned to Judge Ma. Theresa L. Yadao. - On the same day, respondent Lacson filed before the Court of Appeals a petition for certiorari against Judge Pasamba, the Secretary of Justice, the PNP Chief, State Prosecutors Ong and Zacarias, 2nd Assistant City Prosecutor Jamolin, and the People of the Philippines. The said petition was amended to implead as additional party-respondents State Prosecutor Claro Arellano and the RTC, Quezon City, Branch 81 - In the meantime, on June 8, 2001, respondent Lacson also filed with the RTC-QC Branch 81 (presided by Judge Ma. Theresa Yadao), a Motion for Judicial Determination of Probable Cause and in the absence thereof, to dismiss the cases outright. Respondent Lacson, however, filed a Manifestation and Motion dated June 13, 2001 seeking the suspension of the proceedings before the trial court. - The Court of Appeals issued a temporary restraining order enjoining Judge Yadao from issuing a warrant of arrest or conducting any proceeding or hearing in Criminal Cases Nos. 01-101102 to 01-101112. - On August 24, 2001, the Court of Appeals rendered the now assailed Decision. It characterized the termination of Criminal Cases Nos. Q-99-81679 to Q99-81689 as provisional dismissal, and considered Criminal Cases Nos. 01-101102 to 01-101112 as mere revivals of the same. Applying Section 8, Rule 117 of the 2000 Revised Rules of Criminal Procedure, it dismissed the criminal cases against the respondent ISSUE WON Section 8, Rule 117 bars the filing of the eleven (11) informations against the respondent Lacson

committed in relation to, or in discharge of, the official functions of the respondent, as required by R. A. No. 8249. - The Criminal Cases were raffled off to Branch 81 of the Regional Trial Court of Quezon City, then presided by Judge, now Associate Justice of the Court of Appeals, Wenceslao Agnir, Jr. - Before the accused could be arraigned, prosecution witnesses Eduardo de los Reyes, Corazon de la Cruz, Armando Capili and Jane Gomez recanted their affidavits which implicated respondent Lacson in the murder of the KBG members. - On the other hand, private complainants also executed their respective affidavits of desistance declaring that they were no longer interested to prosecute these cases. - Due to these developments, the twenty-six (26) accused, including respondent Lacson, filed five separate but identical motions to: (1) make a judicial determination of the existence of probable cause for the issuance of warrants of arrest (2) hold in abeyance the issuance of the warrants (3) dismiss the cases should the trial court find lack of probable cause. - The records of the case before us are not clear whether the private offended parties were notified of the hearing on March 22, 1999 held by Judge Agnir to resolve the motions filed by respondent Lacson and the other accused. - During the said hearing, the private offended parties who desisted do not appear to have been presented on the witness stand. In their stead, Atty. Godwin Valdez testified that he assisted them in preparing their affidavits of desistance and that he signed said affidavits as witness. On the other hand, Atty. Aurora Bautista of the Philippine Lawyers League presented the affidavits of recantation of prosecution witnesses Eduardo de los Reyes, Armando Capili and Jane Gomez. Only prosecution witness Corazon de la Cruz testified to affirm her affidavit. - On March 29, 1999, Judge Agnir issued a Resolution dismissing the Criminal Cases - On March 27, 2001, PNP Director Leandro R. Mendoza indorsed to the Department of Justice the new affidavits of P/Insp. Ysmael S. Yu and P/S Insp. Abelardo Ramos regarding the Kuratong Baleleng incident for preliminary investigation. On the strength of this indorsement, Secretary of Justice Hernando B. Perez formed a panel to investigate the matter. - On April 17, 2001, the respondent was subpoenaed

Criminal Procedure Rowena Daroy Morales


involving the killing of some members of the Kuratong Baleleng gang. HELD - This rule which took effect on December 1, 2000 provides: SEC. 8. Provisional dismissal.- A case shall not be provisionally dismissed except with the express consent of the accused and with notice to the offended party. The provisional dismissal of offenses punishable by imprisonment not exceeding six (6) years or a fine of any amount, or both, shall become permanent one (1) year after issuance of the order without the case having been revived. With respect to offenses punishable by imprisonment of more than six (6) years, their provisional dismissal shall become permanent two (2) years after issuance of the order without the case having been revived. - Like any other favorable procedural rule, this new rule can be given retroactive effect. However, the Court cannot rule on this issue due to the lack of sufficient factual bases. Thus, there is need of proof of the following facts: (1) whether the provisional dismissal of the cases had the express consent of the accused; (2) whether it was ordered by the court after notice to the offended party, (3) whether the 2year period to revive has already lapsed, and (4) whether there is any justification for the filing of the cases beyond the 2-year period. - There is no uncertainty with respect to the fact that the provisional dismissal of the cases against respondent Lacson bears his express consent. It was respondent Lacson himself who moved to dismiss the subject cases for lack of probable cause before then Judge Agnir, hence, it is beyond argument that their dismissal bears his express consent. - The records of the case, however, do not reveal with equal clarity and conclusiveness whether notices to the offended parties were given before the cases against the respondent Lacson were dismissed by then Judge Agnir. It appears from the resolution of then Judge Agnir that the relatives of the victims who desisted did not appear during the hearing to affirm their affidavits. Their affidavits of desistance were only presented by Atty. Godwin Valdez who testified that he assisted the private complainants in preparing their affidavits and he signed them as a witness. It also appears that only seven (7) persons submitted their affidavits of desistance. From the records of the case before us, it cannot be determined whether there were affidavits of

a2010

page 110

Prof.
to justify the revival of cases beyond the 2-year bar. - In light of the lack of or the conflicting evidence on the various requirements to determine the applicability of Section 8, Rule 117, this Court is not in a position to rule whether or not the re-filing of the cases for multiple murder against respondent Lacson should be enjoined. Fundamental fairness requires that both the prosecution and the respondent Lacson should be afforded the opportunity to be heard and to adduce evidence on the presence or absence of the predicate facts upon which the application of the new rule depends. They involve disputed facts and arguable questions of law. The reception of evidence on these various issues cannot be done in this Court but before the trial court. Dispositive Case remanded

desistance executed by the relatives of the three other victims. The same records do not show whether they were notified of the hearing or had knowledge thereof. To be sure, it is not fair to expect the element of notice to be litigated before then Judge Agnir for Section 8, Rule 117 was yet inexistent at that time. - The fact of notice to the offended parties was not raised either in the petition for prohibition with application for temporary restraining order or writ of preliminary injunction filed by respondent Lacson in the RTC of Manila, to enjoin the prosecutors from reinvestigating the said cases against him. The only question raised in said petition is whether the reinvestigation will violate the right of respondent Lacson against double jeopardy. Thus, the issue of whether or not the reinvestigation is barred by Section 8, Rule 117 was not tackled by the litigants. - Nor was the fact of notice to the offended parties the subject of proof after the eleven (11) informations for murder against respondent Lacson and company were revived in the RTC of Quezon City presided by Judge Yadao. There was hardly any proceeding conducted in the case for respondent Lacson immediately filed a petition for certiorari in the appellate court challenging, among others, the authority of Judge Yadao to entertain the revived informations for multiple murder against him. - The applicability of Section 8, Rule 117 was never considered in the trial court. It was in the Court of Appeals where respondent Lacson raised for the first time the argument that Section 8, Rule 117 bars the revival of the multiple murder cases against him. But even then, the appellate court did not require the parties to elucidate the crucial issue of whether notices were given to the offended parties before Judge Agnir ordered the dismissal of the cases against respondent Lacson and company. - Indeed, the records of this case are inconclusive on the factual issue of whether the multiple murder cases against respondent Lacson are being revived within or beyond the 2-year bar. The reckoning date of the 2-year bar has to be first determined - - whether it is from the date of the Order of then Judge Agnir dismissing the cases or from the dates the Order were received by the various offended parties or from the date of the effectivity of the new rule. - If the cases were revived only after the 2-year bar, the State must be given the opportunity to justify its failure to comply with said timeline. The new rule fixes a timeline to penalize the State for its inexcusable delay in prosecuting cases already filed in courts. It can therefore present compelling reasons

PEOPLE v PANFILO LACSON G.R. No. 149453 April 3, 2003


NATURE Motion for Reconsideration of the Resolution dated May 28, 2002 FACTS - Respondent and his co-accused were charged with multiple murder for the shooting and killing of eleven male persons identified as members of the Kuratong Baleleng Gang. - The Court ruled in the Resolution sought to be reconsidered that the provisional dismissal of Criminal Cases against the accused were with the express consent of the respondent as he himself moved for said provisional dismissal when he filed his motion for judicial determination of probable cause and for examination of witnesses. The Court also held therein that although Section 8, Rule 117 of the Revised Rules of Criminal Procedure could be given retroactive effect, there is still a need to determine whether the requirements for its application are attendant. The trial court was thus directed to resolve the following: ... (1) whether the provisional dismissal of the cases had the express consent of the accused; (2) whether it was ordered by the court after notice to the offended party; (3) whether the 2-year period to revive it has already lapsed; (4) whether there is any justification for the filing of the cases beyond

Criminal Procedure Rowena Daroy Morales


the 2-year period; (5) whether notices to the offended parties were given before the cases of respondent Lacson were dismissed by then Judge Agnir; (6) whether there were affidavits of desistance executed by the relatives of the three (3) other victims; (7) whether the multiple murder cases against respondent Lacson are being revived within or beyond the 2-year bar. - The Court further held that the reckoning date of the two-year bar had to be first determined whether it shall be from the date of the order of then Judge Agnir, Jr. dismissing the cases, or from the dates of receipt thereof by the various offended parties, or from the date of effectivity of the new rule. According to the Court, if the cases were revived only after the two-year bar, the State must be given the opportunity to justify its failure to comply with the said time-bar. It emphasized that the new rule fixes a time-bar to penalize the State for its inexcusable delay in prosecuting cases already filed in court. However, the State is not precluded from presenting compelling reasons to justify the revival of cases beyond the two-year bar. - The petitioners aver that Section 8, Rule 117 of the Revised Rules of Criminal Procedure is not applicable to the Criminal Cases because the essential requirements for its application were not present when Judge Agnir, Jr., issued his resolution of March 29, 1999. The petitioners maintain that the respondent did not give his express consent to the dismissal by Judge Agnir, Jr. of the Criminal Cases. The respondent allegedly admitted in his pleadings filed with the Court of Appeals and during the hearing thereat that he did not file any motion to dismiss said cases, or even agree to a provisional dismissal thereof. Moreover, the heirs of the victims were allegedly not given prior notices of the dismissal of the said cases by Judge Agnir, Jr. According to the petitioners, the respondents express consent to the provisional dismissal of the cases and the notice to all the heirs of the victims of the respondents motion and the hearing thereon are conditions sine qua non to the application of the time-bar in the second paragraph of the new rule. - The petitioners further contend that even on the assumption that the respondent expressly consented to a provisional dismissal of the Criminal Cases and all the heirs of the victims were notified of the respondents motion before the hearing thereon and were served with copies of the resolution of Judge Agnir, Jr. dismissing the eleven cases, the two-year bar in Section 8 of Rule 117 of the Revised Rules of Criminal Procedure should be applied prospectively

a2010

page 111

Prof.
for the same offense or for an offense necessarily included therein. Although the second paragraph of the new rule states that the order of dismissal shall become permanent one year after the issuance thereof without the case having been revived, the provision should be construed to mean that the order of dismissal shall become permanent one year after service of the order of dismissal on the public prosecutor who has control of the prosecution without the criminal case having been revived. The public prosecutor cannot be expected to comply with the timeline unless he is served with a copy of the order of dismissal. - Express consent to a provisional dismissal is given either viva voce or in writing. It is a positive, direct, unequivocal consent requiring no inference or implication to supply its meaning. Where the accused writes on the motion of a prosecutor for a provisional dismissal of the case No objection or With my conformity, the writing amounts to express consent of the accused to a provisional dismissal of the case. The mere inaction or silence of the accused to a motion for a provisional dismissal of the case or his failure to object to a provisional dismissal does not amount to express consent. A motion of the accused for a provisional dismissal of a case is an express consent to such provisional dismissal. If a criminal case is provisionally dismissed with the express consent of the accused, the case may be revived only within the periods provided in the new rule. On the other hand, if a criminal case is provisionally dismissed without the express consent of the accused or over his objection, the new rule would not apply. The case may be revived or refiled even beyond the prescribed periods subject to the right of the accused to oppose the same on the ground of double jeopardy or that such revival or refiling is barred by the statute of limitations. - In this case, the respondent has failed to prove that the first and second requisites of the first paragraph of the new rule were present when Judge Agnir, Jr. dismissed the Criminal Cases. Irrefragably, the prosecution did not file any motion for the provisional dismissal of the said criminal cases. The respondent did not pray for the dismissal, provisional or otherwise of the Criminal Cases. Neither did he ever agree, impliedly or expressly, to a mere provisional dismissal of the cases. - The Court also agrees with the petitioners contention that no notice of any motion for the provisional dismissal or of the hearing thereon was served on the heirs of the victims at least three days before said hearing as mandated by Rule 15, Section

and not retroactively against the State. To apply the time limit retroactively to the criminal cases against the respondent and his co-accused would violate the right of the People to due process, and unduly impair, reduce, and diminish the States substantive right to prosecute the accused for multiple murder. ISSUES 1. WON Section 8, Rule 117 of the Revised Rules of Criminal Procedure is applicable to the Criminal Cases 2. WON the time-bar in said rule should be applied retroactively HELD 1. YES - Section 8, Rule 117 of the Revised Rules of Criminal Procedure reads: Sec. 8. Provisional dismissal. A case shall not be provisionally dismissed except with the express consent of the accused and with notice to the offended party. The provisional dismissal of offenses punishable by imprisonment not exceeding six (6) years or a fine of any amount, or both, shall become permanent one (1) year after issuance of the order without the case having been revived. With respect to offenses punishable by imprisonment of more than six (6) years, their provisional dismissal shall become permanent two (2) years after issuance of the order without the case having been revived. Having invoked said rule before the petitioners-panel of prosecutors and before the Court of Appeals, the respondent is burdened to establish the essential requisites of the first paragraph thereof, namely: 1. the prosecution with the express conformity of the accused or the accused moves for a provisional (sin perjuicio) dismissal of the case; or both the prosecution and the accused move for a provisional dismissal of the case; 2. the offended party is notified of the motion for a provisional dismissal of the case; 3. the court issues an order granting the motion and dismissing the case provisionally; 4. the public prosecutor is served with a copy of the order of provisional dismissal of the case. - The foregoing requirements are conditions sine qua non to the application of the time-bar in the second paragraph of the new rule. The raison d etre for the requirement of the express consent of the accused to a provisional dismissal of a criminal case is to bar him from subsequently asserting that the revival of the criminal case will place him in double jeopardy

Criminal Procedure Rowena Daroy Morales


4 of the Rules of Court. It must be borne in mind that in crimes involving private interests, the new rule requires that the offended party or parties or the heirs of the victims must be given adequate a priori notice of any motion for the provisional dismissal of the criminal case. Such notice may be served on the offended party or the heirs of the victim through the private prosecutor, if there is one, or through the public prosecutor who in turn must relay the notice to the offended party or the heirs of the victim to enable them to confer with him before the hearing or appear in court during the hearing. - In the case at bar, even if the respondents motion for a determination of probable cause and examination of witnesses may be considered for the nonce as his motion for a provisional dismissal of the Criminal Cases, however, the heirs of the victims were not notified thereof prior to the hearing on said motion on March 22, 1999. It must be stressed that the respondent filed his motion only on March 17, 1999 and set it for hearing on March 22, 1999 or barely five days from the filing thereof. Although the public prosecutor was served with a copy of the motion, the records do not show that notices thereof were separately given to the heirs of the victims or that subpoenas were issued to and received by them - Since the conditions sine qua non for the application of the new rule were not present when Judge Agnir, Jr. issued his resolution, the State is not barred by the time limit set forth in the second paragraph of Section 8 of Rule 117 of the Revised Rules of Criminal Procedure. The State can thus revive or refile the Criminal Cases or file new Informations for multiple murder against the respondent. 2. NO - The Court agrees with the petitioners that to apply the time-bar retroactively so that the two-year period commenced to run on March 31, 1999 when the public prosecutor received his copy of the resolution of Judge Agnir, Jr. dismissing the criminal cases is inconsistent with the intendment of the new rule. Instead of giving the State two years to revive provisionally dismissed cases, the State had considerably less than two years to do so. Thus, Judge Agnir, Jr. dismissed the Criminal Cases on March 29, 1999. The new rule took effect on December 1, 2000. If the Court applied the new timebar retroactively, the State would have only one year and three months or until March 31, 2001 within which to revive these criminal cases. The period is short of the two-year period fixed under the new rule. - On the other hand, if the time limit is applied

a2010

page 112

Prof.
and the defendants filed their affidavit and counteraffidavits respectively. - In Jan. 1985, petitioners Bulaong and his counsel de Guzman submitted to City Fiscal of Pasay a replyaffidavit containing statements which are alleged to be libelous. Hence, Vistan and Buenaventura filed a complaint for libel against Bulaong and his counsel de Guzman with the City Fiscal. The latter conducted an investigation, and thereafter, filed an information for libel against petitioners. The said information was later amended on. - Petitioners moved to quash the Information on the ff. grounds: (1) that the facts charged do not constitute an offense; and 2) that the fiscal has no authority to file the Information. They further argue that the reply-affidavit was submitted and sworn to by petitioner Bulaong not only because he was required to do so by the investigating fiscal but also because it was in compliance with his legal and moral duty as complainant in the case for estafa against Vistan and Buenaventura and hence, the reply-affidavit belongs to the class of absolutely privileged communications - The assistant city fiscal filed an opposition to the motion to quash filed by petitioners. RTC of Pasay City denied the motion to quash. Petitioners filed MFR but was denied. Petitioners filed with CA a petition for certiorari, prohibition, mandamus, with preliminary injunction. CA dismissed petition for lack of merit. Hence, the instant petition for review on certiorari was filed ISSUE WON the procedure availed by petitioners after denial by the RTC of the motion to quash (MTQ) was correct HELD NO Ratio: Petitions for certiorari and prohibition are not the correct remedies against an order denying a motion to quash. The defendant should instead, go to trial without prejudice on his part to present the special defenses he had invoked in his motion and, if after trial on the merits, an adverse decision is rendered, to appeal therefrom in the manner authorized by law Reasoning: [a] Sec. 1, Rule 117 of the ROC provides that, upon arraignment, defendant shall immediately either move to quash the complaint or information or plead thereto, or do both and that, if the defendant moves to quash, without pleading, and the motion is withdrawn or overruled, he should

prospectively, the State would have two years from December 1, 2000 or until December 1, 2002 within which to revive the cases. This is in consonance with the intendment of the new rule in fixing the time-bar and thus prevent injustice to the State and avoid absurd, unreasonable, oppressive, injurious, and wrongful results in the administration of justice. The period from April 1, 1999 to November 30, 1999 should be excluded in the computation of the twoyear period because the rule prescribing it was not yet in effect at the time and the State could not be expected to comply with the time-bar. It cannot even be argued that the State waived its right to revive the criminal cases against respondent or that it was negligent for not reviving them within the two-year period under the new rule. To require the State to give a valid justification as a condition sine qua non to the revival of a case provisionally dismissed with the express consent of the accused before the effective date of the new rule is to assume that the State is obliged to comply with the time-bar under the new rule before it took effect. This would be a rank denial of justice. The State must be given a period of one year or two years as the case may be from December 1, 2000 to revive the criminal case without requiring the State to make a valid justification for not reviving the case before the effective date of the new rule. Although in criminal cases, the accused is entitled to justice and fairness, so is the State. Dispositive Motion for Reconsideration is GRANTED

PEOPLE v PANFILO LACSON


October 2003

BULAONG v CA (PEOPLE) 181 SCRA 618 MEDIALDEA; January 30, 1990


NATURE Petition for review on certiorari of the decision of CA FACTS - In March 1984, petitioner Bulaong filed with the RTC of Zambales an action for sum of money against Vistan, Buenaventura and Sta. Maria. Later, Vistan also filed a complaint against Bulaong for rescission of contract with damages. The said cases were consolidated and are pending trial. - In Nov. 1984, Bulaong filed a criminal complaint for estafa with the City Fiscal of Pasay against Vicente Vistan and Leonardo Buenaventura. The complainant

Criminal Procedure Rowena Daroy Morales


immediately plead, which means that trial must proceed. If, after trial on the merits, judgment is rendered adversely to the movant in the MTQ, he can appeal the judgment and raise the same defenses or objections earlier raised in his MTQ which would then be subject to review by the appellate court. [b] An order denying a MTQ, like an order denying a motion to acquit, is interlocutory and not a final order, and thus, not appealable. Neither can it be the subject of a petition for certiorari. Such order of denial may only be reviewed, in the ordinary course of law, by an appeal from the judgment, after trial. [c] In Collins vs. Wolfe and reiterated in Mill vs. Yatco, the accused, after the denial of his MTQ, should have proceeded with the trial of the case in the court below, and if final judgment is rendered against him, he could then appeal, and upon such appeal, present the questions which he sought to be decided by the appellate court in a petition for certiorari.[d] Whether or not the alleged libelous statements in the reply-affidavit are covered within the mantle of absolutely privileged communications, is a defense which petitioners could raise upon the trial on the merits, and, if that defense should fail, they could still raise the same on appeal. The MTQ the information for libel on the ground of qualified privilege, duly opposed by the prosecution, is properly denied, as the prosecution is entitled to prove at the trial that there was malice in fact on the part of the petitioners Dispositive Petition is DENIED. CA decision is AFFIRMED.

a2010

page 113

Prof.
accused cannot be deemed established beyond reasonable doubt. -Consequently, under the circumstances obtaining in this case, the ends of justice require that evidence be presented to determine the culpability of the accused. When a judgment has been entered by consent of an attorney without special authority, it will sometimes be set aside or reopened (Natividad vs. Natividad). Dispositive WHEREFORE, the judgment of respondent Appellate Court is REVERSED and this case is hereby ordered RE-OPENED and REMANDED to the appropriate Branch of the Regional Trial Court of Lucena City, for further reception of evidence. SO ORDERED.

the Stipulation of Facts. The Trial Court convicted petitioner-appellant. -On appeal, respondent Appellate Court upheld the Stipulation of Facts and affirmed the judgment of conviction. 1 ISSUE WON CA erred in affirming the decision of the RTC convicting the petitioner of the offense charged, despite the cold fact that the basis of the conviction was based solely on the stipulation of facts made during the pre-trial on August 8, 1985, which was not signed by the petitioner, nor by his counsel HELD YES. The CA committed a mistake. -The 1985 Rules on Criminal Procedure, which became effective on January 1, 1985, applicable to this case since the pre-trial was held on August 8, 1985, provides: "SEC. 4. Pre-trial agreements must be signed. No agreement or admission made or entered during the pre-trial conference shall be used in evidence against the accused unless reduced to writing and signed by him and his counsel." (Rule 118) The Rule is mandatory. Under the rule of statutory construction, negative words and phrases are to be regarded as mandatory while those in the affirmative are merely directory (McGee vs. Republic). The use of the term "shall" further emphasizes its mandatory character and means that it is imperative, operating to impose a duty which may be enforced (Bersabal vs. Salvador). And more importantly, penal statutes whether substantive and remedial or procedural are, by consecrated rule, to be strictly applied against the government and liberally in favor of the accused (People vs. Terrado). -The conclusion is inevitable, therefore, that the omission of the signature of the accused and his counsel, as mandatorily required by the Rules, renders the Stipulation of Facts inadmissible in evidence. The fact that the lawyer of the accused, in his memorandum, confirmed the Stipulation of Facts does not cure the defect because Rule 118 requires both the accused and his counsel to sign the Stipulation of Facts. What the prosecution should have done, upon discovering that the accused did not sign the Stipulation of Facts, as required by Rule 118, was to submit evidence to establish the elements of the crime, instead of relying solely on the supposed admission of the accused in the Stipulation of Facts. Without said evidence independent of the admission, the guilt of the

PEOPLE v UY 327 SCRA 335 DAVIDE; March 7, 2000


FACTS - Ramon Uy was caught by the PNP in a buy bust operation. 3 informations was filed against him for the illegal sale of 5.8564 grams of methamphetamine hydrochloride or "shabu," and possession of 401 grams of the same drug. - When arraigned, RAMON pleaded not guilty in each case. During the pre-trial, the parties agreed on a joint trial and to dispense with the testimony of Forensic Chemist Loreto F. Bravo. They also agreed on the marking of the exhibits for the prosecution. - During the trial, Uy claimed that he was merely framed. - The trial court gave credence to the prosecutions story of a legitimate buy bust and convicted him of 2 of the 3 charges against him. ISSUE 1. WON there was a legitimate buy bust 2. WON Uy agreed to waive the testimony of the Forensic Chemist during the pre-trial HELD 1. YES. - As has been repeatedly held, credence shall be given to the narration of the incident by the prosecution witnesses especially when they are police officers who are presumed to have performed their duties in a regular manner, unless there be evidence to the contrary; moreover in the absence of proof of motive to falsely impute such a serious crime against appellant, the presumption of

FULE v CA 162 SCRA 446 MELENCIO-HERRERA ; June 22, 1988


NATURE Petition for Review on Certiorari of the Decision of the CA FACTS -Appellate Court affirmed the judgment of the RTC of Lucena City that convicted Fule of Violation of BP 22 (The Bouncing Checks Law) on the basis of the Stipulation of Facts entered into between the prosecution and the defense during the pre-trial conference in the Trial Court. -Only the prosecution presented its evidence. Petitioner-appellant waived the right to present evidence and submitted a Memorandum confirming

Criminal Procedure Rowena Daroy Morales


regularity in the performance of official duty, as well as the findings of the trial court on the credibility of witnesses, shall prevail over appellants self-serving and uncorroborated claim of having been framed - Moreover, the defense of denial or frame-up, like alibi, has been viewed by the court with disfavor for it can just as easily be concocted and is a common and standard defense ploy in most prosecutions for violation of the Dangerous Drugs Act. 2. NO. - Uys premise is that at the pre-trial he did not waive the Forensic Chemists testimony but only "stipulated on the markings of the prosecutions evidence." Indeed, the records disclose that during the pre-trial, conducted immediately after the arraignment on 21 November 1995, Uy, duly represented by counsel de parte Atty. Gerardo Alberto, and the prosecution stipulated on the markings of the prosecutions exhibits, and agreed to dispense with the testimony of Forensic Chemist Loreto F. Bravo. - During the pre-trial, prosecution and defense agreed to stipulate on the markings of the following prosecutions exhibits, thereby dispensing will the testimony of Forensic chemist Loreto E. Bravo. - It may at once be noted that neither Uy nor his counsel made express admission that the contents of the plastic bags to "be marked" contain shabu. That Uy agreed to dispense with the testimony of Forensic Chemist Bravo may not be considered an admission of the findings of Bravo on the contents of the plastic bag. Strictly, from the tenor of the aforequoted portion of the Joint Order, it is clear that Uy and his counsel merely agreed to the marking of the exhibits, and the clause "thereby dispensing with the testimony of forensic Chemist Loreto E. Bravo" must be understood in that context. - Even granting for the sake of argument that Uy admitted during the pre-trial that the plastic bags contained shabu , the admission cannot be used in evidence against him because the Joint Order was not signed by Uy and his counsel. Section 4 of Rule 118 of the Rules of Court expressly provides: SEC. 40. Pre-trial agreements must be signed. No agreement or admission made or entered during the pre-trial conference shall be used in evidence against the accused unless reduced to writing and signed and his counsel. - The purpose of this requirement is to further safeguard the rights of the accused against improvident or unauthorized agreements or admissions which his counsel may have entered into without his knowledge, as he may have waived his presence at the pre-trial conference; eliminate any

a2010

page 114

Prof.
penalty next lower than that prescribed by law shall be imposed, but always in the proper period. Thus, the imposable penalty on James Andrew, by reason of his minority, is 1 degree lower than the statutory penalty. Reasoning - The penalty for the special complex crime of kidnapping and serious illegal detention with homicide and rape being death, one degree lower therefrom is reclusion perpetua. On the other hand, the penalty for simple kidnapping and serious illegal detention is reclusion perpetua to death. One degree lower therefrom is reclusion temporal. There being no aggravating and mitigating circumstance, the penalty to be imposed on James Andrew is reclusion temporal in its medium period. Applying the Indeterminate Sentence Law, he should be sentenced to suffer the penalty of 12 years of prision mayor in its maximum period, as minimum, to 17 years of reclusion temporal in its medium period, as maximum. Dispositive The MFR is GRANTED. For the crime of kidnapping and serious illegal detention with homicide and rape, James Andrew Uy is sentenced to reclusion perpetua; For the crime of simple kidnapping and serious illegal detention, the penalty of 12 years of prision mayor in its maximum period, as minimum, to 17 years of reclusion temporal in its medium period, as maximum.

doubt on the conformity of the accused to the facts agreed upon. - Nevertheless, Uy cannot take advantage of the absence of his and his counsels signatures on the pre trial order. They did not object when the prosecution presented the plastic bags and said that it contained shabu. Uy cannot now raise it for the first time on appeal. Objection to evidence cannot be raised for the first time on appeal. Dispositive Decision affirmed in toto

PEOPLE v LARRAAGA PER CURIAM; January 31, 2006


NATURE MFR filed by brothers James Anthony and James Andrew, both surnamed Uy, praying for the reduction of the penalties imposed upon the latter on the ground that he was a minor at the time the crimes were committed. FACTS - The Uy brothers were convicted of the crimes of special complex crime of kidnapping and serious illegal detention with homicide and rape; and simple kidnapping and serious illegal detention. The Uy brothers claim that James Andrew was only 17 years and 262 days old at the time the crimes were committed. He begs leave and pleads that we admit at this stage of the proceedings his Certificate of Live Birth issued by the NSO, and Baptismal Certificate. He prays that his penalty be reduced, as in the case of his brother James Anthony. - Since the entry in the birth certificate was not legible, the court required the SolGen to secure a clear and legible copy from the Civil Registrar of Cotabato as well as the NSO, and thereafter to file a comment on the issue of James Andrew's minority. The documents showed that James Andrew was indeed a minor when the crimes were committed. The SolGen recommended that the penalty imposed be reduced. ISSUE WON James Anthony's penalty should be reduced because he was a minor at the time the crime was committed HELD YES Ratio Article 68 of the RPC provides: Upon a person over fifteen and under eighteen years of age the

PEOPLE v QUIAZON 78 SCRA 513 FERNANDO; August 31, 1977


NATURE Plea of Acting Solicitor Vicente Mendoza to acquit the accused. FACTS - Judgment of lower court convicted Antonio Quiazon of abduction with rape. Acting Solicitor General Vicente V. Mendoza, instead of filing a brief for appellee, submitted a Manifestation recommending that the judgment of the lower court be reversed and another be entered acquitting him, the need for a thorough study of the record became evident. - Events started in a chance encounter between complainant, Virginia Salazar de la Cruz (Virginia), and appellant while they were passengers in a Baliuag Transit bus, both of them being residents thereof and bound for San Jose City, Nueva Ecija. Virginia was quite friendly, and during the trip was

Criminal Procedure Rowena Daroy Morales


leaning on Quiazon. Encouraged, Quiazon asked if he could visit her at home. Virginia said that instead they could meet in the public market of San Jose City. - Two days thereafter they met. Quiazon brought Virginia home and introduced her to his parents, announcing that she was going to be their daughterin-law. That same day they had sexual intercourse in the house of Quiazon. - After that day, it was not unexpected for such intimacies to be repeated. Every time Virginia visited Quiazon they had sexual intercourse. - May 1973: the accused was eating in the public market with his friend Rogelio Vigilia and the complainant Virginia. Suddenly Virginia stood up and left them because she saw her husband. The following morning, when Rogelio went to visit his brother-in-law, who is a neighbor of the complainant in Barrio Abar, he saw Virginia with contusions and a swollen face. He asked his brother-in-law what had happened, and was told that complainant's husband had beaten her. - Quiazon and Virgina seldom saw each other after the former learned of the latters marriage. However, they wrote each other letters, and even saw each other on countless occasions after the knowledge regarding the marriage surfaced. - Manifestation: "The complainant had earlier introduced herself to the accused as a widow. Antonio did not know that Virginia was in fact married, until sometime during the first week of May, 1973 when they were eating at the restaurant. When he learned that she was married, he told her to avoid him, but she answered that she could not, because she loved him. Antonio did not also try to avoid her because he loved her xxx Even Quiazons parents objected to the relationship, but to no avail. - To avoid being found out, Quiazon and Virginia traveled from barrio to barrio until they reached Barrio Armenia in Tarlac, where they stayed for more than a week. While on their way to the voting precinct which was near a P.C. Detachment, a P. C. soldier, Sgt. Daton, stopped them because he noticed that they were new in the place. The P. C. Officer asked her whether Antonio Quiazon was her husband, and she answered in the negative. At the P.C. Headquarters, Virginia was asked who her husband was, and she answer that her husband is Sgt. Gaudencio de la Cruz, an army man. It was then when the P.C. soldiers became interested in asking her why she was in Tarlac. The complainant answered that she was brought there by the accused and that Antonio abducted and raped her.

a2010

page 115

Prof.
attract attention. In addition, there was a police outpost near the market. Dispositive Decision is reversed, and the accused is acquitted.

- In Cabanatuan City the accused Antonio was detained in jail, by virtue of the complaint brought by Virginia against him. When he was in jail, Virginia visited him. She apologized to him and told him that she did not want what had happened to him, but she had to do it because she was afraid of her husband. ISSUE WON Quiazon is guity of abduction with rape. HELD NO, his guilt was not proven beyond reasonable doubt, and according to the Manifestation of the Acting Solicitor General, the evidence supports Quiazons innocence. Ratio Only if the judge below and the appellate tribunal could arrive at a conclusion that the crime has been committed precisely by the person on trial under such an exacting test should the sentence be one of conviction. It is thus required that every circumstance favoring his innocence be duly taken into account. The proof against him must survive the test of reason; the strongest suspicion must not be permitted to sway judgment. The conscience must be satisfied that on the defendant could be laid the responsibility for the offense charged; that not only did he perpetrate the act but that it amounted to a crime. Moral certainty is required. Reasoning - Art. 3, Section 14 (2) (Constitution) In all criminal prosecutions, the accused shall be presumed innocent until the contrary is proved xxx - It is precisely because of such notorious lack of any persuasive force in the testimony of complainant that the Manifestation asserted most emphatically that appellant could rely on the constitutional presumption of innocence, one of the most valuable rights of an accused person - The complainant alleges that on July 3, 1973 the accused, whom the complainant had never met before, suddenly grabbed her while she was in the public market of San Jose City and forced her to board a tricycle. Then he took her to the house of his parents where he ravished her. The abduction occurred in broad daylight, or at about 10:00 in the morning. The improbability of the complainant's charge is immediately visible from the time and locus where the crime was supposed to have been committed. The market, being a public place, was at its busiest at 10 in the morning. Virginia was also with a niece at the time. Any commotion would easily

CASTILLO v FILTEX 124 SCRA 900 ESCOLIN; September 30, 1983


NATURE Appeal from CFI Rizal decision FACTS -Artemio Castillo, an employee of FILTEX and a member of the Samahan ng Malaya Manggagawa sa Filtex (FFW), was charged together with others in the MTC Makati with the offense of slight physical injuries, for his alleged involvement in a mauling and shining incident which occurred sometime in July 1964 at the height of a strike called by the SAMAHAN. During the pendency of the case, Castillo was suspended from his job. -July 8, 1964: FILTEX and SAMAHAN entered into a Return Work Agreement: >par. 3: company employees against whom court cases are filed or to be filed, shall be suspended by the company upon filing of such cases by the fiscal with the proper courts for as long as the said cases shall remain pending in court >par. 4: in the event said employees are found innocent by the courts, the COMPANY agrees to reinstate them to their respective jobs with back wages minus whatever earnings they earned during the period of suspension; otherwise, if found guilty they shall remain dismissed; -After trial, the MTC Makati found Castillo guilty of slight physical injuries. -CFI Rizal dismissed the case (November 28, 1966) because complainant failed to appear at the scheduled trial. -Castillo asked for reinstatement and back wages. When FILTEX paid no head to his demands, he instituted action in CFI Rizal, claiming that dismissal of the criminal case justified his reinstatement and payment of back wages, pursuant to paragraph 4 of the Return to Work Agreement. -FILTEX filed motion to dismiss; grounds: lack of cause of action and want of jurisdiction, the case being allegedly within the exclusive jurisdiction of CIR. This motion was denied. -Pre-trial: the parties defined the principal issue Is Castillo entitled to reinstatement and back wages

Criminal Procedure Rowena Daroy Morales


after the dismissal of the charge against him in accordance with par. 4 of the "Return to Work Agreement? -Case was submitted for decision on the bases of the parties memoranda and stipulation of facts. CFI Rizal dismissed Castillos complaint, and ordered him to pay FILTEX P1thou as attorney's fees, plus costs. Reasoning: CFI Rizals dismissal of the case was only because of the failure of the complainant to appear at the scheduled trial. The agreement to reinstate an employee expressly states that there must be a finding of innocence by the courts. It did not stipulate that the case should be dismissed. -Hence, this appeal. ISSUE WON Castillo backwages is entitled to reinstatement and

a2010

page 116

Prof.
Petitioners Claim It is the contention of petitioner that the failure to arraign him is violative of his constitutional right to procedural due process, more specifically of his right to be informed of the nature and cause of the accusation against him and of his right to be heard by himself and counsel. - The Solicitor General, when asked to comment, agreed that the procedural defect was of such gravity as to render void the decision of the City Court affirmed by the Court of First Instance. The comment was considered as answer, with the case being submitted for decision. ISSUE WON the accuseds constitutional right to procedural due process was violated. HELD YES. The Constitution requires that the accused be arraigned so that he may be informed as to why be was indicted and what penal offense he has to face, to be convicted only on a showing that his guilt is shown beyond reasonable doubt with full opportunity to disprove the evidence against him. - It is at that stage of arraignment where in the mode and manner required by the Rules, an accused, for the first time, is granted the opportunity to know the precise charge that confronts him. It is imperative that he is thus made fully aware of possible loss of freedom, even of his life depending on the nature of the crime imputed to him. At the very least then, he must be fully informed of why the prosecuting arm of the state is mobilized against him. It is a vital aspect of the constitutional rights guaranteed him. It is not useless formality, much less an idle ceremony. - Petitioner was not arraigned at all and was not represented by counsel throughout the whole proceedings in the respondent City Court. It is indisputable then that there was a denial of petitioner's constitutional right to be heard by himself and counsel. - An equally fatal defect in the proceeding had before respondent Judge Senining was that notwithstanding its being conducted in the absence of petitioner, he was convicted. It was shown that after one postponement due to his failure to appear, the case was reset for hearing. When that date came, without petitioner being present, although his bondsman were notified, respondent Judge, as set forth in the comment of the Solicitor General, "allowed the prosecution to present its evidence. Thereupon, respondent City Court promulgated thedecision.

HELD YES. Since the criminal case was ultimately dismissed, the constitutional presumption of innocence in favor of the appellant should be applied. Castillos innocence need no longer be proved, since under the fundamental law his innocence is presumed. -While it is true that Castillo was convicted of the offense of slight physical injuries by MTC Makati, it is undisputed that on appeal, CFI Rizal dismissed the case for failure of the prosecution witnesses to appear. -ROC Rule 123, Sec 7: Trial de novo on appeal. An appealed case shall be tried in all respects anew in the courts of first instance as if it had been originally instituted in that court. -Applying this rule, the judgment of conviction rendered by MTC Makati was vacated upon perfection of the appeal, to be tried de novo in the CFI as if it were originally instituted therein. The phrase "to vacate" applied to a judgment means "to annul, to render void." -People vs. Dramayo: The starting point is the constitutional presumption of innocence - a right safeguarded the accused. Accusation is not, according to the fundamental law, synonymous with guilt. It is incumbent on the prosecution to demonstrate that culpability lies. Guilt must be shown beyond reasonable doubt. To such a standard this Court has always been committed. -There is need for the most careful scrutiny of the testimony of the state, both oral and documentary, independently of whatever defense is offered by the accused. Only if the judge below and the appellate

tribunal could arrive at a conclusion that the crime had been committed precisely by person on trial under such an exacting test could sentence be one of conviction. -It is thus required that every circumstance favoring his innocence be duly taken into account. The proof against him must survive the test of reason; the strongest suspicion must not be permitted to sway judgment. The conscience must be satisfied that on the defendant could be laid the responsibility for the offense charged; that not only did he perpetrate the act but that it amounted to a crime. So it has been held from the 1903 decision of United States v. Reyes. Interpretation of par. 4 of Return to Work Agreement -FILTEX: said paragraph requires an express finding of innocence by the court in order to entitle an employee to reinstatement and back wages; no such finding of innocence had been made because the criminal case was dismissed on a mere technicality; interpretation of said agreement should not be stretched to include a "mere presumption of innocence under the law." -SC: Constitutional provision on protection to labor constrains courts to interpret the agreement in question in favor of the claim of the laborer and against that of management. Those who are less fortunate in terms of economic well-being should be given preferential attention. States obligation to protect labor is welfare state concept vitalized. (Art. 4, Labor Code. Art. 1700, NCC. ^_^ hehe! ) Dispositive CFI Rizal decision set aside. Remand to Labor Arbiter of NLRC for determination of the amount of back wages.

BORJA v MENDOZA 77 SCRA 422 FERNANDO; June 20, 1977


FACTS - Notwithstanding the absence of an arraignment of petitioner Manuel Borja, accused of slight physical injuries, respondent Judge Romulo R. Senining proceeded with the trial in absentia and found the accused guilty of such offense. - An appeal was filed in the CFI of Cebu presided by respondent Judge Rafael T. Mendoza. - It was then alleged that without any notice to petitioner and without requiring him to submit his memorandum, a decision on the appealed case was rendered affirming the judgment of the City Court.

Criminal Procedure Rowena Daroy Morales


- It is the constitutional right of the accused to be heard in his defense before sentence is pronounced on him. Such "constitutional right is inviolate." There is no doubt that it could be waived, but here there was no such waiver, whether express or implied. - The provision in the present Constitution allowing trial to be held in absentia is unavailing. It cannot justify the actuation of respondent Judge Senining. Its language is clear and explicit. What is more, it is mandatory. Thus: "However, after arraignment, trial may proceed notwithstanding the absence of the accused provided that he has been duly notified and his failure to appear is unjustified." As pointed out then by the Solicitor General, the indispensable requisite for trial in absentia is that it should come "after arraignment." - Without the accused having been arraigned, it becomes academic to discuss the applicability of this exception to the basic constitutional right that the accused should be heard by himself and counsel. - The appeal to the Court of First Instance presided by respondent Judge Mendoza did not possess any curative aspect. Respondent considered the appeal taken by the petitioner as waiver of the defects in the proceedings in the respondent City Court. - Precisely, the appeal itself is tantamount to questioning those defects. In fact, the Memorandum in support of the appeal unmistakably raised as error the absence of petitioner at the arraignment and cited jurisprudence, commentaries and the rules to bolster his position. Specifically, the absence of an arraignment can be invoked at anytime in view of the requirements of due process to ensure a fair and impartial trial. Dispositive The petition was granted.

a2010

page 117

Prof.
- This rule is a restatement of the doctrine laid down in People vs. Apduhan. The searching inquiry of the trial court must be focused on: (1) the voluntariness of the plea, and (2) the full comprehension of the consequences of the plea. - The questions of the trial court failed to show the voluntariness of the plea of guilt of the appellant nor did the questions demonstrate appellant's full comprehension of the consequences of his plea. > The records do not clearly illustrate the personality profile of the accused. > The age, socio-economic status and educational background of the accused were not examined. > With regard to voluntariness, questions regarding the presence or absence of maltreatment of the accused are deemed insufficient when a record of events in the penal facility indicate that Alicando suffered a hematoma from being locked up in a cell with violent inmates upon his arrest. > With regard to comprehension, the trial court inadequately warned Alicando that a plea of guilt would result to a mandatory of penalty of death without explaining to him what mandatory meant. - The rule requires that after a free and intelligent plea of guilt the trial court must require the prosecution to prove the guilt of the appellant and the precise degree of his culpability beyond reasonable doubt. Rule 116, Sec. 3 modifies priorituis prudence that a plea of guilt even in capital offenses is sufficient to sustain a conviction charged in the information without need of further proof. 3. Some prosecution evidence, offered independently of the plea of guilt of the appellant, were inadmissible, yet were considered by the trial court convicting the appellant. Ratio Fruit of the poisonous tree doctrine: once the primary source (the "tree") is shown to have been unlawfully obtained, any secondary or derivative evidence (the "fruit") derived from it is also inadmissible. In other words, illegally seized evidence is obtained as a direct result of the illegal act, whereas the "fruit of the poisonous tree" is the indirect result of the same illegal act. The "fruit of the poisonous tree" is at least once removed from the illegally seized evidence, but it is equally inadmissible. Reasoning - The rule is based on the principle that evidence illegally obtained by the State should not be used to gain other evidence because the originally illegally obtained evidence taints all evidence subsequently obtained.

> At around 5:30 PM, Rebada, one of Penecillas neighbors, spotted Khazie by the window of Alicandos house. Khazie offered to buy yemas from Rebada but Alicando closed the window. Rebada then heard Khazie crying so she approached the house and saw through an opening between the floor and the door that Khazie was being raped. > Khazie did not come home so Romeo and his wife looked for her. Rebada did not tell them what she saw. > In the morning, Khazies corpse was found under the house of Santiago, another neighbor. Rebada then told the Penecillas what she knew. > Alicando was arrested and her verbally confessed his guilt to PO3 Tan without the assistance of counsel. Based on his confession and follow-up interrogations, Khazies slippers were recovered from Alicandos home along with a stained T-shirt and pillow. - June 29, 1994 Alicando was arraigned and pleaded guilty. After the plea of guilt, the trial court ordered the prosecution to present its evidence. - July 20, 1994 The trial court sentenced Alicando to death by electric chair or, if the penal facilities would be available by then, by gas poisoning. ISSUE WON the accused was properly meted the sentence of death HELD NO 1. Arraignment of the accused was null and void Ratio During arraignment, the complaint or the information should be read in a language or dialect which the accused understands. Reasoning - The trial judge failed to follow the procedure outlined in Rule 116 of the RoC. - The information was written in English and it was unknown whether or not the accused could understand English well. It could not be said with certainty that the accused was informed of the nature and cause of the accusation against him. 2. The plea of guilt was null and void. Ratio Rule 116, Sec. 3 provides that in a plea of guilt, the court should ascertain that the accused voluntarily entered into the plea and fully comprehends the ramifications of such a plea and, in addition, the prosecution should also be required to prove his guilt and the precise degree of culpability. Reasoning

PEOPLE v ALICANDO 251 SCRA 293 PUNO; December 12, 1995


NATURE Automatic review FACTS - Alicando was charged of rape with homicide for the death of Khazie Penecilla on June 12, 1994 in Iloilo City. In the process of raping Khazie, he choked her thus causing her death. > Khazies father Romeo was having a drink with two friends in Romeos house. Alicando eventually joined them. At around 4:30 PM, Romeos friends left.

Criminal Procedure Rowena Daroy Morales


- The Court admitted as evidence the things seized in Alicandos house. These are inadmissible evidence for they were gathered by PO3 Tan of as a result of custodial interrogation where appellant verbally confessed to the crime without the benefit of counsel. - This is in violation of Art. 3, Sec. 12 of the 1987 Constitution which requires the assistance of counsel for the accused as well as provides for the right of the accused to remain silent and to be informed of the nature of the accusation against him and that these rights cannot be waived subject to exceptions. A violation of this provision renders the evidence gathered inadmissible. - Even if the evidence gathered were admissible, they are still insufficient as evidence. > The alleged bloodstains on the pillow and shirt were never proven with laboratory tests. > There was no testimony that the shirt in question was worn by the accused when he committed the crime. It was not unnatural for him to have a shirt with bloodstains because he was a butcher. - The burden to prove that an accused waived his right to remain silent and the right to counsel before making a confession under custodial interrogation rests with the prosecution. It is also the burden of the prosecution to show that the evidence derived from confession is not tainted as "fruit of the poisonous tree." Dispositive The Decision convicting accused of the crime of Rape with Homicide and sentencing him to suffer the penalty of death is annulled and set aside and the case is remanded to the trial court for further proceedings.

a2010

page 118

Prof.
the guilt of the accused and the precise degree of his culpability. Nowhere in the rules does it state that an extra-judicial confession is a prerequisite for a conviction based on a plea of guilty. - The physical evidence objected to falls under the exclusionary rule. > The 1987 Constitution's exclusionary rules absolutely forbid evidence obtained from illegal searches and seizures or evidence resulting from uncounseled custodial investigations of accused individuals. > The doctrine is not without its exceptions, and the evidence in dispute in the instant case falls within those exceptions. + The discovery of the victim's body near the house of the accused would have naturally led authorities to undertake a more thorough investigation of the site, particularly in those areas where the victim was last seen. + Under one of the recognized exceptions of the fruit of the poisonous tree doctrine, the more appropriate question in such cases is whether the evidence to which the objection is made would not have been discovered at all but for the illegality or would have been discovered anyway by sources or procedures independent of the illegality. + Another exception refuses to treat the doctrine as absolutely sacred if the evidence in question would have been inevitably discovered under normal conditions. - There is adequate legal evidence to sustain the trial courts conviction with moral certainty. The testimony of a lone witness, free from signs of impropriety or falsehood, is sufficient to convict an accused even if uncorroborated.

that the bulk of proceedings in our trial courts, including the process of arraignment is conducted in the vernacular > Three things which need to be accomplished after the accused in a criminal case enters a plea of guilty to a capital offense, which have all been complied with in this case: 1. the court should conduct a searching inquiry into the voluntariness and full comprehension of the consequences of the accused's plea. There is no rule on conducting inquiry except that in People vs. Dayot, it was held that a searching inquiry ... compels the judge to content himself reasonably that the accused has not been coerced or placed under a state of duress - and that his guilty plea has not therefore been given improvidently - other by actual threats of physical harm from malevolent quarters or simply because of his, the Judge's, intimidating robes. 2. the lower court should require the prosecution to prove the guilt of the accused and the precise degree of his culpability 3. the court should inquire whether or not the accused wishes to present evidence on his behalf and should allow him to do so if he so desires - The plea of guilt was not improvident. > When the appellant pleaded guilty in open court, the appellant was clearly assisted by counsel. > The trial court, on its own, in fact went out of its way to repeatedly inform the defendant of the nature of his plea and the implications of the plea he was making. He was asked a number of times if he was sure of the plea he was making. > The records fail to indicate that appellant questioned his plea of guilty at any stage of the trial. He did not put up any defense with regard to the evidence and the testimonies and even directed the police to the location of the evidence. > The accuseds silence can counter the assertion of the Court that the plea of guilt was improvident. Silence is assent as well as consent, and may, where a direct and specific accusation of crime is made, be regarded under some circumstances as a quasi-confession. An innocent person will defend himself so silence can be understood as a person deferring to do just that. > The absence of an extra-judicial confession does not detract from the efficacy or validity of appellant's plea of guilty. It does not affect the requirement compelling the prosecution to prove

SEPARATE OPINION KAPUNAN [dissent]


- There was substantial compliance with the requirements for arraignment and plea. > There is nothing on the record which would warrant a finding that the information was not read in the language or dialect known to the appellant. > The rule on arraignment and plea does not absolutely require that the same be indicated in the record of every criminal case > Rule 116 contains nothing requiring trial courts, to indicate in the record the fact that the information was read in the language or dialect known to the defendant even if the same was in fact actually complied with by the lower court. And yet, even in Metro Manila alone, one observes

AQUINO v MILITARY COMMISSION 2 63 SCRA 546 ANTONIO; May 9, 1975


FACTS - After Martial Law was proclaimed, Benigno Aquino Jr. was arrested (on Sept 22, 1972), pursuant to General Order No. 2-A of the President for complicity in a conspiracy to seize political and state power in the country and to take over the Government. - On September 25, 1972, he sued for a writ of habeas corpus in which he questioned the legality of the proclamation of martial law and his arrest and detention.

Criminal Procedure Rowena Daroy Morales


- SC issued a writ of habeas corpus and heard the case. SC dismissed the petition and upheld the validity of martial law and the arrest and detention of petitioner. - In the present case, petitioner challenges the jurisdiction of military commissions to try him, alone or together with others, for illegal possession of firearms, ammunition and explosives, for violation of the Anti-Subversion Act and for murder. - When the proceedings before the Military Commission opened, petitioner questioned the fairness of the trial and announced that he did not wish to participate in the proceedings even as he discharged both his defense counsel of choice and his military defense counsel. - For the petitioner's assurance, a Special Committee was created to reinvestigate the charges against petitioner. Petitioner filed supplemental petition questioning the legality of the creation of the Special Committee. - On March 24, 1975, petitioner filed an "Urgent Motion for Issuance of Temporary Restraining Order Against Military Commission No. 2"; praying that said Commission be prohibited from proceeding with the perpetuation of testimony under its Order dated March 10, 1975, the same being illegal, until further orders from the Supreme Court. - On April 14, 1975, this Court also issued a restraining order against respondent Military Commission No. 2, restraining it from further proceeding with the perpetuation of testimony under its Order dated March 10, 1975 until the matter is heard thereto. - When this case was called for hearing, petitioner's counsel presented to this Court a Motion to Withdraw the petition and all other pending matters and/or incidents in connection therewith. ISSUES 1. WON the court has jurisdiction despite petitioners motion to withdraw 2. WON Military Commission No. 2 has been lawfully constituted and validly vested with jurisdiction to hear the cases against civilians, including the petitioner. 3. WON Administrative Order No. 355, creating the Special Committee strips the petitioner of his right to due process 4. WON the denial to an accused of an opportunity to cross-examine the witnesses against him in the preliminary investigation constitutes an infringement of his right to due process,

a2010

page 119

Prof.
of, incident to or in connection with the commission of said crimes" which were pending in the civil courts were ordered transferred to the military tribunals. This jurisdiction of the tribunal, therefore, operates equally on all persons in like circumstances. - The guarantee of due process is not a guarantee of any particular form of tribunal in criminal cases. A military tribunal of competent jurisdiction, accusation in due form, notice and opportunity to defend and trial before an impartial tribunal, adequately meet the due process requirement. Due process of law does not necessarily mean a judicial proceeding in the regular courts. The procedure before the Military Commission, as prescribed in PD No. 39, assures observance of the fundamental requisites of procedural due process, due notice, an essentially fair and impartial trial and reasonable opportunity for the preparation of the defense. - It is asserted that petitioner's trial before the military commission will not be fair and impartial, since the President had already prejudged petitioner's cases and the military tribunal is a mere creation of the President, and "subject to his control and direction." We cannot, however, indulge in unjustified assumptions. Prejudice cannot be presumed, especially if weighed against the great confidence and trust reposed by the people upon the President and the latter's legal obligation under his oath to "do justice to every man". Nor is it justifiable to conceive, much less presume, that the members of the military commission, the Chief of Staff of the AFP, the Board of Review and the Secretary of National Defense, with their corresponding staff judge advocates, as reviewing authorities, through whom petitioner's hypothetical conviction would be reviewed before reaching the President, would all be insensitive to the great principles of justice and violate their respective obligations to act fairly and impartially in the premises. This assumption must be made because innocence, not wrongdoing, is to be presumed. 3. NO - It was precisely because of petitioner's complaint that he was denied the opportunity to be heard in the preliminary investigation of his charges .The President created a Special Committee to reinvestigate the charges filed against him in the military commission. It is intended that the Committee should conduct the investigation with "utmost fairness, impartiality and objectivity" ensuring to the accused his constitutional right to due process, to determine whether "there is reasonable ground to believe that the offenses

5. WON the taking of testimonies and depositions were void 6. WON petitioner may validly waive his right to be present at his trial HELD 1. YES - The court denied the motion, since all matters in issue in this case have already been submitted for resolution, and they are of paramount public interest, it is imperative that the questions raised by petitioner on the constitutionality and legality of proceedings against civilians in the military commissions, pursuant to pertinent General Orders, Presidential Decrees and Letters of Instruction, should be definitely resolved. 2. YES - Military Commission No. 2 has been lawfully constituted and validly vested with jurisdiction to hear the cases against civilians, including the petitioner. Reasoning - The Court has previously declared that the proclamation of Martial Law is valid and constitutional and that its continuance is justified by the danger posed to the public safety. - To preserve the safety of the nation in times of national peril, the President of the Philippines necessarily possesses broad authority compatible with the imperative requirements of the emergency. On the basis of this, he has authorized in GO No. 8 the Chief of Staff of the AFP, to create military tribunals & try and decide cases "of military personnel and such other cases as may be referred to them." In GO No. 12, the military tribunals were vested with jurisdiction "exclusive of the civil courts", among others, over crimes against public order, violations of the Anti-Subversion Act, violations of the laws on firearms, and other crimes which, in the face of the emergency, are directly related to the quelling of the rebellion and preservation of the safety and security of the Republic. - Petitioner is charged with having conspired with certain military leaders of the communist rebellion to overthrow the government, furnishing them arms and other instruments to further the uprising. Under GO No. 12, jurisdiction over this offense has been vested exclusively upon military tribunals. It cannot be said that petitioner has been singled out for trial for this offense before the military commission. Pursuant to GO No. 12, all "criminal cases involving subversion, sedition, insurrection or rebellion or those committed in furtherance of, on the occasion

Criminal Procedure Rowena Daroy Morales


charged were in fact committed and the accused is probably guilty thereof." Petitioner, however, objected by challenging in his supplemental petition before this Court the validity of Administrative Order No. 355, on the pretense that by submitting to the jurisdiction of the Special Committee he would be waiving his right to cross-examination because Presidential Decree No. 77, which applies to the proceedings of the Special Committee, has done away with cross-examination in preliminary investigation. 4. NO - The Constitution "does not require the holding of preliminary investigations. The right exists only, if and when created by statute." It is "not an essential part of due process of law." The absence thereof does not impair the validity of a criminal information or affect the jurisdiction of the court over the case. As a creation of the statute it can, therefore, be modified or amended by law. - It is also evident that there is no curtailment of the constitutional right of an accused person when he is not given the opportunity to "cross-examine the witnesses presented against him in the preliminary investigation before his arrest, this being a matter that depends on the sound discretion of the Judge or investigating officer concerned." 5. NO, the taking of the testimony or deposition was proper and valid. - Petitioner does not dispute respondents' claim that on March 14, 1975, he knew of the order allowing the taking of the deposition of prosecution witnesses on March 31, to continue through April 1 to 4, 1975. - The provisions of PD No. 328, dated October 31, 1973, for the conditional examination of prosecution witnesses before trial, is similar to the provisions of Section 7 of Rule 119 of the Revised Rules of Court. - In Elago,the court said that the order of the court authorizing the taking of the deposition of the witnesses of the prosecution and fixing the date and time thereof is the one that must be served on the accused within a reasonable time prior to that fixed for the examination of the witnesses so that the accused may be present and cross-examine the witness. - 'The opportunity of cross-examination involves two elements: "(1) Notice to the opponent that the deposition is to be taken at the time and place specified, and "(2) A sufficient interval of time to prepare for examination and to reach the place,

a2010

page 120

Prof.
presence at all stages of his trial, there appears, therefore, no logical reason why petitioner, although he is charged with a capital offense, should be precluded from waiving his right to be present in the proceedings for the perpetuation of testimony, since this right, like the others aforestated, was conferred upon him for his protection and benefit. - It is also important to note that under Section 7 of Rule 119 of the Revised Rules of Court (Deposition of witness for the prosecution) the "Failure or refusal on the part of the defendant to attend the examination or the taking of the deposition after notice hereinbefore provided, shall be considered a waiver" - Presidential Decree No. 328 expressly provides that the failure or refusal to attend the examination or the taking of the deposition shall be considered a waiver. "

"(2) The requirements as to the interval of time are now everywhere regulated by statute * * *; the rulings in regard to the sufficiency of time are thus so dependent on the interpretation of the detailed prescriptions of the local statutes that it would be impracticable to examine them here. But whether or not the time allowed was supposedly insufficient or was precisely the time required by statute, the actual attendance of the party obviate any objection upon the ground of insufficiency, because then the party has actually had that opportunity of crossexamination for the sole sake of which the notice was required." 6. YES - Under the present Constitution, trial even of a capital offense may proceed notwithstanding the absence of the accused. It is now provided that "after arraignment, trial may proceed notwithstanding the absence of the accused provided that he has been duly notified and his failure to appear is unjustified." - On the basis of the aforecited provision of the Constitution which allows trial of an accused in absentia, the issue has been raised whether or not petitioner could waive his right to be present at the perpetuation of testimony proceedings before respondent Commission. - As a general rule, subject to certain exceptions, any constitutional or statutory right may be waived if such waiver is not against public policy. The personal presence of the accused from the beginning to the end of a trial for felony, involving his life and liberty, has been considered necessary and vital to the proper conduct of his defense. The "trend of modern authority is in favor of the doctrine that a party in a criminal case may waive irregularities and rights, whether constitutional or statutory, very much the same as in a civil case." - There are, certain rights secured to the individual by the fundamental charter which may be the subject of waiver. The rights of an accused to defend himself in person and by attorney, to be informed of the nature and cause of the accusation, to a speedy and public trial, and to meet the witnesses face to face, as well as the right against unreasonable searches and seizures, are rights guaranteed by the Constitution. They are rights necessary either because of the requirements of due process to ensure a fair and impartial trial, or of the need of protecting the individual from the exercise of arbitrary power. And yet, there is no question that all of these rights may be waived. Considering the aforecited provisions of the Constitution and the absence of any law specifically requiring his

SEPARATE OPINION (on waiver of presence only) CASTRO [concur and dissent]
- My understanding of the provisions of the new Constitution on waiver of presence in criminal proceedings is that such waiver may be validly implied principally in cases where the accused has jumped bail or has escaped, but certainly may not be asserted as a matter of absolute right in cases where the accused is in custody and his identification is needed in the course of the proceedings. - Thus, I voted for qualified waiver.- the accused may waive his presence in the criminal proceedings except at the stages where identification of his person by the prosecution witnesses is necessary. I might agree to the proposition of "total" waiver in any case where the accused agrees explicitly and unequivocally in writing signed by him or personally manifests clearly and indubitably in open court and such manifestation is recorded, that whenever a prosecution witness mentions a name by which the accused is known the witness is referring to him and to no one else.

TEEHANKEE [dissent]
- Petitioners presence at the proceedings could not be compelled by virtue of his express waiver thereof as explicitly allowed by the Constitution and by P.D. No. 328 itself. - Petitioner's submittal that he cannot be compelled to be present at the proceedings even against his will by virtue of his express waiver is meritorious. Whereas previously such right of waiver of the

Criminal Procedure Rowena Daroy Morales


accused's presence in criminal proceedings was generally recognized save in capital cases (leading to the suspension of trial whenever the accused was at large) or where the accused was in custody although for a non-capital offense, the 1973 Constitution now unqualifiedly permits trial in absentia even of capital cases, and provides that "after arraignment, trial may proceed notwithstanding the absence of the accused provided that he has been duly notified and his failure to appear is unjustified," thus recognizing the right of an accused to waive his presence. P.D. No. 328 under which the perpetuation proceedings are being conducted in military commissions (as the counterpart rule for similar proceedings before the regular civil courts, as provided in Rule 119, section 7 of the Rules of Court) explicitly provides that after reasonable notice to an accused to attend the perpetuation proceedings, the deposition by question and answer of the witness may proceed in the accused's absence and "the failure or refusal to attend the examination or the taking of the deposition shall be considered a waiver." Thus, an accused's right of total waiver of his presence either expressly or impliedly by unjustified failure or refusal to attend the proceedings is now explicitly recognized and he cannot be compelled to be present as against his express waiver.

a2010

page 121

Prof.
Jr. vs. Military Commission No. 2 and held that "he cannot be validly compelled to appear and be present during the trial of this case." - Petitioner prays that the order of respondent judge be annulled and set aside and that private respondent Rodolfo Valdez, Jr. be compelled to appear during the trial of the criminal case whenever required to do so by the trial court. - Private respondent claims that Sec 19, Article IV of the 1973 Constitution grants him absolute right to absent himself from the trial of the case filed against him despite the condition of his bail bond that he "will at all times hold himself amenable to the orders and processes of the Court." ISSUE WON the judge erred in granting private respondents manifestation to waive his right to be present during trial HELD YES - Article IV of the 1973 Constitution, Section 19 thereof provides: SEC. 19. In all criminal prosecutions, the accused shall be presumed innocent until the contrary is proved, and shall enjoy the right to be heard by himself and counsel, to be informed of the nature and cause of the accusation against him, to have a speedy, impartial, and public trial, to meet the witnesses face to face, and to have compulsory process to secure the attendance of witnesses and the production of evidence in his behalf. However, after arraignment, trial may proceed notwithstanding the absence of the accused provided that he has been duly notified and his failure to appear is unjustified. - The 1973 Constitution now unqualifiedly permits trial in absentia even of capital offenses, provided that after arraignment he may be compelled to appear for the purpose of Identification by the witnesses of the prosecution, or provided he unqualifiedly admits in open court after his arraignment that he is the person named as the defendant in the case on trial. - The reason for requiring the presence of the accused, despite his waiver, is, if allowed to be absent in all the stages of the proceedings without giving the People's witnesses the opportunity to Identify him in court, he may in his defense say that he was never Identified as the person charged in the information and, therefore, is entitled to an acquittal.

for the authorities to enforce execution of any adverse judgment. But I cannot see why an accused should be compelled to be present at the trial when he prefers perhaps the solitude of his cell to pray either for forgiveness, if he knows he is guilty, or, if he is innocent, for God to illumine the court so there would be unerring justice in his case. (hehehe) - My understanding is that the problem of identification of an accused may be adequately solved without violating the justified wishes of the accused to be left alone. To start with, if he is referred to by the witnesses of the prosecution by name, the court may presume that the amused who has acknowledged his true name at the arraignment is the one indicated.

BORJA v MENDOZA [SUPRA, PAGE 78] PEOPLE v PRESIDING JUDGE OF URDANETA 125 SCRA 269 RELOVA; October 26, 1983
NATURE Petition for certiorari FACTS - Private respondent Rodolfo Valdez, Jr. is charged with murder before the RTC of Pangasinan, in Urdaneta. He is out on a P30,000.00 bail bond which contains the following conditions: The aforenamed, as bondsmen, hereby jointly and severally undertake that the above-mentioned defendant, as principal therein will appear and answer the charge above-mentioned in whatever Court it may be tried, and will at all times hold himself amenable to the orders and processes of the Court, and if convicted, will appear for judgment, and render himself to the execution thereof; or that if he fails to perform any of these conditions will pay to the Republic of the Philippines the sum of Thirty Thousand Pesos (P30,000.00) ... - After his arraignment, Valdez, thru his counsel, manifested orally in open court that he was waiving his right to be present during the trial. The prosecuting fiscal moved that Valdez be compelled to appear and be present at the trial so that he could be identified by prosecution witnesses. Respondent judge sustained the position of private respondent who cited the majority opinion in Aquino,

BARREDO [concur]
- Petitioner has the right to waive his presence at the perpetuation proceedings before the respondent Commission.I find eminent merit in the contention of petitioner that even for identification purposes he cannot be made to be present at the trial against his will. Since under the Constitution, trial of criminal cases in the absence of the accused is allowed, when after the arraignment and in spite of due notice he fails to appear without justification, pursuant to Section 19 of the Bill of Rights or Article IV. - I can understand why an accused has to be present at the arraignment and at the reading of the sentence. In the former, it has to be known to the court that he is indeed the person charged and that he personally understands the accusation against him. More importantly, the plea must be entered by him personally to avoid any misconstruction or misrepresentation, innocent or otherwise. In the latter, it is essential that the accused himself, should be aware from personal knowledge what is the verdict of the court, and if it be conviction, what is the penalty to be served by him. These are matters too personal to permit delegation. At the same time, his presence makes it simpler in the public interest

Criminal Procedure Rowena Daroy Morales


- Furthermore, it is possible that a witness may not know the name of the culprit but can Identify him if he sees him again, in which case the latter's presence in court is necessary. Dispositive petition granted and the assailed order of respondent judge is ANNULLED and SET ASIDE

a2010

page 122

Prof.
prosecution moved that the hearing continue in accordance with the constitutional provision authorizing trial in absentia under certain circumstances. The respondent judge denied the motion, however, and suspended all proceedings until the return of the accused. ISSUE WON the judge erred in suspending the proceedings HELD YES Ratio Under Art.IV Sec.19, the prisoner cannot by simply escaping thwart his continued prosecution and possibly eventual conviction provided only that: a) he has been arraigned; b) he has been duly notified of the trial; and c) his failure to appear is unjustified. Reasoning - The rule is found in the last sentence of Article IV, Section 19, of the 1973 Constitution: In all criminal prosecution, the accused shall be presumed innocent until the contrary is proved and shall enjoy the right to be heard by himself and counsel, to he informed of the nature and cause of the accusation against him, to have a speedy, impartial, and public trial, to meet the witnesses face to face, and to have compulsory process to secure the attendance of witnesses and the production of evidence in his behalf. However, after arraignment, trial may proceed notwithstanding the absence of the accused provided that he has been duly notified and his failure to appear is unjustified. - The purpose of this rule is to speed up the disposition of criminal cases, trial of which could in the past be indefinitely deferred, and many times completely abandoned, because of the defendant's escape. - the fugitive is now deemed to have waived such notice precisely because he has escaped, and it is also this escape that makes his failure to appear at his trial unjustified. Escape can never be a legal justification. - The right to be present at one's trial may now be waived except only at that stage where the prosecution intends to present witnesses who will Identify the accused. - the defendant's escape will be considered a waiver of this right and the inability of the court to notify him of the subsequent hearings will not prevent it from continuing with his trial. He will be deemed to have received due notice.

ISSUE WON the accused, despite having waived his presence at the trial, may still be compelled to be present in the same trial when he is to be identified HELD YES. Stare Decisis. Reasoning - The rule adopted by the Court in the case of Aquino vs. Military Commission No. 2 (supra) is that while the accused may waive his presence at the trial of the case, his presence may be compelled when he is to be identified. The Court said: Since only 6 Justices are of the view that petitioner may waive his right to be present at all stages of the proceedings while five 5 Justices are in agreement that he may so waive such right, except when he is to he identified, the result is that the respondent Commission's Order requiring his presence at all times during the proceedings before it should be modified, in the sense that petitioners presence shall be required only in the instance just indicated. Dispositive Petition GRANTED, orders of respondent Judge ANNULLED and SET ASIDE. Judge is ordered to issue the necessary process to compel the attendance of the accused at the hearing of the criminal case for purposes of identification. Temporary TRO lifted and set aside.

PEOPLE v MACARAEG 141 SCRA 37 CONCEPCION; January 14, 1986


NATURE Petition for certiorari and mandamus with preliminary injunction to review order of CFI of Pangasinan FACTS - Private Respondent Vasco Valdez was charged with Homicide before the CFI of Pangasinan for the death of one Severs Paulo and posted bail for his provisional release. Attached to the bail bond was a waiver stipulating that the trial may proceed in his absence. - When the case was called for trial, the prosecution presented Welino Paulo, as its 1st witness, who when asked if he could identify the accused, answered in the affirmative. Since the accused was not present in court, the prosecution asked the court to order the presence of the accused so that he could be identified. Counsel for accused objected to the motion by invoking the waiver in the bail bond and contended that the absence of the accused is part of his defense. - Respondent Judge Daniel Macaraeg of the CFI, invoking the case of Aquino v Military Commission No.2, denied the motion: The issue at bar was one of those squarely raised in the Aquino case where six out of ten Justices voted that the accused may not be compelled to be present during the trial when he is to be identified by the witnesses of the prosecution while four voted that the accused may be compelled in this instance. The reason of the majority is that the accused must not be compelled to assist the prosecution in proving its case. - The prosecution moved for reconsideration but respondent Judge denied the motion. Prosecution then filed this petition with prayer for a TRO. The SC granted the petition and issued a TRO, restraining the respondent Court from further proceeding with the criminal case.

PEOPLE v SALAS (ABONG, DE LEON, ET AL) 143 SCRA 163 CRUZ; July 29, 1986
NATURE Certiorari and Mandamus FACTS - Mario Abong was originally charged with homicide in the CFI of Cebu but before he could be arraigned the case was reinvestigated on motion of the prosecution. As a result of the reinvestigation, an amended information was filed, with no bail recommended, to which he pleaded not guilty. - While trial was in progress, the prisoner, taking advantage of the first information for homicide, succeeded in deceiving the city court of Cebu into granting him bail and ordering his release; and so he escaped. - Respondent judge Salas, learning later of the trickery, cancelled the illegal bail bond and ordered Abong's re-arrest. But he was gone. Nonetheless, the

Criminal Procedure Rowena Daroy Morales


Dispositive the order of the trial court denying the motion for the trial in absentia of the accused is set aside.

a2010

page 123

Prof.
August 22, 1973 and in the said arraignment he pleaded not guilty. He was also informed of the scheduled hearings set on September 18 and 19, 1973 and this is evidenced by his signature on the notice issued by the lower Court. It was also proved by a certified copy of the Police Blotter that private respondent escaped from his detention center. No explanation for his failure to appear in court in any of the scheduled hearings was given. Even the trial court considered his absence unjustified - The contention of the respondent judge that the right of the accused to be presumed innocent will be violated if a judgment is rendered as to him is untenable. He is still presumed innocent. A judgment of conviction must still be based upon the evidence presented in court. Such evidence must prove him guilty beyond reasonable doubt. Also, there can be no violation of due process since the accused was given the opportunity to be heard. INTENT OF THE LEGISLATURE: . . . The Constitutional Convention felt the need for such a provision as there were quite a number of reported instances where the proceedings against a defendant had to be stayed indefinitely because of his non- appearance. What the Constitution guarantees him is a fair trial, not continued enjoyment of his freedom even if his guilt could be proved. With the categorical statement in the fundamental law that his absence cannot justify a delay provided that he has been duly notified and his failure to appear is unjustified, such an abuse could be remedied. That is the way it should be, for both society and the offended party have a legitimate interest in seeing to it that crime should not go unpunished. 3. NO - The 1985 Rules on Criminal Procedure, particularly Section 1 (c) of Rule 115 clearly reflects the intention of the framers of our Constitution, to wit: ... The absence of the accused without any justifiable cause at the trial on a particular date of which he had notice shall be considered a waiver of his right to be present during that trial. When an accused under custody had been notified of the date of the trail and escapes, he shall be deemed to have waived his right to be present on said date and on all subsequent trial dates until custody in regained.... - An escapee who has been duly tried in absentia waives his right to present evidence on his own behalf and to confront and cross-examine witnesses who testified against him. Dispositive The judgment of the trial court in so far as it suspends the proceedings against the private

GIMENEZ v NAZARENO 160 SCRA 1 GANCAYCO; April 15 1988


NATURE Petition for certiorari and mandamus FACTS - Accused Samson Suan, Alex Potot, Rogelio Mula, Fernando Cargando, Rogelio Baguio and the herein private respondent Teodoro de la Vega Jr., were charged with the crime of murder on August 3, 1973. On August 22, 1973 all the above-named. accused were arraigned and each of them pleaded not guilty to the crime charged. Following the arraignment, the respondent judge, Hon. Ramon E. Nazareno, set the hearing of the case for September 18, 1973 at 1:00 o'clock in the afternoon. All the acused including private respondent, were duly informed of this. - Before the scheduled date of the first hearing the private respondent escaped from his detention center and on the said date, failed to appear in court. This prompted the fiscals handling the case (the petitioners herein) to file a motion with the lower court to proceed with the hearing of the case against all the accused praying that private respondent de la Vega, Jr. be tried in absentia invoking the application of Section 19, Article IV of the 1973 Constitution which provides: SEC. 19. In all criminal prosecution, the accused shall be presumed innocent until the contrary is proved, and shall enjoy the right to be heard by himself and counsel, to be informed of the nature and cause of the accusation against him, to have a speedy, impartial, and public trial, to meet the witnesses face to face, and to have compulsory process to the attendance of witnesses and the production of evidence in his behalf. However, after arraignment trial may proceed notwithstanding the absence of the accused provided that he has been duly notified and his failure to appear is unjustified. - Pursuant to the above-written provision, the lower court proceeded with the trial of the case but nevertheless gave the private respondent the opportunity to take the witness stand the moment he shows up in court. After due trial, or on November 6,1973, the lower court rendered a decision

dismissing the case against the five accused while holding in abeyance the proceedings against the private respondent. On November 16,1973 the petitioners filed a Motion for Reconsideration questioning the above-quoted dispositive portion on the ground that it will render nugatory the constitutional provision on "trial in absentia" cited earlier. However, this was denied by the lower court in an Order dated November 22, 1973. Hence, this petition. ISSUES 1. WON a court loses jurisdiction over an accused who after being arraigned, escapes from the custody of the law 2. WON trial in absentia is warranted 3. WON under Section 19, Article IV of the 1973 Constitution, an accused who has been duly tried in absentia retains his right to present evidence on his own behalf and to confront and cross-examine witnesses who testified against him HELD 1. NO - It is not disputed that the lower court acquired jurisdiction over the person of the accused-private respondent when he appeared during the arraignment on August 22,1973 and pleaded not guilty to the crime charged. In cases criminal, jurisdiction over the person of the accused is acquired either by his arrest for voluntary appearance in court. Such voluntary appearance is accomplished by appearing for arraignment as what accused-private respondent did in this case. Jurisdiction once acquired is not lost upon the instance of parties but continues until the case is terminated. To capsulize the foregoing discussion, suffice it to say that where the accused appears at the arraignment and pleads not guilty to the crime charged, jurisdiction is acquired by the court over his person and this continues until the termination of the case, notwithstanding his escape from the custody of the law. 2. YES - Going to the second part of Section 19, Article IV of the 1973 Constitution aforecited a "trial in absentia"may be had when the following requisites are present: (1) that there has been an arraignment; (2) that the accused has been notified; and (3) that he fails to appear and his failure to do so is unjustified.In this case, all the above conditions were attendant calling for a trial in absentia. As the facts show, the private respondent was arraigned on

Criminal Procedure Rowena Daroy Morales


respondent Teodoro de la Vega, Jr. is reversed and set aside. The respondent judge is hereby directed to render judgment upon the innocence or guilt of the herein private respondent Teodoro de la Vega, Jr. in accordance with the evidence adduced and the applicable law.

a2010

page 124

Prof.
The defendant was found guilty on count 4 as well as counts 1, 2, 3 and 7 and was sentenced to death and to pay a fine of P20,000. ISSUE WON the judgment must be reversed because of the trial court's failure to appoint "another attorney de oficio for the accused in spite of the manifestation of the attorney de oficio (who defended the accused at the trial) that he would like to be relieved for obvious reasons." HELD NO - The appellate tribunal will indulge reasonable presumptions, in favor of the legality and regularity of all the proceedings of the trial court, including the presumption that the accused was not denied the right to have counsel. (U. S. vs. Labial, 27 Phil., 82.) It is presumed that the procedure prescribed by law has been observed unless it is made to appear expressly to the contrary. (U. S. vs. Escalante, 36 Phil., 743.) The fact that the attorney appointed by the trial court to aid the defendant in his defense expressed reluctance to accept the designation because, as the present counsel assumes, he did not sympathize with the defendant's cause, is not sufficient to overcome this presumption. The statement of the counsel in the court below did no necessarily imply that he did not perform his duty to protect the interest of the accused. As a matter of fact, the present counsel "sincerely believes that the said Attorney Carin did his best, although it was not the best of a willing worker." We do not discern in the record any indication that the former counsel did not conduct the defense to the best of his ability. If Attorney Carin did his best as a sworn member of the bar, as the present attorney admits, that was enough; his sentiments did not cut any influence in the result of the case and did not imperil the rights of the appellant.

PEOPLE v SALAS CRUZ; July 29, 1986


NATURE Special civil actions, certiorari and mandamus. FACTS - ABONG was originally charged with homicide in CFI Cebu but before he could be arraigned the case was reinvestigated. An amended information was filed as a result, with no bail recommended, to which ABONG pleaded not guilty. During the trial, ABONG, taking advantage of the first information for homicide, succeeded in deceiving the court into granting him bail and ordering his release; and so he escaped. Judge SALAS, learning later of the trickery, cancelled the illegal bail bond and ordered ABONG's re-arrest. Meanwhile, the prosecution moved that the hearing continue in accordance with the constitutional provision authorizing trial in absentia. SALAS denied the motion, however, and suspended all proceedings until the return of ABONG. Hence, the present petitions. ISSUE WON J. SALAS is correct in disallowing trial in absentia of ABONGs case HELD NO - The purpose of the constitutional rule that after arraignment, trial may proceed notwithstanding the absence of the accused provided that he has been duly notified and his failure to appear is unjustified,10 is to speed up the disposition of criminal cases, trial of which could in the past be indefinitely deferred, and many times completely abandoned, because of the defendant's escape. Now, the prisoner cannot by simply escaping thwart his continued prosecution provided only that: a) he has been arraigned; b) he has been duly notified of the trial; and c) his failure to appear is unjustified. - J. SALAS was probably still thinking of the old doctrine when he ruled that trial in absentia of the
10

escapee could not be held because he could not be duly notified. He forgets that the fugitive is now deemed to have waived such notice precisely because he has escaped, and it is also this escape that makes his failure to appear at his trial unjustified. The right to be present at one's trial may now be waived except only at that stage where the prosecution intends to present witnesses who will identify the accused.11 Under [Sec.14(2), 1987 Const.], the defendant's escape will be considered a waiver of this right and the inability of the court to notify him of the subsequent hearings will not prevent it from continuing with his trial. He will be deemed to have received due notice. The same fact of his escape will make his failure to appear unjustified because he has, by escaping, placed himself beyond the pale, and protection, of the law. - ABONG should be prepared to bear the consequences of his escape, including forfeiture of the right to be notified of the subsequent proceedings and of the right to adduce evidence on his behalf and refute the evidence of the prosecution, not to mention a possible or even probable conviction. Dispositive Order of J. SALAS was SET ASIDE, and he was directed to continue hearing ABONGs case in absentia as long as he has not reappeared, until it is terminated.

PEOPLE v PRIETO (alias EDDIE VALENCIA) 80 Phil 138 TUASON: January 29, 1948
NATURE APPEAL from a judgment of the People's Court FACTS - The appellant was prosecuted in the People's Court for treason on 7 counts. After pleading not guilty he entered a plea of guilty to counts 1, 2, 3 and 7, and maintained the original plea as to counts 4, 5 and 6. The special prosecutor introduced evidence only on count 4, stating with reference to counts 5 and 6 that he did not have sufficient evidence to sustain them. - The attorney de officio manifested that he would like to be relieved from his assignment.

JOHNSON v ZERBST 304 US 458 BLACK; May 23, 1938


NATURE Appeal from the decision of the District Court denying the petition for habeas corpus which the Court of Appeals affirmed FACTS

11

1973 Const, ART. IV, Sec.19. Now, ART. III, Sec.14(2), 1987 Const.

Citing Aquino v. Mil. Commission No. 2 and People v. Presiding Judge. See p.9 of outline.

Criminal Procedure Rowena Daroy Morales


- Petitioner and one Bridwell were arrested in Charleston, S.C., November 21, 1934, charged with feloniously uttering and passing four counterfeit twenty-dollar Federal Reserve notes and possessing twenty-one such notes. Both were then enlisted men in the United States Marine Corps, on leave. They were bound over to await action of the United States Grand July, but were kept in jail due to inability to give bail. January 21, 1935, they were indicted; January 23, 1935, they were taken to court and there first give notice of the indictment; immediately were arraigned, tried, convicted, and sentenced that day to four and one-half years in the penitentiary; and January 25, were transported to the Federal Penitentiary in Atlanta. While counsel had represented them in the preliminary hearings before the commissioner in which they-some two months before their trial-were bound over to the Grand Jury, the accused were unable to employ counsel for their trial. Upon arraignment, both pleaded not guilty, said that they had no lawyer, and-in response to an inquiry of the court-stated that they were ready for trial. They were then tried, convicted, and sentenced, without assistance of counsel. - It appears from the opinion of the District Judge denying habeas corpus that he believed petitioner was deprived, in the trial court, of his constitutional right under the provision of the Sixth Amendment, that, 'In all criminal prosecutions,the accused shall enjoy the right ... to have the Assistance of Counsel for his defense.' However, he held that proceedings depriving petitioner of his constitutional right to assistance of counsel were not sufficient 'to make the trial void and justify its annulment in a habeas corpus proceeding, but that they constituted trial errors or irregularities which could only be corrected on appeal.' - The Court of Appeals affirmed ISSUE WON the remedy of habeas corpus render the conviction of the petitioner void when there is a violation of the right to counsel, the sixth amendment HELD YES - Compliance with this constitutional mandate is an essential jurisdictional prerequisite to a federal court's authority to deprive an accused of his life or liberty. When this right is properly waived, the assistance of counsel is no longer a necessary element of the court's jurisdiction to proceed to

a2010

page 125

Prof.
his conviction to a sufficient extent to test the jurisdiction of the state court to proceed to judgment against him. ... '... it is open to the courts of the United States, upon an application for a writ of habeas corpus, to look beyond forms and inquiry into the very substance of the matter ....' - If this requirement of the Sixth Amendment is not complied with, the court no longer has jurisdiction to proceed. The judgment of conviction pronounced by a court without jurisdiction is void, and one imprisoned thereunder may obtain release by habeas corpus. A judge of the United States-to whom a petition for habeas corpus is addressed-should be alert to examine 'the facts for himself when if true as alleged they make the trial absolutely void.' - It must be remembered, however, that a judgment cannot be lightly set aside by collateral attack, even on habeas corpus. When collaterally attacked, the judgment of a court carries with it a presumption of regularity. Where a defendant, without counsel, acquiesces in a trial resulting in his conviction and later seeks release by the extraordinary remedy of habeas corpus, the burden of proof rests upon him to establish that he did not competently and intelligently waive his constitutional right to assistance of Counsel. If in a habeas corpus hearing, he does meet this burden and convinces the court by a preponderance of evidence that he neither had counsel nor properly waived his constitutional right to counsel, it is the duty of the court to grant the writ. Dispositive The cause is reversed and remanded to the District Court for determination whether petitioner did not competently and intelligently waive his right to counsel. If court finds for petitioner the decision of the district court convicting petitioner must be declared void.

conviction and sentence. If the accused, however, is not represented by counsel and has not competently and intelligently waived his constitutional right, the Sixth Amendment stands as a jurisdictional bar to a valid conviction and sentence depriving him of his life or his liberty. Ratio The purpose of the constitutional guaranty of a right to counsel is to protect an accused from conviction resulting from his own ignorance of his legal and constitutional rights, and the guaranty would be nullified by a determination that an accused's ignorant failure to claim his rights removes the protection of the Constitution. True, habeas corpus cannot be used as a means of reviewing errors of law and irregularities-not involving the question of jurisdiction-occurring during the course of trial; and the 'writ of habeas corpus cannot be used as a writ of error.' These principles, however, must be construed and applied so as to preserve-not destroy-constitutional safeguards of human life and liberty. The scope of inquiry in habeas corpus proceedings has been broadened-not narrowed-since the adoption of the Sixth Amendment. In such a proceeding, 'it would be clearly erroneous to confine the inquiry to the proceedings and judgment of the trial court' and the petitioned court has 'power to inquire with regard to the jurisdiction of the inferior court, either in respect to the subject-matter or to the person, even if such inquiry involves an examination of facts outside of, but not inconsistent with, the record.' Congress has expanded the rights of a petitioner for habeas corpus and the '... effect is to substitute for the bare legal review that seems to have been the limit of judicial authority under the common-law practice, and under the act of 31 Car. II, chap. 2, a more searching investigation, in which the applicant is put upon his oath to set forth the truth of the matter respecting the causes of his detention, and the court, upon determining the actual facts, is to 'dispose of the party as law and justice require.' - 'There being no doubt of the authority of the Congress to thus liberalize the common-law procedure on habeas corpus in order to safeguard the liberty of all persons within the jurisdiction of the United States against infringement through any violation of the Constitution or a law or treaty established thereunder, it results that under the sections cited a prisoner in custody pursuant to the final judgment of a state court of criminal jurisdiction may have a judicial inquiry in a court of the United States into the very truth and substance of the causes of his detention, although it may become necessary to look behind and beyond the record of

PEOPLE v HOLGADO 85 PHIL 752 MORAN; March 22, 1950


FACTS - Appellant Frisco Holgado was charged in the court of First Instance of Romblon with slight illegal detention because according to the information, being a private person, he did "feloniously and without justifiable motive, kidnap and detain one Artemia Fabreag in the house of Antero Holgado for about eight hours thereby depriving said Artemia Fabreag of her personal liberty."

Criminal Procedure Rowena Daroy Morales


- During the trial, he plead guilty as he was without a lawyer, and that a certain Numeriano Ocampo told Holgado to plead guilty. The Court reserved the sentence for a two days despite the fiscals assurances that the certain Numeriano Ocampo has been investigated and found without evidence to link him to the crime - It must be noticed that in the caption of the case as it appears in the judgment above quoted, the offense charged is named SLIGHT ILLEGAL DETENTION while in the body of the judgment if is said that the accused "stands charged with the crime of kidnapping and serious illegal detention." In the formation filed by the provincial fiscal it is said that he "accuses Frisco Holgado of the crime of slight illegal detention." The facts alleged in said information are not clear as to whether the offense is named therein or capital offense of "kidnapping and serious illegal detention" as found by the trial judge in his judgment. Since the accused-appellant pleaded guilty and no evidence appears to have been presented by either party, the trial judge must have deduced the capital offense from the facts pleaded in the information. ISSUE WON the conviction of the lower court is valid HELD NO. It is invalid. - Under the circumstances, particularly the qualified plea given by the accused who was unaided by counsel, it was not prudent, to say the least, for the trial court to render such a serious judgment finding the accused guilty of a capital offense, and imposing upon him such a heavy penalty as ten years and one day of prision mayor to twenty years, without absolute any evidence to determine and clarify the true facts of the case. - rules of Court, Rule 112, section 3, that If the defendant appears without attorney, he must be informed by the court that it is his right to have attorney being arraigned., and must be asked if he desires the aid of attorney, the Court must assign attorney de oficio to defend him. A reasonable time must be allowed for procuring attorney. - the court has four important duties to comply with: 1 It must inform the defendant that it is his right to have attorney before being arraigned; 2 After giving him such information the court must ask him if he desires the aid of an attorney; 3 If he desires and is unable to employ attorney, the court must assign attorney de oficio to defend him; and 4 If

a2010

page 126

Prof.
and whether it had reference to the commission of the offense or to the making of the plea guilty. No investigation was opened by the court on this matter in the presence of the accused and there is now no way of determining whether the supposed instruction is a good defense or may vitiate the voluntariness of the confession. Apparently the court became satisfied with the fiscal's information that he had investigated Mr. Ocampo and found that the same had nothing to do with this case. Such attitude of the court was wrong for the simple reason that a mere statement of the fiscal was not sufficient to overcome a qualified plea of the accused. But above all, the court should have seen to it that the accused be assisted by counsel specially because of the qualified plea given by him and the seriousness of the offense found to be capital by the court. Dispositive The judgment appealed from is reversed and the case is remanded to the Court below for a new arraignment and a new trial after the accused is apprised of his right to have and to be assisted by counsel. So ordered.

the accused desires to procure an attorney of his own the court must grant him a reasonable time therefor. - IN THE CASE, Not one of these duties had been complied with by the trial court. The record discloses that said court did not inform the accused of his right to have an attorney nor did it ask him if he desired the aid of one. The trial court failed to inquire whether or not the accused was to employ an attorney, to grant him reasonable time to procure or assign an attorney de oficio. - The question asked by the court to the accused was "Do you have an attorney or are you going to plead guilty?" Not only did such a question fail to inform the accused that it was his right to have an attorney before arraignment, but, what is worse, the question was so framed that it could have been construed by the accused as a suggestion from the court that he plead guilt if he had no attorney. And this is a denial of fair hearing in violation of the due process clause contained in our Constitution. - One of the great principles of justice guaranteed by our Constitution is that "no person shall be held to answer for a criminal offense without due process of law", and that all accused "shall enjoy the right to be heard by himself and counsel." In criminal cases there can be no fair hearing unless the accused be given the opportunity to be heard by counsel. The right to be heard would be of little avail if it does not include the right to be heard by counsel. Even the most intelligent or educated man may have no skill in the science of the law, particularly in the rules of procedure, and, without counsel, he may be convicted not because he is guilty but because he does not know how to establish his innocence. And this can happen more easily to persons who are ignorant or uneducated. - It is for this reason that the right to be assisted by counsel is deemed so important that it has become a constitutional right and it is so implemented that under our rules of procedure it is not enough for the Court to apprise an accused of his right to have an attorney, it is not enough to ask him whether he desires the aid of an attorney, but it is essential that the court should assign one de oficio if he so desires and he is poor grant him a reasonable time to procure an attorney of his own. - in the instant case, that the accused who was unaided by counsel pleaded guilty but with the following qualification: "but I was instructed by one Mr. Ocampo." The trial court failed to inquire as to the true import of this qualification. the record does not show whether the supposed instructions was real

PEOPLE v NICANDRO 141 SCRA 295 PLANA; February 11, 1986


NATURE Appeal from judgment of CFI Manila FACTS - Pursuant to information regarding the illegal sale of prohibited drugs by Nicandro, the WPD conducted surveillance and organized an entrapment with the confidential informant acting as the buyer of marijuana. With marked money, the informant asked to buy marijuana from Nicandro, and upon delivery of 4 sticks of marijuana cigarettes, the police nabbed Nicandro. The marked bills were recovered from her pockets, as well as marijuana flowering top. - Allegedly, upon being investigated and after having been duly apprised of her constitutional rights, Nicandro orally admitted having sold the marijuana, but refused to reduce her confession to writing. The prosecution relied principally on the testimony of Patrolman Joves, one of the officers who conducted the entrapment. His testimony said when we saw the accused handed the 4 sticks of suspected marijuana cigarettes to our confidential informant and after a prearranged signal was given by the informant that the accused had already sold her the marijuana, we immediately nabbed said suspect and

Criminal Procedure Rowena Daroy Morales


at the same time we identified ourselves as police officers. - When asked how he conducted the investigation, Pat. Joves testified that the first thing I did was I informed the accused of her constitutional rights, then I questioned her about the marijuana that were confiscated xxx and she verbally admitted that she sold the 4 sticks and possessed and owned the other marijuana leaves. CFI convicted her, relying mostly on Nicandros confession as stated in the Joves testimony. She appealed. ISSUES 1. WON court erred in giving probative value to the testimony of the officer 2. WON rights of accused (vs self-incrimination and to confront witness vs her) were violated, thus any evidence obtained therefrom are inadmissible HELD 1. YES Ratio The prosecution evidence leaves much to be desired. It is at best uncertain whether any prosecution witness really saw the alleged sale of marijuana. Reasoning - The court found the testimony of Pat. Joves unreliable as it appears that he himself was unsure of what he saw, first saying that he saw the marijuana being sold openly, but when the improbability of illegal drugs being sold openly was pointed out, he qualified his story by saying that the sale took place secretly. -it is probable that Joves did not really see either the alleged delivery of marijuana or the supposed payment therefor. With his testimony seriously placed in doubt, there is not much left of the prosecution evidence. 2. YES Ratio the right of a person under interrogation to be informed implies a correlative obligation on the part of the investigator to explain, and contemplates an effective communication that results in understanding what is conveyed. Short of this, there is a denial of the right, as it cannot truly be said that the person has been informed of his rights. Reasoning - reliance on oral admission is assailed as violative of Sec20, Art.IV, 1973 Consti (No person shall be compelled
to be a witness vs himself. Any person under investigation for the commission of an offense shall have right to remain silent and to counsel, and to be in. formed of each right. No force, violation, threat, intimidation, or any other means which vitiates the free will shall be used against him. Any

a2010

page 127

Prof.
constitutional rights when he answered the questions, it is idle to talk of waiver of rights. - in this case, Joves did not say what specific rights he mentioned to Nicandro, neither did he state the manner he advised her of her rights so as to make her understand them. This is particularly impt because Nicandro was illiterate and cant be expected to be able to grasp the significance of her rights merely by hearing an abstract statement thereof. - As it is the obligation of the investigator to inform a person under investigation of his rights, so is it the duty of the prosecution to affirmatively establish compliance by the investigator with his said obligation. Absent such affirmative showing, admission or confession made by a person under investigation cannot be admitted in evidence. - Miranda v Arizona: we will not presume that defendant has been effectively apprised of his rights and that his privilege vs self incrimination has been adequately safeguarded on a record that doesnt show that any warnings have been given or any effective alternative has been employed. Nor can a waiver of these rights be assumed on a silent record Dispositive decision SET ASIDE. Acquitted for reasonable doubt

confession obtained in violation of this section shall be inadmissible in evidence).

- above provision is an expanded version of the right vs self-incrimination, formally incorporating the doctrine in Miranda v Arizona: the prosecution may not use statements, whether exculpatory or inculpatory, stemming from custodial interrogation of the defendant unless it demonstrates the use of procedural safeguards effective to secure the privilege against self- incrimination. xxx As for the procedural safeguards to be employed, unless other fully effective means are devised to inform accused persons of their right of silence and to assure a continuous opportunity to exercise it, the ff measures are required. Prior to any questioning, the person must be warned that he has a right to remain silent, that any statement he does make may be used as evidence against him, and that he has a right to the presence of an atty, either retained or appointed. The defendant may waive those rights, provided such is made voluntarily, knowingly & intelligently. If, however, he indicates in any manner & at any stage of the process that he wishes to consult with an atty before speaking, there can be no questioning. Likewise, if the individual is alone & indicates in any manner that he does not wish to be interrogated, the police may not question him. The mere fact that he may have answered some questions or volunteered some statements on his own does not deprive him of the right to refrain from answering any further inquiries until he has consulted with an attorney & thereafter consents to be questioned. (the court points out that the Miranda doctrine rests on the constitutional guarantee that no person shall be compelled to be a witness vs himself) - since right to be informed implies comprehension, degree of explanation required will necessary vary, depending upon the education, intelligence & other relevant personal circumstances of the person under investigation. A simpler & more lucid explanation is needed where the subject is unlettered - Like other constitutional rights, the right vs selfincrimination, including the right of a person under investigation to remain silent & to counsel, and to be informed of such right, may be waived. To be valid, however, a waiver must not only be voluntary; it must be made knowingly & intelligently, which presupposes an awareness or understanding of what is being waived. It stands to reason that where the right has not been adequately explained and there are serious doubts as to whether the person interrogated knew and understood his relevant

BATAAN SHIPYARD & ENGINEERING CO INC (BASECO) v PCGG 150 SCRA 181 NARVASA; May 27, 1987
NATURE SPECIAL CIVIL ACTION for certiorari and prohibition to review the order of the Presidential Commission on Good Government FACTS - Challenged in this special civil action of certiorari and prohibition by a private corporation known as the Bataan Shipyard and Engineering Co., Inc. are: (1) Executive Orders Numbered 1 and 2, promulgated by President Aquino on February 28, 1986 and March 12, 1986 (2) the sequestration, takeover, and other orders issued, and acts done, in accordance with said executive orders by the Presidential Commission on Good Government and/or its Commissioners and agents, affecting said corporation. - BASECO prays that this Court 1) declare unconstitutional and void Executive Orders Numbered 1 and 2;

Criminal Procedure Rowena Daroy Morales


2) annul the sequestration order dated April 14, 1986, and all other orders subsequently issued and acts done on the basis thereof, inclusive of the takeover order of July 14, 1986 and the termination of the services of the BASECO executives. ISSUES 1. WON Executive No s 1, 2 and 14 are unconstitutional 2. WON right against self-incrimination can be invoked by BASECO HELD 1. NO Executive Order No. 1 > stresses the "urgent need to recover all ill-gotten wealth," and postulates that "vast resources of the government have been amassed by former President Ferdinand E. Marcos, his immediate family, relatives, and close associates both here and abroad." Upon these premises, the Presidential Commission on Good Government was created, "charged with the task of assisting the President in regard to (certain specified) matters, - among which was precisely > In relation to the takeover or sequestration that it was authorized to undertake in the fulfillment of its mission, the PCGG was granted "power and authority" to do the following particular acts, to wit: 1. "To sequester or place or cause to be placed under its control or possession any building or office wherein any ill-gotten wealth or properties may be found, and any records pertaining thereto, in order to prevent their destruction, concealment or disappearance which would frustrate or hamper the investigation or otherwise prevent the Commission from accomplishing its task. 2. "To provisionally take over in the public interest or to prevent the disposal or dissipation, business enterprises and properties taken over by the government of the Marcos Administration or by entities or persons close to former President Marcos, until the transactions leading to such acquisition by the latter can be disposed of by the appropriate authorities." 3. "To enjoin or restrain any actual or threatened commission of acts by any person or entity that may render moot and academic, or frustrate or otherwise make ineffectual the efforts of the Commission to carry out its task under this order. " > So that it might ascertain the facts germane to its objectives, it was granted power to conduct investigations, require submission of evidence by

a2010

page 128

Prof.
Philippines or abroad, in their names as nominees, agents or trustees, to make full disclosure of the same to the Commission on Good Government within thirty (30) days from publication of * (the) Executive Order, " Executive Order No. 14 > PCGG is empowered, "with the assistance of the Office of the Solicitor General and other government agencies, * * to file and prosecute all cases investigated by it * * as may be warranted by its findings.'"34 All such cases, whether civil or criminal, are to be filed "with the Sandiganbayan, which shall have exclusive and original jurisdiction thereof." > "(c)ivil suits for restitution, reparation of damages, or indemnification for consequential damages, forfeiture proceedings provided for under Republic Act No. 1379, or any other civil actions under the Civil Code or other existing laws, in connection with * * (said Executive Orders Numbered I and 2) may be filed separately from and proceed independently of any criminal proceedings and may be proved by a preponderance of evidence;" and that, moreover, the "technical rules of procedure and evidence shall not be strictly applied to* *(said) civil cases." 2. NO, there is No Violation of Right against SelfIncrimination Ratio It is elementary that the right against selfincrimination has no application to juridical persons. Reasoning - BASECO contends that its right against selfincrimination and unreasonable searches and seizures had been transgressed by the Order of April 18, 1986 which required it "to produce corporate records from 1973 to 1986 under pain of contempt of the Commission if it fails to do so." The order was issued upon the authority of Section 3 (e) of Executive Order No. 1, treating of the PCGG's power to "issue subpoenas requiring the production of such books, papers,contracts, records, statements of accounts and other documents as may be material to the investigation conducted by the Commission," and paragraph (3), Executive Order No. 2 dealing with its power to "(r)equire all persons in the Philippines holding * *(alleged "ill-gotten") assets or properties, whether located in the Philippines or abroad, in their names as nominees, agents or trustees, to make full disclosure of the same **. " - While an individual may lawfully refuse to answer incriminating questions unless protected by an immunity statute, it does not follow that a corporation, vested with special privileges and franchises, may refuse to show its hand when charged with an abuse of such privileges

subpoenae ad testification and duces tecum; administer oaths; punish for contempt. It was given power also to promulgate such rules and regulations as may be necessary to carry out the purposes of (its creation). " Executive Order No. 2 > gives additional and more specific data and directions respecting "the recovery of ill-gotten properties amassed by the leaders and supporters of the previous regime." It declares that: 1) "* * the Government of the Philippines is in possession of evidence showing that there are assets and properties purportedly pertaining to former Ferdinand E. Marcos, and/or his wife Mrs. Imelda Romualdez Marcos, their close relatives, subordinates, business associates, dummies, agents or nominees which had been or were acquired by them directly or indirectly, through or as a result of the improper or illegal use of funds or properties owned by the government of the Philippines or any of its branches, instrumentalities, enterprises, banks or financial institutions, or by taking undue advantage of their office, authority, influence, connections or relationship, resulting in their unjust enrichment and causing grave damage and prejudice to the Filipino people and the Republic of the Philippines; and 2) " * said assets and properties are in the form of bank accounts, deposits, trust accounts, shares of stocks, buildings, shopping centers, condominiums, mansions, residences, estates, and other kinds of real and personal properties in the Philippines and in various countries of the world." Upon these premises, the President 1) froze "all assets and properties in the Philippines in which former President Marcos and/or his wife, Mrs. Imelda Romualdez Marcos, their close relatives, subordinates, business associates, dummies, agents, or nominees have any interest or participation" 2) prohibited former President Ferdinand Marcos and/or his wife * *, their close relatives, subordinates, business associates, dummies, agents, or nominees from transferring, conveying, encumbering, concealing or dissipating said assets or properties in the Philippines and abroad 3) prohibited "any person from transferring conveying, encumbering or otherwise depleting or concealing such assets and properties or from assisting or taking part in their transfer, encumbrance. concealment or dissipation under pain of such penalties as are prescribed by law;" and 4) required "all persons in the Philippines holding such assets or properties, whether located in the

Criminal Procedure Rowena Daroy Morales


- Oklahoma Press Publishing Co. v. Walling > corporations are not entitled to all of the constitutional protections which private individuals have. They are not at all within the privilege against self-incriminatior, although this court more than once has said that the privilege runs very closely with the 4th Amendment's Search and Seizure provisions. It is also settled that an officer of the company cannot refuse to produce its records in its possession, upon the plea that they will either incriminate him or may incriminate it. - Wilson v. United States > The corporation is a creature of the state. It is presumed to be incorporated for the benefit of the public. It received certain special privileges and franchises, and holds them subject to the laws of the state and the limitations of its charter. Its powers are limited by law. It can make no contract not authorized by its charter. Its rights to act as a corporation are only preserved to it so long as it obeys the laws of its creation. There is a reserve right in the legislature to investigate its contracts and find out whether it has exceeded its powers. It would be a strange anomaly to hold that a state, having chartered a corporation to make use of certain franchises, could not, in the exercise of sovereignty, inquire how these franchises had been employed, and whether they had been abused, and demand the production of the corporate books and papers for that purpose. The defense amounts to this, that an officer of the corporation which is charged with a criminal violation of the statute may plead the criminality of such corporation as a refusal to produce its books. To state this proposition is to answer it. While an individual may lawfully refuse to answer incriminating questions unless protected by an immunity statute, it does not follow that a corporation, vested with special privileges and franchises may refuse to show its hand when charged with an abuse of such privileges. - At any rate, Executive Order No. 14-A, amending Section 4 of Executive Order No. 14 assures protection to individuals required to produce evidence before the PCGG against any possible violation of his right against self-incrimination. It gives them immunity from prosecution on the basis of testimony or information he is compelled to present. As amended, said Section 4 now provides that "The witness may not refuse to comply with the order on the basis of his privilege against selfincrimination; but no testimony or other information compelled under the order (or any

a2010

page 129

Prof.
YES - Sections 1 and 2, Rule 13212 of the rules of Court clearly require that the testimony of a witness shall be given orally in open court. Reasoning - The main and essential purpose of the rule is to secure for the adverse party the opportunity to crossexamine the witness presented. The opponent demands confrontation for the purpose of cross examination which cannot be had except by the direct and personal putting of questions and obtaining immediate answers. - There is also the advantage obtained in the personal appearance of the witness in open court as it affords the judge to assess the weight and value that can be given to any of the testimony based on his perception of the witness countenance, manner, and expression. In deed, the great weight given the findings of fact of the trial judge in the appellate court is based precisely upon the judge having had the opportunity and the assumption that he took advantage of it to ascertain the credibility of the witness. - Rules governing the examination of witnesses are intended to protect the rights of the litigants and to secure orderly dispatch of the business of the courts. Hence only questions directed to the eliciting of testimony which, under the general rules of evidence, is relevant to and competent to prove, the issues of the case, may be propounded to the witness. Dispositive Petition granted. The order of the judge is set aside.

information directly or indirectly derived from such testimony, or other information) may be used against the witness in any criminal case, except a prosecution for perjury, giving a false statement, or otherwise failing to comply with the order." Dispositive petition is dismissed Voting Yap, Fernan, Paras, Gancayco and Sarmiento concur Teehankee, concurs in a separate opinion. Melencio-Herrera, concurs with qualifications in a separation opinion. Gutierrez, Jr. see concurring and dissenting opinion. Cruz dissents in a separate opinion. Feliciano joins M. Herrera's qualified concurring opinion. Padilla see concurring opinion. Bidin joins Gutierrez in his concurring and dissenting opinion. Cortes joins Gutierrez in his concurring and dissenting opinion.

PEOPLE v ESTENSO 72 SCRA 473 ANTONIO; August 27, 1976


NATURE Original Action for Certiorari and prohibition FACTS - In Criminal Case No. 2891, entitled People of the Philippines vs Gregorio Ojoy, of the CFI of Iloilo, the counsel for the accused, after the accused himself had already testified in his own trial, manifested to the Court that he was filing only affidavits for his subsequent witnesses subject to cross-examination by the prosecution on matters stated in the said affidavits and on all matters pertinent and material to the case. - The private prosecutor objected to the manifestation as the same is a violation of Sections 1 and 2 of Rule 132 of the Revised Rules of Court, which require that testimony of the witnesses should be given orally in open court. - The CFI judge granted the motion of the defense. Hence this appeal. ISSUE WON the CFI judge erred in allowing the just the submission of affidavits in lieu of oral testimony HELD

SEPARATE OPINION BARREDO [concur]


- Barredo noted that it was the private prosecutor who objected to the procedure adapted and the

12

Section 1. Testimony to be given in open court.- the testimony of witnesses shall be given orally in open court and under oath or affirmation. Section 2. Testimony in Superior Courts to be reduced in writing.- In superior courts, the testimony of each witness shall be taken in shorthand or stenotype, the name, residence, and occupation of the witness being stated, and all the questions put the witness and his answers thereto being included. If a question put is objected to and the objection is ruled on, the nature of the objection and the ground on which it was sustained or overruled must be stated, or if the witness declines to answer a question out, the fact and the proceedings taken thereon shall be entered in the record. A transcript of the record made by the official stenographer so stenotypist and certified as correct by him shall be prima facie a correct statement of such testimony and proceedings.

Criminal Procedure Rowena Daroy Morales


Solicitor General only commented at the insistence of the Supreme Court. - While concurring with the opinion of the court, Barredo felt that the approach is novel as this may be a solution to speeding up trials in the lower courts. He nevertheless stated that the procedure could probably work only for civil cases. - Barredo anchored his approbation of the procedure on the fact that cross examination may be had even if direct examination is dispensed with. Hence the Constitutional requirement that the opponent be given the opportunity to confront the witness is met. - Barredo suggested certain improvements prior to the adoption of the procedure. Among these are the need for the testimony to be submitted to the prosecution for vetting as to materiality and relevance. Same should be submitted to the judge for his own examination as to materiality and relevance.

a2010

page 130

Prof.
he had already been convicted of rebellion, he cannot now be prosecuted for subversion 4. WON the decision should have been promulgated in the CFI of Rizal, Quezon City Branch, considering that he was then detained or confined at Camp Crame, Quezon City, and not in the Pasig Branch of said Court HELD 1. NO - The Constitution guarantees an accused person the right to meet the witnesses against him face to face. This provision "intends to secure the accused in the right to be tried, so far as facts provable by witnesses are concerned, by only such witnesses as meet him face to face at the trial, who give their testimony in his presence, and give to the accused an opportunity of cross-examination. It was intended to prevent the conviction of the accused upon depositions or ex-parte affidavits, and particularly to preserve the right of the accused to test the recollection of the witnesses in the exercise of the right of cross-examination." - Here, the testimony sought to be made part of the evidence in chief are not ex-parte affidavits, but testimony of witnesses taken down by question and answer during the preliminary investigation in the presence of the accused and his counsel who subjected the said witnesses to a rigid and close cross-examination. The inclusion of said testimony was made subject to the right of the defendant to further cross-examine the witnesses whose testimony are sought to be reproduced and, pursuant to said order, the witnesses were recalled to the stand during the trial and again examined in the presence of the appellant. Upon the facts, there was no curtailment of the constitutional right of the accused to meet the witnesses face to face. 2. NO - Section 7 of Republic Act No. 1700, provides that "No person shall be convicted of any of the offenses penalized herein with prision mayor to death unless on the testimony of at least two witnesses to the same overt act or on confession of the accused in open court." - Appellant's being an officer or ranking leader of the CPP and its military arm, the HMB, is borne out by the testimony of former associates of the appellant in the CPP and the HMB. There is his sworn statement wherein the appellant admitted membership in the Central Committee of the CPP and recounted his prismatic rise in the "Hukbalahap" and later in the HMB, as well as the numerous armed clashes he and

PEOPLE v LIWANAG 73 SCRA 473 CONCEPCION; October 19, 1976


NATURE Appeal from the judgment of the Court of First Instance of Bataan, finding the accused guilty of violating the provisions of Republic Act No. 1700, otherwise known as the Anti-Subversion Act, and sentencing him to suffer the penalty of reclusion perpetua with the accessories of the law, and to pay the costs. FACTS - Liwanag was charged for violating the provisions of RA 1700, otherwise known as the Anti-Subversion Act, in an information filed before the Court of First Instance of Bataan, for having unlawfully and wilfully continued and remained as officer and/or ranking leader of the outlawed CPP and its military arm, the HMB, until his apprehension, without having renounced his aforementioned leadership and/or membership therein within the period prescribed by law, and, while remaining as such leader or highranking member, has taken up arms against the Government by making and conducting raids, ambuscades and armed attacks against civilians, Philippine Constabulary, and local police forces. - Preliminary investigation was conducted by the CFI of Bataan. Finding a prima facie case against the appellant, the Court issued the corresponding

warrant for the arrest and thereafter set the case for trial. - Upon being arraigned, Liwanag, assisted by his counsel, waived the reading of the information and entered a plea of not guilty. In view of the desire of his counsel to file a motion to quash, the court granted Liwanag twenty (20) days within which to do so. - Liwanag filed a motion to quash the information upon the grounds that he has been previously convicted of rebellion based upon the same overt acts as in the instant case, and that Republic Act No. 1700 is an ex post pacto law (bill of attainder) in that it changes the punishment and inflicts a greater punishment or penalty than that annexed to the crime when committed. The court denied the motion. - The case was subsequently set for trial, the prosecution moved that the testimony of the witnesses presented during the preliminary investigation of this case be adopted as part of the evidence in chief of the prosecution. The trial court granted the motion subject to the condition that the witnesses be further cross-examined by counsel for the accused. - At the trial, the witnesses for the prosecution who testified at the preliminary investigation were recalled and were again cross-examined by counsel for the appellant. To bolster their case, the prosecution presented three additional witnesses. The defense, presented the appellant himself who stated that after his apprehension, he was charged with rebellion before the CFI of Pampanga and found guilty thereof; and he was also charged with murder before the CFI of Tarlac and acquitted; and that he surrendered to the PC patrol. However, he admitted membership in the Hukbalahap, and later in the HMB, from 1948 to 1960, and did not take advantage of the amnesty offered in 1948. - The trial court found the accused guilty of the crime of subversion, as charged. ISSUES 1. WON Liwanag was deprived of his fundamental right to confront the witnesses against him when the trial court granted the motion of the Fiscal that the testimony of the witnesses presented during the preliminary investigation be adopted and made part of the evidence for the prosecution 2. WON trial court erred in finding Liwanag guilty 3. WON having been charged with rebellion and subversion based upon the same overt act, and since

Criminal Procedure Rowena Daroy Morales


his men had with the Philippine Constabulary and police forces. There is also the testimony as to the gun battle between a PC patrol and a group of HMB men led by the appellant in Bataan, where the appellant was captured along with his wife. Besides, appellant admitted in court that he was a member of the "Hukbalahap" and the HMB and fought against the government. 3. NO - Violation of Republic Act No. 1700, or subversion, is a crime distinct from that of actual rebellion. - The crime of rebellion is committed by rising publicly and taking up arms against the Government for any of the purposes specified in Article 134 of the Revised Penal Code; while the Anti-Subversion Act punishes affiliation or membership in a subversive organization as defined therein. In rebellion, there must be a public uprising and the taking of arms against the Government; whereas, in subversion, mere membership in a subversive association is sufficient, and the taking up of arms by a member of a subversive organization against the Government is but a circumstance which raises the penalty to be imposed upon the offender. - In the rebellion case, the appellant and several others were charged and convicted of rebellion for having risen publicly and taken up arms against the Government for the purpose of removing the allegiance of the Republic of the Philippines or its laws, the territory of the Philippines, and in furtherance thereof, engaged in combat against the forces of the Government, destroyed property, and committed serious violence during the period from May 28, 1946 to June 19, 1957. - The accused is prosecuted under RA 1700 for having remained a high ranking member of the CPP and its military arm, the HMB, from January, 1946 to June 21, 1960, without having renounced his membership in said organizations; and, being a member or officer of said subversive association, has taken up arms against the Government. - Although the information charges the appellant with having taken up arms against the Government, the same is not specific as to the period covered by it. But, since the appellant is prosecuted for violation of Republic Act No. 1700 it is deducible that the period covered is that from June 20, 1957, when the Act took effect, up to June 21, 1960, when the appellant was captured. Inasmuch as the rebellion case covered the period up to June 19, 1957 and the period covered in the instant case is from June 20, 1957 to June 21, 1960, the claim of having been put

a2010

page 131

Prof.
because Basilio, Talino and Macadangdang were granted separate trials and they did not cross examine Ulat because, as a matter of fact, they were not even required to be present when the other accused were presenting their defenses, the latter's testimonies can not now be considered against said three accused. - The grant of a separate trial rests in the sound discretion of the court and is not a matter of right to the accused, especially where, as in this case, it is sought after the presentation of the evidence of the prosecution. The rule in every case is that the trial court should exercise the utmost circumspection in granting a motion for separate trial, allowing the same only after a thorough study of the claimed justification therefor, if only to avoid the serious difficulties that may arise, such as the one encountered and regretted by the respondent court, in according the accused the right of confrontation. - The right of confrontation is one of the fundamental rights guaranteed by the Constitution to the person facing criminal prosecution who should know, in fairness, who his accusers are and must be given a chance to cross-examine them on their charges. No accusation is permitted to be made against his back or in his absence nor is any derogatory information accepted if it is made anonymously, as in poison pen letters sent by persons who cannot stand by their libels and must shroud their spite in secrecy. In United States v. Javier confrontation is essential because cross-examination is essential. A second reason for the prohibition is that a tribunal may have before it the deportment and appearance of the witness while testifying. ISSUE WON the decision of the court violates Talinos right of confrontation as guaranteed by the Constitution HELD NO - The court have carefully studied the decision under challenge and find that the respondent court did not consider the testimony given by Ulat in convicting Talino. The part of that decision finding Talino guilty made no mention of Ulat at all but confined itself to the petitioner's own acts in approving the questioned vouchers as proof of his complicity in the plot to swindle the government. Talino makes much of the statement in the Comment that the petitioner's guilt could be deduced "from the evidence for the prosecution and from the testimony of Pio Ulat," but that was not the respondent court speaking. That

twice in jeopardy for the same act cannot be sustained. 4. NO - The records show that he had been confined at Fort Bonifacio (then known as Fort William Mckinley), Makati, Rizal, since November 20, 1962 and continued to be detained therein during the continuation of the trial, up to its termination. Dispositive UPON THE FOREGOING, the decision appealed from should be, as it is, hereby affirmed, with costs.

TALINO v SANDIGANBAYAN 148 SCRA 598 CRUZ; March 16, 1987


FACTS - Talino, along with several others, were charged in four separate informations with estafa through falsification of public documents for having allegedly conspired to defraud the government in the total amount of P26,523.00, representing the cost of repairs claimed to have been undertaken, but actually not needed and never made, on four government vehicles, through falsification of the supporting papers to authorize the illegal payments. The cases were tried jointly for all the accused until Genaro Basilio, Alejandro Macadangdang and petitioner Talino asked for separate trials, which were allowed. At one of the proceedings, Pio Ulat gave damaging testimony against Talino. The Sandiganbayan rendered its decision in all the four cases finding Talino, Basilio, Macadangdang Ulat and Renato Valdez guilty beyond reasonable doubt of the crimes charged while absolving the other defendants for insufficient evidence. This decision is now challenged by Talino on the ground that it violates his right of confrontation as guaranteed by the Constitution. - In its decision, the court made the ff remarks: The peculiarity of the trial of these cases is the fact that We allowed, upon their petition, separate trials for the accused Basilio and Talino and Macadangdang. This being the case, We can only consider, in deciding these cases as against them, the evidence for the, prosecution as wen as their own evidence. Evidence offered by the other accused can not be taken up. It would really have been simpler had there been no separate trial because the accused Pio Ulat said so many incriminatory things against the other accused when he took the stand in his own defense. But

Criminal Procedure Rowena Daroy Morales


was the Solicitor General's analysis. As far as the Sandiganbayan was concerned, the said testimony was inadmissible against the petitioner because he "did not cross examine Ulat and was not even required to be present when the latter was testifying. In fact, the respondent court even expressed the wish that Ulat had been presented as rebuttal witness in the separate trial of the petitioner as there would then have been "no impediment to the use of his testimony against the other accused. " As it was not done, the trial court could not and did not consider Ulat's testimony in determining the petitioner's part in the offenses. Dispositive judgment appealed from is AFFIRMED, with costs against the petitioner.

a2010

page 132

Prof.
such a case is from the interpretation thereof which is given by another person. - The prosecution should have presented Emateo himself to testify on what actually transpired. The lower court ignored the right of the accused to meet the witness face to face. - Prosecution revealed not to know anything about the informants background. Although there is a policy of non-disclosure of an informants identity, it cannot be invoked in this case. The informants failure to take the witness to stand to confirm the correctness of his interpretations not only rendered the testimonies as hearsay and inadmissible, but also deprived the appellant of his right to cross-examine him. - Non-presentation of an informer is a privilege that has its own inherent limitation. Where in the disclosure of an informers identity is relevant and helpful to the defense of the accused, or is essential to the proper disposition of the case, the privilege must give way. - Although the identity of the informer was disclosed, prosecution failed to present him as witness on the assertion that his whereabouts are unknown. No subpoena has been issued by the prosecution to Emateo, the presumption that evidence willfully suppressed would be adverse if produced (Sec 5(e), Rule 131) arises. - The appellants claim that the ownership of the sack of marijuana was previously deposited by Emateo was never contradicted by prosecution. Bare assertion of Baganos delivery of the bag does not, by itself indicate ownership nor even illegal possession absent any other evidence. - Apellants signature appearing on the sack and individual bundles containing marijuana do not signify, much less evidence, guilt for they are mere procedural steps undertaken after arrest. Furthermore, it appearing that appellant was not informed of his right to counsel at the time he affixed his signature, the same has been obtained in violation of his right as a person under custodial investigation for the commission of an offense and is therefore inadmissible. - It is a cardinal rule that in order to merit conviction, the prosecution must rely on the strength of its own evidence and not on the weakness of evidence presented by the defense. An accused must always be deemed innocent until the contrary is proved beyond reasonable doubt. In the instant case, the prosecution failed to so establish the guilt of herein appellant.

told appellant that he will pay for the marijuana after weighing it in the hotel, to which the appellant agreed. On the way back to the hotel in Baguio, the NBI was signaled that the transaction took place. The NBI team blocked Bosticks car and arrested the appellant and Emateo. - Prosecution claims that according to Emateo, appellant would only sell to a foreigner, preferably an American. Suspects Claim - He went to the house of Emateo to collect P4000, which the latter borrowed him. Emateo told him to wait for Emateos visitor, from whom he would get the money to pay for his debt. After being introduced to Bostick, they proceeded to Irisan to get the bag Emateo would five the American as gift. Emateo asked Bagano to get the bag because Emateo did not want to get wet. On the way back to Baguio, the NBI team came out and made the arrests. - He claims that the court erred: in finding that he agreed to sell 10kgs of marijuana when he had no knowledge of the alleged sale. In not holding as hearsay the alleged conversation between Bolstick and the appellant when Emateo, the informant who interpreted the conversation was never presented to testify In appreciating the bag and not the sack against the appellant that which Emateo owned and deposited in the quarter of the appellant at irisan earlier the same day of the alleged buy-bust operation. In not appreciating the defense that it was Emateo who owns the Marijuana in question ISSUE WON the Bolstick testimonies were hearsay HELD YES - From Bolsticks testimonies as principal witness for the prosecution, such were mere translations and/or interpretations of what Bagano supposedly said in the dialect and interpreted by Emateo. The only exception is the testimony on what Bolstick saw. - Where a witness is offered to testify to statements of another person, spoken in a language not understood by him, but translated to his by an interpreter, such witness is not qualified, because he does not speak from personal knowledge. All that he can know as to the testimony which is in fact given in

PEOPLE v BAGANO 181 SCRA 747 BIDIN; February 5, 1990


NATURE Appeal on the decision of RTC convicting the accused of the violation of RA6426 as amended, otherwise known as the Dangerous Drugs Act and imposing the penalty of P20K. FACTS - A buy-bust operation was conducted by NBI on a suspected narcotics dealer. Bostick, a special agent of the US Air Force was introduced to Doming Bagano by an informer, Clayton Emateo. - An information for the violation of RA6425, also known as the Dangerous Drugs Act, was filed against the accused. It stated that the accused willfully, unlawfully and feloniously attempt to sell to another ten (10) kilos, more or less, of dried marijuana leaves, a dangerous drug, for P800.00 per kilo. - Upon arraignment, Bagano pleaded not guilty. His motion to admit bail was deferred and later denied. - After trial on the merits, TC judge rendered decision sentencing Bagano with reclusion perpetua. Solicitor Generals Claims - Bostic and Emateo went to the residence of the latter in Baguio City to meet the appellant. Emateo introduced Bagano and an unnamed friend to Bostick. Negotiations for the purchase of 10kilos of marijuana between Bostick and Bagano was done, Emateo being the interpreter. After agreeing for the price of P800/kg, the group proceeded to Baganos house in Irisan Benguet where he kept the marijuana. At Irisan, Bagano left Bostick and Emateo in the car, returning 15mins later carrying a nylon sack. Bostick

Criminal Procedure Rowena Daroy Morales


Dispositive challenged judgment is REVERSED and appellant is hereby ACQUITTED on the ground of reasonable doubt.

a2010

page 133

Prof.
the law excuses no one. The right against selfincrimination is not automatically operational but must be claimed. Failure to claim it is an implied waiver of said right. - The second right, or rather, group of rights, are a persons rights in custodial interrogation, which means questioning initiated by law enforcement officers after a person has been taken into custody or otherwise deprived of his freedom of action in any significant way. These section can be broken down into 3 rights: (1) the right to remain silent and to counsel and to be informed of such right (2) no force, violence, threat, intimidation or any other means which vitiates the free will shall be used against him (3) any confession obtained in violation of this shall be inadmissible in evidence. - The accused must be informed of these rights prior to any questioning, after which the individual may knowingly and intelligently waive these rights and agree to answer or make a statement. Statements not made under custodial interrogation are not protected. - It is important now to inquire whether the rights mentioned apply to persons under preliminary investigation or already charged in court for a crime. It is evident that a defendant under preliminary investigation is not under custodial interrogation, and there is thus no occasion to speak of such rights under custodial interrogation; however, the accused still possesses the right against self-incrimination. - Under the Rules of Court, the accused occupies a different tier of protection from an ordinary witness and is entitled, among others: (1) not be a witness against himself (2) to testify as a witness on his own behalf; but if he offers himself as a witness, he may be cross-examined as any other witness; his neglect of refusal to be a witness shall not in any manner prejudice or be used against him. Thus, unlike an ordinary witness, the accused may refuse to take the witness stand, be sworn, or answer any question altogether. The accused, if he chooses to testify, may refuse to answer only questions which could incriminate him of a crime for which he isnt charged. - It appears that respondent Judge mistakenly applied the rights set forth in Sec 20 Art. IV of the 1973 Constitution. It is clear from the undisputed facts that Ramos was not in any sense under custodial interrogation, and thus his constitutional rights in relation thereto dont apply. Also, Ramos had voluntary answered the questions posed to him on the first day of the administrative investigation and agreed that the proceedings be recorded and filed as exhibits A and K, spontaneously offering to

US v GARCIA 11 PHIL 384 July 24, 1908


THEFT; SUFFICIENCY OF PROOF.-From the Court of First Instance of Ambos Camarines. The uncorroborated testimony of the prosecuting witness held insufficient to prove that the accused was guilty of taking money from a trunk, as alleged, while searching the house of the witness for another person. Judgment reversed and defendant acquitted. Note: The case is really this short only. I did not omit a single word.

rights to remains silent and have counsel. The private prosecutors filed an MFR, but respondent Judge justified his order citing the constitutional precept that the rights in custodial investigation cannot be waived except in writing and in the presence of counsel. - Said orders are now assailed in this petition for certiorari and prohibition. The Court required respondent Judge Ayson and Ramos to comment, and directed the issuance of a TRO enjoining respondents from further proceeding with the trial/hearing. The Solgen, who was also required to comment, sided with petitioner, praying that respondent judges orders be set aside and further ordering the admittance of exhibits A and K of the prosecution. ISSUE WON it was grave abuse of discretion for respondent judge to exclude exhibits A and K HELD YES - At the core of the controversy is Sec. 20, Art. IV of the 1973 Constitution which provides: No person shall be compelled to be a witness against himself. Any person under investigation for the commission of an offense shall have the right to remain silent and to counsel, and to be informed of such right. No force, violence, threat, intimidation, or any other means which vitiates the free will shall be used against him. Any confession obtained in violation of this section shall be inadmissible in evidence. - There are 2 rights dealt with in the section: the right against self-incrimination (now embodied in Sec. 17 Art. 3 of the 1987 Constitution) and the rights of a person in custodial interrogation (Sec 12 Art. 3). - The first right, against self-incrimination, is NOT to be compelled to be a witness against himself and applies to any person testifying in any proceeding, civil, criminal or administrative. It prescribes an option of refusal to answer incriminating questions and not a prohibition of inquiry. However, it can only be claimed when the specific question, incriminatory in character, is actually put to the witness and cannot be claimed at any other time. The witness thus may not disregard a subpoena or refuse to testify altogether. - The provision of the 1973 Constitution does not impose on the judge or any other presiding officer, any affirmative obligation to advise a witness of this right, which the witness should know, as ignorance of

PEOPLE v AYSON G.R. No. 85215 NARVASA; July 7, 1989


FACTS - Private respondent Felipe Ramos was a ticket flight clerk of PALs Baguio station. PAL management notified him of an investigation due to his alleged involvement in irregularities in the sales of plane tickets. On the day before the investigation, Ramos gave his superiors a handwritten note (exhibit K) expressing his willingness to settle the irregularities (in the amount of P76k). At the investigation, Ramos admitted his non-disclosure of the tickets mentioned, that the proceeds had been misused by him, and that although he planned on paying back the money, he had been prevented from doing so out of shame. He also stated his willingness to settle the obligation on a staggered basis, the amount of which would be known at the next investigation which he desired to be held at Baguio CTO, that he be represented by Nieves Blanco, and that he was willing to sign his statement (exhibit A). - 2 months later, an information was filed against Ramos charging him with estafa against PAL. On arraignment he pleaded not guilty. At the close of the case, private prosecutors presented Ramos (above mentioned) statement, including his handwritten admission as evidence. Defendants attorneys objected, particularly as regards the handwritten confession as it was taken without Ramos having counsel. Respondent Judge declared exhibits A and K inadmissible as evidence as it appears he was not reminded of his constitutional

Criminal Procedure Rowena Daroy Morales


compromise his liability. Said exhibits may not be excluded as the so-called Miranda rights had not been accorded to Ramos. Dispositive the writ of certiorari is granted annulling and setting aside the Orders of respondent Judge, and he is hereby ordered to admit in evidence exhibits A and K

a2010

page 134

Prof.
fourteen separate occasions without objection on their part, and without an iota of evidence to substantiate any claim as to any other person so minded being excluded from the premises. It is thus evident that what took place in the chambers of the city court judge was devoid of haste or intentional secrecy. - The 1935 Constitution which was in force at the time of this petition explicitly enumerated the right to a public trial to which an accused was entitled. As a matter of fact, that was one constitutional provision that needed only a single, terse summation from Justice Jose P. Laurel, to gain acceptance. As was stressed by him: "Trial should also be public in order to offset any danger of conducting it in an illegal and unjust manner." It would have been surprising if its proposed inclusion in the Bill of Rights had provoked any discussion, much less a debate. It was merely a reiteration what appeared in the Philippine Autonomy Act of 1916, popularly known as the Jones Law. Earlier, such a right found expression in the Philippine Bill of 1902, likewise an organic act of the then government of this country as an unincorporated territory of the United States. Historically as was pointed out by Justice Black, in the leading case of In re Oliver: This nation's accepted practice of guaranteeing a public trial to an accused has its roots in the English common law heritage, but it likely evolved long before the settlement of the US as an accompaniment of the ancient institution of jury trial. The guarantee to an accused of the right to a public trial appeared in a state constitution in 1776. Later it was embodied in the Sixth Amendment of the Federal Constitution ratified in 1791. Today almost without exception every state by constitution, statute, or judicial decision, requires that all criminal trials be open to the public. - The Constitution guarantees an accused the right to a public trial. There is no ambiguity in the words employed. The trial must be public. It possesses that character when anyone interested in observing the manner a judge conducts the proceedings in his courtroom may do so. There is to be no ban on such attendance. His being a stranger to the litigants is of no moment. No relationship to the parties need be shown. The thought that lies behind this safeguard is the belief that thereby the accused is afforded further protection, that his trial is likely to be conducted with regularity and not tainted with any impropriety. Accdg to J. Laurel, the importance of this right is its being a deterrence to arbitrariness. It is thus understandable why such a right is deemed

GARCIA v DOMINGO 52 SCRA 143 FERNANDO; July 25, 1973


NATURE Petition for certiorari and prohibition FACTS - On January 16, 1968, in the City Court of Manila presided over by petitioner Judge Gregorio Garcia, 8 informations were filed against respondents Edgardo Calo, and Simeon Carbonnel and Francisco Lorenzana, for slight physical injuries, maltreatment, for violation of Sec. 887 of the Revised Ordinances of Manila (resisting an officer); and for slander. The trial of the aforementioned cases was jointly held on March 4, 18, 23, 30, 1968; April 17, & 20, 1968, May 4 & 11, 1968, June 1, 15, 22 & 29, 1968, August 3 & 10, 1968. All the 14 trial dates except March 4 and 18, and April 17, 1968 fell on a Saturday. This was arranged by the parties and the Court upon the insistence of respondents Calo and Carbonnel who, as police officers under suspension because of the cases, desired the same to be terminated as soon as possible and as there were many cases scheduled for trial on the usual criminal trial days (Monday, Wednesday and Friday), Saturday was agreed upon as the invariable trial day for said eight (8) criminal cases. The trial of the cases in question was held, with the conformity of the accused and their counsel, in the chambers of Judge Garcia. During all the 14 days of trial, spanning a period of several months, the accused were at all times represented by their respective counsel, who acted not only in defense of their clients, but as prosecutors of the accusations filed at their clients' instance. There was only 1 day (April 20) when Atty. Consengco, representing respondent Calo and Carbonnel, was absent. But at the insistence of Carbonnel, the trial proceeded, and said respondent cross-examined one of the witnesses presented by the adverse party. In any case, no pretense has been made by the respondents that this constituted an irregularity correctible on certiorari. At the conclusion of the hearings the

accused, thru counsel, asked for and were granted time to submit memoranda. Respondents Calo and Carbonnel, thru counsel, Atty. Rafael Consengco, submitted a 14-page memorandum in support of their prayer for exoneration, and conviction of petitioner Lorenzana in respect of their countercharges against the latter. It is worthy of note that up to this late date, said respondents Calo and Carbonnel had not objected to or pointed out any supposed irregularity in the proceedings thus far; the memorandum submitted in their behalf is confined to a discussion of the evidence adduced in, and the merits of the cases. - The promulgation of judgment scheduled on Sep 23, 1968 was postponed to Sep 28, 1968 at the instance of Atty. Consengco, , and again to Oct 1, 1968. The applications for postponement were not grounded upon any supposed defect or irregularity of the proceedings. - However, on October 1, 1968, Calo and Carbonnel, thru their counsel, filed with the CFI of Manila a petition for certiorari and prohibition, with application for preliminary prohibitory and mandatory injunction alleging jurisdictional defects. Respondent Judge Felix Domingo issued a restraining order thus causing the deferment of the promulgation of the judgment. After proceedings duly had, there was an order from him declaring that 'the constitutional and statutory rights of the accused' had been violated, adversely affecting their 'right to a free and impartial trial, noting 'that the trial of these cases lasting several weeks held exclusively in chambers and not in the court room open the public';" and ordering the city court Judge Garcia, "to desist from reading or causing to be read or promulgated the decisions he may have rendered already in the criminal cases pending in his Court, until further orders of this Court. - The MR was denied. Hence, on January 28, 1969, the matter was elevated to the SC by means of the present suit for certiorari and prohibition. ISSUE WON respondent Judge commit a grave abuse of discretion in ruling that the holding of the trial of the accused inside the chambers of petitioner ,city court Judge Gregorio Garcia, as violative of the constitutional right to public trial HELD YES - The procedure had been agreed to beforehand by accused. The hearings have been thus conducted on

Criminal Procedure Rowena Daroy Morales


embraced in procedural due process. Where a trial takes place, as is quite usual, in the courtroom and a calendar of what cases are to be heard is posted, no problem arises. It the usual course of events that individuals desirous of being present are free to do so. There is the well recognized exception though that warrants the exclusion of the public where the evidence may be characterized as "offensive to decency or public morals." - What did occasion difficulty in this suit was that for the convenience of the parties, and of the city court Judge, it was in the latter's air-conditioned chambers that the trial was held. There is no showing that the public was thereby excluded. It is to be admitted that the size of the room allotted the Judge would reduce the number of those who could be our present. Such a fact though is not indicative of any transgression of this right. Courtrooms are not of uniform dimensions. Some are smaller than others. Moreover, as admitted by Justice Black in his masterly In re Oliver opinion, it suffices to satisfy the requirement of a trial being public if the accused could "have his friends, relatives and counsel present, no matter with what offense he may be charged." - It is an undisputed fact that at least fourteen hearings had been held in chambers of the city court Judge, without objection on the part of respondent policemen. - There is much to be said of course for the concern displayed by respondent Judge to assure the reality as against the mere possibility of a trial being truly public. If it were otherwise, such a right could be reduced to a barren form of words. To the extent then that the conclusion reached by him was motivated by an apprehension that there was an evasion of a constitutional command, he certainly lived up to what is expected of a man of the robe. Further reflection ought to have convinced him though that such a fear was unjustified. An objective appraisal of conditions in municipal or city courts would have gone far in dispelling such misgivings. The crowded daily calendar, the nature of the cases handled, civil as well as criminal, the relaxed attitude on procedural rules not being strictly adhered to all make for a less tense atmosphere. As a result the attendance of the general public is much more in evidence; nor is its presence unwelcome. When it is remembered further that the occupants of such courts are not chosen primarily for their legal acumen, but taken from that portion of the bar more considerably attuned to the pulse of public life, it is not to be rationally expected that an accused would be denied whatever solace and comfort may come

a2010

page 135

Prof.
WON the right of the petitioners to a speedy trial has been accorded HELD NO Ratio. The constitutional right to a speedy trial means one free from vexatious, capricious and oppressive delays [Acebedo v. Sarmiento, Conde v. Rivera]. Thus, if the person accused were innocent, he may within the shortest time possible be spared from anxiety and apprehension arising from a prosecution, and if culpable, he will not be kept long in suspense as to the fate in store for him, within a period of course compatible with his opportunity to present any valid defense. -"The Government should be the last to set an example of delay and oppression in the administration of justice and it is the moral and legal obligation of this court to see that the criminal proceedings against the accused come to an end and that they be immediately discharged from the custody of the law." [People v. Castaeda] - An accused person is entitled to a trial at the earliest opportunity. . . . He cannot be oppressed by delaying the commencement of trial for an unreasonable length of time. If the proceedings pending trial are deferred, the trial itself is necessarily delayed. xxx The Constitution does not say that the right to a speedy trial may be availed of only where the prosecution for crime is commenced and undertaken by the fiscal. It does not exclude from its operation cases commenced by private individuals. Where once a person is prosecuted criminally, he is entitled to a speedy trial, irrespective of the nature of the offense or the manner in which it is authorized to be commenced. [Mercado v. Santos] - remedies available to the accused: The remedy in the event of a non-observance of this right is by habeas corpus if the accused were restrained of his liberty, or by certiorari, prohibition, or mandamus for the final dismissal of the case. [Acebedo v. Sarmiento] Reasoning - In the absence of any valid decision, the stage of trial has not been completed. Thus, when they moved to dismiss in the CA, they could contend that they had not been accorded their right to be tried as promptly as circumstances permit. It was not the pendency in the Court of Appeals of their cases that should be deemed material. It is at times unavoidable that appellate tribunals cannot, even with due diligence, put an end to suits elevated to

from the knowledge that a judge, with the eyes of the alert court alert to his demeanor and his rulings, would run the risk of being unjust, unfair, or arbitrary. Nor does it change matters, just because, as did happen here, it was in the air-conditioned chambers of a city court judge rather than in the usual place that the trial took place. Dispositive Writ of certiorari is granted.

FLORES v PEOPLE 61 SCRA 331 FERNANDO, December 10, 1974


NATURE Petition for certiorari FACTS - December 1951: Francisco Flores and Francisco Angel were accused for robbery - November 1955: TC found them guilty of robbery - December 1955: the 2 petitioners filed a notice of appeal - February 1958: CA issued a resolution remanding the records of the case to the lower court for a rehearing of the testimony of a certain witness deemed material for the disposition of the case. - August 1959: CA granted motion of petitioners to set aside the TC decision so that evidence for the defense on certain new facts or matters may be received, and that a new decision be rendered. Case was returned to TC but for about a year nothing happened because the offended party failed to appear for about 6 or 7 times. And when the offended party took the witness stand, he could no longer remember the details of the crime, even failed to identify the 2 petitioners. TC just sent back the records to CA - May 1965: petitioners sought the dismissal of their case due to the inordinate delay in their disposition (for almost 10 yrs), invoking constitutional right to a speedy trial - January 1966: CA denied motion for reconsideration Respondents defense: (1) case was not properly captioned, the CA not being made a party to the petition (2) it was not adequately shown that the right to a speedy trial had been violated, CA taking all the steps necessary to complete the transcript of stenographic notes of the original trial. ISSUE

Criminal Procedure Rowena Daroy Morales


them. What is decisive is that with the setting aside of the previous decision in the resolution of August 5, 1959, petitioners could validly premise their plea for dismissal on this constitutional safeguard. *the procedural issue on the CA not being made the party respondent, Court considered the substantial issues over this technicality. Dispositive petition for certiorari is granted, and the order of the Court of Appeals in CA-GR No. 16641-R entitled, People v. Francisco Flores, et al., of September 28, 1965 denying the motion to dismiss as well as its order of January 8, 1966 denying the motion for reconsideration, and the order of January 28, 1966 denying the second motion for reconsideration are hereby set aside, nullified, and considered of no force and effect. The criminal case against petitioners in the aforesaid CA-GR No. 16641R are ordered dismissed.

a2010

page 136

Prof.
subsided. Indeed we cannot say that the trial of the petitioners was being held under circumstances which did not permit the observance of those imperative decencies of procedure which have come to be identified with due process. - Even granting the existence of "massive" and "prejudicial" publicity, since the petitioners here do not contend that the respondents have been unduly influenced but simply that they might be by the "barrage" of publicity. 2. YES Ratio Each of the 23 petitioners (accused before the general court-martial) is entitled to one peremptory challenge, irrespective of the number of specifications and/or charges and regardless of whether they are tried jointly or in common. The right to challenge is in quintessence the right to reject, not to select. If from the officers who remain an impartial military court is obtained, the constitutional right of the accused to a fair trial is maintained. Reasoning - It is the submission of the petitioners that "for every charge, each side may exercise one peremptory challenge," and therefore because there are eleven charges they are entitled to eleven separate peremptory challenges. The respondents argue that although there are actually a total of eleven specifications against the petitioners, three of these should be considered as merged with two other specifications, "since in fact they allege the same offenses committed in conspiracy, thus leaving a balance of eight specifications." The general courtmartial thereof takes the position that all the 23 petitioners are entitled to a total of only eight peremptory challenges. (a) A peremptory challenge is afforded to an accused who, whether rightly or wrongly, honestly feels that the member of the court peremptorily challenged by him cannot sit in judgment over him, impartially. Every accused person is entitled to a fair trial. It is not enough that objectively the members of the court may be fair and impartial. It is likewise necessary that subjectively the accused must feel that he is being tried by a fair and impartial body of officers. Because the petitioners may entertain grave doubts as to the fairness or impartiality of distinct, separate and different individual members of the court-martial, it follows necessarily that each of the accused is entitled to one peremptory challenge. (b) Article of War 18 does not distinguish between common trials and joint trials, nor does it make the nature or number of specifications and/or charges a

MARTELINO v ALEJANDRO 32 SCRA 106 CASTRO; March 25, 1970


NATURE Petition for certiorari and prohibition, to nullify the orders of the court-martial denying their challenges, both peremptory and for cause FACTS - There are ongoing court-martial proceedings against the petitioner, Major Eduardo Martelino, alias Abdul Latif Martelino, of the AFP, and the officers and men under him, for violation of the 94th and 97th Articles of War, as a result of the alleged shooting of some Muslim recruits then undergoing commando training on the island of Corregidor. - Initially there was a question of jurisdiction: WON the general court-martial, convened to try the case, acquired jurisdiction over the case despite the fact that about a month earlier, a complaint for frustrated murder had been filed in the fiscal's office of Cavite City. Proceedings had to be suspended until SC finally ruled in favor of the jurisdiction of the military court. - After that, Martelino sought the disqualification of the President of the general court-martial, following the latter's admission that he read newspaper stories of the Corregidor incident. He contended that the case had received such an amount of publicity in the press and was being exploited for political purposes in connection with the upcoming 1969 presidential elections as to imperil his right to a fair trial.

- They then raised peremptory challenges against Col. Alejandro, as president of the court-martial, and Col. Olfindo, Lt. Col. Camagay, Lt. Col. Valones, Lt. Col. Blanco and Col. Malig, as members. There was also an issue as to the number of peremptory challenges that can be raised by each accused. - SC then restrained court-martial from proceeding with the case. Respondents asserted that despite the publicity which the case had received, no proof has been presented showing that the court-martial's president's fairness and impartiality have been impaired. As a preliminary consideration, the respondents urge this Court to throw out the petition on the ground that it has no power to review the proceedings of the court-martial, "except for the purpose of ascertaining whether the military court had jurisdiction of the person and subject matter, and whether, though having such jurisdiction, it had exceeded its powers in the sentence pronounced." ISSUES 1. WON the publicity given to the case against the petitioners was such as to prejudice their right to a fair trial 2. WON each accused was entitled to one peremptory challenge HELD 1. NO Reasoning - The trial courts must take strong measures to ensure that the balance is never weighed against the accused. And appellate tribunals have the duty to make an independent evaluation of the circumstances. Where there is a reasonable likelihood that prejudicial news prior to trial will prevent a fair trial, the judge should continue the case until the threat abates, or transfer it to another county not so permeated with publicity. If publicity during the proceeding threatens the fairness of the trial, a new trial should be ordered. - The spate of publicity in this case before us did not focus on the guilt of the petitioners but rather on the responsibility of the Government for what was claimed to be a "massacre" of Muslim trainees. If there was a "trial by newspaper" at all, it was not of the petitioners but of the Government. Absent here is a showing of failure of the court-martial to protect the accused from massive publicity encouraged by those connected with the conduct of the trial either by a failure to control the release of information or to remove the trial to another venue or to postpone it until the deluge of prejudicial publicity shall have

Criminal Procedure Rowena Daroy Morales


determinant. (c) A perceptive analysis of the companion articles convinces us that the word, "each side," as used in the said article in reference to the defense, should be construed to mean each accused person. Dispositive Subject to our pronouncement that each of the 23 petitioners is entitled to one separate peremptory challenge, the present petition is DENIED. The temporary restraining order issued by this Court is hereby lifted.

a2010

page 137

Prof.
- In the main, appellant's contention in this appeal is that the evidence of the prosecution is weak and insufficient to establish his guilt beyond reasonable doubt, particularly in the manner he was Identified as one of the alleged perpetrators of the four (4) crimes herein charged. He would then argue that such weakness of the state evidence would commensurately strengthen his defense of alibi which, as he claims, the trial court erroneously rejected. - What is indubitably clear is that the state witnesses who pointed to appellant as among the culprits are the ones with no motive sufficient to urge them to testify falsely against appellant who would be punished with no less than death in consequence of their perjured testimony. - First to Identify appellant was Antonio Bejic the lone survivor in the carnage against the Bejic family and household. He Categorically stated that he recognized the voice that shouted: "Hoy", to be that of appellant whom he had known very well because appellant used to sleep in Antonio's grandfather with whom he was residing. Moreover, Antonio also testified having seen appellant stab his cousin, Roberto Bejic He was however candid enough to admit that he did not see who actually killed the rest of his relatives, nor who actually burned the house. This fact would tend to negate appellant's claims that the testimony of the state witnesses were scripted and merely rehearsed. - That the appellant and his companions were the perpetrators of the killing of Jesus Limen and also the burning of his house was positively established by the testimony of Ramon Jimenez. Ramon was with Jesus Limen when the latter was killed and his house burned. That he was taken along by appellant apparently as a hostage to prevent him from reporting to the authorities, and so he was arrested with appellant together is, undeniably, one circumstance strongly lending credence to all that Ramon Jimenez testified to, particularly on appellant's complicity in the commission of the four (4) crimes. - The testimony of Ramon Jimenez of course did not escape the very keenly scrutinizing examination to which appellant's counsel subjected the testimonies of all the other state witnesses, in a zealous effort to show their want of credibility with the inconsistencies, improbabilities and contradictions they supposedly contained. Suffice it to say that the alleged contradictions and inconsistencies were on minor and inconsequential details that would not in any way affect Ramon's credibility.

PEOPLE v ORSAL 113 SCRA 226 PER CURIAM; March 29, 1982
NATURE Automatic review FACTS - Vicente Orsal is one of four accused in four separate cases filed in the Court of First Instance of Zamboanga City - The three (3) other accused are Ramon Gutierrez, Generoso Abapo and Romeo Flores, but the two (2) last named accused having gone at large, only appellant Vicente Orsal and Ramon Gutierrez stood trial - The cases stemmed from the ff acts: *On April 13, 1974 appellant, together with Generoso Abapo, Ramon Gutierrez and Romeo Flores, killed Crisanto Bejic, his wife Eduarda, as well as his grandchildren Atanacia Legazpi and Roberto Bejic, and burned his house *On April 14, 1974, the same accused ransacked the house of Jesus Limen, taking his shoes and clothes, one paltik revolver, two paltik shotguns and a hand grenade, killed Jesus Limen, and burned his house - The court found Orsal and Gutierrez guilty beyond reasonable doubt as principals of the crimes, attended by two (2) aggravating circumstances, namely, that the crimes were committed at nighttime and by a band, without any mitigating circumstances to offset the same, and sentenced them accordingly: (1) Criminal Case No. 471 (1183) for Arson-penalty of Ten (10) years and One (1) day of Prision Mayor as the minimum to Seventeen (17) years, Four (4) months and One (1) day of Reclusion Temporal as the maximum, to indemnify Francisco Limen the amount of P8,000.00, and to pay of the costs of this suit (2) Criminal Case No. 472 (1184) for Arson- penalty of Ten (10) years and One (1) day of Prision Mayor as

the minimum to Seventeen (17) years, Four (4) months and One (1) day of Reclusion Temporal as the maximum, to indemnify Francisco Limen the amount of P8,000.00, and to pay of the costs of this suit (3) Criminal Case No. 473 (1185) for Robbery in Band with Multiple Homicide- four separate and distinct imprisonment's of Ten (10) years and One (1) day of Prision Mayor as the minimum to Seventeen (17) years and Five (5) months of Reclusion Temporal as the maximum, to indemnify the heirs of the deceased Cristino Bejic, Eduarda Bejic, Roberto Bejic and Atanasia Legazpi the total amount of P48,000.00 and to pay of the costs of this suit in each of these four cases (4) Criminal Case No. 432 (1157) for Robbery in Band with Homicide- Supreme penalty of 'DEATH', to indemnify the heirs of the deceased Jesus Limen the amount of P12,000.00, and to pay one fourth of the costs in each of these four cases - In the case of Gutierrez, owing to the fact that he is a youthful offender, as he is only 18 years old, the Court ordered his commitment to the Department of Social Services and Development, Region IX, Zamboanga City - The sentence of death having been imposed in Criminal Case No. 432 (1157), which was heard jointly with the other three (3) cases, the single decision rendered for all the four (4) cases is before the SC for automatic review. ISSUES 1. WON the appellant has been denied his constitutional right to a speedy trial 2. WON the evidence is sufficient to establish his guilt beyond reasonable doubt HELD 1. NO - Appellant first contends that he has been denied his constitutional right of speedy trial because the information was filed only about nine (9) months after his arrest and investigation. - There was no such denial. As correctly set forth by the Solicitor General, the test of violation of the right to speedy trial has always been to begin counting the delay from the time the information is filed, not before the filing. The delay in the filing of the information, which in the instant case has not been without reasonable cause, is, therefore, not to be reckoned with in determining whether there has been a denial of the right to speedy trial. 2. YES

Criminal Procedure Rowena Daroy Morales


- As earlier stated, no sufficient motive was shown why the two (2) eyewitnesses, Antonio Bejic and Ramon Jimenez, would perjure themselves in pointing to appellant as one of the perpetrators of very grave and heinous crimes. That appellant's mother may have accused Ramon Jimenez of theft would, therefore, not serve as motive to induce the two (2) aforenamed witnesses to give the testimony that named and pointed to not only appellant but three (3) other persons including Ramon Gutierrez who stood trial with appellant, as the malefactors. Even as to Ramon Jimenez alone, the alleged motive is assuredly not enough for him to charge falsely appellant, who is his cousin, with such grievous offenses as the killing of four (4) persons, burning down of two (2) houses and stealing of valuable personal belongings. - From how appellant's identity as one of the perpetrators of the four (4) crimes just mentioned has been established, his defense of alibi is futile. The barrio captain of New Sagay,Jesus Agabon, with whom appellant claimed to be with when the crimes were being committed, was presented to corroborate appellant's alibi by testifying that in accordance with the regulation in his place, he registered appellant's name in a notebook as a transient therein. His testimony became worthless when despite sufficient opportunity given him to produce the notebook in Court, by postponing the hearing not just once but twice, the defense witness never again appeared in Court. If his salvation hanged on the credibility of this witness, appellant should have done everything and resorted to even the coercive process of the Court to make said witness go to Court and present the desired document, or explain his inability to do so. Dispositive Decision affirmed in toto.

a2010

page 138

Prof.
- The right to a speedy trial means that the accused is free from vexatious, capricious, and oppressive delays, its salutary objective being to assure that an innocent person may be free from anxiety and expense of a court litigation or, if otherwise, of having his guilt determined within the shortest possible time compatible with the presentation and consideration of whatever legitimate defense he may interpose. - The delays in the prosecution of the offenses were all caused by the accused so he cannot invoke constitutional right to speedy trial. By his own deliberate acts, he is deemed to have waived or abandoned his right to a speedy trial 2. NO - The dismissal of the criminal cases against the accused by the respondent court on the ground that his right to speedy trial had been violated was devoid of factual and legal basis. - In order that the protection against double jeopardy may inure to the benefit of an accused, the following requisites must be present in the first prosecution: (a) a valid complaint or information; (b) a competent court; (c) the defendant had pleaded to the charge; and (d) the defendant was acquitted, or convicted, or the case against him was dismissed or otherwise terminated without his express consent. - The last requisite is not present because the order of the CFI judge was null and void. Dispositive Petition granted. The criminal cases are reinstated and the proper regional trial court is ordered to proceed with all deliberate speed in these cases.

PEOPLE v JARDIN 124 SCRA 167 GUTIERREZ JR; August 17, 1983
NATURE Petition for certiorari on decision of CFI Quezon dismissing the criminal cases against accused Demetrio Jardin because his constitutional right to speedy trial was allegedly violated. FACTS - The criminal prosecutions originated from a lettercomplaint of the Provincial Auditor of Quezon requesting the Provincial Fiscal to file the necessary criminal action under Article 217 of the Revised Penal

Code against Demetrio Jardin for malversation of public funds thru falsification of public documents on six counts. (1967) - {This case is full of delaying tactics} - PI 1: accused moved to postpone 4 times, and failed to appear everytime. - PI was nevertheless conducted. And the six criminal informations were filed in CFI. - AR 1: accused moved to postpone 4 time, never appeared; counsel asked for reinvestigation on the ground that the accused was not given the opportunity to present his defense during the preliminary investigation. Court granted motion. - PI 2: accused moved to postpone many times, failed still to appear. When he finally appeared with his counsel, they asked for 15 days to file memorandum. The memorandum was never filed, so the investigating fiscal filed a manifestation before the court that the records of these cases be returned and the trial on the merits of the same be set. - The court transferred the case to new branch of CFI Quezon without acting on manifestation. Arraignment date was set. - AR 2: more postponements at instance of accused; moved for reinvestigation again. Court granted. - PI 3: reset because no show. Counsel then asked for 5 days to file written sworn statement of accused as defense. No statement was submitted so the records of the case were returned to court. A date was set for arraignment. - AR 3: accused asked for postponement. - Arraignment finally happened on Sept 8, 1970. Accused pleaded NOT GUILTY and asked for trial to be postponed. On postponed date, accused asked for another postponement. - Oct 1970, accused and counsel were at trial; but no one appeared for prosecution, except for a state witness. Counsel moved (orally) for dismissal, invoking accused right to a speedy trial. Court granted motion and dismissed the cases. ISSUES 1. WON accused can invoke right to speedy trial 2. WON this appeal places the accused in double jeopardy HELD 1. NO - The respondent court committed a grave abuse of discretion in dismissing the cases and in basing the dismissal on the constitutional right of the accused to speedy trial.

PEOPLE v ANG GIOC 73 PHIL 336 ABAD SANTOS; October 31, 1941
NATURE Petition for a writ of certiorari FACTS - Ang Gioc, together with Sio Go, Gang Kan, Kee Ya and Chua Chui, was charged with the crime of frustrated murder in the Court of First Instance of Manila. He was released on bail. After a protracted trial, which lasted several months, Ang Gioc and one of his co-accused, Sio Go, were found guilty and sentenced to twelve years and one day of cadena temporal. Ang Gioc and his sureties were duly notified to appear before the court for the reading of

Criminal Procedure Rowena Daroy Morales


the sentence, but the former failed to appear and thereupon the trial judge ordered his arrest and the confiscation of the bond furnished for his temporary release. - All attempts to arrest him proved futile. He was, however, finally arrested after nearly thirteen years from the date fixed for the reading of the sentence. He was subsequently brought before the court and the sentence was read to him, from which he appealed to the Court of Appeals where, against the objection of the Solicitor General, he was allowed to file a bond for his temporary release. - In perfecting the record on appeal it was found that the stenographic notes taken during the trial were not transcribed and that the two stenographers who took the notes were already dead. The matter was referred to several stenographers who stated that they could not transcribe the notes because the deceased had used systems known only to themselves. In this situation, Ang Gioc petitioned the Court of Appeals to remand the cause to the court below for a new trial. - CA remanded for new trial ISSUE WON the CA acquired jurisdiction of the appeal filed by him HELD NO - The accused has rights, one of which is the right of appeal; but this is a purely statutory, not a constitutional, right and this is not one of those fundamental rights which cannot be waived. This right is granted solely for the benefit of the accused. He may avail of it or not, as he pleases. He may waive it either expressly or by implication. When the accused flees after the case has been submitted to the court for decision, he will be deemed to have waived his right to appeal from the judgment rendered against him. Such was the situation with reference to the the respondent Ang Gioc. He was duly notified to appear before the trial court for the reading of the sentence, but failed to do so; and when an order was issued for his arrest, the warrant could not be served on him because he could not be found. Whether or not he escaped to China is immaterial for our present purpose. The fact remains that he succeeded in evading arrest for nearly thirteen years. The record shows that upon his failure to appear for the reading of the sentence, the trial court declared the confiscation of the bond filed by Ang Gioc, and later issued the corresponding order of

a2010

page 139

Prof.
in crimes that shock the conscience. This concern cannot be diluted. - We have always reviewed the imposition of the death penalty regardless of the will of the convict. Our unyielding stance is dictated by the policy that the State should not be given the license to kill without the final determination of this Highest Tribunal whose collective wisdom is the last, effective hedge against an erroneous judgment of a one-judge trial court. This enlightened policy ought to continue as our beacon light for the taking of life ends all rights, a matter of societal value that transcends the personal interest of a convict. The importance of this societal value should not be blurred by the escape of a convict which is a problem of law enforcement. Neither should this Court be moved alone by the outrage of the public for the rise in statistics of heinous crimes for our decisions should not be directed by the changing winds of the social weather. Let us not for a moment forget that an accused does not cease to have rights just because of his conviction. This principle is implicit in our Constitution which recognizes that an accused, to be right, while the majority, even if overwhelming, has no right to be wrong.

execution. This action of the court amounted to a judicial declaration that Ang Gioc was a fugitive from justice, and such declaration cannot after the lapse of nearly thirteen years be controverted by proof aliunde. A contrary view would encourage accused persons to trifle with the administration of justice, and provide means for guilty parties to escape punishment. We reject it without the least hesitation by declaring that Ang Gioc had waived his right to appeal from the judgment rendered against him. The law will not allow a person to take advantage of his own wrong. - Ang Gioc has waived his right of appeal. Court of Appeals acquired no jurisdiction of the appeal filed by him, except to dismiss it; and that court acted in excess of its jurisdiction when it ordered the cause to be remanded to the court of origin for a new trial Dispositive remanding order must be set aside, and the judgment of the Court of First Instance of Manila declared final and executory.

PEOPLE v ESPARAS 260 SCRA 539 PUNO; August 20, 1996


FACTS - Accused Esparas was charged with violation of RA No. 6425 as amended by RA No. 759 for importing into the country 20 kilograms of "shabu" in Criminal Case No. 94-5897 before the RTC of Pasay City, Br. 114. After arraignment, the accused escaped from jail and was tried in absentia. On March 13, 1995, the trial court found her guilty as charged and imposed on her the death penalty. The accused remains at large up to the present time. ISSUE WON the Court will proceed to automatically review her death sentence HELD YES - The power of this Court to review a decision imposing the death penalty cannot be waived either by the accused or by the courts. Ours is not only the power but the duty to review all death penalty cases. No litigant can repudiate this power which is bestowed by the Constitution. The power is more of a sacred duty which we have to discharge to assure the People that the innocence of a citizen is our concern not only in crimes that slight but even more,

SAMSON v CA [SUPRA, PAGE 84] PEOPLE v CITY COURT OF MANILA [SUPRA, PAGE 72] GALMAN v SANDIGANBAYAN [SUPRA, PAGE 82] CAES v IAC 179 SCRA 54 CRUZ; November 6, 1989
FACTS - Joel Caes was charged in 2 separate informations with ILLEGAL POSSESSION OF FIREARMS AND ILLEGAL POSSESSION OF MARIJUANA before the CFI of Rizal. (SHORT version: this guy stayed in prison for 3 yrs coz nothing was happening with his case-trial was postponed 11 times-the court then decided to dismiss it provisionally but was later on revived. ***But if maam wants details, read on!) - The cases were consolidated on Dec 10, 1981.

Criminal Procedure Rowena Daroy Morales


- Arraignment was originally scheduled on January 11, 1982, but was for some reason postponed. - August 31, 1982, Caes was arraigned and pleaded not guilty. - Trial was scheduled for October 13, 1982, but this was reset upon agreement of the parties. - Nov 15, 1982: the trial was again postponed for reasons that do not appear in the record. - Dec 20, 1982: the trial was again postponed because the prosecution witnesses were absent. - Jan 19, 1983: the third resetting of the case was also canceled, no reason appearing in the record. - Feb 21, 1983: no trial could be held again, because witnesses being absent. - March 21, 1983: the trial was reset once more, again because the prosecution witnesses were absent. - April 19, 1983: the trial of the case had not yet started. It was reset because the prosecution witnesses were again absent. - June 3, 1983, a sheriffs return informed the trial court that the prosecution witnesses, namely, Capt. Carlos Dacanay and Sgt. Bonifacio Lustado had been personally served with subpoena to appear and testify at the hearing scheduled on June 6, 1983. - June 6, 1983: the trial was again postponed, this time because there was no trial fiscal. - July 12, 1983: trial was reset for lack of material time. - Sept 6, 1983: The trial was once more reset by agree-judgment of the parties. - Oct 19, 1983: the trial was reset to November 14, 1983. - Nov 14, 1983: the prosecution moved for the provisional dismissal of the case because its witnesses had not appeared. - On the same date, Judge Gorgonio ordered the case Provisionally Dismissed - Jan 9, 1984: a motion to revive the cases was filed by Maj. Dacanay (he had been promoted in the meantime) and Sgt. Lustado who alleged that they could not attend the hearing scheduled on November 14, 1983, for lack of notice. - Copy of the motion was furnished the City Fiscal of Caloocan City but not the petitioner. Said motion was granted by J. Gorgonio -Caes filed a MR but was denied and the revived cases were set from hearing on Nov. 19, 1984. - Caes questioned the judge's order on certiorari with this Court, which referred his petition to the IAC. - IAC dismissed it for lack of merit on May 20, 1986, and reconsideration was denied on June 17, 1986.

a2010

page 140

Prof.
motion was granted just the same, and ex parte at that and without hearing, and the petitioner's subsequent objection was brushed aside. 2. YES - Fittingly described as "res judicata in prison grey," the right against double jeopardy prohibits the prosecution of a person for a crime of which he has been previously acquitted or convicted. The purpose is to set the effects of the first prosecution forever at rest, assuring the accused that he shall not thereafter be subjected to the danger and anxiety of a second charge against him for the same offense. - People v. Ylagan, Mendoza v. Almeda Lopez, People v. Obsania ~ To constitute double jeopardy, there must be: (a) a valid complaint or information; (b) filed before a competent court; (c) to which the defendant had pleaded; and (d) of which he had been previously acquitted or convicted or which was dismissed or otherwise terminated without his express consent. - There is no question that the first three requisites are present in the case at bar. WHAT IS THE EFFECT OF THE PROV. DISMISSAL? - People v Ylagan: It is settled that a case may be dismissed if the dismissal is made on motion of the accused himself or on motion of the prosecution with the express consent of the accused. Such a dismissal is correctly denominated provisional. But a dismissal is not provisional even if so designated if it is shown that it was made without the express consent of the accused. This consent cannot be presumed nor may it be merely implied from the defendant's silence or his failure to object. - Pendatum v. Aragon, People v. Hinaut, Solis v. Agloro: Such consent must be express, so as to leave no doubt as to the defendant's conformity. Otherwise, the dismissal will be regarded as final, i.e., with prejudice to the refiling of the case. - There are instances in fact when the dismissal will be held to be final and to dispose of the case once and for all even if the dismissal was made on motion of the accused himself. - The first is where the dismissal is based on a demurrer to the evidence filed by the accused after the prosecution has rested. Such dismissal has the effect of a judgment on the merits and operates as an acquittal. - The other exception is where the dismissal is made, also on motion of the accused, because of the denial of his right to a speedy trial. This is in effect a failure to prosecute. - SC said that this case is similar to Conde v. Rivera (so this doctrine applies in this case too) where a prosecuting officer, without good cause, secures

ISSUES 1. WON the motion to revive the cases was invalid because it was not filed by the proper party nor was a copy served to CAES 2. WON the revival of the cases would place the petitioner double jeopardy in violation of the Bill of Rights HELD 1. YES - The trial judge erred in ordering the revival of the cases against Caes and that CFI also erred in affirming that order. Caes having been denied his constitutional right to a speedy trial, and not having expressly consented to the "provisional" dismissal of the cases against him, he was entitled to their final dismissal under the constitutional prohibition against double jeopardy. Reasoning - Rule 110, See. 5, par.1: It is axiomatic that the prosecution of a criminal case is the responsibility of the govt prosecutor and must always be under his control. - Herrero v. Diaz: This is true even if a private prosecutor is allowed to assist him and actually handles the examination of the witnesses and the introduction of other evidence. - The witnesses, even if they are the complaining witnesses, cannot act for the prosecutor in the handling of the case. Although they may ask for the filing of the case, they have no personality to move for its dismissal or revival as they are not even parties thereto nor do they represent the parties to the action. Their only function is to testify. - In a criminal prosecution, the plaintiff is represented by the government prosecutor, or one acting under his authority, and by no one else. - It follows that the motion for the revival of the cases filed by prosecution witnesses (who never even testified) should have been summarily dismissed by the trial judge. - The mere fact that the government prosecutor was furnished a copy of the motion and he did not interpose any objection was not enough to justify the action of these witnesses. -The prosecutor should have initiated the motion himself if he thought it proper. The presumption that he approved of the motion is not enough, especially since we are dealing here with the liberty of a person who had a right at least to be notified of the move to prosecute him again. - The fact that he was not so informed made the irregularity even more serious. It is curious that the

Criminal Procedure Rowena Daroy Morales


postponements of the trial of a defendant against his protest beyond a reasonable period of time, as in this instance for more than a year, the accused is entitled to relief ... - The circumstance that the dismissal of the cases against the petitioner was described by the trial judge as "provisional" did not change the nature of that dismissal. As it was based on the "lack of interest" of the prosecutor and the consequent delay in the trial of the cases, it was final and operated as an acquittal of the accused on the merits. - No less importantly, there is no proof that Caes expressly concurred in the provisional dismissal. Implied consent, as we have repeatedly held, is not enough; neither may it be lightly inferred from the presumption of regularity, for we are dealing here with the alleged waiver of a constitutional right. Any doubt on this matter must be resolved in favor of the accused. - Whos fault was it then? The responsibility clearly lies with the Office of the City Prosecutor of Caloocan City for its negligence and ineptitude. Dispositive PETITION IS GRANTED. DISMISSAL OF THE CRIMINAL CASES declared as FINAL.

a2010

page 141

Prof.
- Rivera pleaded not guilty to all 13 cases. A pre-trial was conducted. Prosecution presented its witnesses and offered documentary exhibits. - September 29, 2003: The defense was scheduled to present evidence; however, during the hearing, petitioners former counsel, Atty. Benjamin C. Belarmino, Jr., informed the court that they have not yet received the resolution on the prosecutions Formal Offer of Exhibits, further manifesting that upon receipt of the resolution, they will ask for leave of court to file demurrer to evidence. - The court directed Atty. Belarmino to file a demurrer to evidence even without leave of court but the latter manifested that he would still discuss the matter with his collaborating counsel. However, in the order issued by the Sandiganbayan, it was stated that petitioner, through counsel, manifested that he would be filing a demurrer to evidence without leave of court within 10 days. - October 20, 2003: Demurrer to Evidence was filed without leave of court - October 27, 2003: Prosecution filed its Opposition. - Pursuant to Section 23, Rule 119 of The Revised Rules of Criminal Procedure, the Sandiganbayan considered the right of petitioner to present evidence waived and deemed the case submitted for judgment on the basis of the evidence for the prosecution. - May 3, 2004: the Sandiganbayan found petitioner guilty in all 13 cases. - May 17, 2004: Petitioner moved for reconsideration of the decision and further moved that he be allowed to present evidence. - June 10, 2004: MFR denied ISSUE WON the decision and resolution of the Sandiganbayan should be set aside to allow petitioner to present evidence despite the demurrer to evidence filed

- Father of victim filed Petition for Certiorari before CA. CA nullified the orders of TC and held that Godoys Motion to Acquit was a demurrer to evidence. ISSUE WON the Motion to Acquit is the same as a motion to dismiss on demurrer to evidence HELD NO - Section 15 Rule 119 of the 1985 Rules on Crim Pro is relevant. The rule is explicit that in filing a Motion to Dismiss on ground of insufficiency of evidence, an accused waives the right to present evidence. There is no material difference between the Motion to Acquit by Godoy and a demurrer to evidence. A different label doesnt change the true nature of pleading. - The rationale is that when accused moves for dismissal on ground of insufficiency of evidence, he does so in belief that evidence is insufficient to convict and any need for him to present evidence is negated. Accused cannot be allowed to wager on outcome of judicial proceedings by espousing inconsistent viewpoints. - HOWEVER, in this case, this rule is inapplicable. First, because the prosecution has not yet rested its case. The documentary exhibits are still in issue. Second, the TC lost no time in denying Motion to Acquit and Godoy was ready to present evidence but prosecution moved to disqualify him. The practice sought to be avoided by the rule is inexistent.

PEOPLE v PANFILO LACSON [SUPRA, PAGE 74] GODOY v CA MELENCIO-HERRERA; August 30, 1988
NATURE Petition for certiorari to review the decision of the CA FACTS - Godoy is one of 6 accused of homicide. All pleaded not guilty. Trial proceeded until prosecution concluded presentation of evidence after which prosecution formally offered documentary exhibits. Before defense submitted objections to offer, petitioner filed Motion to Acquit on ground of lack of evidence proving guilt beyond reasonable doubt. - TC denied Motion to Acquit and admitted Formal Offer of Documentary Exhibits for prosecution. - Prosecution moved for disqualification of Godoy from presenting evidence as well as his exclusion from proceedings on ground that the Motion to Acquit is equivalent to a demurrer to the evidence so he had already waived his right to present evidence. TC denied the disqualification and allowed Godoy to present evidence. Prosecution filed MFR but this was denied.

RIVERA v PEOPLE YNARES-SANTIAGO; June 9, 2005


NATURE Petition for review on certiorari under Rule 45 of the Rules of Court FACTS - Juan G. Rivera and Eric O. Garcia, municipal mayor and disbursement officer, respectively, of Guinobatan, Albay, were charged before the Sandiganbayan with 12 counts of falsification of public documents and 1 count of malversation of public funds involving the amount of P1,936,798.64 given to the Municipality of Guinobatan as calamity fund for the victims of the Mayon volcanic eruption. Garcia died on August 25, 2001 and was accordingly dropped from the amended information.

HELD YES - A demurrer to evidence is defined as an objection by one of the parties in an action, to the effect that the evidence which his adversary produced is insufficient in point of law, whether true or not, to make out a case or sustain the issue. The party demurring challenges the sufficiency of the whole evidence to sustain a verdict. In passing upon the sufficiency of the evidence raised in a demurrer, the court is merely required to ascertain whether there is

Criminal Procedure Rowena Daroy Morales


competent or sufficient proof to sustain the indictment or to support a verdict of guilt. - The order dated September 29, 2003, inaccurately stated that Atty. Belarmino manifested that he will be filing a demurrer to evidence even without leave of court when the records show no such manifestation was made. On the contrary, the records show that Atty. Belarmino asked for leave of court to file a demurrer to evidence and for time to discuss the same with his co-counsel but was instead ordered by the court to file the same without leave of court within ten days. - Atty. Belarmino did not cite any ground when he moved for leave of court to file demurrer to evidence; neither did the Sandiganbayan make any inquiry thereon before issuing the order, directing the petitioner to file a demurrer to evidence even without leave of court. This is contrary to the provisions of Section 23, Rule 119 of the Revised Rules of Criminal Procedure which specifically instructs that the motion for leave of court to file demurrer to evidence shall specifically state its grounds. - Petitioner was not consulted nor did his counsel confer with him and ask whether he understood the significance of filing a demurrer to evidence. Atty. Belarmino was not given the opportunity to discuss with petitioner the consequences of filing a demurrer to evidence without leave of court. - People v. Bodoso: Henceforth, to protect the constitutional right to due process of every accused in a capital offense and to avoid any confusion about the proper steps to be taken when a trial court comes face to face with an accused or his counsel who wants to waive his clients right to present evidence and be heard, it shall be the unequivocal duty of the trial court to observe, as a prerequisite to the validity of such waiver, a procedure akin to a searching inquiry as specified in People v. Aranzado when an accused pleads guilty, particularly 1. The trial court shall hear both the prosecution and the accused with their respective counsel on the desire or manifestation of the accused to waive the right to present evidence and be heard. 2. The trial court shall ensure the attendance of the prosecution and especially the accused with their respective counsel in the hearing which must be recorded. Their presence must be duly entered in the minutes of the proceedings. 3. During the hearing, it shall be the task of the trial court to

a2010

page 142

Prof.
Riveras were personally encashed by her while she was allegedly accompanied by Garcia. Witnesses who owned the forged receipts testified that they handed the blank receipts to Garcia and not to petitioner. Then, after receipt of the cash, the disbursement vouchers and other forms required to liquidate the amount were allegedly prepared by Lim and thereafter, transmitted to Rivera for approval, and finally to the provincial Government. It has not been satisfactorily established whether petitioner has appropriated, taken or misappropriated, or has consented to the taking by another person, of such funds. - The presentation of evidence by the defense would resolve any doubt as to petitioners complicity and avoid possible miscarriage of justice. - Clearly, when transcendental matters like life, liberty or State security are involved, suspension of the rules is likely to be welcomed more generously. The Rules on procedure are merely tools designed to facilitate the attainment of justice. When they are rigid and strict in application, resulting in technicalities that tend to frustrate rather than promote justice, the Court is empowered to suspend the rules. Dispositive Petition is GRANTED. Sandiganbayan resolutions SET ASIDE. Records of Criminal Case Nos. 26686-98 REMANDED to the Sandiganbayan for further proceedings.

a. ask the defense counsel a series of question to determine whether he had conferred with and completely explained to the accused that he had the right to present evidence and be heard as well as its meaning and consequences, together with the significance and outcome of the waiver of such right. If the lawyer for the accused has not done so, the trial court shall give the latter enough time to fulfill this professional obligation. b. inquire from the defense counsel with conformity of the accused whether he wants to present evidence or submit a memorandum elucidating on the contradictions and insufficiency of the prosecution evidence, if any, or in default theory, file a demurrer to evidence with prior leave of court, if he so believes that the prosecution evidence is so weak that it need not even be rebutted. If there is a desire to do so, the trial court shall give the defense enough time to this purpose. c. elicit information about the personality profile of the accused, such as his age, socio-economic status, and educational background, which may serve as a trustworthy index of his capacity to give a free and informed waiver. d. all questions posed to the accused should be in a language known and understood by the latter, hence, the record must state the language used for this purpose as well as reflect the corresponding translation thereof in English. - People v. Flores: Though the Rules require no such inquiry to be undertaken by the court for the validity of such waiver or any judgment made as result of the waiver, prudence, however, requires the Court to ascertain the same to avoid any grave miscarriage of justice. Any lawyer worth his salt ought to know that the filing of a demurrer to evidence with leave of court has the beneficial effect of reserving the movants right to present evidence if the demurrer is denied by the court. Thus, a counsel who files a demurrer with leave of court, but at the same time expressly waives his right to present evidence should put a judge on guard that said counsel may not entirely comprehend the consequences of the waiver. - The evidence on record do not clearly show where and to whom the allegedly malversed money were given after it was encashed. What is clear is that the calamity fund was released to Almeda O. Lim, the Municipal Treasurer of Guinobatan, Albay to which Official Receipt No. 8749242H was issued. Thereafter, checks bearing her signature and that of

HUN HYUNG PARK v EUNG WON CHOI CARPIO-MORALES; February 12, 2007
FACTS - Eung Won Choi, was charged for violation of BP 22, otherwise known as the Bouncing Checks Law, for issuing PNB Check No. 0077133 postdated August 28, 1999 in the amount of P1,875,000 which was dishonored for having been drawn against insufficient funds. He pleaded not guilty. - After the prosecution rested its case, respondent filed a Motion for Leave of Court to File Demurrer to Evidence to which he attached his Demurrer, asserting that the prosecution failed to prove that he received the notice of dishonor, hence, the presumption of the element of knowledge of insufficiency of funds did not arise. - (2/27/03) The MeTC of Makati, Branch 65 granted the demurrer and dismissed the case. The prosecutions motion for reconsideration was denied. - Park appealed the civil aspect of the case to the RTC of Makati, contending that the dismissal of the criminal case should not include its civil aspect. The

Criminal Procedure Rowena Daroy Morales


RTC held that while the evidence presented was insufficient to prove Chois criminal liability, it did not altogether extinguish his civil liability. It accordingly granted Parks appeal and ordered Choi to pay him P1,875,000 with legal interest. - Upon Chois motion for reconsideration, however, the RTC set aside its decision and ordered the remand of the case to the MeTC for further proceedings, so that Choi may adduce evidence on the civil aspect of the case. Parks motion for reconsideration of the remand of the case having been denied, he elevated the case to the CA which dismissed his petition. ISSUES 1. WON the CA erred in dismissing the petition for not fully complying with verification requirements 2. WON the CA erred in dismissing the petition on the ground that it was not accompanied by copies of certain pleadings and other material portions of the record as would support the allegations of the petition 3. WON the CA erred in dismissing the petition for failure to implead the People of the Philippines as a party 4. WON the respondent has a right to present evidence on the civil aspect of the case in view of his demurrer HELD 1. NO Ratio Verification is not an empty ritual or a meaningless formality. Its import must never be sacrificed in the name of mere expedience or sheer caprice. For what is at stake is the matter of verity attested by the sanctity of an oath to secure an assurance that the allegations in the pleading have been made in good faith, or are true and correct and not merely speculative. Reasoning - Section 4 of Rule 7 of the RoC: Verification Except when otherwise specifically required by law or rule, pleadings need not be under oath, verified or accompanied by affidavit. A pleading is verified by an affidavit that the affiant has read the pleading and that the allegations therein are true and correct of his personal knowledge or based on authentic records. - A pleading required to be verified which contains a verification based on information and belief, or upon knowledge, information and belief, or lacks a proper verification shall be treated as an unsigned pleading.

a2010

page 143

Prof.
accordance with the prescribed procedure to insure an orderly and speedy administration of justice. Reasoning - The materiality of those documents is very apparent since the civil aspect of the case, from which Park is appealing, was likewise dismissed by the trial court on account of the same Demurrer. The Rules require that the petition must be accompanied by clearly legible duplicate original or true copies of the judgments or final orders of both lower courts, certified correct by the clerk of court [Sec 2(d) Rule 42]. - The only duplicate original or certified true copies attached as annexes to the petition are the RTC Order granting respondents MFR and the RTC Order denying petitioners MFR. The copy of the September 11, 2003 RTC Decision, which petitioner prayed to be reinstated, is not a certified true copy and is not even legible. Petitioner later recompensed though by appending to his MFR a duplicate original copy. - While petitioner averred before the CA in his MFR that the February 27, 2003 MeTC Order was already attached to his petition as Annex G, Annex G bares a replicate copy of a different order. It was to this Court that petitioner belatedly submitted an uncertified true copy of the said MeTC Order as an annex to his Reply to respondents Comment. The copy of the other MeTC Order, dated May 5, 2003, which petitioner attached to his petition before the CA is similarly uncertified as true. Since both Orders were adverse to him even with respect to the civil aspect of the case, petitioner was mandated to submit them in the required form. 3. YES Reasoning - The MeTC acquitted respondent. As a rule, a judgment of acquittal is immediately final and executory and the prosecution cannot appeal the acquittal because of the constitutional prohibition against double jeopardy. Either the offended party or the accused may, however appeal the civil aspect of the judgment despite the acquittal of the accused. The public prosecutor has generally no interest in appealing the civil aspect of a decision acquitting the accused. The acquittal ends his work. The case is terminated as far as he is concerned. The real parties in interest in the civil aspect of a decision are the offended party and the accused. 4. YES Reasoning - In case of a demurrer to evidence filed with leave of court, the accused may adduce countervailing evidence if the court denies the demurrer. Such

- Park argues that the word or is a disjunctive term signifying disassociation and independence, hence, he chose to affirm in his petition he filed before the court a quo that its contents are true and correct of my own personal knowledge, and not on the basis of authentic documents. On the other hand, Choi counters that the word or may be interpreted in a conjunctive sense and construed to mean as and, or vice versa, when the context of the law so warrants. - A pleading may be verified under either of the two given modes or under both. The veracity of the allegations in a pleading may be affirmed based on either ones own personal knowledge or on authentic records, or both, as warranted. The use of the preposition or connotes that either source qualifies as a sufficient basis for verification and, needless to state, the concurrence of both sources is more than sufficient. Bearing both a disjunctive and conjunctive sense, this parallel legal signification avoids a construction that will exclude the combination of the alternatives or bar the efficacy of any one of the alternatives standing alone. - However, the range of permutations is not left to the pleaders liking, but is dependent on the surrounding nature of the allegations which may warrant that a verification be based either purely on personal knowledge, or entirely on authentic records, or on both sources. Authentic records as a basis for verification bear significance in petitions where the greater portions of the allegations are based on the records of the proceedings in the court of origin, and not solely on the personal knowledge of the petitioner. - To sustain petitioners explanation that the basis of verification is a matter of simple preference would trivialize the rationale and diminish the resoluteness of the rule. It would play on predilection and pay no heed in providing enough assurance of the correctness of the allegations. 2. NO Ratio Procedural rules are tools designed to facilitate the adjudication of cases. Courts and litigants alike are thus enjoined to abide strictly by the rules. And while the Court, in some instances, allows a relaxation in the application of the rules, this, we stress, was never intended to forge a bastion for erring litigants to violate the rules with impunity. The liberality in the interpretation and application of the rules applies only in proper cases and under justifiable causes and circumstances. While it is true that litigation is not a game of technicalities, it is equally true that every case must be prosecuted in

Criminal Procedure Rowena Daroy Morales


denial bears no distinction as to the two aspects of the case because there is a disparity of evidentiary value between the quanta of evidence in such aspects of the case. In other words, a court may not deny the demurrer as to the criminal aspect and at the same time grant the demurrer as to the civil aspect, for if the evidence so far presented is not insufficient to prove the crime beyond reasonable doubt, then the same evidence is likewise not insufficient to establish civil liability by mere preponderance of evidence. - On the other hand, if the evidence so far presented is insufficient as proof beyond reasonable doubt, it does not follow that the same evidence is insufficient to establish a preponderance of evidence. For if the court grants the demurrer, proceedings on the civil aspect of the case generally proceed. The only recognized instance when an acquittal on demurrer carries with it the dismissal of the civil aspect is when there is a finding that the act or omission from which the civil liability may arise did not exist. Absent such determination, trial as to the civil aspect of the case must perforce continue. - In the instant case, the MeTC granted the demurrer and dismissed the case without any finding that the act or omission from which the civil liability may arise did not exist. Choi did not assail the RTC order of remand. He thereby recognized that there is basis for a remand. - Park posits that Choi waived his right to present evidence on the civil aspect of the case (1) when the grant of the demurrer was reversed on appeal, citing Section 1 of Rule 33, and (2) when respondent orally opposed petitioners motion for reconsideration pleading that proceedings with respect to the civil aspect of the case continue. - Petitioners citation of Section 1 of Rule 33 is incorrect. Where a court has jurisdiction over the subject matter and over the person of the accused, and the crime was committed within its territorial jurisdiction, the court necessarily exercises jurisdiction over all issues that the law requires it to resolve. One of the issues in a criminal case being the civil liability of the accused arising from the crime, the governing law is the Rules of Criminal Procedure, not the Rules of Civil Procedure which pertains to a civil action arising from the initiatory pleading that gives rise to the suit. - As for petitioners attribution of waiver to respondent, it cannot be determined with certainty from the records the nature of Chois alleged oral objections to Parks motion for reconsideration of the grant of the demurrer to evidence. Any waiver of the

a2010

page 144

Prof.
reception of further evidence after the parties have closed their evidence, such action is addressed to the sound discretion of the court, to be exercised only on valid and justifiable reasons (which are absent in this case). REASONING: The failure of the accused to complete his testimony was of his own making, on the initiation, confirmation and reiteration of his own counsel. Verily, the present stance of the accused is a blatant disregard of solemn agreements submitted to and approved by a court of justice and would make a mockery of the judicial process. Coming now to the conclusion of the trial court that the accused raped and, on the occasion thereof, killed Teresita Gumapay, the Court has painstakingly scrutinized the record, with the concomitant calibration of the evidence and the consequent determination as to whether the quantum thereof passes the test of moral certainty of guilt. There is no doubt that it was the accused who killed Teresita Gumapay, the evidence thereon being capped by his own written confession of the same before the investigating officers. The authenticity of and the fact that he and the witnesses thereto knowingly affixed their signatures on said extrajudicial confession were never questioned. Buttressing the foregoing evidence is the positive identification of the accused at the situs and during the occurrence of the crime. We agree with the well-reasoned opinion of the trial court that the accused is not entitled to the exempting circumstance of insanity. Against the effete efforts in the accused's afterthought to create an insanity defense is the whole weight of the presumption of sanity provided by law, amply supported by convincing circumstances laudably pointed out by the trial court. DISPOSITION WHEREFORE, with the modifications that the death sentence imposed by the trial court is reduced to reclusion perpetua pursuant to Section 19(l), Article III of the Constitution, and the indemnification for the death of Teresita Gumapay is hereby increased from P12,000.00 to P30,000.00, consonant with present jurisprudence.

right to present evidence must be positively demonstrated. Any ambiguity in the voluntariness of the waiver is frowned upon; hence, courts must indulge every reasonable presumption against it. Dispositive Petition is DENIED.

PEOPLE v CRUZ 177 SCRA 451 REGALADO; SEPTEMBER 13, 1989


FACTS The information for rape with homicide (based on witness testimonies, postmortem report, police interrogation, and admission by the accused) was filed against Danilo Gole Cruz with the then CFI. Accused pleaded not guilty in the arraignment and trial on the merits followed wherein several witnesses were presented. The suspension of the direct examination of the accused was at his instance and as moved by his counsel. Upon medical examination, accused was found to be suffering from schizophrenia, and was admitted to a mental hospital. He however tried to escape. Later, Dr. Maaba recommended on March 22, 1982 the discharge of the accused from the mental hospital and for his return to the provincial jail of Bulacan, he having been found fit to stand trial. This unrebutted fact notwithstanding, the accused refused to take the witness stand without any plausible justification. In addition, it was the defense itself which moved to terminate the testimony of the accused, which fact became the basis for such testimony being stricken from the records for lack of cross-examination. In fact, when the former presiding judge thereafter ordered the reopening of the case sua sponte, it was the defense that objected to the same and insisted that the case be deemed submitted for decision. Defense then motioned for a reopening of the case which was denied for lack of merit. Trial Court found the accused guilty beyond reasonable doubt of the crime of rape with homicide and found his defense of insanity as an exempting circumstance unavailing. ISSUE WON accused was denied of his constitutional right to be heard and to defend himself. (NO.) HELD RATIO: The mere filing of a motion to reopen a case must not in any way automatically vacate an agreement and order submitting the case for decision. While the court may reopen a case for

PEOPLE v CONCEPCION 84 PHIL 787 PARAS; October 25, 1949


NATURE

Criminal Procedure Rowena Daroy Morales


Appeal from a judgment of the Peoples Court finding the appellant, Concepcion, guilty of treason, and sentencing him to life imprisonment and to pay a fine of P10,000. FACTS (the case is really short, there arent many facts or anything) - Appellant was found guilt of treason on 3 counts: - Apprehension on December 7, 1944 of Basilio Severino. - Apprehension on December 3, 1944, of Clemente Chica - Apprehension on January 9 of Gavino Moras - the three individuals were apprehended by the appellant or at his instigation, due to their guerrilla connections, all in Cebu City, and the appellant was accompanied by Japanese during all 3 apprehensions ISSUE WON the lower court committed an error by allowing the prosecution to present evidence of appellants Filipino citizenship after the prosecution had rested its case and the defense had moved for dismissal HELD NO Reasoning - The matter of reopening a case for the reception of further evidence after either the prosecution or the defense has rested is within the discretion of the trial court. Dispositive The appealed judgment, being in conformity with the facts and the law, is affirmed.

a2010

page 145

Prof.
According to Judge Maceda, since the prosecution was not able to present its evidence on the first four hearing dates and there was either no return on the subpoenas subsequently issued or there was no subpoena issued at all to Pedrosa and Dr. Salen, the prosecution should have been given a last chance to present the alleged eyewitness and the doctor. - Judge Maceda denied Cabarless MFR and set the case for hearing on May 8, 2003 to hear the testimonies of Pedrosa and Dr. Salen. The subpoena issued to Pedrosa for that hearing was duly served, but service upon Dr. Salen failed since the doctor was no longer assigned to the SPD Crime Laboratory. Notwithstanding the service upon Pedrosa, the prosecution still failed to present a witness during the May 8, 2003 hearing. Nonetheless, Judge Maceda, upon motion, again decided to extend to the prosecution another chance, giving the People June 19 and July 3, 2003 as additional hearing dates. Finally, on June 19, 2003, Pedrosa took the witness stand and completed her direct examination. A few days thereafter, Cabarles filed the present petition questioning Judge Macedas order, alleging that it was issued with grave abuse of discretion. Since trial in the lower court continued, on July 3, 2003, the Public Attorneys Office conducted its crossexamination of Pedrosa. Cabarles was then given a chance to adduce further evidence on his behalf.1avvphi1.net On August 9, 2004, Judge Maceda deferred the promulgation of judgment and ordered the case archived pending this Courts resolution of the case. ISSUES 1. WON respondent judge acted with grave abuse of discretion in issuing the assailed order 2. WON petitioners right to due process and speedy disposition of his case was violated HELD 1. YES Ratio Section 24, Rule 119 and existing jurisprudence stress the following requirements for reopening a case: (1) the reopening must be before the finality of a judgment of conviction; (2) the order is issued by the judge on his own initiative or upon motion; (3) the order is issued only after a hearing is conducted; (4) the order intends to prevent a miscarriage of justice; and (5) the presentation of additional and/or further evidence should be terminated within thirty days from the issuance of the order. Reasoning

CABARLES v MACEDA QUISUMBING; February 20, 2007


NATURE Petitioner seeks to annul the Order issued by respondent Judge Bonifacio Sanz Maceda in Criminal Case No. 99-0878, entitled People of the Philippines v. Rene "Nonoy" Cabarles y Adizas, for murder, filed with the Regional Trial Court of Las Pias City, Branch 275. FACTS - On June 18, 1999, Cabarles was charged with murder under the following information: The undersigned Prosecutor II accuses RENE "NONOY" CABARLES Y ADIZAS of the crime of Murder, committed as follows:

That on or about the 25th day of April, 1999, in the City of Las Pias, Philippines and within the jurisdiction of this Honorable Court, the above-named accused, without justifiable motive with intent to kill and by means of treachery and evident premeditation, did then and there willfully, unlawfully and feloniously attack, assault, and stab with a deadly weapon (fan knife) one Antonio Callosa, which directly caused his death. - Cabarles pleaded not guilty. The trial court scheduled the case for hearing on the following dates, to wit: pre-trial on November 22, 2000; presentation of prosecutions evidence on April 18, May 4, 11, 18, and 23, 2001; and presentation of defense evidence on June 20 and 27, July 4 and 18, and August 1, 2001. -The prosecution had subpoenas issued to its witnesses: Flocerfina Callosa, the mother of the deceased; Imelda Pedrosa, the alleged eyewitness; Carlos Callosa, brother of the deceased; and Dr. Romeo T. Salen, Police Senior Inspector of the Southern Police District (SPD) Crime Laboratory to testify on the contents of the death certificate of Antonio Callosa. - Through no fault of its own, the prosecution was unable to present its evidence on the first four hearing dates. - A day before the scheduled promulgation of judgment on April 2, 2003, Judge Maceda motu proprio issued the questioned order reopening the case. In it, he observed that the prosecution may not have been given its day in court resulting in a miscarriage of justice. He explained that because there was a mix-up in the dates specified in the subpoena and the hearing dates of when the case was actually heard, the prosecution was unable to present its evidence on the first four of the five hearing dates: April 18, May 4, 11 and 18, 2001 assigned to it. Judge Maceda found that there was no hearing conducted on April 18, 2001. Thereafter, the subpoena issued to Pedrosa required her to appear on April 11, 2001, which was not a date assigned for the prosecution but May 11, 2001. Also, Judge Maceda noted that another subpoena was issued to Pedrosa and Dr. Salen requiring them to appear on May 11 and June 20, 2001. But, the May 11, 2001 hearing was reset to May 25, 2001 because the judge was indisposed, and insofar as the June 20, 2001 setting was concerned, it was not one of the days set by the court for the prosecution. Judge Maceda further observed that the May 18, 2001 hearing was never scheduled and May 25, 2001 was likewise not a hearing date set by the court.

Criminal Procedure Rowena Daroy Morales


- Generally, after the parties have produced their respective direct proofs, they are allowed to offer rebutting evidence only. However, the court, for good reasons, in the furtherance of justice, may allow new evidence upon their original case, and its ruling will not be disturbed in the appellate court where no abuse of discretion appears. A motion to reopen may thus properly be presented only after either or both parties had formally offered and closed their evidence, but before judgment is rendered, and even after promulgation but before finality of judgment and the only controlling guideline governing a motion to reopen is the paramount interest of justice. This remedy of reopening a case was meant to prevent a miscarriage of justice. - However, while Judge Maceda is allowed to reopen the case before judgment is rendered, Section 24 requires that a hearing must first be conducted. Judge Maceda issued the April 1, 2003 Order without notice and hearing and without giving the prosecution and accused an opportunity to manifest their position on the matter. This failure, to our mind, constitutes grave abuse of discretion and goes against the due process clause of the Constitution which requires notice and opportunity to be heard. The issuance of the said order, without the benefit of a hearing, is contrary to the express language of Section 24, Rule 119. 2. YES Ratio The concept of speedy disposition is relative or flexible. A mere mathematical reckoning of the time involved is not sufficient. Particular regard must be taken of the facts and circumstances peculiar to each case. Reasoning - The right to a speedy disposition of a case, like the right to speedy trial, is deemed violated when the proceeding is attended by vexatious, capricious, and oppressive delays; or when unjustified postponements of the trial are asked for and secured; or when without cause or justifiable motive, a long period of time is allowed to elapse without the party having his case tried. - Cabarles invokes the jurisdiction of this Court in the interest of speedy justice since the information against him was filed way back in June 1999, and almost eight years thereafter, no judgment has yet been rendered. Any further delay in the resolution of the instant petition will be prejudicial to Cabarles. Also, the Court has full discretionary power to take cognizance of the petition filed directly to it for compelling reasons or if warranted by the nature of the issues raised. Since Section 24 is a new

a2010

page 146

Prof.
(6) months' imprisonment and to pay complainant the amount of P200.00, plus costs the

provision, and considering the irregularities in the issuance of the April 1, 2003 Order, it is necessary to resolve the issues raised in this petition. - In fine, we are not unmindful of the gravity of the crime charged; but justice must be dispensed with an even hand. Regardless of how much we want to punish the perpetrators of this ghastly crime and give justice to the victim and her family, the protection provided by the Bill of Rights is bestowed upon all individuals, without exception, regardless of race, color, creed, gender or political persuasion whether privileged or less privileged - to be invoked without fear or favor. Hence, the accused deserves no less than an acquittal; ergo, he is not called upon to disprove what the prosecution has not proved. Dispositive instant petition is GRANTED. We hold that the assailed Order dated April 1, 2003 was issued with grave abuse of discretion. Said Order is hereby ANNULLED and SET ASIDE. Accordingly, any evidence received and offered in this case as a result of the April 1, 2003 Order is hereby stricken off the record. Let the records of this case be REMANDED immediately to the trial court concerned for its appropriate action without further delay.

ISSUES 1. WON the application of the Rules on Summary procedure was valid 2. WON he was afforded due process HELD NO - The Rule on Summary Procedure in Special Cases applies only to criminal cases where the penalty prescribed by law for the offense charged does not exceed six (6) months imprisonment or a fine of one thousand pesos (P1,000.00) or both - The crime of Theft as charged herein is penalized with arresto mayor in its medium period to prision correccional in its minimum period, or, from two (2) months and one (1) day to two (2) years and four (4) months. 6 Clearly, the Rule on Summary Procedure is inapplicable - But even assuming that the case falls under the coverage of said Rule, the same does not dispense with trial "Section 11. When case set for arraignment and trial.- Should the court, upon a consideration of the complaint or information and the affidavits submitted by both parties, find no cause or ground to hold the defendant for trial, it shall order the dismissal of the case; otherwise, the court shall set the case for arraignment and trial. "Section 14. Procedure of Trial.- Upon a plea of not guilty being entered, the trial shall immediately proceed. The affidavits submitted by the parties shall constitute the direct testimonies of the witnesses who executed the same. Witnesses who testified may be subjected to cross-examination. Should the affiant fail to testify, his affidavit shall not be considered as competent evidence for the party presenting the affidavit, but the adverse party may utilize the same for any admissible purpose. "No witness shall be allowed to testify unless he had previously submitted an affidavit to the court in accordance with Sections 9 and 10 hereof." - since petitioner-accused had pleaded not guilty, trial should have proceeded immediately. But not only was petitioner unrepresented by counsel upon arraignment; he was neither accorded the benefit of trial

COMBATE v SAN JOSE 135 SCRA 693 MELENCIO-HERRERA; April 15, 1985
NATURE Petition for Certiorari FACTS - petitioner was charged with the crime of Theft of one (1) Rooster [Fighting Cock] color red, belonging to Romeo Posada worth P200.00. - Following the procedure laid down in the Rule on Summary Procedure in Special Cases, respondent Judge required petitioner and his witnesses to submit counter-affidavits to the supporting affidavits of the complainant - On June 5, 1984, petitioner was subpoenaed to appear before respondent Judge and was arraigned without the assistance of counsel. He pleaded not guilty. - Subsequently, in an Order dated July 5, 1984, respondent Judge deemed the case submitted for resolution purportedly pursuant to the Rule on Summary Procedure. - In a Decision promulgated on July 16, 1984, without benefit of trial, petitioner was sentenced to suffer six

Criminal Procedure Rowena Daroy Morales


Dispositive Judge's Decision promulgated on July 16, 1994, is hereby ANNULLED for having been issued with grave abuse of discretion. The case is remanded to the Municipal Circuit Trial Court of Magarao-Canaman, Camarines Sur, for proceedings strictly in accordance with law.

a2010

page 147

Prof.
(e) Said accused has not at any time been convicted of any offense involving moral turpitude. Evidence adduced in support of the discharge shall automatically form part of the trial. If the court denies the motion for discharge of the accused as state witness, his sworn statement shall be inadmissible in evidence. - The power to prosecute includes the initial discretion to determine who should be utilized by the government as a state witness. The prosecution has gathered the evidence against the accused and is in a better position to decide the testimonial evidence needed by the State to press its prosecution to a successful conclusion. Under our Rules, however, it is the courts that will finally determine whether the requirements have been satisfied to justify the discharge of an accused to become a witness for the government. - The testimony of dela Cruz was an absolute necessity. - Neither does dela Cruz appear to be the most guilty of the accused. The trial court held that dela Cruz was not privy to the kidnap plan and was merely taken in later by the group because they suspected that she already knew too much. - Did the lower courts properly consider the testimony of dela Cruz? It is a jurisprudential rule that the testimony of a self-confessed accomplice or co-conspirator imputing the blame to or implicating his co-accused cannot, by itself and without corroboration, be regarded as proof with a moral certainty that the latter committed or participated in the commission of the crime. The testimony must be substantially corroborated in its material points by unimpeachable testimony and strong circumstances and must be to such an extent that its trustworthiness becomes manifest. The testimony of dela Cruz was substantially corroborated by no less than the victim himself, Oliver, as well as Pedro. - As noted by the trial court, there may have been inconsistencies in the narration of dela Cruz. These, however, were minor details and simply could be attributed to the frailty of human memory. It cannot be expected that her testimony would be entirely flawless. Inconsistencies as to minor details and collateral matters do not affect the credibility of the witnesses nor the veracity or weight of their testimonies. Such minor inconsistencies may even serve to strengthen their credibility as they negate any suspicion that the testimonies have been rehearsed. Moreover, the testimony of dela Cruz coincides with that of Oliver and Pedro relating to the

PEOPLE v RODRIGO TINGA; January 23, 2007


NATURE Automatic review FACTS - On 10 September 1996, Oliver Caparas, then 13 years of age, was waiting for a ride to school in a corner near his house in Matimbo, Malolos, Bulacan, when four (4) men forcibly seized and boarded him into a car. While inside the car, he was blindfolded. He was later transferred to a van. The van, tailed by a car, traveled to Baguio. While there, they slept overnight inside the van in a parking lot. - The following day, Eleazar Caparas, the father of Oliver, received a call from the kidnappers initially asking for P10 million ransom - The kidnappers proceeded to Bonitas Resort in Pangasinan. Oliver was then brought to a room and his blindfold removed. He stayed inside the room for one week. During his stay, a woman, later identified as Lanie dela Cruz, took care of him by feeding him three times a day. - After three days of negotiation, the kidnappers agreed to lower the ransom to P1.7 million. On 17 September 1996, Pedro Navarro, an uncle of Oliver, was instructed by Eleazar Caparas to deliver the ransom money. After receiving a call from the kidnappers, he proceeded to follow the instructions on the drop-off. - Later that night, Oliver was made to board the same van and brought to the Petron Gas Station in Meycauayan Highway. Upon alighting from the van, he was given P500.00 and was told that he would be fetched by his uncle inside a canteen in the gas station. At around 1:00 a.m. of 18 September 1997, the kidnappers called Eleazar again and asked them to go to the Petron Gas Station located between Meycauayan and Marilao along the Expressway. Upon arriving at the Petron Station at 3:00 a.m, Pedro Navarro saw Oliver eating inside the canteen and brought him home where he was reunited with his father. - After the kidnapping incident, an investigation was conducted by the Intelligence Section of the

Philippine National Police. It appears that one of the suspects was a member of an NPA rebel returnee group headed by Armando Rodrigo, Jr. Upon the killing of Bert Liwanag, his girlfriend, dela Cruz, who was a suspected member of the group, was invited for questioning. On that occasion, she admitted her participation in the kidnapping of Oliver Caparas and implicated appellants. - An Information was filed on 11 March 1997 against appellants Plata, Fajardo and Rodrigo, together with dela Cruz, Armando Rodrigo, Helen Joven, Boyong Catindig, Jun Parubrob, and a John Doe. - Four of the accused were apprehended, namely: Plata, Rodrigo, Fajardo and dela Cruz. The rest remained at large. The trial court, upon motion of the prosecution, discharged Dela Cruz to serve as state witness. - On arraignment, appellants pleaded not guilty. - On 31 May 2000, the RTC rendered its decision finding all appellants guilty beyond reasonable doubt. - Appellants elevated the case to the Court of Appeals. - The appellate court affirmed the trial courts decision except that it acquitted Rodrigo. - Appellants Plata and Fajardo submitted their individual appeal briefs. ISSUE WON Dela Cruz was eligible to be a state witness HELD - Section 17, Rule 119 of the Rules of Court provides: When two or more persons are jointly charged with the commission of any offense, upon motion of the prosecution before resting its case, the court may direct one or more of the accused to be discharged with their consent so that they may be witnesses for the state when, after requiring the prosecution to present evidence and the sworn statement of each proposed state witness at a hearing in support of the discharge, the court is satisfied that: (a) There is absolute necessity for the testimony of the accused whose discharge is requested; (b) There is no other direct evidence available for the proper prosecution of the offense committed, except the testimony of said accused; (c) The testimony of said accused can be substantially corroborated in its material points; (d) Said accused does not appear to be the most guilty; and

Criminal Procedure Rowena Daroy Morales


principal occurrence and the positive identification of appellants. - Plata insists that dela Cruz harbored a grudge against him because he was apparently a member of the Armando Rodrigo group, the lone suspect in the murder of Bert Liwanag, dela Cruzs boyfriend. Platas effort to impute ill-motive on the part of de la Cruz to falsely testify against him does not hold water. Even granting that De la Cruz may have an axe to grind is of no moment. Plata was positively identified by Oliver. His statement was corroborated by dela Cruz. Motive becomes essential only when the identity of the culprit is in doubt and not when he is positively identified by a credible witness. Dispositive Affirmed with modification

a2010

page 148

Prof.
right to prosecute vests the prosecutor with a wide range of discretion, the discretion of whether, what and whom to charge, the exercise of which depends on a smorgasbord of factors which are best appreciated by prosecutors. We thus hold that it is not constitutionally impermissible for Congress to enact R.A. No. 6981 vesting in the Department of Justice the power to determine who can qualify as a witness in the program and who shall be granted immunity from prosecution. Section 9 of Rule 119 does not support the proposition that the power to choose who shall be a state witness is an inherent judicial prerogative. Under this provision, the court, is given the power to discharge a state witness only because it has already acquired jurisdiction over the crime and the accused. The discharge of an accused is part of the exercise of jurisdiction but is not a recognition of an inherent judicial function. Moreover, the Rules of Court have never been interpreted to be beyond change by legislation designed to improve the administration of our justice system. R.A. No. 6981 is one of the much sought penal reform laws to help government in its uphill fight against crime, one certain cause of which is the reticence of witnesses to testify. - The rationale for the law is well put by the Department of Justice, viz.: "Witnesses, for fear of reprisal and economic dislocation, usually refuse to appear and testify in the investigation/prosecution of criminal complaints/cases. Because of such refusal, criminal complaints/cases have been dismissed for insufficiency and/or lack of evidence. For a more effective administration of criminal justice, there was a necessity to pass a law protecting witnesses and granting them certain rights and benefits to ensure their appearance in investigative bodies/courts." Petitioner Webb's challenge to the validity of R.A. No. 6981 cannot therefore succeed. Dispositive Petitions dismissed

WEBB v DE LEON 247 SCRA 652 PUNO; August 23, 1995


NATURE Petitions for the issuance of the extraordinary writs of certiorari, prohibition and mandamus FACTS - Petitioners Hubert Webb, Michael Gatchalian, Antonio J. Lejano and six (6) other persons were charged with the crime of Rape with Homicide (Vizconde massacre). - Petitioners fault the DOJ Panel for not including Alfaro in the Information considering her alleged conspiratorial participation in the crime of rape with homicide. It is urged that this constitutes ". . . an intrusion into judicial prerogative for it is only the court which has the power under the Rules on Criminal Procedure to discharge an accused as a state witness" based on Section 9, Rule 119 which gives the court the prerogative to approve the discharge of an accused to be a state witness. ISSUE WON Alfaro should be included as one of the accused in the information HELD - The non-inclusion of Alfaro is anchored on Republic Act No. 6981, entitled "An Act Providing For A Witness Protection, Security And Benefit Program And For Other Purposes" enacted on April 24, 1991. Alfaro qualified under its Section 10, which provides: Sec. 10. State Witness. Any person who has participated in the commission of a crime and

desires to a witness for the State, can apply and, if qualified as determined in this Act and by the Department, shall be admitted into the Program whenever the following circumstances are present: (a) the offense in which his testimony will be used is a grave felony as defined under the R.P.C. or its equivalent under special laws; (b) there is absolute necessity for his testimony; (c) there is no other direct evidence available for the proper prosecution of the offense committed; (d) his testimony can be substantially corroborated on its material points; (e) he does not appear to be most guilty; and (f) he has not at anytime been convicted of any crime involving moral turpitude. An accused discharged from an information or criminal complaint by the court in order that he may be a State Witness pursuant to Sections 9 and 10 of Rule 119 of the Revised Rules of Court may upon his petition be admitted to the Program if he complies with the other requirements of this Act. Nothing in this Act shall prevent the discharge of an accused so that he can be used as a Witness under Rule 119 of the Revised Rules of Court. - Upon qualification of Alfaro to the program, Section 12 of the said law mandates her non-inclusion in the criminal Complaint or Information, thus: Sec. 12. Effect of Admission of a State Witness into the Program. The certification of admission into the Program by the Department shall be given full faith and credit by the provincial or city prosecutor who is required NOT TO INCLUDE THE WITNESS IN THE CRIMINAL COMPLAINT OR INFORMATION and if included therein, to petition the court for his discharge in order that he can be utilized as a State Witness. The court shall order the discharge and exclusion of the said accused from the information. - Admission into the Program shall entitle such State Witness to immunity from criminal prosecution for the offense or offenses in which his testimony will be given or used and all the rights and benefits provided under Section 8 hereof - Petitioner's argument lacks appeal for it lies on the faulty assumption that the decision whom to prosecute is a judicial function, the sole prerogative of courts and beyond executive and legislative interference. In truth, the prosecution of crimes appertains to the executive department of government whose principal power and responsibility is to see that our laws are faithfully executed. A necessary component of this power to execute our laws is the right to prosecute their violators. The

PEOPLE v BUBAN [SUPRA, PAGE 6] PEOPLE v SAPAL 328 SCRA 417 KAPUNAN; December 22, 1997
NATURE Automatic Review of the Decision FACTS - The RTC of Manila sentenced accused-appellant

Criminal Procedure Rowena Daroy Morales


Jimmy Sapal to DEATH after he was found guilty beyond reasonable doubt of the crime of unlawful possession of three (3) kgs. of marijuana. - The prosecution presented two witnesses, namely P03 Jesus Gomez and Renee Checa, a forensic chemist. Gomez testified that the office of the Drug Enforcement Unit received a call that accused who had a standing warrant of arrest had been seen at Jocson St., Sampaloc Manila. - Their group spotted the car frequently used by the accused. The police operatives approached the car, told the accused and his wife, along with two other companions, to get down of the car. Forthwith, Gomez conducted a search of the vehicle and in the course thereof, allegedly found a light green plastic bag in the back seat containing three bricks of marijuana. - The accused testified that the police operatives took his wallet which contained cash and several ATM cards and that the latter coerced him into divulging the PIN numbers of the ATM cards. - Accused gave them the correct PIN number to his Far East Bank account but purposely mixed up the other PIN numbers to his other bank accounts. - The accused and his companions were brought to the WPD headquarters where their male companion, Jerry, was mauled to force him to admit that drugs were recovered from their group. The accuseds wife and her other companion were likewise coerced to admit the same. However, they all insisted that no illegal drugs were recovered from any of them. - After trial, the trial court rendered the decision under review. ISSUE WON the guilt of the accused was proven beyond reasonable doubt to warrant the supreme penalty of death HELD The Court finds for the accused. - While the Court is mindful that law enforcers enjoy the presumption of regularity in the performance of their duties, this presumption cannot prevail over the constitutional right of the accused to be presumed innocent and it cannot, by itself, constitute proof of guilt beyond reasonable doubt. - In the present case, there is sufficient evidence to show that the manner by which the law enforcers effected the arrest of the accused was highly irregular and suspect. - Gomez claimed that they arrested accused pursuant to the warrant issued by Judge Barrios

a2010

page 149

Prof.
NATURE Petition for certiorari and prohibition FACTS - Petitioner Tarcisio Icao was a provincial guard employed by the Province of Zamboanga del Norte. His chief function was to guard prisoners confined in the provincial jail located in Dipolog City. He was charged with the felony of infidelity in the custody of prisoners in the CFI Dipolog City, and after due arraignment and trial, was convicted. - On the same day that the judgment was promulgated, private respondent Icao filed a petition for probation pursuant to the provisions of the Probation Law of 1976, and was released from custody on his own recognizance. He never thereafter sought to take an appeal or have the verdict reversed or modified. According to Sol-Gen., Icao's application for probation was approved. Nothing in the record clearly supports this assertion. Whether or not probation was granted is not, however, material. The case will be resolved on other considerations. - A month later, the respondent Judge's attention was drawn to a letter of the Probation Officer of Dipolog City, replying to an inquiry of the Office of the Provincial Governor, stating that pending final action on his petition for probation, Icao could continue performing his duties as provincial guard in accordance with the spirit and intent of the Probation Law. The respondent Judge issued an order, announcing his amendment of the judgment of conviction by specifying the period of temporary special disqualification of Icao, and requiring the latter's presence for the promulgation of the amended decision. - Icao moved for reconsideration, arguing that the Court had already lost jurisdiction over the case, the judgment having become final, and the alteration by the respondent Judge of the decision under the circumstances would place him in double jeopardy. His motion was denied, as was, too, a second MFR. Hence, this petition for certiorari and prohibition now before this Court. ISSUE WON respondent Judge had the authority to modify the judgment of conviction HELD NO Ratio A judgment of conviction may be modified or set aside by the court rendering it before the

which explicitly stated that said accused Jimmy Sapal be brought before him as soon as possible. However, contrary to the clear directive of the warrant, the law enforcers never brought him before the said judge. Gomez himself admitted the same and did not offer any convincing explanation for this omission. - Moreover, the records reveal that the documents relating to the arrest of the accused and his wife were prepared three (3) days after the arrest. - Further, the case was submitted to the inquest prosecutor only on April 25, 1995 and the information against accused and his wife was subsequently filed on April 26, 1995. - It was not likewise shown that accused was fully apprised of his rights under custodial arrest. Specifically, accused was not assisted by counsel when he was under custodial investigation. - Admittedly, accused is deemed to have waived his right to question the irregularities attending his arrest for his failure to raise the same before he entered his plea. Nonetheless, the peculiar factual circumstances surrounding the case effectively destroy the presumption of regularity in the performance by Gomez and his colleagues of their duties. Such being the case, the presumption of regularity cannot be made the sole basis of the conviction of the accused. - It has been sufficiently established that several withdrawals were made from the Far East Bank account of accused through ATM on April 22, 1995 and these withdrawals could not have been made by the accused and his wife because they were then already under arrest. - The Court cannot completely disregard this piece of evidence as it strongly corroborates the testimony of accused that law enforcers were able to withdraw money from is Far East Bank account through ATM. - It is well-settled that where the circumstances shown to exist yield two (2) or more inferences, one of which consistent with the presumption of innocence while the other or others may be compatible with the finding of guilt, the court must acquit the accused: for the evidence does not fulfill the test or moral certainty and is insufficient to support a judgment of conviction. Dispositive Reversed.

ICAO v APALISOK 180 SCRA 680 NARVASA; December 29, 1989

Criminal Procedure Rowena Daroy Morales


judgment has become final or appeal has been perfected. A judgment in a criminal case becomes final after the lapse of the period for perfecting an appeal, or when the sentence has been partially or totally satisfied or served, or the defendant has expressly waived in writing his right to appeal. ( Sec. 7, Rule 120 of the Rules of Court of 1964) Reasoning - Under said rule, the respondent Judge had clearly lost the authority to modify the judgment of conviction. [a] The judgment in this case became final and executory because the 15-day period of appeal provided by law had lapsed without an appeal being taken. A judgment which has become final and executory can no longer be amended or corrected except only as regards clerical errors. Hence, even the subsequent discovery of an erroneous imposition of a penalty will not justify correction of the judgment after it has become final. [b] Under Art. 44 of RPC, the penalty of arresto imposed on Icao carries with it that of suspension of the right to hold office and the right of suffrage during the term of the sentence. The plain implication would appear to be that courts have no power to fix a longer term for that accessory penalty of disqualification Dispositive Petition GRANTED. Challenged Orders annulled and set aside.

a2010

page 150

Prof.
constitutes a grave abuse of discretion which calls for relief from this Court. - We do not question the correctness of the findings of the Court of Appeals that the evidence sought to be presented by the petitioner do not fall under the category of newly-discovered evidence because the same his alleged appointment as an agent of the Philippine Constabulary and a permit to possess a handgrenade were supposed to be known to petitioner and existing at the time of trial and not discovered only thereafter. - It is indeed an established rule that for a new trial to be granted on the ground of newly discovered evidence, it must be shown that (a) the evidence was discovered after trial; (b) such evidence could not have been discovered and produced at the trial even with the exercise of reasonable diligence; (c) the evidence is material, not merely cumulative, corroborative, or impeaching; and (d) it must go to the merits as ought to produce a different result if admitted. - However, petitioner herein does not justify his motion for a new trial on newly discovered evidence, but rather on broader grounds of substantial justice under Sec. 11, Rule 124 of the Rules of Court which provides: "Power of appellate court on appeal. Upon appeal from a judgment of the Court of First Instance, the appellate court may reverse, affirm, or modify the judgment and increase or reduce the penalty imposed by the trial court, remand the case to the Court of First Instance for new trial or retrial, or dismiss the case." - Petitioner asserts, and correctly so, that the authority of respondent appellate court over an appealed case is broad and ample enough to embrace situations as the instant case where the court may grant a new trial or a retrial for reasons other than that provided in Section 13 of the same Rule, or Section 2, Rule 121 of the Rules of Court. While Section 13, Rule 124, and Section 2, Rule 121, provide for specific grounds for a new trial, i.e. newly discovered evidence, and errors of law or irregularities committed during the trial, Section 11, Rule 124 quoted above does not so specify, thereby leaving to the sound discretion of the court the determination, on a case to case basis, of what would constitute meritorious circumstances warranting a new trial or retrial. - Thus, admittedly, courts may suspend its own rules or except a case from them for the purposes of justice or, in a proper case, disregard them. In this jurisdiction, in not a few instances, this Court ordered

JOSE v CA 70 SCRA 257 MUOZ PALMA; March 31, 1976


FACTS - Jose was convicted of illegal possession of explosives (handgrenade) and sentenced to suffer imprisonment of five years. He seeks a new trial but was denied by the CFI of Pampanga and affirmed by the CA. - Jose was arrested by the local police for illegal discharge of firearm, robbery and illegal possession of explosives. Hon. Romero acquitted accused Jose of illegal discharge of firearm and robbery, but convicted him for illegal possession of the handgrenade that was found on his person at the time of his arrest. - After promulgation of the judgment, petitioner on that same day, filed his notice of appeal. Petitioner filed a motion praying that the case be reopened to permit him to present, pursuant to a reservation he had made in the course of the trial, a permit to possess the handgrenade in question. The trial court denied the motion mainly on the ground that it had

lost jurisdiction over the case in view of the perfection of the appeal by the accused on the very date the decision was promulgated. - The records were then elevated to the Court of Appeals where petitioner as accused-appellant raised the issues of (1) an erroneous conviction for illegal possession of explosives when there was no proof of an essential element of the crime, and (2) erroneous denial of his motion to reopen the case for the reception of his permit to possess the handgrenade. Jose prayed for his acquittal or in the alternative for the remand of the case back to the trial court for a new trial. CA affirmed RTC. - A motion for reconsideration and/or new trial was filed but was denied. - Jose filed before the SC but was denied. Thus this Motion for Reconsideration. - Manifestation was submitted by the Solicitor General informing the Court that in view of the "persistence of accused petitioner Lorenzo Jose both before this Honorable Court and respondent Court of Appeals as to his alleged existing appointment as PC Agent and/or authority to possess handgrenade," in the interest of justice, he was constrained to make pertinent inquiries from the PC Chief, Gen. Fidel V. Ramos who in reply sent his letter dated December 27, 1974 that states that Mr. Lorenzo Jose was appointed as PC Agent. The Solicitor General now concedes that the interests of justice will best be served by remanding this case to the court of origin for a new trial. ISSUE WON CA committed an error of law and gravely abuse its discretion when it denied petitioner's motion for new trial "for the reception of (1) the written permit of petitioner to possess and use handgrenade, and (2) the written appointment of petitioner as PC agent with Code No. P-36-68 and Code Name 'Safari' (both documents are dated 31 January 1968)" HELD YES - This is a situation where a rigid application of rules of procedure must bow to the overriding goal of courts of justice to render justice where justice is due to secure to every individual all possible legal means to prove his innocence of a crime of which he is charged. The failure of the Court of Appeals to appreciate the merits of the situation, involving as it does the liberty of an individual, thereby closing its ear to a plea that a miscarriage of justice be averted,

Criminal Procedure Rowena Daroy Morales


a new trial in criminal cases on grounds not mentioned in the statute, viz: retraction of witness, negligence or incompetency of counsel, improvident plea of guilty, disqualification of an attorney de oficio to represent the accused in the trial court, and where a judgment was rendered on a stipulation of facts entered into by both the prosecution and the defense. - Characteristically, a new trial has been described as a new invention to temper the severity of a judgment or prevent the failure of justice. - Petitioner cites certain peculiar circumstances obtaining in the case now before Us which may be classified as exceptional enough to warrant a new trial if only to afford human opportunity to establish his innocence of the crime charged. - Thus petitioner was facing a criminal prosecution for illegal possession of a handgrenade in the court below. He claimed to be an agent of the Philippine Constabulary with a permit to possess explosives such as the handgrenade in question. However, he found himself in a situation where he had to make a choice reveal his identity as an undercover agent of the Philippine Constabulary assigned to perform intelligence work on subversive activities and face possible reprisals or even liquidation at the hands of the dissidents considering that Floridablanca, the site of the incident, was in the heart of "Huklandia", or ride on the hope of a possible exoneration or acquittal based on insufficiency of the evidence of the prosecution. Without revealing his identity as an agent of the Philippine Constabulary, he claimed before the trial judge that he had a permit to possess the handgrenade and prayed for time to present the same. The permit however could not be produced because it would reveal his intelligence work activities. Came the judgment of conviction and with it the staggering impact of a five-year imprisonment. The competent authorities then realized that it was unjust for this man to go to jail for a crime he had not committed, hence, came the desired evidence concerning petitioner's appointment as a Philippine Constabulary agent and his authority to possess a handgrenade for the protection of his person, but, it was too late according to the trial court because in the meantime the accused had perfected his appeal. Dispositive PREMISES CONSIDERED, We hereby set aside the judgment of conviction of the herein petitioner, Lorenzo Jose, and remand the case to the court a quo for a new trial only for the purpose of allowing said accused to present additional evidence in his defense. The trial court shall inform this Court

a2010

page 151

Prof.
- The respondents appealed to the DOJ. The DOJ reversed the resolution and ordered that the informations be withdrawn. The corporation moved to reconsider but was denied by the DOJ. They then assailed the DOJ order before the CA. - In the meantime, respondents filed a motion to dismiss the criminal cases. Judge Baybay granted the motion. ISSUES 1. WON the judge correctly dismissed the criminal case 2. WON the only remedy for the petitioners was a petition fro certiorari, not an ordinary appeal HELD 1. NO - As to what mode of review petitioners may avail of after a court grants an accuseds motion to withdraw information and/or to dismiss the case, Section 1 of Rule 122 of the 2000 Revised Rules of Criminal Procedure instructs: "Any party may appeal from a judgment or final order, unless the accused will be placed in double jeopardy." - In availing of the remedy of certiorari before the RTC, petitioners claim that they had no plain, adequate and speedy remedy to question the MeTCs grant of the motion. -The records of the cases show, however, that the motion was granted by the MeTC before respondents were arraigned. Thus, the prohibition against appeal in case a criminal case is dismissed as the accused would be placed in double jeopardy does not apply. - Petitioners not having availed of the proper remedy to assail the dismissal of the cases, the dismissal had become final and executory. On this score alone, the present petition must fail. 2. YES - The judge made a finding independent of that of the DOJs. - The trial court did stress in its December 3, 2002 Order denying the motion for reconsideration that it was bound to make, as it did, a preliminary finding independently of those of the Secretary of Justice. - The trial judge need not state with specificity or make a lengthy exposition of the factual and legal foundation relied upon by him to arrive at his decision. It suffices that upon his own personal evaluation of the evidence and the law involved in the case, he is convinced that there is no probable cause to indict the accused. Dispositive Petition denied

of the final outcome of the case within a reasonable time.

FIRST WOMENS CREDIT v BAYBAY CARPIO MORALES; January 31, 2007


FACTS

First Womens Credit Corp filed a petition before the Securities and Exchange Commission (SEC) against the corporations officers Jacinto, Colayco, Sangil and Cruz, for alleged mismanagement of the corporation. - The SEC, in SEC Case No. 11-97-5816, created an Interim Management Committee (IMC) for the corporation by Order of November 17, 1999. The Order was upheld by the SEC en banc on July 4, 2000. - The IMC thereupon issued directives to the corporations president Antonio Tayao and corporate secretary and treasurer Glicerio Perez. - Allegedly in conspiracy with Jacinto and Colayco, Tayao and Perez defied the implementation of the SEC November 17, 1999 Order6 when IMC attempted to enter the main office of the corporation in Makati on December 3, 1999, December 29, 1999 and January 28, 2000 - The IMC then preventively suspended Tayao and Perez. However, Tayao and Perez continued to issue memoranda to the employees to disobey the IMC. Later, the IMC dismissed them both. - The corporation, represented by Katayama (minority stockholder), filed before the Makati City Prosecutor criminal complaints against Jacinto, Colayco, Tayao and Perez for: a) Article 151 which punishes resistance and disobedience to person in authority or the agents of such person (20 counts); b) Article 154 which punishes the unlawful use of means of publication and unlawful utterances (2 counts); c) Article 172(2) which punishes falsification by private individuals and use of falsified documents (2 counts); d) Article 315, paragraph 2(a) Estafa by falsely pretending to be officers of FWCC (23 counts) - The investigating prosecutor found no probable cause for violations under A151, A154 and A315. However, it found probable cause for 2 counts of violation of A 172(2) against Jacinto, Colayco and Perez, and 3 counts of grave coercion against Tayao and 3 secuirty guards.. The City prosecutor approved the investigating prosecutors resolution.

Criminal Procedure Rowena Daroy Morales


GALMAN v SANDIGANBAYAN [SUPRA, PAGE 82] PEOPLE v BAYOTAS 236 SCRA 239 ROMERO; September 2, 1994
FACTS - Rogelio Bayotas was charged with Rape and eventually convicted thereof. Pending appeal of his conviction, Bayotas died at the National Bilibid Hospital. Consequently, the Supreme Court dismissed the criminal aspect of the appeal. However, it required the Solicitor General to file its comment with regard to Bayotas' civil liability arising from his commission of the offense charged. In his comment, the Solicitor General expressed his view that the death of accused-appellant did not extinguish his civil liability as a result of his commission of the offense charged. - Counsel for the accused-appellant argues that the death of the accused while judgment of conviction is pending appeal extinguishes both his criminal and civil penalties. Said counsel invoked the ruling of the CA in People v. Castillo and Ocfemia which held that the civil obligation in a criminal case takes root in the criminal liability and, therefore, civil liability is extinguished if accused should die before final judgment is rendered. ISSUE WON death of the accused pending appeal of his conviction extinguishes his civil liability HELD YES Ratio Article 89 of the Revised Penal Code is the controlling statute. It reads, in part: Criminal liability is totally extinguished By the death of the convict, as to the personal penalties; and as to the pecuniary penalties liability therefor is extinguished only when the death of the offender occurs before final judgment. The term final judgment employed in the RPC means judgment beyond recall. Really, as long as a judgment has not become executory, it cannot be truthfully said that defendant is definitely guilty of the felony charged against him. If the private offended party, upon extinction of the civil liability ex delicto desires to recover damages from the same act or omission complained of, he must, subject to Section 1, Rule 111, file a separate civil action, this

a2010

page 152

Prof.
- Martinez filed a "Motion for Reinvestigation" which was denied by Judge Manuel E. Yuzon. The case was set for arraignment and pre-trial conference on July 31, 1990, but this setting was cancelled in view of Judge Yuzon's retirement. - Martinez filed a petition with the DOJ seeking review of the resolution of the City Prosecutor finding a prima facie case of libel against him. Accordingly, 3rd Asst. City Prosecutor Lourdes C. Tabanag filed before the trial court a motion to suspend proceedings pending resolution by the DOJ of Martinez' petition for review, which was granted by Judge Pepito. - Complainant Laurel attempted once more to have the case set for arraignment and trial. No action was taken on his said motion. - August 16, 1991: Acting Justice Secretary Silvestre H. Bello III declared inter alia that while the language used in the article may be unsavory and unpleasant to complainant, the same was not actionable as libel, as it embodied merely an opinion protected as a privileged communication under Article 354 of the RPC. The appealed resolution was set aside and the City Prosecutor was directed to cause the dismissal of the information filed against Manuel F. Martinez. Consequently, a motion to dismiss was filed on August 26, 1991 and set for hearing on December 17, 1991. At the hearing, upon manifestation of complainant's counsel, as private prosecutor, that he had received no copy of the motion to dismiss, the trial court directed the case prosecutor to furnish said counsel the desired copy, giving the latter ten (10) days to respond thereto. - Motion to dismiss was granted by Judge Roberto Barrios. - Laurel went to CA ascribing error to the lower court. - CA issued a Resolution granting the appeal and remanding the case for arraignment of the accused and trial on the merits. The Appellate Court ruled that private complainant had "sufficient personality and a valid grievance against the order of dismissal before arraignment" and that the remedy of appeal was property available because the order of dismissal was a final order which terminated all proceedings in the case. - The fault or error tainting the order of dismissal of the lower court consists in its failure to observe procedural due process and to exercise its discretion properly and judiciously. - The dismissal was based merely on the findings of the Acting Secretary of Justice that no libel was committed. The trial judge did not make an independent evaluation or assessment of the merits of the case.

time predicated not on the felony previously charged but on other sources of obligation. We summarize our ruling herein: 1. Death of the accused pending appeal of his conviction extinguishes his criminal liability as well as the civil liability based solely thereon. 2. Corollarily, the claim for civil liability survives notwithstanding the death of accused, if the same may also be predicated on a source of obligation other than delict. 3. Where the civil liability survives, an action for recovery therefor may be pursued but only by way of filing a separate civil action 4. The private offended party need not fear a forfeiture of his right to file this separate civil action by prescription, in cases where during the prosecution of the criminal action and prior to its extinction, the private-offended party instituted together therewith the civil action. In such case, the statute of limitations on the civil liability is deemed interrupted during the pendency of the criminal case, conformably with provisions of Article 1155 of the Civil Code, that should thereby avoid any apprehension on a possible privation of right by prescription. Applying this set of rules to the case at bench, we hold that the death of appellant Bayotas extinguished his criminal liability and the civil liability based solely on the act complained of, i.e., rape. Dispositive The appeal is DISMISSED.

MARTINEZ v CA (LAUREL) 237 SCRA 575 NARVASA; October 13, 1994


NATURE This petition for review prays for the reversal of the resolutions of the Court of Appeals. FACTS - Manuel P. Martinez actually seeks the dismissal of the information for libel filed against him in the Trial Court. - On complaint of then Vice-President Salvador H. Laurel, an Information was filed before the RTC of Manila by Assistant Prosecutor Antonio J. Ballena, charging Manuel P. Martinez with libel arising from the allegedly derogatory and scurrilous imputations and insinuations against Laurel contained in Martinez' article entitled "The Sorrows of Laurel" published on January 8, 1990 in his Manila Times column.

Criminal Procedure Rowena Daroy Morales


- The grant of the motion to dismiss was based upon considerations other than the judge's own personal individual conviction that there was no case against the accused. The trial judge must himself be convinced that there was indeed no sufficient evidence against the accused, and this conclusion can be arrived at only after an assessment of the evidence in the possession of the prosecution. Petitioners Claim - Martinez moved to dismiss the appeal on the ground that no appeal lies from the dismissal of a criminal case, and certainly not by the private complainant, particularly where dismissal was at the instance of the City Prosecutor upon orders of the Department of Justice. - If any remedy was available to private complainant, it was a petition for certiorari, not an appeal. ISSUE WON complainant is allowed to file an appeal HELD YES Ratio The right to appeal from a final judgment or order in a criminal case is granted to "any party", except when the accused is placed thereby in double jeopardy. Reasoning Section 2, Rule 122 RCP "Who may appeal. Any party may appeal from a final judgment or order, except if the accused would be placed thereby in double jeopardy. - Court ruled that the word "party" must be understood to mean not only the government and the accused, but also other persons who may be affected by the judgment rendered in the criminal proceeding. Thus, the party injured by the crime has been held to have the right to appeal from a resolution of the court which is derogatory to his right to demand civil liability arising from the offense. (People v. Guido) - Offended parties in criminal cases have sufficient interest and personality as 'person(s) aggrieved' to file the special civil action of prohibition and certiorari under Sections 1 and 2 of Rule 65 in line with the underlying spirit of the liberal construction of the Rules of Court - The procedural recourse of appeal taken by private complainant Laurel is correct because the order of dismissal was a final order. It finally disposed of the pending action so that nothing more could be done with it in the lower court.

a2010

page 153

Prof.
- Warden of Bilibid Prison says Cruz is now serving the subsidiary imprisonment on account of his failure to pay the P1,000 fine in case#1. At the rate of P2.50 a day, said subsidiary imprisonment will expire about the 9th of July, 1911. - Oct 21, 1910: writ as prayed for was issued. Hearing was set the next day. ISSUE WON that part of the sentence of the CFI condemning Cruz to subsidiary imprisonment in case of insolvency in the payment of the P1,000 fine is legal HELD NO - CFI had jurisdiction of the offense complained of. It had jurisdiction of the prisoner who was properly brought before it. It had jurisdiction to hear and decide upon the defense offered by him, but it did not have power to sentence the petitioner to subsidiary imprisonment in case of insolvency in the payment of the fine imposed. It is therefore clear that that part of the judgment is void. - Act No. 1732 (effective November 1, 1907): when a fine is imposed as a whole, or as any part of the punishment for any criminal offense made punishable by any Act of the Philippine Commission, the court shall also sentence the guilty person to subsidiary imprisonment until the fine is satisfied; provided that such subsidiary imprisonment shall not, in any case, exceed one year; but in case the court imposes both a fine and imprisonment the subsidiary imprisonment shall not exceed one-third of the term of imprisonment imposed by such sentence. - This provision is not applicable to Cruz because the penalty was imposed upon him long before Act No. 1732 went into effect. Penal statutes can not have a retroactive effect for the reason that such effect would not be beneficial to the petitioner. (Art. 22, Penal Code; US v Macasaet) - Prior to the passage of Act No. 1732, CFIs had no authority to impose subsidiary imprisonment for failure to pay fines in cases of conviction for violations of the Acts issued by the Philippine Commission. Such errors (regarding authority to impose the penalty of subsidiary imprisonment in case of insolvency) when committed have been corrected by SC in those cases which were appealed: US v Hutchinson, US v Lineses, and US v Macasaet, among them.

- The remedy against such a judgment is an appeal, regardless of the questions sought to be raised on appeal, whether of fact, or of law, whether involving jurisdiction or grave abuse of discretion of the Trial Court. . . . (T)he party aggrieved . . . did not have the option to substitute the special civil action of certiorari under Rule 65 for the remedy of appeal provided for in Rule 41. Indeed, the existence and availability of the right of appeal are antithetical to the availment of the special civil action of certiorari. - The rule therefore in this jurisdiction is that once a complaint or information is filed in Court any disposition of the case as its dismissal or the conviction or acquittal of the accused rests in the sound discretion of the Court. Although the fiscal retains the direction and control of the prosecution of criminal cases even while the case is already in Court he cannot impose his opinion on the trial court. The Court is the best and sole judge on what to do with the case before it. (Crespo v. Mogul) Dispositive Petition is denied.

CRUZ v DIRECTOR OF PRISONS 17 PHIL 269 TRENT; November 3, 1910


NATURE Petition for writ of habeas corpus FACTS - Juan M. Cruz was tried, convicted, and sentenced by CFI Manila for violation of the provisions of Act No. 292 by the Phil Commission (effective Nov 4, 1901). - Case#1: conspiracy against the government. He was sentenced to 3 yrs imprisonment, and fine of P1,000 with subsidiary imprisonment in case of insolvency in the payment of the fine. - Case#2: sedition. He was sentenced to imprisonment of 2 yrs, and fine of US $2,000 (=PhP4000). This last case was appealed to, and affirmed by SC, without subsidiary imprisonment. - Nov 15, 1905: Cruz commenced to serve these sentences. - Nov 15, 1910: supposed expiry of 5yr total prison term. - June 4, 1910: expiry of prison term on account of good conduct allowance, as provided by Act No. 1533 (Cruz was not allowed the full time for good conduct on account of certain violations of prison regulations). - Oct 14, 1910: petition for writ oh habeas corpus was filed in behalf of Cruz.

Criminal Procedure Rowena Daroy Morales


- CFI did not have power to sentence Cruz to subsidiary imprisonment in case of insolvency in the payment of the fine imposed. - SC at this time has no power to correct this error committed by the court below, neither has it power to remand the case to the trial court for that purpose. The fact that Cruz did not appeal can not affect the question as the two penalties imposed are separate and distinct. - The courts uniformly hold that where a sentence imposes a punishment in excess of the power of the court to impose, such sentence is void as to the excess, and some of the courts hold that the sentence is void in toto; but the weight of authority sustains the proposition that such a sentence is void only as to the excess imposed in case the parts are separable, the rule being that the petitioner is not entitled to his discharge on a writ of habeas corpus unless he has served out so much of the sentence as was valid. - Warden agrees Cruz has served out the entire part of the sentences which CFI had power to impose, and adhering to the rule that that part of the sentences imposed by the court below in excess of its jurisdiction is void, the petitioner is entitled to his release. Dispositive Cruz ordered discharged from custody.

a2010

page 154

Prof.
exemplary damages to deter other fathers with perverse tendencies or aberrant sexual behavior from sexually abusing their own daughters Dispositive Judgment appealed from was affirmed with the modification.

PEOPLE v CULA 329 SCRA 101 MELO; March 28, 2000


FACTS - Accused-appellants Manuel Cula Y Bandilla and Joselito Lopez Y Roco were charged for raping the formers 16-year old daughter, Maricel Cula. - Accused-appellants were found guilty beyond reasonable doubt of the crime of rape as charged in the Information. - Manuel Cula was sentenced to suffer the penalty of death while Joselito Lopez, the penalty of Reclusion Perpetua. ISSUE WON the court erred in imposing the penalty of death HELD YES - It is a well-established rule in criminal procedure that an appeal in a criminal proceeding throws the whole case open for review and it becomes the duty

of the appellate court to correct an error in the appealed judgment, whether this is assigned as an error or not. - In the case at bar, the trial court, pursuant to Section 11 of Republic 7659, imposed the penalty of death upon accused-appellant Manuel Cula, taking into account the minority of Maricel as she is said to have been only 16 years old at the time of the rape incident, as well as the relationship of father and daughter between them. - People vs. Javier: However it is significant to note that the prosecution failed to present the birth certificate of the complainant. Although the victims age was not contested by the defense, proof of age of the victim is particularly necessary in this case considering that the victim's age which was then 16 years old is just two years less than the majority age of 18. - At all events, it is the burden of the prosecution to prove with certainty the fact that the victim was below 18 when the rape was committed in order to justify the imposition of the death penalty. The record of the case is bereft of any independent evidence, such as the victim's duly certified Certificate of Live Birth, accurately showing private complainant's age. - The fact that accused-appellant Manuel has not denied the allegation in the complaint that Maricel was 16 years old when the crime was committed cannot make up for the failure of the prosecution to discharge its burden in this regard. - Because of this lapse, as well as the corresponding failure of the trial court to make a categorical finding as to the minority of the victim, we hold that the qualifying circumstance of minority under Republic Act No. 7659 cannot be appreciated in this case, and accordingly the death penalty cannot be imposed. - The award of damages made by the trial court should likewise be modified. As regards the civil indemnity, this Court has to date consistently ruled that if, in the crime of rape, the death penalty is imposed, the indemnity ex delicto for the victim should be in the amount of P75,000.00; and if the death penalty is not decreed by the Court, the victim would instead be entitled to P50,000. Accordingly, accused~appellants shall each pay the amount of P50,000.00 as civil indemnity for each count of rape. In addition, as held in People vs. Prades, both accused-appellants must each indemnify the victim the amount of P50,000.00 as moral damages for each count of rape without the need of pleading or proof as the basis thereof. Lastly, accused~appellant Manuel is also liable to pay the sum of P20,000.00 as

You might also like